intranet.tdmu.edu.uaintranet.tdmu.edu.ua/data/kafedra/internal/infect_desease... · web viewbest...

573
Actual problems of infectious diseases and HIV infection Tests 1. Drug of choice for cholera prophylaxis is: A. *oxytetracycline B.chloramphenicol C. erythromycin D. penicillin E. none of these 2. The function of glucose in ORS (oral rehydration solution): A. increase Na+ absorption by Co-transport B.gives sweet taste to ORS C. increase osmalality of ORS D. *increase Na+ K- pump activity E. increase Ca+ absorption 3. El-Tor vibrio may be differentiated from classical vibrio by the fact that El-Tor vibrio: A. agglutinate chicken and sheep RBC B.resistant to classical phage IV C. resistant to polymixin B-5 unit disc D. *all of the above E. none of these 4. Chemo-prophylaxis for cholera is administrating: A. *doxycycline 300 mg once B.metrogyl 400 mg 3 tablets C. vancomycin 1 mg stat D. kanamycin 500 mg stat E. lincomycin 1 g 5. The average incubation period of cholera is: A. 24 hours B.*48 hours C. 72 hours D. 96 hours E. 12 hours 6. Which is not essential in cholera epidemic: A. notification B.oral rehydration therapy and tetracycline C. chlorination of well every week

Upload: lythuy

Post on 20-Apr-2018

252 views

Category:

Documents


1 download

TRANSCRIPT

Page 1: intranet.tdmu.edu.uaintranet.tdmu.edu.ua/data/kafedra/internal/infect_desease... · Web viewBest method to treat diarrhoea in child is: intra venous fluide *ORS antibiotics bowel

Actual problems of infectious diseases and HIV infectionTests1. Drug of choice for cholera prophylaxis is:

A.*oxytetracyclineB.chloramphenicolC.erythromycinD.penicillinE.none of these

2. The function of glucose in ORS (oral rehydration solution):A.increase Na+ absorption by Co-transportB.gives sweet taste to ORSC.increase osmalality of ORSD.*increase Na+ K- pump activityE. increase Ca+ absorption

3. El-Tor vibrio may be differentiated from classical vibrio by the fact that El-Tor vibrio:

A.agglutinate chicken and sheep RBCB.resistant to classical phage IVC.resistant to polymixin B-5 unit discD.*all of the aboveE.none of these

4. Chemo-prophylaxis for cholera is administrating:A.*doxycycline 300 mg onceB.metrogyl 400 mg 3 tabletsC.vancomycin 1 mg statD.kanamycin 500 mg statE. lincomycin 1 g

5. The average incubation period of cholera is:A.24 hoursB.*48 hoursC.72 hoursD.96 hoursE.12 hours

6. Which is not essential in cholera epidemic:A.notificationB.oral rehydration therapy and tetracyclineC.chlorination of well every weekD.*isolationE.chemo-prophylaxis

7. Oral rehydration therapy does not contain:A.sodium chlorideB.*calcium lactateC.bicarbonateD.glucoseE.none of these

8. Best method to treat diarrhoea in child is:

Page 2: intranet.tdmu.edu.uaintranet.tdmu.edu.ua/data/kafedra/internal/infect_desease... · Web viewBest method to treat diarrhoea in child is: intra venous fluide *ORS antibiotics bowel

A.intra venous fluideB.*ORSC.antibioticsD.bowel bindersE. lavage of stomach

9. ORS contains how much potassium:A.*20B.30C.40D.10E.50

10. Certificate to cholera vaccination is valid after:A.5 daysB.*10 daysC.15 daysD.20 daysE.25 days

11. Drug of choice for treating cholera in a pregnant women is:A.tetracyclineB.doxycyclineC.*furazolidoneD.cotrimoxozoleE.none of these

12. Best emergency sanitary measure to control cholera is:A.disinfection of stoolB.mass vaccinationC.*provision of chlorinated waterD.chemoprophylaxisE.none of these

13. Drug of choice in cholera treatment is:A.*tetracyclineB.sulphadiazineC.erythromycinD.ampicillinE.none of these

14. A contact carrier in cholera has following characteristic:A.gall bladder is infectedB.stool is not positive for vibrio choleraC.does not play any role in spread of infectionD.*duration of carrier state is less than 10 daysE.none of these

15. Quantity of NaCl in an ORS packet for making 1 litre of oral rehydration fluid is:A.*3,5 gramB.2,5 gramC.1,5 gramD.2 gram

Page 3: intranet.tdmu.edu.uaintranet.tdmu.edu.ua/data/kafedra/internal/infect_desease... · Web viewBest method to treat diarrhoea in child is: intra venous fluide *ORS antibiotics bowel

E.3 gram16. A freshly prepared oral rehydration solution should not be used after:

A.4 hoursB.6 hoursC.12 hoursD.*24 hoursE.48 hours

17. Regarding cholera vaccine which one of following is true:A.it is given at interval of 6 monthsB.long lasting immunityC.not useful in epidemicsD.*not given orallyE. is high effective

18. ORS rehydration fluid does not contain:A.NaClB.*calcium lactateC.bicarbonateD.glucoseE.none of these

19. What is the transport medium for cholera:A.tellurinate mediumB.chacko-nair mediumC.*venkatraman-ramakrishna mediumD.Mc-Leods mediumE.none of these

20. Which of the following about cholera is true:A.inavasiveB.endotoxin is releasedC.*vibriocidal antibody titre measure prevalenceD.all of theseE.none of these

21. Vibrio cholera was discovered by:A.*KochB.MechnicovC.JohnsnowD.VirchowE.Jenner

22. The characteristic feature of El-Tor cholera are all except:A.more of subclinical casesB.mortality is lessC.*secondary attack rate is high in familyD.El-Tor vibrio is harder and able to survive longerE. severity is less

23. The growth factor required for growth of vibrio paraheamolyticus is:A.*salineB.tryptophan

Page 4: intranet.tdmu.edu.uaintranet.tdmu.edu.ua/data/kafedra/internal/infect_desease... · Web viewBest method to treat diarrhoea in child is: intra venous fluide *ORS antibiotics bowel

C.bileD.citrateE. sugar

24. True about vibrio cholera is:A.*very resistant to alkaline PHB.nutritionally fastidiousC.best growth at 24 oCD.rod shaped bacilliE.all of these

25. The following are true about vibrio cholera except:A.*produces indole and reduces nitraresB.dies rapidly at low temperatureC.synthesises neuraminidasesD.vaccine confirms long immunityE.none of these

26. True about epidemiology of cholera is:A.*chemoprophylaxis is not effectiveB.boiling of water can’t destroy organismC.food can transport diseaseD.vaccination give 90 % protectionE. rehydration is not effective

27. What percentage of fluid loss will be in IV degree of dehydration?A.4-8 % of body weightB.6-9 % of body weightC.3-6 % of body weightD.*Over 10 % of body weightE.Over 15 % of body weight

28. At what percent of fluid loss will be I degree of dehydration?A.*3-6 % of body weightB.6-9 % of body weightC.1-3 % of body weightD.0,5-2 % of body weightE.2-7 % of body weight

29. At I degree of dehydration the loss of liquid is:A.0,5-1,5 % of body weightB.6-9 % of body weightC.3-6 % of body weightD.5-8 % of body weightE.*1-3 % of body weight

30. At what percent of fluid loss will be II degree of dehydration?A.3-6 % of body weightB.Over 10 % of body weightC.*6-9 % of body weightD.4-8 % of body weightE.10-15 % of body weight

31. At what degree of dehydration, there will be “metabolic violation”:

Page 5: intranet.tdmu.edu.uaintranet.tdmu.edu.ua/data/kafedra/internal/infect_desease... · Web viewBest method to treat diarrhoea in child is: intra venous fluide *ORS antibiotics bowel

A.SubcompensatedB.*NegativeC.IrreversibleD.Moderate metabolic acidosisE. Insignificant metabolic alkalosis

32. What time is it necessary to complete primary rehydration at dehydration shock?A.3-5 hrsB.0.5 hrsC.2-3 hrsD.*1-1.5 hrsE.4-6 hrs

33. What from the below mentioned preparations, can be used for the treatment of primary rehydration?

A.*RehydronB.AcesaltC.KhlosaltD.KvartasaltE.Lactosalt

34. What from the below mentioned preparations, can be used for the treatment of primary rehydration?

A.AcesaltB.TrisaltC.*OralitD.CryoplasmaE.Lactosalt

35. What from the below mentioned preparations can be used for the treatment of primary rehydration?

A.LactosaltB.*DisaltC.AcesaltD.TrisaltE.Khlosalt

36. What clinically atypical forms of cholera do you know?A.*Very rapid of the children and elderly personsB.“Choleric typhoid”, acute subclinical, for the children and elderly personsC.Dry, very rapid, “choleric typhoid”, subclinical for the children and elderly personsD.Very rapid “choleric typhoid”, acute, subclinical, for the children and elderly

persons37. In a settlement was found out a few cases of cholerA. Who must be insulated?

A.with dysfunction of intestineB.Patients with choleraC.CarriersD.*Persons contact with the sick patientE.Persons with hyperthermia

38. Who must be admitted in the hospital from the focus of cholera?A.Carriers

Page 6: intranet.tdmu.edu.uaintranet.tdmu.edu.ua/data/kafedra/internal/infect_desease... · Web viewBest method to treat diarrhoea in child is: intra venous fluide *ORS antibiotics bowel

B.Patients with choleraC.*Persons with dysfunction of intestineD.Contact personsE.Persons with high temperature

39. In the break out of cholera it is necessary to carry out such measures, except:A.Hyper chlorination of drinking waterB.An active discovery of patients by roundsC.Obligatory hospitalization, inspection and treatment of patients and vibrio testsD.Revealing and isolation of contact personsE.*Vaccine prophylaxis

40. With the purpose of specific prophylaxis of cholera is used:A.*Cholerogen-toxoidB.VaccineC.NitrofuranesD.ImmunoproteinE.Antibiotics

41. In the different places of settlement found out a few cases of disease of cholerA. Who from the contacts of cholera patient is sent in an insulator?

A.Vibrio positiveB.Patients with choleraC.*Contact with the patient personsD.Persons with dysfunction of intestineE.Persons with high temperature

42. Which from the below is a complication of cholera?A.CollapseB.Infectious-toxic shockC.Acute renal insufficiencyD.*Dehydration shockE.Status typhosis

43. What salt solutions do not contain potassium?A.TrisaltB.LactosaltC.*DisaltD.QudrosaltE.Khlosalt

44. For the rehydration in dehydration shock it is necessary to conduct the permanent careful account of all losses of liquid in each:

A.4 hrsB.30 hrsC.3 hrsD.*2 hrsE.5 hrs

45. In 1 liter of Trisalt solution, the concentration of potassium chloride is:A.3 g/lB.1.5 g/lC.*1.0 g/l

Page 7: intranet.tdmu.edu.uaintranet.tdmu.edu.ua/data/kafedra/internal/infect_desease... · Web viewBest method to treat diarrhoea in child is: intra venous fluide *ORS antibiotics bowel

D.2 g/lE.2.5 g/l

46. The essential therapy for cholera is.A.DietB.Antibacterial preparationsC.Correction of dysbacteriosisD.DesintoxicationE.*Primary rehydration

47. The main principle of therapy for re-hydration in cholera is.A.Determining the definitive degree of dehydration from clinical dataB.Amount of lost liquid which was preceded at time of hospitalizationC.Application of isotonic polyglucal solutionD.Simultaneous introduction of liquid in more than one vesselE.*All are correct

48. Duration of therapy of primary rehydration in cholera is.A.30 minutesB.*2 hoursC.6 hoursD.12 hoursE.1 days

49. Amount of solutions necessary for the primary rehydration in cholera is.A.*Accordingly to the degree of dehydration at time of hospitalizationB.In accordance with the loss of liquidC.2 lD.5 lE.10 l

50. Amount of solutions necessary for the secondary rehydration in cholera is.A.*Accordingly to the degree of dehydration at the time of hospitalizationB.In accordance with the loss of liquidC.2 lD.5 lE.10 l

51. What from is the given measures during the secondary rehydration?A.Determining degree of dehydration from clinical dataB.Amount of lost liquid, which was preceded at the time of hospitalizationC.Application of isotonic crystalloid solutionsD.Simultaneous introduction of liquid in a few vesselsE.*Amount of liquid loss

52. What solutions must be applied for compensatory rehydration in cholera?A.ColloidB.Hypertensive epitonic polyionic crystalloidC.*Isotonic polyionic crystalloidD.ReosorbilactE. Isotonic solution of glucose

53. Method of etiotropic therapy of cholera is.A.Glucocorticoids

Page 8: intranet.tdmu.edu.uaintranet.tdmu.edu.ua/data/kafedra/internal/infect_desease... · Web viewBest method to treat diarrhoea in child is: intra venous fluide *ORS antibiotics bowel

B.AntiviralC.*AntibioticsD.RehydrationE.Vaccine

54. In the different places of settlement it is found out a few cases of cholerA. Who from such place is directed to an insulator?

A.Patients with a choleraB.TransmittersC.*Persons who had contact with the patientD.Persons with dysfunction of gastro-intestinal tractE.Persons who left the place on infection

55. For cholera prophylaxis drug is:A.erythromycinB.ampicillinC.*tetracyclineD.biseptolE.penicillin

56. What mechanism is typical for salmonellosis.A.*Fecal-oralB.ContactC.TransmissiveD.Air-dropE.All possible

57. In order to prevent salmonellosis should be.A.DisinfectionB.VaccinationC.ChemoprophylaxisD.*Sanitary and epidemiological control over foodE.All these measures are not undertaken

58. What group of infectious diseases salmonellosis belong to?A.SapronozB.*ZoonosisC.AntroponozD.ZooantroponozE.The group is not defined

59. Salmonella is classified by.A.*O-antigen and H-antigenB.O-antigen and Vi-antigenC.H-antigen and Vi-antigenD.O-antigen, H-antigen and Vi-antigenE.O-antigen, H-antigen, Vi-antigen and HBsAg

60. What salmonella is adapted to humans.A.*S. typhiB.S. newportC.S. cholerae-suisD.S. abortus-ovis

Page 9: intranet.tdmu.edu.uaintranet.tdmu.edu.ua/data/kafedra/internal/infect_desease... · Web viewBest method to treat diarrhoea in child is: intra venous fluide *ORS antibiotics bowel

E.S. gallinarum-pullorum61. What is the level of morbidity of salmonellosis nowadays in Ukraine.

A.Not registeredB.EpidemicC.*SporadicD.Annual outbreaksE. In endemic focus only

62. How often chronic carriering formed after salmonellosis.A.Not formedB.*in 0,1-1 %C.in 8-10 %D.in 20-30 %E. in 50-80 %

63. The source of agent in salmonellosis is.A.CatsB.*Farm animalsC.RodentsD.SoilE.Feces of patients

64. Greatest epidemiological role in spreding of salmonella belong to.A.*CattleB.Gray ratsC.MiceD.FishE.Man

65. What is mechanism of transmission of salmonellosis.A.VerticalB.ParenteralC.Air-dropD.ContactE.*Fecal-oral

66. What is most important factor in salmonellosis transmission.A.Boiled meatB.FishC.WaterD.SexE.*Eggs

67. What route of transmission is not inherent to Salmonella typhimurium.A.MilkB.Contact homeC.WaterD.*SexE.Food

Page 10: intranet.tdmu.edu.uaintranet.tdmu.edu.ua/data/kafedra/internal/infect_desease... · Web viewBest method to treat diarrhoea in child is: intra venous fluide *ORS antibiotics bowel

68. What typical dietary factor in spreading of salmonellosis.A.JuicesB.AlcoholC.*Meat productsD.SaladE.Water

69. What season is typical for salmonellosis.A.SpringB.Winter and springC.AutumnD.WinterE.*Summer-autumn

70. What is the kind of immunity after salmonellosis.A.InhereditedB.*Type specificC.Short termD.Not formedE.Passive

71. What type of outbreaks appear in salmonellosis.A.WaterB.HomeC.FarmD.*FoodE.Milk

72. What preventive and antepidemic activities in salmonella focused on the first link of epidemic process.

A.*Veterinarian measuresB.Revealing, hospitalization and treatment of sick peopleC.Systematic sanitary-hygienic controlD.DisinfectionE.Vaccination

73. The rules of discharging of salmonellosis patients from a hospital .A.One-time negative bacteriological investigation of stoolB.*Three negative bacteriological investigation of stoolC.14 days normal body temperature and the double negative bacteriological study

stool and urineD.Clinical recovery and normalization rectomanoscopy pictureE.Normalization rectomanoscopy picture and in the absence of antiserum to RNGA

74. Demands according more than 3 months salmonella carrier who are working in food production.

A.DyspanserizationB.Recently released from workC.RehospitalizationD.*Do not allow to workE.Do nothing

Page 11: intranet.tdmu.edu.uaintranet.tdmu.edu.ua/data/kafedra/internal/infect_desease... · Web viewBest method to treat diarrhoea in child is: intra venous fluide *ORS antibiotics bowel

75. All laboratory and instrumental tests are needed to confirming the diagnosis of food poisoning, except:

A.*General blood analysisB.CoprogramC.Occupied emptyingD.Occupied sourcesE.Serum researches with the autoculture of substance

76. The etiologic diagnosis of acute intestinal infections can be confirmed thus, except for:

A.Separation of pathogen from patients and from remainder of suspicious productB.To obtain identical cultures of bacteria from a few patients from those which

consumed that mealC.*Separation of identical cultures from different materials (washings, vomiting

mass, excrement) at one patient at the bacterial semination them no less than 105/g and diminishing of this index in the process of convalescence

D.Presence at the selected culture of Escherichia’s and staphylococcus enterotoxinE.Positive agglutination reaction or other immunological reactions with autoshtames

of possible pathogen, which testify to growth of title of antibodies on the blood serum of patient in the dynamics of disease

77. What is necessary for bacteriological confirmation of clostridial gastroenterocolitis diagnosis?

A.Endo‘s medium, thermostatB.Ploskirev‘s medium and blood agarC.Blaurock‘s medium, thermostatD.Endo‘s medium, anaerostatE.*Blaurock‘s media, anaerostat

78. Which from the listed products can become the causal factor of toxic food-borne infection?

A.Decorative cakesB.*GalantineC.CheeseD.Fresh breadE.Tea

79. What inoculums material should be taken to discharge the toxins?A.*Suspected foodB.UrineC.StoolD.Vomiting massE.Medullar

80. What is the duration of incubation period in food poisoning?A.*2 hours – 24 hoursB.3 daysC.1 weekD.1 monthE.1 years

81. What methods can confirm the diagnosis of food poisoning?

Page 12: intranet.tdmu.edu.uaintranet.tdmu.edu.ua/data/kafedra/internal/infect_desease... · Web viewBest method to treat diarrhoea in child is: intra venous fluide *ORS antibiotics bowel

A.Diagnostic confirmation requires isolating staphyloccocci from the urineB.Diagnostic confirmation requires isolating staphyloccocci from the stoolC.Diagnostic confirmation requires isolating staphyloccocci from the liquorD.*Diagnostic confirmation requires isolating staphyloccocci from the suspected

foodE.All above it

82. When the specific complication of typhoid fever like intestinal bleeding may appier?

A.On the 1st week of illnessB.On the 2nd week of illnessC.*On the 3rd week of illnessD.On the 4th week of illnessE.On the any week of illness

83. When the specific complication of typhoid fever like perforation of a bowel may appier?

A.On the 1st week of illnessB.On the 2nd week of illnessC.*On the 3rd week of illnessD.On the 4th week of illnessE.On the any week of illness

84. Who is the source of typhoid fever?A.*Sick peopleB.Sick agricultural animalsC.Sick rodentsD.SoilE.Defecating of patients

85. Salmonella typhi contains:A.Only O-antigen and Н-antigenB.Only O-antigen and Vi-antigenC.Only H-antigen and Vi-antigenD.*O-antigen, H-antigen and Vi-antigenE.O-antigen, H-antigen, Vi- antigen and HBsAg

86. When is it possible to stop etiotropic treatment in typhoid fever?A.At once after normalization of temperature of bodyB.After normalization of sizes of liver and spleenC.After disappearance of roseollasD.In 10 days after disappearance of roseollasE.*After the 10th day of normalisation of body temperature

87. By what method is it possible to find out bacterial carriering in typhoid fever?A.CoprocultureB.Reaction of agglutination of VidallC.Indirect hemaglutination test with О-аntigenD.Indirect hemaglutination test with a Н-antigenE.*Indirect hemaglutination test with a Vi-antigen

88. Typhoid bacilli are usually cultured from:A.*Blood, stool, urine

Page 13: intranet.tdmu.edu.uaintranet.tdmu.edu.ua/data/kafedra/internal/infect_desease... · Web viewBest method to treat diarrhoea in child is: intra venous fluide *ORS antibiotics bowel

B.Blood, urine, sputumC.Stool, liquor, urineD.Blood, stool, sputumE.Stool, liquor, sputum

89. A suspected case of typhoid fever of 1st week is admitted in the hospital. What examination (laboratory diagnosis) do you suggest for this patient?

A.CoprocultureB.Reaction of agglutination of VidallC.Indirect hemaglutination test with О-, Н-, Vi-аntigensD.UrinocultureE.*Hemoculture

90. A suspected case of typhoid fever of 3nd week is admitted in the hospital. What examination (laboratory diagnosis) do you suggest for this patient?

A.CoprocultureB.Reaction of agglutination of VidallC.UrinocultureD.HemocultureE.*All about it

91. What from the transferred signs is not characteristic for a typhoid rash?A.*Papular, disappears together with normalization of temperature of bodyB.Appears on a 7-10th day, roseolla-typeC.Located mainly on a abdomen and lateral surfaces of trunk, observed at the half of

patientsD.The amount of elements is limited, pours in additionE.Roseolla-type, sometimes saved longer than fever

92. What symptom is not typical for typhoid on the second week of illness?A.ConstipationB.HeadacheC.FeverD.Relative bradycardiaE.*Cramps

93. What changes in general blood analysis are typical for typhoid fever?A.*Leykopenia, aneosiniphilia, lymph-, monocytosis, enhanceable RSEB.Leykocytosis, hypereosinophilia, thrombocytosis, enhanceable RSEC.Hypochromatic anaemia, leycocytosis, appearance of young forms, RSE is not

changedD.Hyperchromatic anaemia, leycocytosis, appearance of young forms, enhanceable

RSEE.Leykopenia, lymphopenia, thrombocytosis, enhanceable RSE

94. What does the diagnostic titre of reaction of Vі-haemaglutination testify to?A.*Typhoid bacterial-carrierB.Epidemic typhusC.MeningococcaemiaD.MalariaE.Brucellosis

95. What is the duration of observation in typhoid fever?

Page 14: intranet.tdmu.edu.uaintranet.tdmu.edu.ua/data/kafedra/internal/infect_desease... · Web viewBest method to treat diarrhoea in child is: intra venous fluide *ORS antibiotics bowel

A.14 daysB.*21 daysC.7 daysD.30 daysE.Does not look after

96. Reconvalescents of typhoid fever may discharged from a clinic after:A.Non-permanent negative bacteriologic examination of defecatingB.*21th day of normal temperature of body and 3-multiple negative bacteriologic

examination of excrement and urineC.of 14th day of normal temperature of body and 2-multiple negative bacteriologic

examination of excrement and urineD.Clinical convalescence and normalization of rectal manoscopic pictureE.Normalisations of rectal manoscopic picture and in default of title of antibodies in

RNGA97. In focus of typhoid fever doing, except for:

A.ThermometryB.CoprocultureC.Vidal reactionD.*HaemocultureE.Urine culture

98. What from the adopted ways of transmission is characteristic for typhoid fever?A.*AlimentaryB.ContactC.TransmissionD.Air-dropE.Vertical

99. What environments do typhoid sticks grow on well?A.Chicken embryosB.water-whey nourishing environmentC.*Bilious clear soupD.Meat-peptone gelose + cistinE.To the Bismute-sulfate gelose

100. Name of the basic factors of pathogen of typhoid stick?A.*Vi-antigen and endotoxinB.ExotoxinC.Vi-antigenD.Enzymes of pathogenicityE.Endotoxin

101. Duration of latent period of typhoid fever?A.3-7 daysB.*9-21 dayC.From a few hovers to 2-3 daysD.From 12 to 100 daysE.From a few hovers to 17 days

102. The place of reproduction of typhoid bacterium in the organism of human is:A.Stomach

Page 15: intranet.tdmu.edu.uaintranet.tdmu.edu.ua/data/kafedra/internal/infect_desease... · Web viewBest method to treat diarrhoea in child is: intra venous fluide *ORS antibiotics bowel

B.*Lymphatic formations of colonC.BloodD.Bilious waysE.Mucous membrane of colon

103. What from the indicated pathology anatomic phases is not characteristic for typhoid?

A.*Catarrhal inflammationB.Cerebral-type of swellingC.NecrosisD.UlcersE.Clean ulcers

104. Food poisoning due to Staphylococcus aureus has an incubation period of:A.*1 - 6 hoursB.6 – 12 hoursC.12 – 18 hourD.more then 18 hoursE. less then 1 hour

105. Material which should be taken for bacteriological examination in case of food poisoning include:

A.Suspicion food productsB.Vomiting massC.Stool of patientD.Washing massE.*All of above

106. The immediate treatment for toxic food borne infection is:A.Gastric lavageB.SorbentsC.AntibioticsD.*Both A & BE.All of above

107. Which of the following is not a causative agent of food poisoning:A.Staphylococcus aureusB.Bacillus cereusC.Streptococcus haemoliticusD.Clostridium perfringensE.*Yersinia pestis

108. What is the main clinical symptom of food poisoning:A.HeadacheB.High feverC.ConstipationD.DiarrheaE.*Vomiting

109. Pathogenesis of food borne infections involves the production of:A.*EnterotoxinsB.EndotoxinsC.Necrotoxins

Page 16: intranet.tdmu.edu.uaintranet.tdmu.edu.ua/data/kafedra/internal/infect_desease... · Web viewBest method to treat diarrhoea in child is: intra venous fluide *ORS antibiotics bowel

D.All of aboveE.None of above

110. The following are characteristic features of Staphylococcus aureus food poisoning except:

A.*Optimum temperature for toxin formation is 37B.Intra dietetic toxinns are responsible for intestinal symptomsC.Toxins can be destroyed by boiling for 30 minD.Incubation period is 1-6 hoursE.All of above

111. Differential diagnosis of food poisoning is done with:A.CholeraB.ShigellosisC.SalmonellosisD.Rota viral infectionE.*All of above

112. Which of the following is frequent complication of food poisoning:A.*Acute heard insufficiencyB.Acute renal insufficiencyC.Acute lung insufficiencyD.Acute brain insufficiencyE.All of above

113. Etiology agent of botulism is:A.CampylobacterB.Balantidium coliC.*Cl. botulinumD.Cl. perfrigensE.St. Aureus

114. What groups of symptoms are occurs in the clinic of botulism?A.Vomiting, high temperatureB.*Dysfunction of speech and vision, breathing, dysphagiaC.Sickness, general weaknessD.High temperature, diarrhea, speech dysfunctionE.Diarrhea, vomiting, dysfunction of vision

115. What special treatment used in beginning of the botulism?A.*Antibiotic therapyB.Disintoxication therapyC.Hormonal therapyD.Sulfonamides therapyE.Vaccine therapy

116. What material should be taken to find botulotoxin?A.StoolB.UrineC.*BloodD.Vomiting massesE.All above it

117. What specific treatment used in botulism?

Page 17: intranet.tdmu.edu.uaintranet.tdmu.edu.ua/data/kafedra/internal/infect_desease... · Web viewBest method to treat diarrhoea in child is: intra venous fluide *ORS antibiotics bowel

A.Antibiotic therapyB.Disintoxication therapyC.Hormonal therapyD.*Serum therapyE.Vaccine therapy

118. What is transmissive factor of botulism?A.WaterB.MilkC.AirD.*SoilE.Animals

119. Drug of choice for sanation of the carriers of amoeba cysts can be.A.MonomicynB.DelagilC.TetracyclineD.*YatrenE.Ursosan

120. What is the duration of clinical reconvalescense in acute amoebiasis.A.3-6 monthsB.1-3 monthsC.*6-12 monthsD.12-18 monthsE.18-24 months

121. What agent causes balantidiasis.A.*B. ColiB.B. EnterocoliticaC.S. DerbyD.S. Boydi E.L. Canicola

122. What group of pathogens balantidiasis belong to.A.HlamidiaB.MycoplasmaC.*SimplestD.WormingE.Rickettsiae

123. Who is the reservour of the causative agent in balantidiasis.A.*PigB.CowC.SheepD.GoatE.Human

124. In what disease mucous, erythrocytes, eosinophils, plasma cells and crystals Charcot-Leiden were in stool analysis reveals.

Page 18: intranet.tdmu.edu.uaintranet.tdmu.edu.ua/data/kafedra/internal/infect_desease... · Web viewBest method to treat diarrhoea in child is: intra venous fluide *ORS antibiotics bowel

A.*Intestinal amoebiasisB.Intestinal yersiniosisC.ShigellosisD.BalantidiasisE.Enterohaemorrhagic esherichiosis

125. What is the main method of taking of material for parasitological examination in case of intestinal amoebiasis.

A.In the next day of defecationB.After processing of desinfectantsC.*Immediately after the defecationD.1-2 hours after processing with Lyugol solutionE.2-3 hours after processing with iron hematoxylin

126. What is the pathological changes in intestine in balantidiasis. A.No changesB.UlcerC.*Hyperemia, edemaD.EdemaE.Hyperemia without edema

127. What is the incubation period in balantidiasis.A.7-14 daysB.5-10 daysC.*1-3 monthsD.1-3 weeksE.3-6 weeks

128. What complication is typical for balantidiasis.A.Intestinal bleedingB.CachexiaC.Perforated ulcerD.Abscess of bowelE.*All the above mentioned

129. What are the stages of life-cycle of balantidium.A.CystB.Vegetative and sporeC.*Vegetative and cystD.SporeE.Vegetative

130. What is the mechanism of transmision in balantidiasis.A.ContactB.*Fecal-oralC.Air-dropD.TransmissiveE.Vertical

131. What group of infectious diseases balantidiasis belongs to.A.*IntestinalB.Blood infectionC.Sapronosis

Page 19: intranet.tdmu.edu.uaintranet.tdmu.edu.ua/data/kafedra/internal/infect_desease... · Web viewBest method to treat diarrhoea in child is: intra venous fluide *ORS antibiotics bowel

D.External coversE.Antroponosis

132. What clinical forms of balantidiasis most often may appear.A.*MildB.AcuteC.SubclinicalD.ChronicE.All of the above

133. The complications of balantidiasis can be all except:A.Hypochromic anemiaB.Intestinal bleedingC.Perforation ulcersD.*Liver abscessE.Cachexia

134. When a patient can be discharged from a hospital after the treatment of balantidiasis?

A.*After clinical recovering and two negative results of microscopic study of stoolB.After clinical recovering and two negative results of bacteriological research of

stoolC.After clinical recovering and a negative result microscopic study of stoolD.After clinical recovering and normalization in the total bloodE.After clinical recovery, indicators of the overall normalization of blood and two

negative results of bacteriological research stool135. How long a recoveres after balantidiasis should be observed and followed-up?

A.3 monthsB.6 monthsC.*1 yearD.2 yearsE.5 years

136. Name the cause of giardiasis:A.L. CanicolaB.B. ColiC.*L. ІntestinalisD.S. BoidiiE.L. pomona

137. For verification of diagnosis balantidiasis use:A.BacterioscopyB.BacteriologyC.FluoroexamD.*ProtozooscopyE.USD

138. Name the cause of amoebiasis.A.*E. HistolyticaB.L. CanicolaC.S. DerbyD.S. Boydii

Page 20: intranet.tdmu.edu.uaintranet.tdmu.edu.ua/data/kafedra/internal/infect_desease... · Web viewBest method to treat diarrhoea in child is: intra venous fluide *ORS antibiotics bowel

E.B. Enterocolitica139. What is the group of pathogens of amoebiasis?

A.MycoplasmaB.ChlamidaiC.*The simplestD.RickettsiaeE.Worms

140. Who is a source of the causative agents of amoebiasis?A.*PeopleB.CowsC.SheepD.PigsE.Camels

141. What part of lower GI tract is affected with amoebiasis most often?A.*Descending and ascending colonB.Sigmoid and rectumC.Duodenun and jejunumD.Transversal colonE.Small intestine

142. The causative agent of amoebiasis during life cycle can be in any form except:A.Mature cystB.Immature cystC.TrophozoitD.Quadrinucleate cystE.*Spore

143. What group pathogenic agents of giardiasis belongs to:A.*SimplestB.WormingC.RickettsiaeD.MycoplasmaE.Hlamidia

144. Most of vegitative forms of amoebiasis can be found in stool of:A.*Patients with acute intestinal amoebiasisB.Reconvalences after acute intestinal amoebiasisC.Patients with amebic liver abscessesD.Patients with chronic recurrent intestinal amoebiasis in remission stageE.All the above

145. In what form the causative agent of acute intestinal amoebiasis could be found in stool:

A.Minor vegetative formB.Loomenal formC.CystD.*Tissue formE.All the above listed forms

146. What ulcers are specific for amoebiasis:

Page 21: intranet.tdmu.edu.uaintranet.tdmu.edu.ua/data/kafedra/internal/infect_desease... · Web viewBest method to treat diarrhoea in child is: intra venous fluide *ORS antibiotics bowel

A.*Edematose ulcers with undermining, surrounded by aflushing zone located on the intact mucosa

B.Smooth ulcers with undermining, located on the hyperemic mucosaC.Necrotic ulcers, located on the hyperemic mucosaD.Edematose ulcers with undermining located on the intact mucosaE.Small lesions on the basis of infiltration covered with white coat

147. What kind of mucous membrane can be found between amoebiatic ulcers of colon:

A.Hyperemia without edemaB.Lividity, without edemaC.Hyperemia, edemaD.Regular colored edemaE.*Do not change

148. What is the duration of incubation period for intestinal amoebiasis:A.3-5 daysB.4-6 daysC.*From 1 week to several monthsD.1-2 yearsE.From 3 months to 1 year

149. What are the known clinical forms of amoebiasis, except:A.EntericB.SkinC.Liver abscessD.*MyocarditisE.Lung abscess

150. Intestinal amoebiasis can be characterized by such complications, ecxept:A.AmoebomaB.Intestinal bleedingC.Perforation of ulcersD.*MeningitisE.Stricture of colon

151. What is the mechanism of amoebiasis transmission:A.Vector borneB.*Fecal-oralC.Air-droppingD.WoundE.Vertical

152. Which group does delagil belong to:A.Indirect ant amoeboidB.Direct anti amoeboidC.*Tissue ant amoeboidD.Product with universal effectE.Do not belong to any of these groups

153. How long clinical recovering lasts in acute amoebiasis.A.3-6 monthsB.1-3 months

Page 22: intranet.tdmu.edu.uaintranet.tdmu.edu.ua/data/kafedra/internal/infect_desease... · Web viewBest method to treat diarrhoea in child is: intra venous fluide *ORS antibiotics bowel

C.*6-12 monthsD.12-18 monthsE.18-24 months

154. Drug of choice for sanation of the carriers of amoeba cysts can be.A.MonomicynB.DelagilC.TetracyclineD.*YatrenE.Ursosan

155. In what disease mucous, erythrocytes, eosinophils, plasma cells and crystals Charcot-Leiden in stool analysis were reveals.

A.*Intestinal amoebiasisB.Intestinal yersiniosisC.ShigellosisD.BalantidiasisE.Enterohaemorrhagic esherichiosis

156. What is the main method of material taking for parasitological examination in intestinal amoebiasis.

A.In the next day of defecationB.After processing of desinfectantsC.*1-2 hours after processing with Lyugol solutionD.Immediately after the defecationE.2-3 hours after processing with iron hematoxylin

157. Name the cause of amoebiasis.A.L. CanicolaB.*E. HistolyticaC.S. DerbyD.S. BoydiiE.B. enterocolitica

158. What group of pathogens the agent of amoebiasis belongs to?A.MycoplasmaB.ChlamidiaC.*SimplestD.RickettsiaeE.Worms

159. What is a source of the causal agent of amoebiasis?A.*PeopleB.CowsC.SheepD.PigsE.Camels

160. What part of GI tract is affected in amoebiasis most often?A.*Descending and ascending colonB.Sigmoid and rectumC.Doudenum and jejunumD.Transversal colon

Page 23: intranet.tdmu.edu.uaintranet.tdmu.edu.ua/data/kafedra/internal/infect_desease... · Web viewBest method to treat diarrhoea in child is: intra venous fluide *ORS antibiotics bowel

E.Small intestine161. The causal agent of amoebiasis during life cycle can be in any form except:

A.Mature cystB.Immature cystC.TrophosoitD.Quadrinucleate cystE.*Spore

162. Most of vegitative forms of amoebiasis can be found in stool of:A.Reconvalence acute intestinal amoebiasisB.Cystonic after use laxativeC.*The patient acute intestinal amoebiasisD.Patients with chronic recurrent intestinal amoebiasis in remission stageE.Patients with amebic liver abscesses

163. In what form the causative agent is found in stool in acute intestinal amoebiasis:A.*Tissue formsB.Minor vegetative formC.Loomenal formD.CystsE.All the above listed forms

164. What ulcers are specific for amebiasis:A.Smooth ulcers with undermining, located on the hyperemic mucosaB.Necrotic ulcers, located on the hyperemic mucosaC.*Edematose ulcers with undermining, surrounded by aflushing zone located on the

intact mucosaD.Edematose ulcers with undermining located on the intact mucosaE.Small lesions on the basis of infiltration covered with white coat

165. What kind of ulcers are present at аmebiasis?A.Smooth sharp edges, placed on a hyperemic mucus membraneB.*Fillings out sharp edges, surrounded by the area of hyperemia, are placed on the

unchanged mucus membraneC.Even edges, placed on a hyperemic mucus membraneD.Fillings out sharp edges, placed on the unchanged mucus membraneE.Fillings out sharp edges, surrounded by the area of hyperemia, are placed on the

unchanged mucus membrane166. What are the known clinical forms of amoebiasis, except:

A.EntericB.SkinC.Liver abscessD.*MyocarditisE.Lung abscess

167. When it is possible to discharge the patients after amebiasis from infectious hospital?

A.*After clinical convalescence, in default of in incandescence of mucous, еosinophils, crystals of Charkot-Leiden and two negative results of parasitological research of excrements

Page 24: intranet.tdmu.edu.uaintranet.tdmu.edu.ua/data/kafedra/internal/infect_desease... · Web viewBest method to treat diarrhoea in child is: intra venous fluide *ORS antibiotics bowel

B.After clinical convalescence, in default of in incandescence of blood and three negative results of parasitological research of excrements

C.After clinical convalescence, in default of leukocytosis and two negative results of parasitological research of excrements

D.After clinical convalescence and normalization of indexes of global analysis of blood

E.After clinical convalescence, normalization of indexes of global analysis of blood and two negative results of bacteriological examination of excrement

168. How often treatment of cyst amebae carrier should be done?A.Three times a yearB.Once a yearC.*Twice a yearD.Do not doneE.Four times a year

169. How to increase posibility of lamblia cysts in fresh feces and vegetative forms in duodenal content revealing?

A.Cultivation in thermostatB.By the method of floatation in bilious clear soupC.Cultivation in anaerobic chamberD.*By the applications of phase-contrast and lumencense microscopy with the help of

methylen-orangeE.To sow on a nourishing environment

170. What drug is more effective in treatment of giardiasis?A.UrsoholB.DelagilC.*OrnidazolD.TetracyclinE.Enteroseptol

171. What agent causes balantidiasis.A.*B. ColiB.B. EnterocoliticaC.S. DerbyD.S. boydi E.L. canicola

172. What group of pathogens balantidiasis belong to.A.HlamidiumB.MycoplasmasC.*SimplestD.WormingE.Rickettsiae

173. Who is the source in balantidiasis.A.*PigB.CowC.SheepD.GoatE.Bear

Page 25: intranet.tdmu.edu.uaintranet.tdmu.edu.ua/data/kafedra/internal/infect_desease... · Web viewBest method to treat diarrhoea in child is: intra venous fluide *ORS antibiotics bowel

174. What is the pathological changes in intestine in balantidiasis. A.No changesB.UlcerC.*Hyperemia, edemaD.EdemaE.Hyperemia without edema

175. What is the incubation period in balantidiasis.A.7-14 daysB.5-10 daysC.1-3 monthsD.*1-3 weeksE.3-6 weeks

176. What complication is typical for balantidiasis.A.Intestinal bleedingB.CachexiaC.Perforated ulcerD.Abscess of bowelE.*All the above mentioned

177. What are the forms of balantidium.A.CystB.Vegetative and sporeC.*Vegetative and cystD.SporeE.Vegetative

178. What is the transmissive mechanism in balantidiasis.A.ContactB.*Fecal-oralC.Air-dropD.TransmissiveE.Vertical

179. What group of infectious diseases balantidiasis belongs to.A.*IntestinalB.Blood infectionC.SapronosisD.External coversE.Antroponosis

180. What clinical forms of balantidiasis are seen most often.A.AcuteB.SubclinicalC.*MildD.ChronicE.All of the above

181. What is the possible complications of balantidiasis, except:A.Hypochromic anaemiaB.EnterorrhagiaC.Perforation of ulcer

Page 26: intranet.tdmu.edu.uaintranet.tdmu.edu.ua/data/kafedra/internal/infect_desease... · Web viewBest method to treat diarrhoea in child is: intra venous fluide *ORS antibiotics bowel

D.*Abscess of liverE.Cachexia

182. When would you discharge a patient with balandiasis from a hospital?A.*After clinical recovery and two negative results of parasitological research of

excrementB.After clinical recovery and two negative results of bacteriological examination of

excrementC.After clinical recovery and one negative result of parasitological research of

excrementD.After clinical recovery and normalization of indexes of global analysis of bloodE.After clinical recovery, normalization of indexes of global analysis of blood and

two negative results of bacteriological examination of excrement183. How long does lasts recovering after a balantidiasis?

A.6 monthsB.3 monthsC.*1 yearD.2 yearsE.5 years

184. With what serum reactions it is possible to confirm the diagnosis of balantidiasis?

A.*Complement link reaction, reaction in gel precipitation, reaction of immobilization

B.Reaction of indirect gemagglutination, immune fluorescent reactionC.Complement link reaction, reaction of indirect hemagglutinationD.Complement link reaction, immune fluorescent reaction, reaction of indirect

gemagglutinationE.Complement link reaction, reaction of indirect gemagglutination

185. What test is more frequently used for verification of balantidias:A.Virusological B.Bacteriological C.X-raysD.*Research on protozoaE.Ultrasound

186. The clinical forms of balantidiasis are all, except?A.*Mild rapidB.AcuteC.SubclinicalD.Chronic continuesE.Chronic recurrent

187. What group of pathogenic agents giardiasis belongs to?A.WormingB.*The simplestC.RickettsiaeD.MycoplasmasE.Hlamidii

Page 27: intranet.tdmu.edu.uaintranet.tdmu.edu.ua/data/kafedra/internal/infect_desease... · Web viewBest method to treat diarrhoea in child is: intra venous fluide *ORS antibiotics bowel

188. What from the given preparations can applied for etiotropic therapy of amebiosis?

A.Osarsol B.MetronidazolC.TetracyclineD.DelagilE.*All are correct

189. Choose the universal drug for amebiasis treatment.A.OsarsolB.*MetronidazoleC.TetracyclineD.DelagilE.All are correct

190. What is the mechanism of ascariasis transmission?A.PercutaneousB.TransmissiveC.*Fecal-oralD.ParenteralE.Air-drop

191. What is the phase of ascariasis pathogenesis?A.BacteremiaB.*Early (migratory)C.ExtraintestinalD.Toxinemia E.Parenchymatous diffusion

192. What is the phase of ascariasis pathogenesis?A.BacteremiaB.ToxinemiaC.ExtraintestinalD.*Late (intestinal)E.Parenchymatous diffusion

193. What is the epidemiology of enterobiosis?A.ZoonosisB.Wound helminthiasisC.Percutaneous helminthiasisD.*Contagious helminthiasisE.Transmissive helminthiasis

194. What is the place of the parazitising of the agent in strongyloidosis? A.*Upper sections of a small intestine B.Large intestine (sigmoid)C.Large intestine (caecum)D.Bile ductsE.Liver

195. What is the source on the invasion in enterobiasis?A.Wild animals B.Dogs, cats

Page 28: intranet.tdmu.edu.uaintranet.tdmu.edu.ua/data/kafedra/internal/infect_desease... · Web viewBest method to treat diarrhoea in child is: intra venous fluide *ORS antibiotics bowel

C.*HumansD.Mollusks E.Fish

196. What are the ways of the infection in ancylostomosis?A.*Peroral B. Parentral

C. Air-dropD. TransmissiveE. All are corect

197. What are the main clinical sighs of the early stage of ancylostostomiasis?A.Dermatitis (polymorphic rash, itch)B.Damage of the respiratory tract (bronchitis, laryngotracheitis, eosinophilic

infiltrates)C.Fever D.Eosinophilia (30-60 %) E.*All are corect

198. What are the principal clinical sighs of the chronic stage of ancylostostomiasis?A.Hepasplenomegaly B.B 12 deficiency anemia C.*Nausea, vomiting, diarrhea, abdominal pain (gastroduodenitis) D.Fever with attacks in 48 hours E.Pericarditis

199. What are the principal clinical sighs of the chronic stage of ancylostostomiasis?A.Constipation, abdominal painB.Hepasplenomegaly C.*Iron deficiency anemia D.Fever with attacks in 48 hours E.Pericarditis

200. What is epidemiology of strongyloidiasis?A.AnthroponosisB.GeogelminthiasisC.The way of the infection is peroralD.The way of the infection is percutaneousE.*All right

201. What are the clinical forms of the late stage of strongyloidiasis?A.Gastrointestinal form, allergic-toxic formB.Allergic-toxic form, meningoencephalitic formC.*Gastrointestinal form, allergic-toxic form, mixed formD.Meningeal, meningoencephalitic formE.Acute hepatic insufficiency, gastrointestinal form,

202. What is the agent of pork worm disease?A.*Taenia soleumB.Hymenolepis nanaC.Taeniarhynchus saginatus D.Trichinella spiralisE.Dyphylobotrium latum

Page 29: intranet.tdmu.edu.uaintranet.tdmu.edu.ua/data/kafedra/internal/infect_desease... · Web viewBest method to treat diarrhoea in child is: intra venous fluide *ORS antibiotics bowel

203. What are the principal clinical syndromes in pork worm disease?A.Cholestatic syndromeB.Renal syndrome C.*Dyspeptic syndromeD.Meningeal syndromeE.Respiratory syndrome

204. What are the principal clinical syndromes in pork worm disease?A.Renal syndromeB.Cholestatic syndromeC.*General toxic syndrome D.Meningeal syndromeE.Respiratory syndrome

205. What is the epidemiology of cystecercosis?A.The infection is an endogenic (autoinvasion)B.The human is the definitive hostC.The pigs are an intermediate hostD.The human is an intermediate hostE.*All are correct

206. What methods are used for diagnostics of cystecercosis?A.*OphthalmoscopeB.Biochemistry methodsC.Cardiogram D.Biological testE. Intracutaneous test

207. What methods are used for diagnostics of cystecercosis?A.CardiogramB.Biochemistry methodsC.*Computer tomography D.Biological testE. Intracutaneous test

208. What methods are used for diagnostics of cystecercosis?A.Biological testB.Biochemistry methodsC.CardiogramD.*RoentgenogramsE. Intracutaneous test

209. What is epidemiology of fascioliasis?A.It is anthroponosisB.*It is zoonosis C.The definitive host is camel and cattleD.The intermediate hosts are the pigsE.The intermediate hosts are mice

210. What is epidemiology of fascioliasis?A.It is anthroponosisB.It is sapronosis C.*The definitive host is human and cattle

Page 30: intranet.tdmu.edu.uaintranet.tdmu.edu.ua/data/kafedra/internal/infect_desease... · Web viewBest method to treat diarrhoea in child is: intra venous fluide *ORS antibiotics bowel

D.The intermediate hosts are the pigsE.The intermediate hosts are mice

211. What is epidemiology of fascioliasis?A.It is anthroponosisB.It is sapronosisC.The definitive host is mice and cattleD.The intermediate hosts are the pigsE.*The intermediate hosts are mollusks

212. The invasion of the human with Dyphillobotirum latum is possible in use into the food:

A.*Fresh water fish B.Sea fish C.PorkD.BeefE.Milk

213. What is the duration of the life of the adult forms in echinoccosis?A.*It is continued from 6 months till 1 yearB.It is continued till 1 month C.It is continued from 1 year till 3 years D.It is continued more than 3 years E. It is continued 7 days

214. What is epidemiology of hymenolepidosis?A.It is percutaneous helminthiasis B.It is not contagious helminthiasis C.*It is peroral helminthiasis D.It is transmissive helminthiasis E. It is especially dangerous disease

215. What is epidemiology of hymenolepidosis?A.It is not peroral helminthiasis B.It is percutaneous helminthiasis C.*It is contagious helminthiasis D.It is transmissive helminthiasis E. It is especially dangerous disease

216. What is the possible localization of hydatid cyst?A.LiverB.LungsC.BrainD.OvariesE.*All are correct

217. How to increase frequency of findings of cyst of lamblias in fresh incandescence and vegetative forms in duodenal content?

A.Cultivation in thermostatB.By the method of floatation in bilious clear soupC.Cultivation in anaerobic chamberD.*By the applications of phase-contrast and lumencense microscopy with the help of

methylen-orange

Page 31: intranet.tdmu.edu.uaintranet.tdmu.edu.ua/data/kafedra/internal/infect_desease... · Web viewBest method to treat diarrhoea in child is: intra venous fluide *ORS antibiotics bowel

E.To sow on a nourishing environment218. What preparation is it more expedient to apply for giardiasis treatment?

A.UrsoholB.DelagilC.*OrnidazolD.TetracyclinE.Enteroseptol

219. Name the cause of giardiasis:A.L. CanicolaB.B. ColiC.*L. ІntestinalisD.S. BoidiiE.L. pomona

220. The most effective means of filariasis control will be:A.Yatren therapyB.Insecticidal measures against culex mosquitoesC.Provision of underground drainageD.*Personal prophylaxixE.All mentioned above

221. Causative agent of Giardiasis is:A.G. lambliaB.G. intestinalisC.*G. lamblia and G. intestinalisD.B. coliE.None of the above

222. Cryptosporidiosis is:A.Blood borne infection B.Respiratory infection C.*Intestinal infection D.Infection of external coversE.Helminthiasis

223. Balantidiasis is caused by: A.*B. coliB.B. hominisC.E. intestinalisD.Cyclospora cayetanensisE.B. melitensis

224. Trichinellosis develops after:A.Bite of a tickB.Drinking of contaminated waterC.*Ingestion of the infected meatD.Bite of a dogE.All of the above

225. Which of the following species of Trichinella are distributed world wide:A.T. nelsoniB.T. spiralis

Page 32: intranet.tdmu.edu.uaintranet.tdmu.edu.ua/data/kafedra/internal/infect_desease... · Web viewBest method to treat diarrhoea in child is: intra venous fluide *ORS antibiotics bowel

C.T. nativaD.*All mentioned aboveE.None

226. What is the main symptom of the Trichinellosis:A.RashB.Muscle painC.Edema of eyelidsD.Nodules in musclesE.*All mentioned above

227. Which of the following is the largest intestinal helminthes in human:A.*D. latumB.S. stercoralisC.Anisakis simplexD.E. vermicularisE.T. saginatus

228. Loffler’s syndrome in ascariasis is due to: A.Inflitration of payer’s patches by eosinophillsB.Invasion of gallblader by A.lumbricoides pathogenC.*inflitration of lung tissue by eosinophillsD.Inflitration of liver by eosinophillsE.None of the above

229. Which drug can be used in pregnancy in case of ascariasis?A.AlbendazoleB.MebendazoleC.Pyrantel pamoateD.IvermectinE.*Piperasin adipinat

230. Which of the following is known as pinworm?A.*E. vermicularisB. E. duodenaleC.N. americanusD.T. soliumE.All of the above

231. What is larva currents?A.Dead larvaB.Floating larvaC.*Running larvaD.Slipping larvaE.None of the above

232. All of the following are the blood flukes except:A.Schistosoma japonicumB.Fasciola giganticaC.Clonorchis sinensisD.Fasciola hepaticaE.*Echinococcus granulosis

233. Chyluria is the complication of:

Page 33: intranet.tdmu.edu.uaintranet.tdmu.edu.ua/data/kafedra/internal/infect_desease... · Web viewBest method to treat diarrhoea in child is: intra venous fluide *ORS antibiotics bowel

A.*Lymphatic filariasisB.Abdominal angiostrongyliasisC.EnterobiasisD.TrichuriasisE.Amebiasis

234. Drug of choice for the treatment of the lymphatic filariasis is:A.AlbendazoleB.GlucorticoidsC.*DiethylcarbamazineD.MetronidazoleE.Chloramphenicol

235. The distinctive pattern of movement of filarial worms in lymphatic vessels is known as:

A.Filaria jumping signB.*Filaria dance signC.Filaria swim signD.Filaria escape signE.Filaria flying sign

236. Onchoceriasis is also known as:A.Tropical pulmonary eosinophilliaB.*River blindnessC.Guinea worm infectionD.African eye worm diseaseE.Ricketsiosis

237. B-12 deficiency is cause by which of the following: A.Echinococus granulosisB.T. saginataC.E. multilocularisD.*Diphyllobothrium latumE.Ascaris lumbricoideus

238. Ascaridiosis is:A.Bacterial infectionB.Viral infectionC.ProtozoosisD.Fungal infectionE.*Helminthosis

239. Ascaridiosis belongs to:A.*NematodosisB.TrematodosisC.CestodosisD.RicketsiosisE.Mycosis

240. Toxocarosis is:A.Bacterial infectionB.Viral infectionC.Protozoosis

Page 34: intranet.tdmu.edu.uaintranet.tdmu.edu.ua/data/kafedra/internal/infect_desease... · Web viewBest method to treat diarrhoea in child is: intra venous fluide *ORS antibiotics bowel

D.Fungal infectionE.*Helminthosis

241. Toxocarosis belongs to:A.TrematodosisB.CestodosisC.NematodosisD.RicketsiosisE.Mycosis

242. Enterobiosis is:A.Bacterial infectionB.Viral infectionC.ProtozoosisD.Fungal infectionE.*Helminthosis

243. Enterobiosis belongs to:A.TrematodosisB.*Nematodosis C.CestodosisD.RicketsiosisE.Mycosis

244. Trichinosis is:A.Bacterial infectionB.Viral infectionC.ProtozoosisD.Fungal infectionE.*Helminthosis

245. Trichinosis belongs to:A.*NematodosisB.TrematodosisC.CestodosisD.RicketsiosisE.Mycosis

246. Strongyloidosis is:A.Bacterial infectionB.Viral infectionC.ProtozoosisD.Fungal infectionE.*Helminthosis

247. Strongyloidosis belongs to:A.TrematodosisB.*NematodosisC.CestodosisD.RicketsiosisE.Mycosis

248. Schistosomosis is:A.Bacterial infection

Page 35: intranet.tdmu.edu.uaintranet.tdmu.edu.ua/data/kafedra/internal/infect_desease... · Web viewBest method to treat diarrhoea in child is: intra venous fluide *ORS antibiotics bowel

B.Viral infectionC.ProtozoosisD.Fungal infectionE.*Helminthosis

249. Schistosomosis belongs to:A.NematodosisB.*TrematodosisC.CestodosisD.RicketsiosisE.Mycosis

250. Fasciolosis is:A.Bacterial infectionB.Viral infectionC.ProtozoosisD.Fungal infectionE.*Helminthosis

251. Fasciolosis belongs to:A.NematodosisB.*TrematodosisC.CestodosisD.RicketsiosisE.Mycosis

252. Opisthorchosis is:A.Bacterial infectionB.Viral infectionC.*HelminthosisD.ProtozoosisE.Fungal infection

253. Opisthorchosis belongs to:A.NematodosisB.*TrematodosisC.CestodosisD.RicketsiosisE.Mycosis

254. Echinococcosis is:A.Bacterial infectionB.Viral infectionC.ProtozoosisD.Fungal infectionE.*Helminthosis

255. Echinococcosis belongs to:A.NematodosisB.TrematodosisC.*CestodosisD.RicketsiosisE.Mycosis

Page 36: intranet.tdmu.edu.uaintranet.tdmu.edu.ua/data/kafedra/internal/infect_desease... · Web viewBest method to treat diarrhoea in child is: intra venous fluide *ORS antibiotics bowel

256. Alveococcosis is:A.Bacterial infectionB.*HelminthosisC.Viral infectionD.ProtozoosisE.Fungal infection

257. Alveococcosis belongs to:A.NematodosisB.TrematodosisC.*CestodosisD.RicketsiosisE.Mycosis

258. Diphyllobothriosis is:A.Bacterial infectionB.Viral infectionC.*HelminthosisD.ProtozoosisE.Fungal infection

259. Diphyllobothriosis belongs to:A.NematodosisB.TrematodosisC.*CestodosisD.RicketsiosisE.Mycosis

260. Teniarinchosis is:A.Bacterial infectionB.Viral infectionC.ProtozoosisD.Fungal infectionE.*Helminthosis

261. Teniarinchosis belongs to:A.NematodosisB.TrematodosisC.*CestodosisD.RicketsiosisE.Mycosis

262. Teniosis is:A.Bacterial infectionB.Viral infectionC.*HelminthosisD.ProtozoosisE.Fungal infection

263. Teniosis belongs to:A.NematodosisB.TrematodosisC.*Cestodosis

Page 37: intranet.tdmu.edu.uaintranet.tdmu.edu.ua/data/kafedra/internal/infect_desease... · Web viewBest method to treat diarrhoea in child is: intra venous fluide *ORS antibiotics bowel

D.RicketsiosisE.Mycosis

264. Cycticercosis is:A.Bacterial infectionB.*HelminthosisC.Viral infectionD.ProtozoosisE.Fungal infection

265. Cycticercosis belongs to:A.NematodosisB.TrematodosisC.*CestodosisD.RicketsiosisE.Mycosis

266. What clinical forms of balantidiasis are seen most often.A.*MildB.AcuteC.SubclinicalD.ChronicE.All of the above

267. Give recommendations for a patient in reconvalensent period of viral hepatitis during a clinical supervision.

A.A medical supervision during 6 monthB.Biochemical inspectionC.Abstain from hard physical loadD.Temporal contra-indications for prophylactic inoculationsE.*All the above

268. What is etiotropic therapy of viral hepatitis.A.RibavirinB.InterferonC.Inductors of interferonD.ZefixE.*All the above

269. Choose the remedies for etiotropic therapy of viral hepatitis.A.*InterferonsB.VaccineC.Normal human immune globulinD.HepatoprotectorsE.Glucocorticoids

270. Choose the remedies for etiotropic therapy for viral hepatitis.A.AntibioticsB.*InterferonsC.ProbioticsD.VaccineE.Normal human immunoprotein

271. The criteria for application of etiotropic therapy in viral hepatitis is.

Page 38: intranet.tdmu.edu.uaintranet.tdmu.edu.ua/data/kafedra/internal/infect_desease... · Web viewBest method to treat diarrhoea in child is: intra venous fluide *ORS antibiotics bowel

A.Protracted motion of HBV, HVDB.Any form of HVC.Biochemical activityD.Presence of virus replication E.*All the above

272. The criteria for application of etiotropic therapy for the patient with HCV.A.Clinical displays are insignificantB.Icterus is absentC.Moderate biochemical activityD.There is anti-HCV in bloodE.*RNA of HCV +

273. Factors which are indications of successful interferon therapy in HV infections are all, except.

A.Level of ALaT not more than 2-3 normB.Low titre of HCV after the treatmentC.Absence of cholestasisD.2th and 4th genotypes of HCVE.*Expressed fibrosis

274. Indirect action of interferon therapy.A.Influenza-like syndromeB.NauseaC.ItchingD.Para-hypnosisE.*All the above||

275. .Give recommendation for a patient in reconvalensent period of viral hepatitis during a clinical supervision after isolation.

A.*Medical supervision during 6 months, periodic biochemical inspections.B.Control bacteriological examinationsC.Full labor investigationD.To continue prophylactic inoculationsE.Supervision is not needed

276. Indirect action of interferons.A.FlatulenceB.DiarrheaC.NauseaD.DepressionE.*All the above

277. Indirect action of interferon therapy are all except.A.Influenza-like syndromeB.NauseaC.DepressionD.Intensification of autoimmune diseasesE.*Progress of fibrosis

278. . Basic principles of antiviral therapy for viral hepatitis.A.Individual selection of dose and rhythm of application of preparationsB.Duration of introduction of preparations

Page 39: intranet.tdmu.edu.uaintranet.tdmu.edu.ua/data/kafedra/internal/infect_desease... · Web viewBest method to treat diarrhoea in child is: intra venous fluide *ORS antibiotics bowel

C.Control of amount of erytrocytes, leucocytes and thrombocytes, in bloodD.Control of iron level in bloodE.*All the above

279. . Contra-indications for antiviral therapy of viral hepatitis.A.Decompensatory cirrhosis of liverB.Thrombocytopenia <50000 in 1 мм3 C.Psychic disordersD.Leucocytopenia <1500 in 1 мм3E.*All the above

280. Contra-indications for antiviral therapy of viral hepatitis.A.A.Decompensatory cirrhosis of liverB.Autoimmune diseaseC.Alcoholism and other drug addictionsD.D.Coinfection by HIVE.*All the above

281. Choose the indexes of efficiency of interferon therapy.A.*Disappearance| of markers of viral replicationB.Improvement of the general stateC.Normalization of the liver size D.Disappearance of icterusE.All the above

282. Choose the indexes of efficiency of interferon therapy.A.Improvement of the general stateB.*Normalization of activity of ALaTC.Normalization of the liver size D.Disappearance of icterusE.All the above

283. Types of answer for interferon therapy are.A.Stable remissionB.UnsteadyC.Partial answerD.Absence of answerE.*All the above

284. The characteristic of an unsteady answer of interferon therapy are.A.Disappearance of markers of viral replication upon completion of course of therapyB.Normalization of activity of ALaT during the course of therapy C.An origin of relapse in next 6 monthsD.Disappearance of icterusE.*All the above

285. That characteristic of a partial answer of interferon therapy are all, except.A.*Disappearance of markers of viral replicationB.Normalization of activity of ALaT is upon completion of course of therapyC.Disappearance of icterus

Page 40: intranet.tdmu.edu.uaintranet.tdmu.edu.ua/data/kafedra/internal/infect_desease... · Web viewBest method to treat diarrhoea in child is: intra venous fluide *ORS antibiotics bowel

D.Normalization the state of patientE.Normalization of the size of liver

286. When is interferon therapy effective in the the patient.A.Normalization of the state of patientB.Normalization of activity of ALaT upon completion of course of therapyC.Disappearance of icterusD.Normalization of the size of liverE.*The markers of viral replication, are determined upon completion of course of

therapy287. What laboratory work-up is needed for confirming the diagnosis of viral

hepatitis.A.Total analysis of bloodB.Determination of level of bilirubinC.Determination of activity of aminotransferaseD.*Determination of markers of HV in IFAE.All the above

288. What laboratory and instrumental examinations are needed for confirming the diagnosis of viral hepatitis.

A.Complete analysis of bloodB.Ultrasound of abdominal regionC.Determination of activity of aminotransferaseD.*Determination of antigen of virusesE.Duodenal probing

289. What is incubation period for hepatitis B:A.45 daysB.*180 daysC.360 daysD.90 daysE.25 days

290. All the hepatitis have parenteral route of transmission except:A.*AB.BC.CD.DE.TTV

291. Chronic course is common for viral hepatitis except: A.*AB.BC.CD.DE.B+C

292. All the following medicines are interferons except:A.IntronB.RoferonC.ReaferonD.Leukinferon

Page 41: intranet.tdmu.edu.uaintranet.tdmu.edu.ua/data/kafedra/internal/infect_desease... · Web viewBest method to treat diarrhoea in child is: intra venous fluide *ORS antibiotics bowel

E.*Cycloferon293. All the following medicines are hepatoprotective agents except:

A.CarsilB.SiliborC.LegalonD.*LomusolE.Arginine

294. What is mechanism of transmission in viral hepatitis A:A.ContactB.TransmissiveC.VerticalD.*Fecal-oralE.Air-drop

295. What is mechanism of transmission in viral hepatitis B:A.*ContactB.TransmissiveC.AlimentaryD.Fecal-oralE.Air-drop

296. What is mechanism of transmission in viral hepatitis C:A.*ContactB.TransmissiveC.AlimentaryD.Fecal-oralE.Air-drop

297. What is mechanism of transmission in viral hepatitis D:A.*ContactB.TransmissiveC.AlimentaryD.Fecal-oralE.Air-drop

298. What is mechanism of transmission in viral hepatitis E:A.ContactB.SexualC.AlimentaryD.*Fecal-oralE.Air-drop

299. At inspection of a 8 weeks term pregnant woman HBsAg was found. Level of bilirubin of blood and activity of ALAT were normal. What is necessary to do?

A.*To save pregnancy and conduct the inoculation to newborn against hepatitis BB.Termination of pregnancyC.Termination of pregnancy and conduct treatment by interferonD.To save pregnancy and conduct treatment by lamivudinE.To save pregnancy and ultrasonic inspection of the fetus

300. Diagnosed a patient: chronic hepatitis in the stage of integration. What markers will be in

Page 42: intranet.tdmu.edu.uaintranet.tdmu.edu.ua/data/kafedra/internal/infect_desease... · Web viewBest method to treat diarrhoea in child is: intra venous fluide *ORS antibiotics bowel

A.patient in this stage disease?B.HBeAgC.Antibodies to HBeAgD.DNA OF HBVE.Viral DNA-polimeraseF.*HBsAg, anti-НBе

301. As etiotropic therapy of sharp and chronic viral hepatitis B utillize:A.CorticosteroidB.Immunomodulate preparationsC.CytostaticsD.AntibioticsE.*Antiviral preparations

302. When did order a patient with virus hepatitis?A.Non-permanent negative virologic research of defecatingB.21 days of normal temperature of bodyC.Of normalization of level of bilirubinumD.Clinical convalescence and normalization of level of transaminasisE.*Clinical convalescence and no more than triple increase of level of transaminasis

303. Give recommendation for a patient in reconvalensent period of viral hepatitis during a clinical supervision after isolation.

A.A medical supervision during 6 monthB.Biochemical inspectionC.Abstain from hard physical loadD.Temporal contra-indications for prophylactic inoculationsE.*All the above

304. Give recommendation for a patient in reconvalensent period of viral hepatitis during a clinical supervision after isolation.

A.*Medical supervision during 6 months, periodic biochemical inspections.B.Control bacteriological examinationsC.Full labor investigationD.To continue prophylactic inoculationsE.Supervision is not needed

305. Etiotropic therapy of viral hepatitis is.A.RibavirinB.InterferonC.Inductors of interferonD.ZefixE.*All the above.

306. Choose the remedies for etiotropic therapy of viral hepatitis.A.*RibavirinB.VaccineC.Normal human immunoproteinD.HepatoprotectorE.Glucocorticoid

307. Choose the remedies for etiotropic therapy for viral hepatitis.A.Antibiotics

Page 43: intranet.tdmu.edu.uaintranet.tdmu.edu.ua/data/kafedra/internal/infect_desease... · Web viewBest method to treat diarrhoea in child is: intra venous fluide *ORS antibiotics bowel

B.*InterferonC.ProbioticsD.VaccineE.Normal human immunoprotein

308. The criteria for application of etiotropic therapy in viral hepatitis is.A.Protracted motion of HBV, HVDB.Any form of HVC.Biochemical activityD.Presence of virus replication E.*All the above.

309. The criteria for application of etiotropic therapy for the patient with HCV.A.Clinical displays are insignificantB.Icterus is absentC.Moderate biochemical activityD.There is anti-HCV in bloodE.*RNA of HCV +

310. Factors which are indications of successful interferon therapy in HV infections are all, except.

A.Level of ALaT not more than 2-3 normB.Low titre of HCV after the treatmentC.Absence of cholestasisD.2th and 4th genotypes of HCVE.*Expressed fibrosis

311. Indirect action of interferon therapy.A.Influenza-like syndromeB.NauseaC.ItchingD.Para-hypnosisE.*All the above

312. Indirect action of interferons.A.FlatulenceB.DiarrheaC.NauseaD.DepressionE.*All the above

313. Indirect action of interferon therapy are all except.A.Influenza-like syndromeB.NauseaC.DepressionD.Intensification of autoimmune diseasesE.*Progress of fibrosis

314. Basic principles of antiviral therapy for viral hepatitis.A.Individual selection of dose and rhythm of application of preparationsB.Duration of introduction of preparationsC.Control of amount of erytrocytes, leucocytes and thrombocytes, in bloodD.Control of iron level in blood

Page 44: intranet.tdmu.edu.uaintranet.tdmu.edu.ua/data/kafedra/internal/infect_desease... · Web viewBest method to treat diarrhoea in child is: intra venous fluide *ORS antibiotics bowel

E.*All the above315. Contra-indications for antiviral therapy of viral hepatitis.

A.Decompensatory cirrhosis of liverB.Thrombocytopenia <50000 in 1 мм3 C.Psychic disordersD.Leucocytopenia <1500 in 1 мм3E.*All the above

316. Contra-indications for antiviral therapy of viral hepatitis.A.Decompensatory cirrhosis of liverB.Autoimmune diseaseC.Alcoholism and other drug addictionsD.Coinfection by HIVE.*All the above

317. Choose the indexes of efficiency of interferon therapy.A.*Disappearance| of markers of viral replicationB.Improvement of the general stateC.Normalization of the liver size D.Disappearance of icterusE.All the above

318. Choose the indexes of efficiency of interferon therapy.A.Improvement of the general stateB.*Normalization of activity of ALaTC.Normalization of the liver size D.Disappearance of icterusE.All the above

319. Types of answer for interferon therapy are.A.Stable remissionB.UnsteadyC.Partial answerD.Absence of answerE.*All the above

320. The characteristic of an unsteady answer of interferon therapy are.A.Disappearance of markers of viral replication upon completion of course of therapyB.Normalization of activity of ALaT during the course of therapy C.An origin of relapse in next 6 monthsD.Disappearance of icterusE.*All the above

321. That characteristic of a partial answer of interferon therapy are all, except.A.*Disappearance of markers of viral replicationB.Normalization of activity of ALaT is upon completion of course of therapyC.Disappearance of icterusD.Normalization the state of patientE.Normalization of the size of liver

322. When is interferon therapy effective in the the patient.A.Normalization of the state of patientB.Normalization of activity of ALaT upon completion of course of therapy

Page 45: intranet.tdmu.edu.uaintranet.tdmu.edu.ua/data/kafedra/internal/infect_desease... · Web viewBest method to treat diarrhoea in child is: intra venous fluide *ORS antibiotics bowel

C.Disappearance of icterusD.Normalization of the size of liverE.*The markers of viral replication, are determined upon completion of course of

therapy323. What laboratory work-up is needed for confirming the diagnosis of viral

hepatitis.A.General blood analysisB.Determination of level of bilirubinC.Determination of activity of aminotransferaseD.*Determination of markers of HV in IFAE.All the above

324. What laboratory and instrumental examinations are needed for confirming the diagnosis of viral hepatitis.

A.General blood analysisB.Ultrasound of abdominal regionC.Determination of activity of aminotransferaseD.*Determination of antigen of virusesE.Duodenal probing

325. What is maximal incubation period for hepatitis B:A.45 daysB.*180 daysC.360 daysD.90 daysE.25 days

326. All the hepatitis have parenteral route of transmission except:A.*AB.BC.CD.DE.TTV

327. Chronic course is common for viral hepatitis except: A.*AB.BC.CD.DE.B+C

328. All the following medicines are interferons except:A.IntronB.RoferonC.ReaferonD.LeukinferonE.*Cycloferon

329. All the following medicines are hepatoprotective agents except:A.CarsilB.SiliborC.Legalon

Page 46: intranet.tdmu.edu.uaintranet.tdmu.edu.ua/data/kafedra/internal/infect_desease... · Web viewBest method to treat diarrhoea in child is: intra venous fluide *ORS antibiotics bowel

D.*LomusolE.Arginine

330. What is incubation period for hepatitis A:A.*45 daysB.180 daysC.360 daysD.90 daysE.25 days

331. What group of infectious diseases hepatitis A belong to:A.External coversB.*IntestinalC.BloodD.WoundE.Transmissive

332. What group of infectious diseases hepatitis B belong to:A.*External coversB.IntestinalC.BloodD.WoundE.Transmissive

333. What group of infectious diseases hepatitis C belong to:A.*External coversB.IntestinalC.BloodD.WoundE.Transmissive

334. What group of infectious diseases hepatitis D belong to:A.*External coversB.IntestinalC.BloodD.WoundE.Transmissive

335. What group of infectious diseases hepatitis E belong to:A.External coversB.*IntestinalC.BloodD.WoundE.Transmissive

336. What is transmissive factor for hepatitis A:A.BloodB.SemenC.*WaterD.AirE.Milk of mother

337. What is transmissive factor for hepatitis B:A.Food

Page 47: intranet.tdmu.edu.uaintranet.tdmu.edu.ua/data/kafedra/internal/infect_desease... · Web viewBest method to treat diarrhoea in child is: intra venous fluide *ORS antibiotics bowel

B.MilkC.WaterD.AirE.*Milk of mother

338. What recommendations for a patient in reconvalensent period of viral hepatitis.A.A medical supervision during 6 monthB.Biochemical inspectionC.Abstain from hard physical loadD.Temporal contra-indications for prophylactic inoculationsE.*All the above

339. Medicines for etiotropic therapy of viral hepatitis.A.RibavirinB.InterferonC.Inductors of interferonD.ZefixE.*All the above

340. What drugs is etiotropic therapy of viral hepatitis.A.*InterferonsB.VaccineC.Normal human immune globulinD.HepatoprotectorsE.Glucocorticoids

341. What criteria for application of etiotropic therapy in viral hepatitis is.A.Protracted motion of HBV, HVDB.Any form of HVC.Biochemical activityD.Presence of virus replication E.*All the above

342. What criteria for application of etiotropic therapy for the patient with HCV.A.Clinical displays are insignificantB.Icterus is absentC.Moderate biochemical activityD.There is anti-HCV in bloodE.*RNA of HCV +

343. Signs of efficiency of interferon therapy in HV infections are all, except.A.Level of ALaT not more than 2-3 normB.Low titre of HCV after the treatmentC.olestasisD.2th and 4th genotypes of HCVE.*Expressed fibrosis

344. What is indirect action of interferon therapy.A.Influenza-like syndromeB.NauseaC.ItchingD.Para-hypnosisE.*All the above||

Page 48: intranet.tdmu.edu.uaintranet.tdmu.edu.ua/data/kafedra/internal/infect_desease... · Web viewBest method to treat diarrhoea in child is: intra venous fluide *ORS antibiotics bowel

345. What recommendation for viral hepatitis patient in recovering period.A.*Medical supervision during 6 months, periodic biochemical inspections.B.Control bacteriological examinationsC.Full labor investigationD.To continue prophylactic inoculationsE.Supervision is not needed

346. What is indirect action of interferons.A.FlatulenceB.DiarrheaC.NauseaD.DepressionE.*All the above

347. Indirect action of interferon therapy are all except.A.Influenza-like syndromeB.NauseaC.DepressionD.Intensification of autoimmune diseasesE.*Progress of fibrosis

348. What is basic principles of antiviral therapy for viral hepatitis.A.Individual selection of dose and rhythm of application of preparationsB.Duration of introduction of preparationsC.Control of amount of erytrocytes, leucocytes and thrombocytes, in bloodD.Control of iron level in bloodE.*All the above

349. What is contra-indications for antiviral therapy of viral hepatitis.A.Decompensatory cirrhosis of liverB.Thrombocytopenia <50000 in 1 мм3 C.Psychic disordersD.Leucocytopenia <1500 in 1 мм3E.*All the above

350. What is contra-indications for antiviral therapy of viral hepatitis.A.A.Decompensatory cirrhosis of liverB.B. Autoimmune disease

351. Alcoholism and other drug addictionsA.D.Coinfection by HIVB.*E. All the above

352. Choose the indexes of efficiency of interferon therapy.A.*Disappearance of markers of viral replicationB.Improvement of the general stateC.Normalization of the liver size D.Disappearance of icterusE.All the above

353. Choose the indexes of efficiency of interferon therapy.A.Improvement of the general stateB.*Normalization of activity of ALaTC.Normalization of the liver size

Page 49: intranet.tdmu.edu.uaintranet.tdmu.edu.ua/data/kafedra/internal/infect_desease... · Web viewBest method to treat diarrhoea in child is: intra venous fluide *ORS antibiotics bowel

D.Disappearance of icterusE.All the above

354. What is types of answer for interferon therapy.A.Stable remissionB.UnsteadyC.Partial answerD.Absence of answerE.*All the above

355. What is organism answer of interferon therapy.A.Disappearance of markers of viral replication upon completion of course of therapyB.Normalization of activity of ALaT during the course of therapy C.Relapse in next 6 monthsD.Disappearance of icterusE.*All the above

356. Answer of interferon therapy are all, except.A.*Disappearance of markers of viral replicationB.Normalization of ALaT activityC.Disappearance of icterusD.Normalization of patient general conditionE.Normalization of a liver size

357. What investigation for confirming of viral hepatitis diagnosis.A.Total analysis of bloodB.Determination of bilirubin level C.Determination of aminotransferases activityD.*Determination of HV markers by IFAE.All the above

358. What examination is helpful in confirming of viral hepatitis diagnosis.A.Complete analysis of bloodB.Ultrasound of abdominal regionC.Determination of activity of aminotransferaseD.*Determination of antigen of virusesE.Duodenal probing

359. What is duration of incubation period for B hepatitis :A.45 daysB.*180 daysC.360 daysD.90 daysE.25 days

360. Parenteral way of transmission is present in spreding of hepatitis except:A.*AB.BC.CD.DE.TTV

361. Chronic course is possible for viral hepatitis except: A.*A

Page 50: intranet.tdmu.edu.uaintranet.tdmu.edu.ua/data/kafedra/internal/infect_desease... · Web viewBest method to treat diarrhoea in child is: intra venous fluide *ORS antibiotics bowel

B.BC.CD.DE.B+C

362. All the following medicines are interferons except:A.IntronB.RoferonC.ReaferonD.LeukinferonE.*Cycloferon

363. All the following medicines are hepatoprotective agents except:A.CarsilB.SiliborC.LegalonD.*LomusolE.Arginine

364. What is mechanism of viral hepatitis A transmission:A.ContactB.TransmissiveC.VerticalD.*Fecal-oralE.Air-drop

365. What is mechanism of hepatitis B transmission:A.*ContactB.TransmissiveC.AlimentaryD.Fecal-oralE.Air-drop

366. What is mechanism hepatitis C transmission:A.*ContactB.TransmissiveC.AlimentaryD.Fecal-oralE.Air-drop

367. What rash is present in case of haemorrhagic fevers with kidneys syndrome?A.RoseolarB.Maculo-papularC.PunctuateD.*PetechialE.Rash is not present

368. What rash is present in case of Congo hemorrhagic fever?A.RoseolaB.Maculo-papularC.PunctulateD.*PetechialE.Rashes not is characteristic

Page 51: intranet.tdmu.edu.uaintranet.tdmu.edu.ua/data/kafedra/internal/infect_desease... · Web viewBest method to treat diarrhoea in child is: intra venous fluide *ORS antibiotics bowel

369. What rashes is present in case of Crimea hemorrhagic fever?A.RoseolarB.Maculo-papularC.PunctulateD.*PetechialE.Rashes not is characteristic

370. How long the rash is present in case of haemorrhagic fever with kidneys syndrome?

A.*During all feverish periodB.Before the reconvalescenseC.Before development of clinical features of kidneys insufficiencyD.During whole diseaseE.Appears yet in a latent period and disappears in the period of early reconvalescense

371. A kidney syndrome at haemorrhagic fever with kidneys syndrome shows up usually:

A.Only laboratory changesB.Only on BRIDLESC.*By pain in lumbar area, positive Pasternatsky symptom, development of oliguriaD.By fever, polyuria, dyspepsiaE.By paradoxical ischuria

372. What changes in biochemical blood test inherent for haemorrhagic fever with kidneys syndrome?

A.Increase level of urea and bilirubinB.The level of urea and kreatinine fallsC.The level of kreatinine grows and urea fallsD.The level of urea grows and kreatinine fallsE.*The level of urea and kreatinine grows

373. What changes in haemogram is typical for haemorrhagic fever with kidneys syndrome?

A.Normochromic anaemia, leucocytosis with atypical mononucleosis, thrombocytopenia enhanceable ESR

B.Erythrocytosis, lymphocytosis,ESR is enhanceableC.Normochromic anaemia, leucopenia with neutrophylosis, thrombocytopenia

enhanceable ESRD.*Hypochromic anaemia, leucocytosis with neutrophylosis, thrombocytopenia

enhanceable ESRE.Hyperchromic anaemia, leucocytosis with neutrophylosis, thrombocytopenia

mionectic ESR374. Polyuria in haemorrhagic fever with kidneys syndrome is a sign of:

A.*RecoveringB.. Chronic processC.Unfavorable flow of illnessD.Development of complicationsE.Complete convalescence

375. In most patients with Congo hemorrhagic what type of fever is:A.Wunderlich’s type

Page 52: intranet.tdmu.edu.uaintranet.tdmu.edu.ua/data/kafedra/internal/infect_desease... · Web viewBest method to treat diarrhoea in child is: intra venous fluide *ORS antibiotics bowel

B.Botkin’s typeC.UndulatingD.IntermittentE.*Two-humped

376. What changes in haemogram inherent Congo hemorrhagic fever?A.Normochomic anaemia, leucocytosis mononuclearB.Erythrocytosis, lymphocytosisC.*Hypochromic anemia, erythrofilosisD.Hypochromic anemia, neutrofilosisE.Hyperchromic anemia, neutrofilosis

377. What is typical for the Lassa hemorrhagic fever:A.Effect of cardiovascular systemB.Development of acute hepatic insufficiencyC.Hundred-per-cent lethalityD.*Defeat of breathing organsE.Development of paresis and paralysis

378. Confirm diagnosis of haemorrhagic fever with kidneys syndrome by a way of:A.Only virological methodsB.Only bacteriological methodsC.Bacteriological and serum methodsD.Proper epidemiological informationE.*Virologic and serum methods

379. Confirm the diagnosis of Lassa hemorrhagic fever by a way of:A.Only virological methodsB.Only bacteriological methodsC.Bacteriological and serum methodsD.Proper epidemiological informationE.*Virologic and serum methods

380. Confirm the diagnosis of Congo hemorrhagic fever by a way of:A.Only virological methodsB.Only bacteriological methodsC.Bacteriological and serum methodsD.Proper epidemiological informationE.*Virologic and serum methods

381. Confirm the diagnosis of Ebola fever by a way of:A.Growth of viruses on chicken embryonsB.Only bacteriological methodsC.Bacteriological and serum methodsD.Proper epidemiological informationE.*Selection of virus on the Vero culture

382. Confirm the diagnosis of Omsk fever by a way of:A.Growth of virus on chicken embryonsB.Only bacteriological methodsC.Bacteriological and serum methodsD.Proper epidemiological informationE.*Selection of virus on the Vero culture

Page 53: intranet.tdmu.edu.uaintranet.tdmu.edu.ua/data/kafedra/internal/infect_desease... · Web viewBest method to treat diarrhoea in child is: intra venous fluide *ORS antibiotics bowel

383. Confirm the diagnosis of Marburg fever by a way of:A.Growth on chicken embryosB.Only bacteriological methodsC.Bacteriological and serum methodsD.Proper epidemiologys informationE.*Selection of virus on the Vero culture

384. What etiothropic means use at treatment of haemorrhagic fever with kidneys syndrome:

A.BenzylpenicillinB.DopamineC.*VirolexD.DexamethazoneE.Etamsylatum

385. What etiothropic means use at treatment of patients with Lassa fever:A.BenzylpenicillinB.DopamineC.*RibavirinD.DexamethazoleE.Etamsylatum

386. What etiothropic means use at treatment of patients with Marburg fever:A.BenzypenicillinB.DopamineC.*RibavirinD.DexamethazoneE.Etamsylatum

387. What etiothropic means use at treatment of patients with Congo fever:A.BenzylpenicillinB.DopamineC.*RibavirinD.DexamethazoneE.Etamsylatum

388. What etiothropic means use at treatment of patients with Ebola fever:A.BenzylpenicillinB.DopamineC.*VirolexD.DexamethazoneE.Etamsylatum

389. What etiothropic means use at treatment of patients with Crimea fever:A.BenzylpenicillinB.DopamineC.*RibavirinD.DexamethazoneE.Etamsylatum

390. Specific prevention of hemorrhagic fevers may perform by:A.Live vaccineB.Killed vaccine

Page 54: intranet.tdmu.edu.uaintranet.tdmu.edu.ua/data/kafedra/internal/infect_desease... · Web viewBest method to treat diarrhoea in child is: intra venous fluide *ORS antibiotics bowel

C.Specific immunoglobulinD.*Do not developE.Polivalent vaccine

391. Who is the source of the causal agent in the Crimean-Congo haemorrhagic fever?A.Rodents, cattle, birdsB.TikesC.*Rodents, cattle, birds, sick peopleD.Sick man, reconvalescent, bacteriocarrierE.Rodents, cattle, birds, sick people, bacteriocarries

392. The source of infection of Omsk‘s hemorrhagic fever are muskrat, water rats and other rodents. Who are the carriers?

A.Bee and fleaB.*Pliers and fleaC.MosquitoesD.FlyE.Pliers and mosquitoes

393. Specific prevention of Crimean-Congo haemorrhagic fever are:A.*Vaccine and human immunoglobulinB.SerumC.Serum and human immunoglobulinD.Do not developedE.Antibacterial drugs

394. How to prevent malaria?A.Follow the rules of personal hygieneB.Do not drink raw waterC.VaccinationD.*ChemoprophylaxisE.Heating of food

395. What group of infectious diseases malaria belong to?A.IntestinalB.RespiratoryC.External coversD.TransmissiveE.*Blood

396. What is the treatment of malaria attacks.A.AntibioticsB.Serum transferC.DelagilD.PrimaquineE.*Delagil + Primaquine

397. In a survey of donor blood microhametosis are found. Assign treatment. A.DelagilB.*PrimaquineC.AntibioticsD.FluoroquinoloneE.Sulfanilamide

Page 55: intranet.tdmu.edu.uaintranet.tdmu.edu.ua/data/kafedra/internal/infect_desease... · Web viewBest method to treat diarrhoea in child is: intra venous fluide *ORS antibiotics bowel

398. Malaria must be differentiated primarily with such diseases:A.PyelonephritisB.SepsisC.Viral hepatitisD.LeptospirosisE.*All of the above

399. The diagnosis of malaria can be confirmed by:A.Microscopy of urineB.HaemocultureC.Bacteriology of stoolD.Serological reactionsE.*Parazitoscopy of blood

400. Rules of hospitalization of patients with malaria:A.*In separate roomB.In a respiratory infections departmentC.In the Meltser’s wardD.Patients are not hospitalizedE. In a intestinal infections department

401. Why early relapse in malaria develops?A.The immune deficiencyB.*Due to erythrocytic forms of shizontsC.Releasing of tissue shizonts to the blood stream D.Fresh contaminationE.Availability of a blood gamonts

402. Why late relapses in malaria develops?A.Due to erythrocytic forms of shizontsB.*Releasing of tissue shizonts to the blood stream C.Fresh contaminationD.Availability of a blood gamontsE.The immune deficiency

403. By what drug is possible to prevent early recurrence of malaria:A.*DelagilB.ImmunoglobulinC.AntibioticsD.PrimahinE.Glucocorticoids

404. By what drug is possible to prevent relapse of malaria:A.DelagilB.ImmunoglobulinC.AntibioticsD.*PrimahinE.Glucocorticoids

405. Radical treatment of malaria include:A.Primaquine within 2 weeksB.GlucocorticoidsC.*Delagil + primahin

Page 56: intranet.tdmu.edu.uaintranet.tdmu.edu.ua/data/kafedra/internal/infect_desease... · Web viewBest method to treat diarrhoea in child is: intra venous fluide *ORS antibiotics bowel

D.Serum transfer406. Indications for the appointment of hematoshizotrop antimalarial drugs:

A.*Attack of malariaB.Preventing late relapseC.Prophylactic course after returning from areas difficult to malariaD.Antyretsidiv course for the rekonvalescentsE.During a check-up

407. Indications for the appointment of histoshizotrop antimalarial drugs:A.Attack of malariaB.*Prevention of late relapseC.Complications of malariaD.Malaria chemoprophylaxis for a period of stay in endemic areasE.During a check-up

408. What do you need to give to the patients with malaria‘s coma?A.GlucocorticoidsB.Antishok treatmentC.*Intravenous delagilD.Blood transfusionE.Oxygen

409. Specific complications of malaria, except:A.Hemoglobinuria feverB.Spleen destructionC.Malaria‘s comaD.*Intestinal perforation E.Hemolytic anaemia

410. Reconvalences after malaria may descharged from a clinic no earlier than:A.After 2 weeks till complete clinical recoveryB.After 2 weeks of a radical course of therapyC.*After a radical course of therapy with a negative results of parazitoscopyD.After 3 weeks with a negative blood culture resultsE.After 3 weeks, if the negative results of planting feces

411. Activities on contact with import cases of malaria:A.Parasitoscopy of bloodB.The direction of the contact in the detention facility for 5 daysC.ChemoprophylaxisD.VaccinationE.*Do not hold

412. What is malaria prevention for those who have returned from disadvantaged areas:

A.Introduction of human immunoglobulinB.InterferonC.6-days-prevention by streptomycin or tetracyclineD.*Primaquine E.All the above

413. What measures must be taken in relation to people who had a contact with malaria case:

Page 57: intranet.tdmu.edu.uaintranet.tdmu.edu.ua/data/kafedra/internal/infect_desease... · Web viewBest method to treat diarrhoea in child is: intra venous fluide *ORS antibiotics bowel

A.Microscopy of periferal blood B.Isolation during 5 daysC.ChimioprophylaxisD.Houses roundsE.*Does not conduct

414. Who would be a subject for inspection on malaria?A.Recovering of malaria personsB.Persons, returning from endemic regions of malariaC.Patients with fever more than 5 daysD.Patients with spleenomegalyE.*All above enumerated

415. Methods for identification of sources of malaria:A.Stool culture testB.HaemocultureC.*Microscopic assessment of bloodD.Byurne test E.All above enumerated

416. When should begin treatment of malaria patients?A.*Immediately after hospitalizationB.After taking of material for researchC.After of final diagnosisD.After laboratory and instrumental researchE.All answers are correct

417. Delagil appoint in case of malaria in such doses:A.0,5 g 3 times a day 3 daysB.0,5 g during a weekC.*In the first day 1 g, then 0,5 g every 6 hD.0,5 g a day during a monthE.0,5 g 2 times a day 3 days

418. There are etiotropic drugs of malaria, except:A.DelagilumB.PrymahinC.*CeresynD.QuinineE.Fansydar

419. The radical course of treatment of malaria includes:A.Five-day therapy of delagilumB.Prymahin during 2 weeksC.C. Delagilum + prymahin + fansydarD.*Delagilum + prymahinE.Delagilum + fansydar

420. What do you need for treatment of chlorochyn resistent forms of malaria?A.BactrimumB.Dapson C.FansydarD.Meflohin

Page 58: intranet.tdmu.edu.uaintranet.tdmu.edu.ua/data/kafedra/internal/infect_desease... · Web viewBest method to treat diarrhoea in child is: intra venous fluide *ORS antibiotics bowel

E.*All enumerated421. The most effective antibiotics at the treatment of patients with leptospirosis are:

A.*PenicillinB.MacrolidsC.CefalosporinsD.FtorhinolonsE.Sulfanilamids

422. What daily doses of penicillin for leptospirosis treatment:A.2-3 million unitsB.*3-12 million unitsC.10-20 million unitsD.20-40 million unitsE.Over 40 million units

423. All of these is an epidemic dangerous to leptospirosis except:A.Farm animalsB.Wide rodentsC.Domestic animalsD.FoxesE.*Humans

424. How long lasts the incubation period of leptospirosis:A.2 monthB.1-7 daysC.*7-14 daysD.14-21 daysE.2-3 days

425. Which serotypes of leptospirosis caused the disease more frequent:A.L. interogansB.L. grippotyphosaC.L. canicolaD.*L. icterohaemorrhagiaE.L. Pomona

426. Agglutinines in leptospirosis arrive at a maximal titre:A.On the third day of illness B.*On the third week of illness and laterC.On the fourth week of illness D.On the second month of illness E.On the second week of illness

427. Rules of hospitalization of patients with infectious mononucleosis:A.Patients are not hospitalizedB.In a ward for the infections of respiratory tractsC.*In a separate wardD.In a ward for the infections of external coversE. In a ward for intestinal infections

428. What symptoms are not characteristic of infectious mononucleosis?A.HepatomegalyB.*Oliguria

Page 59: intranet.tdmu.edu.uaintranet.tdmu.edu.ua/data/kafedra/internal/infect_desease... · Web viewBest method to treat diarrhoea in child is: intra venous fluide *ORS antibiotics bowel

C.LimfocytosisD.TonsillitisE.Splenomegaly

429. How much atypical mononucleares present in infection mononucleosis?A.Less than 5 cellsB.*10 and more cellsC.5 and more cellsD.Less than10 cellsE.50 % cells

430. What is most possible complication occur in infectious mononucleosis?A.OliguriaB.Autoimmune processC.MeningitisD.*Spleen ruptureE.Pneumonia

431. What additional inspections must be conducted to the patient with infectious mononucleosis?

A.*IFA on HIV-infection, bacteriology inspection on diphtheriaB.IFA on HIV-infection, bacteriology inspection on a scarlet feverC.Bacteriology inspection on diphtheria and typhoid feverD.Burne and Rihth-Heddlson reactions E.Paul-Bunnel reaction and punction of lymphatic node

432. What from the following symptoms are not characteristic of infectious mononucleosis?

A.FeverB.*Defeat of kidneysC.LymphadenopathyD.TonsillitisE. Increasing of liver and spleen

433. For what disease characterize changes in a blood (presence of lymphomonocytes and atypical mononuclears?

A.FluB.*Infectious mononucleosisC.MeaselsD.AIDSE.Diphtheria

434. In a survey of donor blood found microhametosis. Assign treatment. A.DelagilB.*PrimaquineC.AntibioticsD.FluoroquinoloneE.Sulfanilamide

435. Diagnosed a patient: chronic hepatitis in the stage of integration. What markers will be in patient in this stage disease?

A.HBeAgB.Antibodies to HBeAg

Page 60: intranet.tdmu.edu.uaintranet.tdmu.edu.ua/data/kafedra/internal/infect_desease... · Web viewBest method to treat diarrhoea in child is: intra venous fluide *ORS antibiotics bowel

C.DNA OF HBVD.Viral DNA-polimeraseE.*HBsAg, anti-НBе

436. As etiotropic therapy of sharp and chronic viral hepatitis B utillize:A.CorticosteroidB.Immunomodulate preparationsC.CytostaticsD.AntibioticsE.*Antiviral preparations

437. Give recommendation for a patient in reconvalensent period of viral hepatitis during a clinical supervision after isolation.

A.*Medical supervision during 6 months, periodic biochemical inspections.B.Control bacteriological examinationsC.Full labor investigationD.To continue prophylactic inoculationsE.Supervision is not needed

438. Etiotropic therapy of viral hepatitis is.A.RibavirinB.InterferonC.Inductors of interferonD.ZefixE.*All the above

439. Choose the remedies for etiotropic therapy of viral hepatitis.A.*RibavirinB.VaccineC.Normal human immunoproteinD.HepatoprotectorE.Glucocorticoid

440. Choose the remedies for etiotropic therapy for viral hepatitis.A.AntibioticsB.*InterferonC.ProbioticsD.VaccineE.Normal human immunoprotein

441. The criteria for application of etiotropic therapy in viral hepatitis is.A.Protracted motion of HBV, HVDB.Any form of HVC.Biochemical activityD.Presence of virus replication E.*All the above

442. The criteria for application of etiotropic therapy for the patient with HCV.A.Clinical displays are insignificantB.Icterus is absentC.Moderate biochemical activityD.There is anti-HCV in bloodE.*RNA of HCV +

Page 61: intranet.tdmu.edu.uaintranet.tdmu.edu.ua/data/kafedra/internal/infect_desease... · Web viewBest method to treat diarrhoea in child is: intra venous fluide *ORS antibiotics bowel

443. Factors which are indications of successful interferon therapy in HV infections are all, except.

A.Level of ALaT not more than 2-3 normB.Low titre of HCV after the treatmentC.Absence of cholestasisD.2th and 4th genotypes of HCVE.*Expressed fibrosis

444. Indirect action of interferon therapy.A.Influenza-like syndromeB.NauseaC.ItchingD.Para-hypnosisE.*All the above||

445. Indirect action of interferons.A.FlatulenceB.DiarrheaC.NauseaD.DepressionE.*All the above

446. Indirect action of interferon therapy are all except.A.Influenza-like syndromeB.NauseaC.DepressionD.Intensification of autoimmune diseasesE.*Progress of fibrosis

447. Basic principles of antiviral therapy for viral hepatitis.A.Individual selection of dose and rhythm of application of preparationsB.Duration of introduction of preparationsC.Control of amount of erytrocytes, leucocytes and thrombocytes, in bloodD.Control of iron level in bloodE.*All the above

448. Contra-indications for antiviral therapy of viral hepatitis.A.Decompensatory cirrhosis of liverB.Thrombocytopenia <50000 in 1 мм3 C.Psychic disordersD.Leucocytopenia <1500 in 1 мм3E.*All the above

449. Contra-indications for antiviral therapy of viral hepatitis.A.Decompensatory cirrhosis of liverB.Autoimmune diseaseC.Alcoholism and other drug addictionsD.Coinfection by HIVE.*All the above

450. Choose the indexes of efficiency of interferon therapy.A.*Disappearance| of markers of viral replicationB.Improvement of the general state

Page 62: intranet.tdmu.edu.uaintranet.tdmu.edu.ua/data/kafedra/internal/infect_desease... · Web viewBest method to treat diarrhoea in child is: intra venous fluide *ORS antibiotics bowel

C.Normalization of the liver size D.Disappearance of icterusE.All the above

451. Choose the indexes of efficiency of interferon therapy.A.Improvement of the general stateB.*Normalization of activity of ALaTC.Normalization of the liver size D.Disappearance of icterusE.All the above

452. Types of answer for interferon therapy are.A.Stable remissionB.UnsteadyC.Partial answerD.Absence of answerE.*All the above

453. The characteristic of an unsteady answer of interferon therapy are.A.Disappearance of markers of viral replication upon completion of course of therapyB.Normalization of activity of ALaT during the course of therapy C.An origin of relapse in next 6 monthsD.Disappearance of icterusE.*All the above

454. That characteristic of a partial answer of interferon therapy are all, except.A.*Disappearance of markers of viral replicationB.Normalization of activity of ALaT is upon completion of course of therapyC.Disappearance of icterusD.Normalization the state of patientE.Normalization of the size of liver

455. When is interferon therapy effective in the the patient.A.Normalization of the state of patientB.Normalization of activity of ALaT upon completion of course of therapyC.Disappearance of icterusD.Normalization of the size of liverE.*The markers of viral replication, are determined upon completion of course of

therapy456. What laboratory work-up is needed for confirming the diagnosis of viral

hepatitis.A.Total analysis of bloodB.Determination of level of bilirubinC.Determination of activity of aminotransferaseD.*Determination of markers of HV in IFAE.All the above

457. What laboratory and instrumental examinations are needed for confirming the diagnosis of viral hepatitis.

A.Complete analysis of bloodB.Ultrasound of abdominal regionC.Determination of activity of aminotransferase

Page 63: intranet.tdmu.edu.uaintranet.tdmu.edu.ua/data/kafedra/internal/infect_desease... · Web viewBest method to treat diarrhoea in child is: intra venous fluide *ORS antibiotics bowel

D.*Determination of antigen of virusesE.Duodenal probing

458. What is incubation period for hepatitis B:A.45 daysB.*180 daysC.360 daysD.90 daysE.25 days

459. All the hepatitis have parenteral route of transmission except:A.*AB.BC.CD.DE.TTV

460. Chronic course is common for viral hepatitis except: A.*AB.BC.CD.DE.B+C

461. All the following medicines are interferons except:A.IntronB.RoferonC.ReaferonD.LeukinferonE.*Cycloferon

462. All the following medicines are hepatoprotective agents except:A.CarsilB.SiliborC.LegalonD.*LomusolE.Arginine

463. Phage symptom in case of yellow fever is:A.Pain in right iliac areaB.Enanthema on a soft palateC.*Replacement of tachicardia on expressed bradicardiaD.Hemorrhages in a conjunctivaE.Yellow hands

464. Hemograme in the second period of yellow fever:A.Leukocytosis B.Normal global analysis of bloodC.*Leukopenia, neutropeniaD.Leukopenia, neutrophilosisE.Leukocytosis, lymphomonocytosis

465. Whatever complication meets at the yellow fever:A.*Liver insufficiency

Page 64: intranet.tdmu.edu.uaintranet.tdmu.edu.ua/data/kafedra/internal/infect_desease... · Web viewBest method to treat diarrhoea in child is: intra venous fluide *ORS antibiotics bowel

B.Kidney insufficiencyC.Infectious-toxic shockD.MyocarditisE.Edema of lungs

466. Unlike leptospirosis in case of yellow fever is absent:A.Hemorrhagic syndromeB.Kidney insufficiencyC.Іntoxication syndromeD.Міalglic syndromeE.*Hepatic insufficiency

467. For confirmation of yellow fever diagnosis use:A.Bacteriological analysis of bloodB.Bacteriological examination of urine C.*Virological hemanalysisD.Biochemical blood testE.Global analysis of blood

468. In the initial period of hemorrhagic fever with a kidney syndrome a characteristic sign is:

A.High temperaturesB.Pains in gastrocnemius muscles and positive Pasternatsky symptomC.*Pains in joints and positive Pasternatsky symptom D.Hemorragic syndromeE.Dyspepsia phenomena

469. An initial period at the hemorrhagic fever with a kidneys syndrome lasts:A.Few hoursB.One dayC.*Three daysD.one weekE.Two weeks

470. When a violations of diuresis at patients with hemorrhagic fever with a kidneys syndrome appear:

A.In initial periodB.Don’t appearC.During all periods of diseaseD.*In climax periodE. In recovering period

471. General view of patient with the hemorrhagic fever with a kidneys syndrome:A.Skinning coversB.*Pallor of nasolabial triangle, hyperemia of neck and overhead half of trunkC.Hyperemia of person, scleritis, conjunctivitisD.Grayish color of personE. Icteric color of skin

472. In a biochemical blood test of patients with the hemorrhagic fever with a kidneys syndrome not typically:

A.High level of ureaB.Decline of potassium level

Page 65: intranet.tdmu.edu.uaintranet.tdmu.edu.ua/data/kafedra/internal/infect_desease... · Web viewBest method to treat diarrhoea in child is: intra venous fluide *ORS antibiotics bowel

C.*BilirubinemiaD.Increasing of kreatinineE. Increasing of nitrogen

473. For confirmation of diagnosis of hemorragic fever with a kidney syndrome use:A.Bacteriological methodB.Virological methodC.*Reaction of immunofluorescenceD.Reaction of braking of hemagglutinationE.Research of blood drop under a microscope

474. For treatment of patients with the hemorrhagic fever with a kidney syndrome does not use:

A.GlucocorticoidsB.Anabolic steroidC.Disintoxication facilitiesD.*Dihydration facilitiesE.Antihistaminics

475. For the initial period of the Congo hemorrhagic fever not characteristically:A.FeverB.Pains in joints and musclesC.Severe pain of headD.*OliguriaE.Dizziness

476. At an objective review for the Congo hemorrhagic fever characteristically:A.*Mucosal hyperemia of personB.Pallor of personC.Puffiness of personD.Ochrodermia of personE.Exanthema on face

477. The most characteristic symptom in the climax period of the Congo hemorrhagic fever is:

A.*Hemorrhagic syndromeB.Hepatic insufficiencyC.Dyspepsia phenomenaD.Sharp kidney insufficiencyE.Мeningeal syndrome

478. In a case of Congo fever in a general blood analysis is not typical:A.. LeukocytosisB.*LeukopeniaC.NeutropeniaD.ThrombocytopeniaE. Increasing of ESR

479. What laboratory and instrumental examinations should used for flu diagnosis?A.Complete analysis of bloodB.X-ray of organs of thoraxic cavityC.Analysis sputumD.*Determination of viruses by the method of immunofluorescence

Page 66: intranet.tdmu.edu.uaintranet.tdmu.edu.ua/data/kafedra/internal/infect_desease... · Web viewBest method to treat diarrhoea in child is: intra venous fluide *ORS antibiotics bowel

E.Biochemical blood test480. Virus causing hemorrhagic cystitis, diarrhea and conjunctivitis:

A.RSVB.RhinovirusC.*AdenovirusD.RotavirusE.Flu

481. What is conduct specific passive immunnoprophylaxis of flu?A.Live antenuated vaccineB.Inactive parenteral vaccineC.*Immune proteinD.RemantadinE.Antibiotics of wide spectrum of action|

482. Duration of isolation of patient with influenza complications?A.4 daysB.7 daysC.*10 daysD.17 daysE.20 days

483. How is the urgent prophylaxis of scarlet fever conducted?A.VaccinationB.*Isolation of children, who had contact with a patientC.ChemioprophylaxisD.DisinfectionE.Non-admission of contact with carrier of B-streptococcus

484. What level is necessary to reduce the temperature of patient’s body with hyperthermia?

A.39 °C B.*38 °CC.37,5 °CD.37 °CE.38,5 °C

485. What made specific passive flu immunization?A.Living intranasal vaccineB.Parenteral inactivated vaccineC.* Immune globulinD.RemantadinE.Antibiotic

486. What pathogen causes severe acute respiratory syndrome?A.BocavirusB.RheovirusC.MetapneumovirusD.AdenovirusE.*Coronavirus

487. Which family owned influenza?A.Pallidum

Page 67: intranet.tdmu.edu.uaintranet.tdmu.edu.ua/data/kafedra/internal/infect_desease... · Web viewBest method to treat diarrhoea in child is: intra venous fluide *ORS antibiotics bowel

B.LegionellaC.TohovirusD.* OrtomixovirusE.Rickettsiae

488. The intensity of intoxication caused by flu depends on?A.HemagglutininB.NeuraminidaseC.* RybonucleoproteidD.Membrane proteinsE.S-protein

489. What determs the immunosuppressive action of influenza virus?A.*HemagglutininB.S-proteinC.NeuraminidaseD.RNA polymeraseE.The membrane protein

490. What is antigenic shift of influenza virus?A.*Antigenic changes of rybonucleoproteidB.Recombination hemagglutinin and neuraminidaseC.Antigenic changes of the virus within serovarsD.Genetic recombination between different strains of influenza virusE.Variability neuraminidase

491. What medium used for culturing influenza virus?A.*Chicken embryos and cell cultureB.Gall brothC.Meat pepton agarD.Medium containing bloodE.Water-serum culture medium

492. Which of these protein of influenza virus is capable to hemolise red blood cells?A.RNA polymerase B.S-proteinC.NeuraminidaseD.Membrane proteinE.* Hemagglutinin

493. Specify contagious period in flu?A.The end of the incubation period + the entire period of the diseaseB.End of the incubation periodC.Only during the heightD.* The period of convalescenceE.The crisis period + 7 days normal body temperature

494. What is a gateway for the flu virus?A.*Cylindrical mucosal epithelium of respiratory tractB.Solitary folliclesC.Mucous of tonsilsD.Epithelial cells of the skinE.Mucous membranes of the digestive tract

Page 68: intranet.tdmu.edu.uaintranet.tdmu.edu.ua/data/kafedra/internal/infect_desease... · Web viewBest method to treat diarrhoea in child is: intra venous fluide *ORS antibiotics bowel

495. What is antigenic drift of influenza virus?A.*Partial change in antigenic specificity of hemagglutininB.Recombination hemagglutinin and neuraminidaseC.Antigenic changes of the virus within serovarsD.Genetic recombination between different strains of influenza virusE.Variability neuraminidase

496. What is the main preventive measure in case of contact with sick with the flu?A.VaccinationB.* ChemoprophylaxisC.VitaminoprophylaxisD.Admission antipyreticsE.Admission immunostimulators

497. Which of these vaccines is the least reactivity?A.FullvirionB.Split vaccinesC.* SubunitD.LiveE. Inactivated

498. What is the most common form of adenovirus infection?A.TracheobronchitisB.LaryngitisC.PneumoniaD.* Pharyngoconjunctive feverE.Rhinitis

499. What is most often clinically manifested flu?A.NasopharyngitisB.BronchospasmC.RhinitisD.Acute respiratory failureE.* Laryngotracheitis

500. What is most evident parainfluenza infection?A.RhinitisB.* LaryngitisC.TracheobronchitisD.LymphadenopathyE.Bronchospasm

501. Which agents of acute respiratory infections contain DNA?A.Respiratory virus syntytsialnyyB.Influenza virusC.Parainfluenza virusD.* AdenovirusE.Rhinovirus

502. Which of acute respiratory diseases transmitted not only through airborne, but the fecal-oral transmission mechanism?

A.Rhinoviral diseaseB.Parainfluenza

Page 69: intranet.tdmu.edu.uaintranet.tdmu.edu.ua/data/kafedra/internal/infect_desease... · Web viewBest method to treat diarrhoea in child is: intra venous fluide *ORS antibiotics bowel

C.* Adenoviral diseaseD.Respiratory syntytsial disease E.Flu

503. What is the duration of incubation period of adenoviral infection?A.Several hours - 1 dayB.1-2 daysC.* 1-14 daysD.2-3 daysE.14-21 day

504. What is characteristic of adenoviral conjunctivitis unlike diphtheria eyes?A.Conjunctiva of eyelids bright red tape snug, hard shotB.Complete loss of vision due panoftalmitC.* Conjunctiva of eyelids bright red, covered with films that are easily removedD.Photophobia, pain during palpationE.Conjunctiva with haemorrhages

505. Which of the following is characteristic of adenovirus infection?A.BronchiolitisB.Bronchiolitis and pneumoniaC.Real croup, pneumoniaD.* Membranous conjunctivitis and pharyngitisE.Bronchiolitiasis and false croup

506. Which family pathogen of parainfluenza belong to?A.PneumoviridaeB.* ParamyxoviridaeC.OrtomyxoviridaeD.AdenoviridaeE.Reoviridae

507. What is theduration of incubation period in parainfluenza?A.Several hours - 2 daysB.7-9 daysC.9-14 daysD.* 7-2 daysE.14-21 day

508. What is most often damaged in parainfluenza?A.*The mucous membrane of the larynx and tracheaB.Lower respiratory tractC.Maxillary sinusD.ConjunctivaE.Nasal mucosa

509. What complication is typical for parainfluenza?A.Real croupB.* False croupC.SinusitisD.PneumoniaE.Pharyngitis

510. Which family pathogen of respiratory syncytial infection belong to?

Page 70: intranet.tdmu.edu.uaintranet.tdmu.edu.ua/data/kafedra/internal/infect_desease... · Web viewBest method to treat diarrhoea in child is: intra venous fluide *ORS antibiotics bowel

A.PicornaviridaeB.OrthomyxoviridaeC.AdenoviridaeD.ReoviridaeE.* Paramyxoviridae

511. Which of acute respiratory disease is characterized by the development of pulmonary edema?

A.Adenovirus infectionB.* FluC.Respiratory syncytial infectionD.ParainfluenzaE.Psittacosis

512. What changes in general blood analisis is characteristic in rhinovirus infection?A.*The number of leukocytes and ESR are not changedB.Significant leukocytosis and increased ESRC.Leukocytosis with neutrocytosisD.Leukopenia with lymphocytosisE.Leukocytosis, anemia, thrombocytopenia

513. What is the possible complication of respiratory syncytial infection?A.EtmoiditisB.OtitisC.PneumoniaD.SinusitisE.* All of the above

514. Which family agent of rhinovirus infection belong to?A.RetroviridaeB.ParamyxoviridaeC.*PicornaviridaeD.AdenoviridaeE.Reoviridae

515. What is the duration of incubation period of rhinovirus infection?A.Several hours-1 dayB.1-6 daysC.*7-10 daysD.10-14 daysE.14-20 days

516. Which family pathogen of metapnevmovirus infection owned to?A.RetroviridaeB.* ParamyxoviridaeC.PicornaviridaeD.AdenoviridaeE.Reoviridae

517. What is the most effective drug in the treatment of metapneumovirus infection?A.GanacyclovirB.AcyclovirC.Tamiflu

Page 71: intranet.tdmu.edu.uaintranet.tdmu.edu.ua/data/kafedra/internal/infect_desease... · Web viewBest method to treat diarrhoea in child is: intra venous fluide *ORS antibiotics bowel

D.RemantadinE.* Ribavirin

518. What transfer factor is not typical for coronavirus infection?A.AirB.WaterC.* SemenD.BloodE.Urine

519. People of what ages is most sensitive for bocavirus infection?A.*Children from neonatal period up to 1 yearB.Children from 1 to 5 yearsC.Children from 6 to 10 yearsD.TeensE.Adults

520. For what viral infection convulsions may occur?A.Adenovirus infectionB.Respiratory syntytsialnaC.Rynovirusna infectionD.ParainfluenzaE.* Flu

521. What are the symptoms of croup syndrome?A.*Inspiratory dyspneaB.Expiratory wheezeC.Gruff "barking" coughD.Spastic coughE.Availability of films in the oropharynx

522. The usage of what drug is not appropriate in mild and moderate forms of flu?A.AntiviralB.ExpectorantC.* AntibioticsD.ImmunostimulatorsE.Desensitizing

523. What is the most effective drug in the treatment of influenza?A.GanacyclovirB.AcyclovirC.* IngavirinD.RemantadinE.Amizon

524. For what infection meningeal syndrome is most typical?A.Adenovirus infectionB.ParainfluenzaC.Rhinovirus infectionD.* FluE.Respiratory syncytial disease

525. What infectious disease is characterized by the local reaction of the nasal mucosa with hyperemia, edema and significant secretion?

Page 72: intranet.tdmu.edu.uaintranet.tdmu.edu.ua/data/kafedra/internal/infect_desease... · Web viewBest method to treat diarrhoea in child is: intra venous fluide *ORS antibiotics bowel

A.*Rhinovirus diseaseB.Typhoid feverC.MalariaD.VaricellaE.Flu

526. What viruse more often cause nosocomial infection?A.AdenovirusesB.Parainfluenza virusC.RhinovirusD.Influenza virusesE.* Coronavirus

527. Methods of laboratory diagnosis of acute respiratory viral infections except:A.VirologicalB.SerumC.* Blood cultureD.PCRE.Flu

528. Specific methods of laboratory diagnosis of influenza:A.Cultivation of the virus in chicken eggs or tissue culturesB.Detection of virus antigens by immunofluorescence in nasal washings fromC.Detection of antibodies to the virus in paired sera of bloodD.Detection of virus antigens by fluorescent microscopy in smears from the nasal

mucosaE.* All of the above

529. What laboratory tests of influenza?A.Immunofluorescence method of nasopharyngeal swabsB.General bloodC.Bacteriological investigation of sputumD.Biological test on laboratory animalsE.*All of the above

530. What are the indications for antibiotics therapy in flu?A.Very severe courseB.Presence of complicationsC.Some age groups (children, elderly people age)D.The presence of foci of chronic bacterial infectionsE.* All of the above

531. What drug is used as etiotropic therapy in adenoviral infection?A.ParacetamolB.AspirinC.CeftriaxoneD.* DezoxirybonukleazE.All of the above

532. What indicates possible complications in flu?A.Duration of hyperthermia over 5 daysB.LeukocytosisC.Netrophilosis

Page 73: intranet.tdmu.edu.uaintranet.tdmu.edu.ua/data/kafedra/internal/infect_desease... · Web viewBest method to treat diarrhoea in child is: intra venous fluide *ORS antibiotics bowel

D.Accelerated ESRE.* All of the above

533. What is color in fluorescent microscope preparation in case of influenza?A.RedB.* GreenC.BlueD.YellowE.Violet

534. What is the entrance gate at infectious mononucleosis? A.Mucosa of colonB.Mucosa of digestive tract C.Epithelial cells of skin D.Peyer‘s plate and follicles E.*Mucosa of nazo-pharig

535. What is preparation of specific therapy for viral neuro infection? A.* Acyclovir B.Cefataxime C.Ceftriaxone D.Gentamycin E.Furazolidon

536. What is preparation for specific therapy of viral neiroinfection? A.Lazix B.Cefotaksim C.Ceftriakson D.* Acyclovir E.Prednisolon

537. What is drug for specific therapy of widespread form of diphtheria of nasopharynx.

A.Macrolids per os B.Penicillin i/m C.Cortycosteroid D.* Antidiphterial serum i/v E.Antitoxic therapy

538. What is the first dose of antidiphterial serum for a 6 years child with a diphtherial widespread croup:

A.* 40 AО B.15 AО C.20 AО D.80 AО E.60 AО

539. What is the first dose of antitoxic antidiphtherial serum for a patient with diphtheria of pharynx?

A.120 thousand of AО B.80 thousand of AО C.* 30 thousand of AО D.50 thousand of AО

Page 74: intranet.tdmu.edu.uaintranet.tdmu.edu.ua/data/kafedra/internal/infect_desease... · Web viewBest method to treat diarrhoea in child is: intra venous fluide *ORS antibiotics bowel

E.150 thousand of AО 540. At the end of treatment of patients with tonsillitis it is recommended to enter:

A.500 000 of Bicyllin-3 intramuscular B.* 1 500 000 of Bicyllin-5 intramuscular C.1 000 000 of Bicyllin-5 intramuscular D.1 500 000 of Bicyllin-3 intramuscular E.500 000 of Bicyllin-3 intramuscular

541. Before revaccination from diphtheria of adult persons, are recommended: A.* To explore an immune type B.To use antibiotics C.To use antihistamines D.5 years after last revaccination E.10 years after last revaccination

542. Before revaccination from diphtheria of adult persons, are recommended: A.* To explore an immune type B.To use antibiotics C.To use antihistamines D.5 years after last revaccination E.10 years after last revaccination

543. What is the exciter of tonsillitis (angina). A.* Streptococcus of group A B.Streptococcus of group B C.Streptococcus of group C D.Streptococcus of group D E.Streptococcus of group E

544. Choose, what changes are characteristic for diphtheria of tonsils. A.Tonsils enlarged, edematous, on-the-spot of tonsills are some heaved up

subephithelial abscesses yellow-white color B.In lacunes of tonsils are a pus as yellow-white coat C.Tonsils are hyperemic, hypertrophied, on both are necrotizing areas dark grey

color, after removing layer by layer of which the deep defect of mucus shell appeared with an uneven bottom

D.One tonsils hyperemic and filling out, on its surface there is a grey-white coat, under it coat – bleeding ulcer with a smooth bottom

E.* One tonsil is enlarged, on him dense grey-white color coat, which becomes separated from hardness, mucus bleeds under him

545. Choose, what changes are characteristic for a follicle tonsillitis (angina). A.* Tonsils enlarged, edematous, on-the-spot of tonsils are some heaved up

subephithelial abscesses yellow-white color B.In lacunes of tonsils are a pus as yellow-white coat C.Tonsils are hyperemic, hypertrophied, on both are necrotizing areas dark grey

color, after removing layer by layer of which the deep defect of mucus shell appeared with an uneven bottom

D.One tonsils hyperemic and filling out, on its surface there is a grey-white coat, under it coat – bleeding ulcer with a smooth bottom

Page 75: intranet.tdmu.edu.uaintranet.tdmu.edu.ua/data/kafedra/internal/infect_desease... · Web viewBest method to treat diarrhoea in child is: intra venous fluide *ORS antibiotics bowel

E.One tonsil is enlarged, on him dense grey-white color coat, which becomes separated from hardness, mucus bleeds under him

546. Choose, what changes are characteristic for a lacunars tonsillitis (angina). A.Tonsils enlarged, edematous, on-the-spot of tonsils are some heaved up

subephithelial abscesses yellow-white color B.* In lacunes of tonsils are a pus as yellow-white coat C.Tonsils are hyperemic, hypertrophied, on both are necrotizing areas dark grey

color, after removing layer by layer of which the deep defect of mucus shell appeared with an uneven bottom

D.One tonsils hyperemic and filling out, on its surface there is a grey-white coat, under it coat – bleeding ulcer with a smooth bottom

E.One tonsil is enlarged, on him dense grey-white color coat, which becomes separated from hardness, mucus bleeds under him

547. Choose, what changes are characteristic for a ulcers-necrotic tonsillitis (angina). A.Tonsils enlarged, edematous, on-the-spot of tonsils are some heaved up

subephithelial abscesses yellow-white color B.In lacunes of tonsils are a pus as yellow-white coat C.* Tonsils are hyperemic, hypertrophied, on both are necrotizing areas dark grey

color, after removing layer by layer of which the deep defect of mucus shell appeared with an uneven bottom

D.One tonsils hyperemic and filling out, on its surface there is a grey-white coat, under it coat – bleeding ulcer with a smooth bottom

E.One tonsil is enlarged, on him dense grey-white color coat, which becomes separated from hardness, mucus bleeds under him

548. Choose, what changes are characteristic for a Vensan-Plaut‘s tonsillitis. A.Tonsils enlarged, edematous, on-the-spot of tonsils are some heaved up

subephithelial abscesses yellow-white color B.In lacunes of tonsils are a pus as yellow-white coat C.Tonsils are hyperemic, hypertrophied, on both are necrotizing areas dark grey

color, after removing layer by layer of which the deep defect of mucus shell appeared with an uneven bottom

D.* One tonsils hyperemic and filling out, on its surface there is a grey-white coat, under it coat – bleeding ulcer with a smooth bottom

E.One tonsil is enlarged, on him dense grey-white color coat, which becomes separated from hardness, mucus bleeds under him

549. Complication of diphtheria of larynx is: A.Myocarditis B.Paresis of auditory nerve C.Nephrosonephritis D.* Cereals E.Poliomyelitis

550. Complications of 4-5th week of diphtheria are: A.Encephalitis B.Bulbar disorders, pancreatitis, hepatitis C.* Poliomyelitis, myocarditis D.Nephrosonephritis

Page 76: intranet.tdmu.edu.uaintranet.tdmu.edu.ua/data/kafedra/internal/infect_desease... · Web viewBest method to treat diarrhoea in child is: intra venous fluide *ORS antibiotics bowel

E.Stenotic laryngotracheitis 551. Complications which often develop on the first week of diphtheria of otopharynx

are: A.Poliomyelitis B.Asphyxia C.Insufficiency of glandulars D.hepatospleenomegaly E.* Paresis of soft palate

552. Diphtheria planned vaccination begin in: A.In first days after birth of child B.* In 3 month age C.In 6-month age D.In 1 year E. In 6 years

553. Early complications of diphtheria of otopharynx is: A.* Paresis of soft palate B.Pneumonia C.Asphyxia D.Croup E.Poliomyelitis

554. Especially high titre of ant diptheria antitoxic antibodies testifies in: A.Recovering B.Acute period of diphtheria C.* Bacteriocarriering D.Forming of immunity to diphtheria E.About nothing does not testify

555. Etiology agent of meningitis is: A.* Neisseria meningitides B.Entamoeba histolytica C.Vibro cholerae D.Clostridium botulinum E.Campylobacter pylori

556. For corynebacterium diphtheria is typical: A.Contain endotoxin only B.* Exotoxin production C.Exotoxin does not product D.Enterotoxin production E.Myelotoxin production

557. For the treatment of acidosis in meningococcal meningitis is better to use. A.10-20 % glucose solution B.10 % chloride solution C.* 4 % sodium bicarbonate solution D.Albumen E.Concentrated dry plasma

558. What disease is typical changes in blood (presence of atypical mononucleares)? A.Flu

Page 77: intranet.tdmu.edu.uaintranet.tdmu.edu.ua/data/kafedra/internal/infect_desease... · Web viewBest method to treat diarrhoea in child is: intra venous fluide *ORS antibiotics bowel

B.* Infectious mononucleosis C.Measles D.AIDS E.Diphtheria

559. What etiotropic (antistreptococcal) facilities are the most effective : A.Furazolidonum B.Gentamicin C.* Benzilpenicilin and Oxacillinum D.Benzilpenicilin and Furazolidonum E.Doksiciklin and Gentamicin

560. How is it possible to specify the diagnosis of meningococcal meningitis. A.Meningitis is primary B.Presence of a lot of cells in the CSF C.Presence of gram-negative diplococcus in CSF D.Meningococes from the throat E.* All the above

561. How long is the incubation period of a tonsillitis (angina)? A.From a few hours to 5 days B.From a few hours to 4 days C.From a few hours to 3 days D.* From a few hours to 2 days E.From a few hours to 1 days

562. How long is the period of fever in patients with a tonsillitis (angina)? A.1-2 days B.2-3 days C.* 3-5 days D.5-7 days E. ..More than week

563. Name the exciter of acute tonsillitis:A.α -hemolytic streptococcus of group AB.γ -hemolytic streptococcus of group AC.α -hemolytic streptococcus of group CD.*β -hemolytic streptococcus of group C E.β -hemolytic streptococcus of group A

564. The source of exciter of tonsillitis is:A.patient with anginaB.patient with erysipelasC.patient with the scarlet feverD.healthy transmitter of hemolytic streptococcus E.*all listed above

565. What is the basic mechanism of transmission of tonsillitis?A.*air-dropletB.alimentaryC.contact D.transmisiveE.vertical

Page 78: intranet.tdmu.edu.uaintranet.tdmu.edu.ua/data/kafedra/internal/infect_desease... · Web viewBest method to treat diarrhoea in child is: intra venous fluide *ORS antibiotics bowel

566. How long last the incubation period in tonsillitis? A.from a few hours to 5 days B.from a few hours to 4 daysC.from a few hours to 3 days D.*from a few hours to 2 days E. from a few hours to 1 days

567. Angina begins sharply, from headache, increase of temperature of body, dull ache in joints and chill. What other characteristic syndrome of appears simultaneously (rarer in the end of 1st days)?

A.nauseaB.vomitingC.*pain in the throat D.pain in the stomachE. tachycardia

568. A frequent and early symptom of tonsillitis is an enlargement of lymph nodes, their pain. Which group of lymph nodes enlarge first of all?

A.posterior-neckB.occypital C.supraclavicularD.*submandibularE.anterior-neck

569. What is the average duration of fever period in tonsillitis?A.days B.2-3 daysC.*3-5 daysD.5-7 daysE.More one week

570. What kinds of angina can be distinguish according to the changes in a tonsils? A.Catarrhal, follicle and lacunarB.*Catarrhal, follicle, lacunar and necrotic-ulcerous C.Catarrhal, follicle, lacunar, pellicle and necrotic-ulcerousD.Follicle, lacunar and necrotic-ulcerous E.Follicle, lacunar, pellicle and necrotic-ulcerous

571. What is the medicine for specific therapy of widespread form of nasopharynx diphtheria:

A.Macrolids per osB.Penicillin i/mC.CortycosteroidD.*Antidiphterial serum i/vE.Antitoxic therapy

572. Specify the correct method of serum introduction after the Bezredko method:A.1,0 ml of divorced 1:100 hypodermic – through 30 min. 0,1 ml of undivorced

hypodermic – through 30 min. all dose of intramuscleB.0,1 ml of divorced 1:1 000 endermic – through 30 min. 0,1 ml of divorced 1:10

hypodermic – through 30 min. all dose of intramuscle

Page 79: intranet.tdmu.edu.uaintranet.tdmu.edu.ua/data/kafedra/internal/infect_desease... · Web viewBest method to treat diarrhoea in child is: intra venous fluide *ORS antibiotics bowel

C.0,1 ml of undivorced endermic – through 30 min. 0,1 ml hypodermic – through 30 min. all dose of intramuscle

D.*0,1 ml of divorced 1:100 endermic – through 30 min. 0,1 ml of undivorced hypodermic – through 30 min. all dose of intramuscle

E.1,0 ml of divorced 1:10 hypodermic – through 30 min. 0,1 ml of undivorced hypodermic – through 30 min. all dose of intramuscle

573. What is characteristic signs of raid at diphtheria?A.One-sided, grey-white, on-the-spot crateriform ulcersB.*Grey-white, dense with clear edges and brilliant surfaceC.Yellow-white, fragile, perilacunar is locatedD.One-sided, yellow-white, in lacunasE.White, fragile, is easily taken off by a spatula

574. What is the exciter of diphtheria:A.Virus of Epshtein-BarrB.*Leffler Bacillus C.Corynebacteria ulceransD.Fusiform stickE.Corynebacteria xerosis

575. Etiology agent of meningitis are accept:A.Staphylococci B.Neisseria meningitides C.Mycobacterium tuberculosis D.Viruses E.*Entamoeba histolytica

576. Wich of these symptoms are often present in patients with meningitis?A.Profuse watery diarrhea, vomiting, dehydratation, muscular cramps B.*Fever, headache, stiff neck, vomiting, confusion, irritability C.Headache, dry cough, algor D.Abdominal pain, diarrhea, constipation, flatulence E.Algor, high temperature, headache

577. What measures it’s necessary to perform in the focus of viral meningoencephalitis?

A.Bacteriological inspection of contact B.Phagoprophylaxis C.Supervision during 2 weeks D.Chemoprophylaxis E.*Immunization

578. What is used as specific prophylaxis of viral meningoencephalitis.A.Immune globulin B.*. Vaccine C.Antibioticsnatoxin D.Serum E.Nothing

579. What laboratory methods should be taken for diagnosis of meningoencephalitis?A.Lumbar puncture B.*Biopsy of tissues

Page 80: intranet.tdmu.edu.uaintranet.tdmu.edu.ua/data/kafedra/internal/infect_desease... · Web viewBest method to treat diarrhoea in child is: intra venous fluide *ORS antibiotics bowel

C.Urine examination D.Coprogram E.Serologic detection

580. What clinical forms of tick encephalitis present accept:A.Meningeal B.Meningoencephalitic C.*General D.Feverish E.Meningoencephalopoliemielitic

581. What drugs of choice at etiotropic therapy of viral meningoencephalitis.A.Penicillins B.Amynoglicosides C.*Interferons D.Phtorhinilons E.Nitrofuranes

582. Viral meningoencephalitis and DIC-syndrome require above all things.A.Administration of diuretics B.Administration of antihistaminic preparations C.Administration of vitamins D.*Administration of analgesic E.Administration of heparin

583. Source of meningoencephalitis are accept:A.People B.Ticks C.*Fish D.Fleas E.Mosquitoes

584. How is it possible to specify the diagnosis of viral meningoencephalitis after cerebrospinal puncture.

A.Some increasing of chlorides in CSF B.Presence of a lot of lymphocytes in CSF C.*All the above D.Some increasing of protein in CSF E.Neurolymph under high pressure

585. Etiology agent of meningitis is:A.*Neisseria meningitidesB.Entamoeba histolyticaC.Vibro choleraeD.Clostridium botulinumE.Campylobacter pylori

586. Witch of these symptoms are often present in patients with meningitis?A.Algor, high temperature, headacheB.Profuse watery diarrhea, vomiting, dehydratation, muscular crampsC.Abdominal pain, diarrhea, constipation, flatulenceD.Headache, dry cough, algor

Page 81: intranet.tdmu.edu.uaintranet.tdmu.edu.ua/data/kafedra/internal/infect_desease... · Web viewBest method to treat diarrhoea in child is: intra venous fluide *ORS antibiotics bowel

E.*Prodromal respiratory illness or sore throat, fever, headache, stiff neck, vomiting, confusion, irritability

587. What laboratory methods should be taken to discharge meningitis?A.*Lumbar punctureB.Serologic detectionC.Urine examinationD.CoprogramaE.Biopsy of tissues

588. Source of meningitis is:A.AnimalsB.BirdsC.FishD.Pediculus humanusE.*People

589. How is it possible to specify the diagnosis of meningococcal meningitis.A.Meningitis is primaryB.Presence of a lot of cells in the CSFC.Presence of gram-negative diplococcus in CSFD.Meningococes from the throatE.*All the above

590. What are the rules at taking of smear material on the discovery of meningococal infection?

A.The taken away material at drawing out must not touch only mucus shell of cheeks and tongue

B.The taken away material at drawing out must not touch only teeth and tongueC.The taken away material at drawing out must not touch only teeth, mucus shell of

cheeksD.*The taken away material at drawing out must not touch teeth, mucus shell of

cheeks and tongueE.The taken away material at drawing out can touch teeth, mucus shell of cheeks and

tongue591. What temperature terms is it needed for cultivation of meningococcal on

artificial mediums?A.23-40 °CB.35-43 °CC.*35-37 °CD.23-35 °CE.37-39 °C

592. When does the laboratory give the results of bacteriological examination of smear from throat?

A.On 2th daysB.On 3th daysC.*On 4th daysD.On 5th daysE.On 6th days

593. What is taken for serum research for confirmation of meningococcal infection?

Page 82: intranet.tdmu.edu.uaintranet.tdmu.edu.ua/data/kafedra/internal/infect_desease... · Web viewBest method to treat diarrhoea in child is: intra venous fluide *ORS antibiotics bowel

A.*BloodB.MucusC.UrineD.CSFE.Saliva

594. What antibiotics preparations of choice of etiotropic therapy at a meningococcal infection.

A.* Benzylpenicillin and it derivatives B. GentamycinC. CefazolinD.SulfolamideE.Ciprofloxacin

595. In what dose should| benzyl penicillin be administered at meningococcal meningitis?

A.From a calculation 100-300 thousands unit on 1 kg of mass of body on daysB.*From a calculation 200-500 thousands unit on 1 kg of mass of body on daysC.From a calculation 500-700 thousands unit on 1 kg of mass of body on daysD.From a calculation 700-900 thousands unit on 1 kg of mass of body on daysE.Regardless of mass of body

596. In what daily interval should the dose of benzylpenicillin at meningococcal meningitis administered.

A.2 hrsB.*4 hrsC.6 hrsD.5 hrsE.8 hrs

597. Which preparation has a bacteriostatic action, and is more expedient to begin etiotropic therapy in the case of infectious toxic shock.

A.From benzylpenicillin and its derivativesB.From ciprofloxacinC.From gentamycinD.From ciprofloxacinE.*From levomycitin of succinate

598. A patient is sick with meningococcal meningitis. He take a massive dose of penicillin. 4 days temperature of body 36,6-36,8 °C. Meningeal signs are negative. When is it possible to stop the antibiotic therapy.

A.*At a cytosis in a CSF 100 and less, lymphocytes prevailB.After 10 days from the beginning antibiotic therapyC.After 7 days from the beginning antibiotic therapyD.At a cytosis 100 and less, neutrophil prevailE.From 6 days from the beginning antibiotic

599. A patient with meningococcal meningitis gets penicillin during 7 days. The temperature of body is normal 4 days. Meningeal signs are negative. When is it possible to stop the antibiotic therapy.

A.In default of leucocytosis displacement in bloodB.*. At a cytosis in a neurolymph 100 and less, lymphocyte prevail

Page 83: intranet.tdmu.edu.uaintranet.tdmu.edu.ua/data/kafedra/internal/infect_desease... · Web viewBest method to treat diarrhoea in child is: intra venous fluide *ORS antibiotics bowel

C.At a cytosis in a neurolymph 100 and less, neutrophil prevailD.At a cytosis in a neurolymph 150, lymphocyte prevailE.At once immediately

600. For the treatment of acidosis at meningococcal meningitis is better to use.A.10-20 % glucose solutionB.10 % chloride solutionC.*4 % sodium bicarbonate solutionD.AlbuminE.Concentrated dry plasma

601. Meningococemia and ID-syndrome require above all things.A.Administration of diureticsB.Administration of analgesicC.*Administration of heparinD.Administration of vitaminsE.Administration of antihistaminic preparations

602. What is used as specific prophylaxis in the period of epidemic spreading of meningococcal infection.

A.Immun globulinB.SerumC.*VaccineD.AnatoxinE.Nothing

603. What measures are conducted in the place of meningococcal infection?A.Supervision during 2 weeksB.PhagoprophylaxisC.ImmunizationD.*Bacteriological inspection of contactE.Chemoprophylaxis

604. What complication has developed in patient with diphtheria of mouth pellicle severe form was diagnosed. On the 6th day of disease when pain in the heart region, palpitation were appeared. Pulse – 120 per 1 min, systolic noise on apex of heart. On ECG is incomplete blockade of left leg of Giss bunch?

A.*Early infectious-toxic myocarditisB.Myocardial dystrophyC.Heart attack of myocardiumD.Acute cardio-vessel insufficiencyE.Stenosis of mitral valve

605. What group of infectious diseases diphtheria belong to?A.SapronosisB.ZoonosisC.*AnthroponosisD.ZooanthroponosisE.A group is not certain

606. What is the properties of сorynebacterium diphtheria:A.Contain endotoxin onlyB.*Exotoxin products

Page 84: intranet.tdmu.edu.uaintranet.tdmu.edu.ua/data/kafedra/internal/infect_desease... · Web viewBest method to treat diarrhoea in child is: intra venous fluide *ORS antibiotics bowel

C.Exotoxin does not productD.An enterotoxin productsE.Myelotoxin products

607. The source of infection at diphtheria is:A.*Sick people and carriersB.Sick agricultural animalsC.RodentsD.MosquitoesE.Aerosol of saliva and epipharyngeal mucous of patients

608. What is mechanism of transmission of Corynebacterium diphtheria?A.VerticalB.TransmissiveC.*Air-dropD.ContactE.Parenteral

609. Especially high titre of ant diptherial antitoxic antibodies testifies to:A.IncubationB.Acute period of diphtheriaC.*BacteriocarrieringD.Forming of immunity to diphtheria E.About nothing does not testify

610. What group of infectious diseases by L. Gromashevsky classification diphtheria belong to?

A.External coversB.BloodC.IntestinalD.*Respiratory E.Transmissive

611. What is transmissive factors in diphtheria?A.BloodB.WaterC.*SalivaD.UrineE.Exrements

612. What is seasonal character of diphtheria?A.Spring-summerB.Summer-autumnC.*Autumn-winterD.Winter-springE.Spring-autumn

613. Before revaccination from diphtheria of adult persons, they are recommended:A.*To explore an immune typeB.To use antibioticsC.To use antihistaminesD.5 years after last revaccinationE.10 years after last revaccination

Page 85: intranet.tdmu.edu.uaintranet.tdmu.edu.ua/data/kafedra/internal/infect_desease... · Web viewBest method to treat diarrhoea in child is: intra venous fluide *ORS antibiotics bowel

614. Diphtheria planned vaccination begin in:A.In first days after birth of childB.*In 3 month ageC.In 6-month ageD.In 1 yearE. In 6 years

615. In preschool is case of disease on diphtheriA. What prophylactic measures must be conducted above all things?

A.Urgent hospitalizationB.Urgent vaccinationC.*Quarantines measuresD.Urgent by chemical prophylactic antibioticsE. Introduction of antidiphterial whey

616. At a child 6 years with a diphtherial widespread croup the first dose of antidiphterial serum makes:

A.*40 AОB.15 AОC.20 AОD.80 AОE.60 AО

617. What material it’s necessary to take for bacteriologic examination in suspicion on diphtheria?

A.ExcrementB.BloodC.UrineD.*MucousE.Neurolymph

618. What is main complication of diphtheria of larynx:A.MyocarditisB.Paresis of auditory nerveC.NephrosonephritisD.*CroupE.Poliomyelitis

619. What complications more often develops during the first week of diphtheria of otopharynx:

A.PoliomyelitisB.AsphyxiaC.ParatonsillitisD.HepatospleenomegalyE.*Paresis of soft palate

620. What is early complications of diphtheria of otopharynx:A.*Paresis of soft palateB.PneumoniaC.AsphyxiaD.CroupE.Poliomyelitis

Page 86: intranet.tdmu.edu.uaintranet.tdmu.edu.ua/data/kafedra/internal/infect_desease... · Web viewBest method to treat diarrhoea in child is: intra venous fluide *ORS antibiotics bowel

621. What complications more often develops during 4-5th week of diphtheria:A.EncephalitisB.Bulbar disorders, pancreatitis, hepatitisC.*Poliomyelitis, myocarditisD.NephrosonephritisE.Stenotic laryngotracheitis

622. What are the rules of hospitalization of patients with infectious mononucleosis?A.Patients are not hospitalizedB.In a chamber for the infections of respiratory tractsC.*In a separate chamberD.In a chamber for the infections of external coversE. In a chamber for intestinal infections

623. What additional inspections must be conducted to the patient with infectious mononucleosis?

A.*IFA on HIV-infection, bacteriology inspection on diphtheriaB.IFA on HIV-infection, bacteriology inspection on a rabbit-feverC.Bacteriology inspection on diphtheria and typhoidD.Reaction of Burne and Rihth-HeddlsonE.Reaction of Paul-Bunnel and punction of lymphatic knot

624. What from the following symptoms are not characteristic of infectious mononucleosis?

A.FeverB.*Defeat of kidneysC.LymphadenopathyD.TonsillitisE. Increasing of liver and spleen

625. For what disease characterize changes in a blood (presence of lymphomonocytes and atypical mononuclears)?

A.FluB.*Infectious mononucleosisC.MeaselsD.AIDSE.Diphtheria

626. What additional test should hold for the patient with infectious mononucleosis?A.Burne and Wright-Hadlson‘s reactionsB.ELISA-test, bacteriological test for tularemiaC.Bacteriological test for diphtheria and typhoid feverD.*ELISA-test, bacteriological test for diphtheriaE.Paul-Burne reaction and lymph node puncture

627. What the most possible complication occurs during infectious mononucleosis?A.MeningitisB.Autoimmune alopeciaC.EncephalitisD.*Splenic ruptureE.Obstruction of respiratory tract

628. The source of infection at infectious mononucleosis is:

Page 87: intranet.tdmu.edu.uaintranet.tdmu.edu.ua/data/kafedra/internal/infect_desease... · Web viewBest method to treat diarrhoea in child is: intra venous fluide *ORS antibiotics bowel

A.*Sick people and carriersB.Sick agricultural animalsC.RodentsD.MosquitoesE.Aerosol of saliva and epipharyngeal mucous of patients

629. What is seasonal character of infectious mononucleosis?A.Spring-summerB.Summer-autumnC.Autumn-winterD.*Winter-springE.Spring-autumn

630. After the disease which was accompanied by the fever and pharyngalgias, there were an odynophagia, dysarthria, weakness and violation of motions in hands and feet, hyporeflexia, violation of sensitiveness in extremities to the polyneurotic type. What disease does it follow to think about above all things?

A.Neuropathy of hypoglossusB.*Diphtherial polyneuropathyC.Neuropathy of glossopharyngeus nerveD.Trunk encephalitisE.Pseudobulbar syndrome

631. What is immediately investigation in suspicious of diphtheria:A.Strokes with tonsills, nose or other areas for the exposure of diphtherial stickB.IFAC.*Microscopy (painting by Neiser)D.HaemocultureE.RDHA with a diphtherial diagnosticum

632. Etiology agent of meningitis is:A.*Neisseria meningitidesB.Entamoeba histolyticaC.Vibrio choleraeD.Clostridium botulinumE.Campylobacter pylori

633. Wich of these symptoms are often present in patients with meningitis?A.Algor, high temperature, headacheB.Profuse watery diarrhea, vomiting, dehydratation, muscular crampsC.Abdominal pain, diarrhea, constipation, flatulenceD.Headache, dry cough, algorE.*Prodromal respiratory illness or sore throat, fever, headache, stiff neck, vomiting,

confusion, irritability634. What group of infectious diseases meningococcal infection belong to:

A.IntestinalB.BloodC.*RespiratoryD.TransmissiveE.External covers

635. What is the mechanism of transmission of meningococcal infection?

Page 88: intranet.tdmu.edu.uaintranet.tdmu.edu.ua/data/kafedra/internal/infect_desease... · Web viewBest method to treat diarrhoea in child is: intra venous fluide *ORS antibiotics bowel

A.Fecal-oralB.ContactC.TransmissiveD.*Air-dropE.Vertical

636. What is seasonal character of meningococcal infection?A.Summer-autumnB.Autumn-winterC.*Winter-springD.WinterE.Summer

637. What clinical form of meningococcal infection more often may happened?A.MeningococcemiaB.MeningitisC.MeningoencephalitisD.*NasopharengitisE.Pneumonia

638. What syndrome may appear in severe meningococcemia?A.Paul-BunnelB.Plaut-VincentC.Jarish-GersgeimerD.Gien-BarreE.*Waterhause-Friedrichsen

639. What laboratory methods should be taken to discharge meningitis?A.*Lumbar punctureB.Serologic detectionC.Urine examinationD.CoprogramaE.Biopsy of tissues

640. Source of meningitis is:A.AnimalsB.BirdsC.FishD.Pediculus humanusE.*People

641. How is it possible to specify the diagnosis of meningococcal meningitis.A.Meningitis is primaryB.Presence of a lot of cells in the CSFC.Presence of gram-negative diplococcus in CSFD.Meningococes from the throatE.*All the above

642. What are the rules at taking of smear material on the discovery of meningococal infection?

A.The taken away material at drawing out must not touch only mucus shell of cheeks and tongue

B.The taken away material at drawing out must not touch only teeth and tongue

Page 89: intranet.tdmu.edu.uaintranet.tdmu.edu.ua/data/kafedra/internal/infect_desease... · Web viewBest method to treat diarrhoea in child is: intra venous fluide *ORS antibiotics bowel

C.The taken away material at drawing out must not touch only teeth, mucus shell of cheeks

D.*The taken away material at drawing out must not touch|| teeth, mucus shell of cheeks and tongue

E.The taken away material|| at drawing out can touch|| teeth, mucus shell of cheeks and tongue

643. When does the laboratory give the results of bacteriological examination of smear from throat?

A.On 2th daysB.On 3th daysC.*On 4th daysD.On 5th daysE.On 6th days

644. What is taken for serum research for confirmation of meningococcal infection?A.*BloodB.MucusC.UrineD.CSFE.Saliva

645. What antibiotics preparations of choice of etiotropic therapy at a meningococcal infection.

A.*Benzylpenicillin and it derivatives B.GentamycinC.CefazolinD.SulfolamideE.Ciprofloxacin

646. In what dose should| benzyl penicillin be administered at meningococcal meningitis?

A.From a calculation 100-300 thousands unit on 1 kg of mass of body on daysB.*From a calculation 200-500 thousands unit on 1 kg of mass of body on daysC.From a calculation 500-700 thousands unit on 1 kg of mass of body on daysD.From a calculation 700-900 thousands unit on 1 kg of mass of body on daysE.Regardless of mass of body

647. In what daily interval should the dose of benzylpenicillin at meningococcal meningitis administered.

A.2 hrsB.*4 hrsC.6 hrsD.5 hrsE.8 hrs

648. Which preparation has a bacteriostatic action, and is more expedient to begin etiotropic therapy in the case of infectious toxic shock.

A.From benzylpenicillin and its derivativesB.From ciprofloxacinC.From gentamycinD.From ciprofloxacin

Page 90: intranet.tdmu.edu.uaintranet.tdmu.edu.ua/data/kafedra/internal/infect_desease... · Web viewBest method to treat diarrhoea in child is: intra venous fluide *ORS antibiotics bowel

E.*From laevomycitin succinate649. For the treatment of acidosis at meningococcal meningitis is better to use.

A.10-20 % glucose solutionB.10 % chloride solutionC.*4 % sodium bicarbonate solutionD.AlbumenE.Concentrated dry plasma

650. Meningococсemia and DIC-syndrome require above all things.A.Administration of diureticsB.Administration of analgesicC.*Administration of heparinD.Administration of vitaminsE.Administration of antihistaminic preparations

651. What is used as specific prophylaxis in the period of epidemic spreading of meningococcal infection.

A.Immune globulinB.SerumC.*VaccineD.AnatoxinE.Nothing

652. A patient with meningococcal meningitis gets penicillin during 7 days. Last 4 days temperature of body is normal. Meningeal signs are absent. When is it possible to abolish an antibiotic?

A.*At cytosis in liquor 100 and less, lymphocytes prevailsB.At absence of leukocytosis and stab-nucleus shift in a bloodC.At cytosis in liquor 100 and more less, neutrophils prevailsD.At cytosis in liquor 150, lymphocytes prevailsE.At once

653. What measures are conducted in the place of meningococcal infection?A.Supervision during 2 weeksB.PhagoprophylaxisC.ImmunizationD.*Bacteriological inspection of contactE.Chemoprophylaxis

654. Violations of electrolyte balance show up at leptospirosis:A.*Metabolic acidosisB.By a metabolic alkalosisC.Respirator acidosisD.By a respiratory alkalosisE.All above enumerated

655. The decline of arteriotony at a leptospirosis is not caused:A.By expansion of vessels under the action of toxinB.HypovolemiaC.By myocarditisD.*HypercalcgestyE.By adrenal insufficiency

Page 91: intranet.tdmu.edu.uaintranet.tdmu.edu.ua/data/kafedra/internal/infect_desease... · Web viewBest method to treat diarrhoea in child is: intra venous fluide *ORS antibiotics bowel

656. All are the clinical signs of measles except:A.Acute beginning of high feverB.*Icterus C.Maculo-papular rashD.Sequential appearance of rashE.Scaling

657. For how long a patient with complicated form of measles should be isolated:A.For 4 days from the beginning of rashB.For 7 days from the beginning of rashC.*For 10 days from the beginning of rashD.For 17 days from the beginning E.For 20 days from the beginning of illness

658. Term of contagious period of patient diagnosed with uncomplicated form of measles

A.Until clinical recoveryB.After rash starts disappearingC.Before appearance of rashD.*4 days from the beginning of rashE.10 days from the beginning of illness

659. What is the duration of quarantine in child's establishment in case of rubella?A.5 days after the isolation of the last patientB.11 daysC.21 dayD.10 daysE.No need for quarantineF.*5 days after isolation of the last child

660. What is duration of contagious period for a patient with epidemic parotitis?A.21 daysB.First week of illnessC.First 10 days from the beginning of diseaseD.Whole period of clinical symptomsE.*First 9 days of disease.

661. What measures should be taken in regards to persons, who were in contact with a patient diagnosed with epidemic parotitis?

A.Observation after contact people during a maximal length of incubation periodB.Quorantin in child's establishmentC.Isolation of people who were in contact with ill from 11th to the 21t day of illnessD.Isolation of children up to 10 years old, who were not ill with epidemic parotitis,

for 21 day from a moment of contactE.*All above enumerated

662. What is the duration of contagious period for a patient diagnosed with scarlet fever?

A.10 days from the beginning of illnessB.Until patient is discharged from the hospitalC.Until rash is presentD.*Till the 22d day from the beginning of illness

Page 92: intranet.tdmu.edu.uaintranet.tdmu.edu.ua/data/kafedra/internal/infect_desease... · Web viewBest method to treat diarrhoea in child is: intra venous fluide *ORS antibiotics bowel

E.Not contagious663. What is duration period of supervision after ill with scarlet fever?

A.*7 days from time of contactB.21 day C.Till patient’s rash is presentD.Till patient is discharged from permanent establishmentE.Not conducted

664. Methods of specific prophylaxis of scarlet fever:A.Isolation of illB.VaccinationC.Use of antibioticsD.DisinfectionE.*Does not exist

665. What are the anti epidemic measures in regards to people who were in contact with chicken-pox patient:

A.*Separation and limit of contacts with othersB.VaccinationC.Use of antibioticsD.DisinfectionE.Does not exist

666. Measures of urgent prophylaxis for unvaccinated children who have never been ill with measles in case of exposure to an ill with measles

A.Separation from the sourceB.*VaccinationC.Administration of antibioticsD.Disinfection E.Does not exist

667. Measures of urgent prophylaxis of measles for contacts which have never been ill, but were vaccinated against measles

A.Separation from the sourceB.VaccinationC.Use of antibiotics D.Use of immunoglobulinE.*No need to conduct

668. Measures of urgent prophylaxis of measles for people who had been ill with measles, but never have been vaccinated

A.Separation from the illB.VaccinationC.Use of immunoglobulinD.*Use of antibiotics E.No need to conduct

669. Possible side effects at application of antibiotics are all, except.A.Stomach-acheB.Nausea, vomitC.DiarrheaD.*Fever

Page 93: intranet.tdmu.edu.uaintranet.tdmu.edu.ua/data/kafedra/internal/infect_desease... · Web viewBest method to treat diarrhoea in child is: intra venous fluide *ORS antibiotics bowel

E.Skin rash670. Contra-indication for the application of antibiotics are all, except.

A.An increased sensitiveness to preparationB.Severy disorders of liverC.Severy disorders of kidneysD.Period of pregnancy and lactationE.*Prolonged fever

671. Principles of etiotropic therapy of sepsis.A.Administration of antibiotics quick as possibleB.Administration of antibiotics in maximal therapeutic dosesC.In accordance to credible microbiological diagnosisD.An account of possible of therapeutic concentration of antibiotic in field of

infectionE.*All the above

672. Immune modulator therapy of sepsis are all, except.A.RonkoleykinB.InterferonC.Inductors of interferonD.*VaccineE.Normal human immune globuline

673. Basic principles of antibiotics therapy.A.A selection of antibiotic after the sensitiveness of the selected exciterB.selection of antibiotic according to the diagnosis of certain infectious disease

clinicallyC.Choose the most active preparationD.Choose the least toxic preparationE.*All the above

674. Basic principles of antibiotics therapy of sepsis.A.A selection of preparation from data of bacteriostaticB.Determination of dose, method and multiple of introduction the preparationC.Timeliness and definite duration of introduction input of antibiotic D.D Combining antibiotics between itself for enhancement of antibacterial effectE.*E. All the above

675. Agglutinines at a leptospirosis arrive at a maximal titre:A.On the third day of illness B.*On the third week of illness and laterC.On the fourth week of illness D.On the second month of illness E.To the second week of illness

676. At a leptospirosis the exposure of antibodies is considered reliable in a titre:A.1:70 and anymoreB.1:80 and anymoreC.1:60 and anymoreD.*1:100 and anymore E.1:40 and anymore

677. At treatment of patients with leptospirosis antibiotics consider most effective:

Page 94: intranet.tdmu.edu.uaintranet.tdmu.edu.ua/data/kafedra/internal/infect_desease... · Web viewBest method to treat diarrhoea in child is: intra venous fluide *ORS antibiotics bowel

A.*PenicillinB.MacrolidsC.CefalosporinsD.FtorhinolonsE.Sul'fanilamids

678. Etiotropic therapy of leptospirosis includes:A.OxyhinolonsB.SulfanilamidsC.NitrofuransD.AntitocidnsE.*Antibiotics

679. What from antibiotics more expedient to use for treatment of leptospirosis?A.MacrolidsB.TetracyclinsC.AminoglicosidsD.*PenicillinsE.Metrogil

680. A patient has a severy icteric form of leptospirosis. What from antibiotics is better to appoint as etiotropic therapy?

A.YunidoksB.TetracyclinumC.AzitromicinD.RovamicinE.*Penicillin

681. Treatment of leptospirosis:A.Desintoxication, dehydratation, antibiotics, glukokorticosteroidsB.*Antileptospirosis immunoprotein, antibiotics, nosotropic facilitiesC.Antibiotics, rehadratation, sorbtion preparation, vitaminsD.Diet № 7, antibiotics, desintoxication facilitiesE.Antibiotics, diuretic, interferons

682. Daily allowance doses of penicillin at treatment of leptospirosis:A.2-3 million unitsB.*3-12 million unitsC.20 million unitsD.40 million unitsE.Over 40 million units

683. With the purpose of immunotherapy it is better to apply at treatment of leptospirosis:

A.ImmunodepressantsB.AntihistaminicsC.Horse wheyD.*Antileptospirosis human immunoproteinE.Antileptospirosis neat immunoprotein

684. What from antibiotics are more effective in treatment of icteric form of leptospirosis:

A.*Penicillins

Page 95: intranet.tdmu.edu.uaintranet.tdmu.edu.ua/data/kafedra/internal/infect_desease... · Web viewBest method to treat diarrhoea in child is: intra venous fluide *ORS antibiotics bowel

B.AminoglicozidsC.TetracyclinsD.MacrolidsE.Metrogil

685. For the prophylaxis of leptospirosis use:A.*Active vaccineB.AnavaccineC.ToxoidD.AllE.Antibiotics

686. Vaccinations against leptosprosis perform for:A.AllB.Only villagersC.Only to the habitants of endemic districtsD.*Only to the persons busy in the stock-raisingE. It is not conducted

687. Which serotypes of leptospirosis caused the disease more frequent:A.L. interogansB.L. grippotyphosaC.L. canicolaD.*L. icterohaemorrhagiaE.L. Pomona

688. All of these have an epidemic dangerous exept:A.Farm animalsB.Wide rodentsC.Domestic animalsD.FoxesE.*Human

689. How long lasts the leptospirosis incubation period:A.2 monthB.1-7 daysC.*7-14 daysD.14-21 daysE.2-3 days

690. The main of antibiotics which is used in treatment of leptospirosis:A.TetracyclinB.TetraoleanC.ErythromycinD.StreptomycinE.*Penicillin

691. When there can be such specific complication of typhoid fever, like to intestinal bleeding?

A.On the 1st week of illnessB.On the 2nd week of illnessC.*On the 3rd week of illnessD.On the 4th week of illness

Page 96: intranet.tdmu.edu.uaintranet.tdmu.edu.ua/data/kafedra/internal/infect_desease... · Web viewBest method to treat diarrhoea in child is: intra venous fluide *ORS antibiotics bowel

E.On the any week of illness692. When there can be such specific complication of typhoid fever, like to

perforation of bowel?A.On the 1st week of illnessB.On the 2nd week of illnessC.*On the 3rd week of illnessD.On the 4th week of illnessE.On the any week of illness

693. Who is the source of typhoid fever?A.*Sick peopleB.Sick agricultural animalsC.Sick rodentsD.SoilE.Defecating of patients

694. Salmonella typhi contains:A.Only O-antigen and Н-antigenB.Only O-antigen and Vi-antigenC.Only H-antigen and Vi-antigenD.*O-antigen, H-antigen and Vi-antigenE.O-antigen, H-antigen, Vi- antigen and HBsAg

695. When is it possible to abolish etiotropic preparations in a patient with typhoid fever?

A.At once after normalization of temperature of bodyB.After normalization of sizes of liver and spleenC.After disappearance of roseollasD.In 10 days after disappearance of roseollasE.*After the 10th day of normal temperature of body

696. By which method is it possible to find out bacterial carrier in case of typhoid fever?

A.CoprocultureB.Reaction of agglutination of VidallC.Indirect hemaglutination test with О-аntigenD.Indirect hemaglutination test with a Н-antigenE.*Indirect hemaglutination test with a Vi-antigen

697. Typhoid bacilli are usually cultured from:A.*Blood, stool, urineB.Blood, urine, sputumC.Stool, liquor, urineD.Blood, stool, sputumE.Stool, liquor, sputum

698. A suspected case of typhoid fever of 1st week is admitted in the hospital. What examination (laboratory diagnosis) do you suggest for this patient?

A.CoprocultureB.Reaction of agglutination of VidallC.Indirect hemaglutination test with О-, Н-, Vi-аntigensD.Urinoculture

Page 97: intranet.tdmu.edu.uaintranet.tdmu.edu.ua/data/kafedra/internal/infect_desease... · Web viewBest method to treat diarrhoea in child is: intra venous fluide *ORS antibiotics bowel

E.*Hemoculture699. A suspected case of typhoid fever of 3nd week is admitted in the hospital. What

examination (laboratory diagnosis) do you suggest for this patient?A.CoprocultureB.Reaction of agglutination of VidallC.UrinocultureD.HemocultureE.*All about it

700. What from the transferred signs is not characteristic for a typhoid rash?A.*Papular, disappears together with normalization of temperature of bodyB.Appears on a 7-10th day, rosella-typeC.Located mainly on a abdomen and lateral surfaces of trunk, observed at the half of

patientsD.The amount of elements is limited, pours in additionE. rosella-type, sometimes saved longer than fever

701. What ever symptom is not characteristic for typhoid on the second week of illness?

A.ConstipationB.HeadacheC.FeverD.Relative bradycardiaE.*Cramps

702. What changes in general analysis of blood are characteristic for typhoid?A.*Leykopenia, aneosiniphilia, lymph-, monocytosis, enhanceable RSEB.Leykocytosis, hypereosinophilia, thrombocytosis, enhanceable RSEC.Hypochromatic anaemia, leycocytosis, appearance of young forms, RSE is not

changedD.Hyperchromatic anaemia, leycocytosis, appearance of young forms, enhanceable

RSEE.Leykopenia, lymphopenia, thrombocytosis, enhanceable RSE

703. What does the diagnostic titre of reaction of Vі-haemaglutination testify to?A.*About typhoid bacterial-carrierB.About the period of height of the epidemic typhusC.About meningococcaemiaD.About a malariaE.About the latent period of brucellosis

704. What term of looking after the hearth of typhoid?A.14 daysB.*21 daysC.7 daysD.30 daysE.Does not look after

705. Reconvalescente of typhoid fever may go out from clinic after:A.Non-permanent negative bacteriologic examination of defecatingB.*21th day of normal temperature of body and 3-multiple negative bacteriologic

examination of excrement and urine

Page 98: intranet.tdmu.edu.uaintranet.tdmu.edu.ua/data/kafedra/internal/infect_desease... · Web viewBest method to treat diarrhoea in child is: intra venous fluide *ORS antibiotics bowel

C.of 14th day of normal temperature of body and 2-multiple negative bacteriologic examination of excrement and urine

D.Clinical convalescence and normalization of rectal manoscopic pictureE.Normalisations of rectal manoscopic picture and in default of title of antibodies in

RNGA706. In epydfocus of typhoid fever doing, except for:

A.Daily thermometeryB.Coprologic cultureC.Reaction of VidalD.*HaemocultureE.Urine culture

707. For establishment of transmitter of Salmonella typhi utilize:A.Test of CuverkalovB.RA with O- and Н-antigenC.PCRD.Bacteriologic examination and reaction of VidalE.*Bacteriologic examination and RNGA with a Vi-antigen

708. What from the adopted ways of transmission is characteristic for typhoid?A.*AlimentaryB.ContactC.TransmissionD.Air-dropE.Vertical

709. What environments do typhoid sticks grow on well?A.Chicken embryosB.water-whey nourishing environmentC.*Bilious clear soupD.Meat-peptone gelose + cistinE.To the Bismute-sulfate gelose

710. Name of the basic factors of pathogen of typhoid stick?A.*Vi-antigen and endotoxinB.ExotoxinC.Vi-antigenD.Enzymes of pathogenicityE.Endotoxin

711. Duration of latent period at typhoid?A.3-7 daysB.*9-21 dayC.From a few hovers to 2-3 daysD.From 12 to 100 daysE.From a few hovers to 17 days

712. The part of reproduction of typhoid bacterium in the organism of man is:A.StomachB.*Lymphatic formations of colonC.BloodD.Bilious ways

Page 99: intranet.tdmu.edu.uaintranet.tdmu.edu.ua/data/kafedra/internal/infect_desease... · Web viewBest method to treat diarrhoea in child is: intra venous fluide *ORS antibiotics bowel

E.Mucous membrane of colon713. What from the adopted phases of pathogenesis is not characteristic for typhoid?

A.*Swelling, edema of mucous membrane of overhead respiratory tractsB.Stage of penetrationC.Stage of lymphodefence reactionsD.Stage of bacteriaemiaE.Stage of intoxication

714. What from the indicated pathology anatomic phases is not characteristic for typhoid?

A.*Catarrhal inflammation of amygdalesB.Cerebral-type of swellingC.NecrosisD.UlcersE.Clean ulcers

715. What is entrance gates of typhoid fever agent?A.Mucous membrane of amygdalesB.Mucous membrane of nasopharynxC.Epithelial cells of skinD.Mucous membrane of colonE.*Mucous membrane of digestive tract

716. Who is the source of epidemic typhus?A.Patients with epidemic typhusB.Patients with disease Brill-ZinsserC.*Patients with epidemic typhus and disease Brill-ZinsserD.Patients with Brill-Zinsser disease and Sachs diseaseE.Patients with epidemic typhus and abdominal typhoid

717. During what time the lice can transfer the epidemic typhus?A.Up to 10 daysB.Up to 15 daysC.Up to 20 daysD.*Up to 30 daysE.Up to 40 days

718. What is Brill's disease?A.Vertiacal borneB.*Remote relapse typhusC.Early relapse typhusD.Re-infection rickettsiaE.Self nozological unit

719. Often, in patient with epidemic typhus is tongue‘s tremor when protrusion that sticked on the lower teeth. What term did it call?

A.Symptom of HellerB.ConjunctivitisC.*Govorov-Godele symptomD.Zorohovich-Chiari symptomE.Enantema Rosenberg‘s

720. What is not typical for epidemic typhus exanthema?

Page 100: intranet.tdmu.edu.uaintranet.tdmu.edu.ua/data/kafedra/internal/infect_desease... · Web viewBest method to treat diarrhoea in child is: intra venous fluide *ORS antibiotics bowel

A.*Arise on 7-10-day of illnessB.Has rosy-petehia natureC.Localized mainly on the lateral surface of the torso and limbs flexion surfacesD.It can grab his hands and feet, but never on the faceE.Abundant

721. What is not typical for epidemic typhus exanthema?A.Disappear with the drop in temperatureB.Never appeared another rashC.Roseola saved up to 6 days, petehii – 12D.Leave a little pigmentation and poor peelingE.*Single elements

722. What is not typical of blood in the severe epidemic typhus?A.Neutrofil leukocytosisB.Academy of Sciences or hypleozinofilC.LimphopeniyaD.*AnemiaE.The increase in ESR

723. What is not typical for the analysis of urine in the severe epidemic typhus?A.ProteinuriaB.Single-cylinder hyalineC.*Multiple granular cylindersD.A small number of erythrocytesE.A small number of leukocytes

724. When is serological diagnosis possible in patients with epidemic typhus?A.From the 1st day of illnessB.On the 2nd day illnessC.From the 3-4th day illnessD.From 4-5th day of illnessE.*From the 5 to 7th day of illness

725. What diagnostic titer response agglutination test with typhus rickettsia?A.1:40 and aboveB.1:80 and aboveC.*1:160 or higherD.1:320 or higherE.1:640 or higher

726. In with titre will be positive reaction of agglutination in patient with epidemic typhus in droplets test when Mosing?

A.*1:40 and aboveB.1:80 and aboveC.1:160 or higherD.1:320 or higherE.1:640 or higher

727. For contact persons in the centre of the epidemic typhus establish surveillance over:

A.21 daysB.25 days

Page 101: intranet.tdmu.edu.uaintranet.tdmu.edu.ua/data/kafedra/internal/infect_desease... · Web viewBest method to treat diarrhoea in child is: intra venous fluide *ORS antibiotics bowel

C.*51 daysD.72 daysE.3 months

728. During the contact person with the Brill‘s disease establish surveillance over:A.21 daysB.*25 daysC.51 daysD.72 daysE.3 months

729. In the case of head lice carry out sanitation: hair cut, followed by incineration, processing hair. What kind of insecticide products for this use?

A.0,5 % solution of water emulsions karbofosaB.0,5 % metilatsetofosC.10 % solution of liquid neutral metilatsetofosa soapD.0,5 % water emulsion dikreziluE.*3 % soap RHTSG

730. When patients after epidemic typhus may go out from clinic?A.*After clinical recovery, but not before the 12-day normal temperatureB.After a full clinical recoveryC.After clinical recovery, but not before the 12-day period following the lifting of

antibioticsD.At the 12-day normal body temperatureE.After clinical recovery, but not earlier than the 9-day normal body temperature

731. Whit antibiotics are less effective from the transferred at the epidemic fever?A.TetracyclinB.MetacyclinC.*LevomicetynD.VibramycinumE.Doxyciclin

732. Who is the source of epidemic typhus?A.Patients with epidemic typhusB.Patients with disease Brill-ZinsserC.*Patients with epidemic typhus and disease Brill-ZinsserD.Patients with Brill-Zinsser disease and Sachs diseaseE.Patients with epidemic typhus and abdominal typhoid

733. When sick people gets epidemic typhus infection, which period affects more?A.Over the past 2 days, the incubation period and 2-3 days after lowering temperatureB.All hectic period and 2-3 days after lowering temperatureC.2-3 days after lowering temperatureD.*Over the past 2 days, the incubation period, all febrile period and 2-3 days after

lowering temperatureE.Over the past 2 days, the incubation period and the hectic period

734. On which period the maximal growth of infection occurs during epidemic typhus disease?

A.At the incubation periodB.*At the 1th week of illness

Page 102: intranet.tdmu.edu.uaintranet.tdmu.edu.ua/data/kafedra/internal/infect_desease... · Web viewBest method to treat diarrhoea in child is: intra venous fluide *ORS antibiotics bowel

C.At the 2nd week of illnessD.At the 3rd week of illnessE.At the time of recovery

735. During what time the lice can transfer the epidemic typhus?A.Up to 10 daysB.Up to 15 daysC.Up to 20 daysD.*Up to 30 daysE.Up to 40 days

736. What is Brill's disease?A.Vertiacal borneB.*Remote relapse typhusC.Early relapse typhusD.Re-infection rickettsiaE.Self nozological unit

737. Often, in patient with epidemic typhus arise transition petehies in the conjunctivA. What term did it call?

A.Symptom of HellerB.ConjunctivitisC.Symptom of Govorova-GodeleD.*Symptom of Zorohovich-Chiari-AvtsynaE.Enantema Rosenberg‘s

738. Often, in patient with epidemic typhus arises petehies on mucosal soft palate. What term did it call?

A.Symptom of HellerB.ConjunctivitisC.Symptom of Govorova-GodeleD.Symptom of Zorohovich-KiariE.*Enantema Rosenberg‘s

739. Often, in patient with epidemic typhus is tongue‘s tremor when protrusion that sticked on the lower teeth. What term did it call?

A.Symptom of HellerB.ConjunctivitisC.*Symptom of Govorova-GodeleD.Symptom of Zorohovich-ChiariE.Enantema Rosenberg‘s

740. What is not typical for epidemic typhus exanthema?A.*Arise on 7-10-day of illnessB.Has rosy-petehia natureC.Localized mainly on the lateral surface of the torso and limbs flexion surfacesD.It can grab his hands and feet, but never on the faceE.Abundant

741. What is not typical for epidemic typhus exanthema?A.Disappear with the drop in temperatureB.Never appeared another rashC.Roseola saved up to 6 days, petehii – 12

Page 103: intranet.tdmu.edu.uaintranet.tdmu.edu.ua/data/kafedra/internal/infect_desease... · Web viewBest method to treat diarrhoea in child is: intra venous fluide *ORS antibiotics bowel

D.Leave a little pigmentation and poor peelingE.* Arise on 7-10-day of illness

742. What is not typical of blood in the severe epidemic typhus?A.Neutrofil leukocytosisB.Academy of Sciences or hypleozinofilC.LimphopeniyaD.*AnemiaE.The increase in ESR

743. What is not typical for the analysis of urine in the severe epidemic typhus?A.ProteinuriaB.Single-cylinder hyalineC.*Multiple granular cylindersD.A small number of erythrocytesE.A small number of leukocytes

744. When is serological diagnosis possible in patients with epidemic typhus?A.From the 1st day of illnessB.On the 2nd day illnessC.From the 3-4th day illnessD.From 4-5th day of illnessE.*From the 5 to 7th day of illness

745. What diagnostic titer response agglutination test with typhus rickettsia?A.1:40 and aboveB.1:80 and aboveC.*1:160 or higherD.1:320 or higherE.1:640 or higher

746. In with titre will be positive reaction of agglutination in patient with epidemic typhus in droplets test when Mosing?

A.*1:40 and aboveB.1:80 and aboveC.1:160 or higherD.1:320 or higherE.1:640 or higher

747. In the family of the patient with epidemic typhus, were lice in the children. With the help of any of these events could prevent the subsequent spread of the disease?

A.*Monitoring and complete sanitation of contact in the centreB.The use of chemoprophylaxisC.The use of antibioticsD.Isolation contactE.Check-up

748. When you can stopped etiotropic medications treatment of the patient with epidemic typhus?

A.Immediately after the normalization of body temperatureB.After the normalization of the liver and spleenC.*After a 2-day normal body temperatureD.After the disappearance of roseola

Page 104: intranet.tdmu.edu.uaintranet.tdmu.edu.ua/data/kafedra/internal/infect_desease... · Web viewBest method to treat diarrhoea in child is: intra venous fluide *ORS antibiotics bowel

E.Within 10 days after the disappearance of roseola749. For contact persons in the centre of the epidemic typhus establish surveillance

over:A.21 daysB.25 daysC.*51 daysD.72 daysE.3 months

750. During the contact person with the Brill‘s disease establish surveillance over:A.21 daysB.*25 daysC.51 daysD.72 daysE.3 months

751. In the case of head lice carry out sanitation: hair cut, followed by incineration, processing hair. What kind of insecticide products for this use?

A.0,5 % solution of water emulsions karbofosaB.0,5 % metilatsetofosC.10 % solution of liquid neutral metilatsetofosa soapD.0,5 % water emulsion dikreziluE.*3 % soap RHTSG

752. When patients after epidemic typhus may go out from clinic?A.*After clinical recovery, but not before the 12-day normal temperatureB.After a full clinical recoveryC.After clinical recovery, but not before the 12-day period following the lifting of

antibioticsD.At the 12-day normal body temperatureE.After clinical recovery, but not earlier than the 9-day normal body temperature

753. Phage symptom in case of yellow fever is:A.Pain in right iliac areaB.Enanthema on a soft palateC.*Replacement of tachicardia on expressed bradicardiaD.Hemorrhages in a conjunctivaE.Yellow hands

754. Hemograme in the second period of yellow fever:A.Leukocytosis B.Normal global analysis of bloodC.*Leukopenia, neutropeniaD.Leukopenia, neutrophilosisE.Leukocytosis, lymphomonocytosis

755. Whatever complication meets at the yellow fever:A.*Liver insufficiencyB.Kidney insufficiencyC.Infectious-toxic shockD.MyocarditisE.Edema of lungs

Page 105: intranet.tdmu.edu.uaintranet.tdmu.edu.ua/data/kafedra/internal/infect_desease... · Web viewBest method to treat diarrhoea in child is: intra venous fluide *ORS antibiotics bowel

756. Unlike leptospirosis in case of yellow fever is absent:A.Hemorrhagic syndromeB.Kidney insufficiencyC.Іntoxication syndromeD.Міalglic syndromeE.*Hepatic insufficiency

757. For confirmation of yellow fever diagnosis use:A.Bacteriological analysis of bloodB.Bacteriological examination of urine C.*Virological hemanalysisD.Biochemical blood testE.Global analysis of blood

758. In the initial period of hemorrhagic fever with a kidney syndrome a characteristic sign is:

A.High temperaturesB.Pains in gastrocnemius muscles and positive Pasternatsky symptomC.*Pains in joints and positive Pasternatsky symptom D.Hemorragic syndromeE.Dyspepsia phenomena

759. An initial period at the hemorrhagic fever with a kidneys syndrome lasts:A.Few hoursB.DayC.*To three daysD.WeekE.Two weeks

760. Whether there is violation of diuresis at patients with hemorrhagic fever with a kidneys syndrome:

A.In an initial periodB.It is notC.It is in all periods of diseaseD.*It is in climax periodE. It is in the period of recovering

761. General view of patient with the hemorrhagic fever with a kidneys syndrome:A.Skinning coversB.*Pallor of nasolabial triangle, hyperemia of neck and overhead half of trunkC.Hyperemia of person, scleritis, conjunctivitisD.Grayish color of personE. Icteric color of skin

762. In the biochemical blood test at patients with the hemorrhagic fever with a kidneys syndrome not characteristically:

A.High level of ureaB.Decline of potassium levelC.*BilirubinemiaD.Increasing of kreatinineE. Increasing of nitrogen

763. For confirmation of diagnosis of hemorragic fever with a kidney syndrome use:

Page 106: intranet.tdmu.edu.uaintranet.tdmu.edu.ua/data/kafedra/internal/infect_desease... · Web viewBest method to treat diarrhoea in child is: intra venous fluide *ORS antibiotics bowel

A.Bacteriological methodB.Virological methodC.*Reaction of immunofluorescenceD.Reaction of braking of hemagglutinationE.Research of blood drop under a microscope

764. For treatment of patients with the hemorrhagic fever with a kidney syndrome does not use:

A.GlucocorticoidsB.Anabolic steroidC.Disintoxication facilitiesD.*Dihydration facilitiesE.Antihistaminics

765. For the initial period of the Congo hemorrhagic fever not characteristically:A.FeverB.Pains in joints and musclesC.Severe pain of headD.*OliguriaE.Dizziness

766. At an objective review for the Congo hemorrhagic fever characteristically:A.*Mucosal hyperemia of personB.Pallor of personC.Puffiness of personD.Ochrodermia of personE.Exanthema on face

767. The most characteristic symptom in the climax period of the Congo hemorrhagic fever is:

A.*Hemorrhagic syndromeB.Hepatic insufficiencyC.Dyspepsia phenomenaD.Sharp kidney insufficiencyE.Мeningeal syndrome

768. In the general blood analysis of in case of Congo hemorrhagic fever is not typically:

A.LeukocytosisB.*LeukopeniaC.NeutropeniaD.ThrombocytopeniaE. Increasing of ESR

769. What rashes in case of haemorrhagic fevers with kidneys syndrome?A.RoseolaB.Maculo-papularC.PunctuateD.*PetechialE.Rashes is not characteristic

770. What rashes present in case of Congo hemorrhagic fever?A.Roseola

Page 107: intranet.tdmu.edu.uaintranet.tdmu.edu.ua/data/kafedra/internal/infect_desease... · Web viewBest method to treat diarrhoea in child is: intra venous fluide *ORS antibiotics bowel

B.Maculo-papularC.PunctulateD.*PetechialE.Rashes not is characteristic

771. What rashes present in case of Crimea hemorrhagic fever?A.RoseolaB.Maculo-papularC.PunctulateD.*PetechialE.Rashes not is characteristic

772. How long the rash is present in case of haemorrhagic fever with kidneys syndrome?

A.*During all feverish periodB.Before the reconvalescenseC.Before development of clinical features of kidneys insufficiencyD.During whole diseaseE.Appears yet in a latent period and disappears in the period of early reconvalescense

773. A kidney syndrome at haemorrhagic fever with kidneys syndrome shows up usually:

A.Only laboratory changesB.Only on BRIDLESC.*By pain in lumbar area, positive Pasternatsky symptom, development of oliguriaD.By fever, polyuria, dyspepsiaE.By paradoxical ischuria

774. What changes in biochemical blood test inherent for haemorrhagic fever with kidneys syndrome?

A.Increase level of urea and bilirubinB.The level of urea and kreatinine fallsC.The level of kreatinine grows and urea fallsD.The level of urea grows and kreatinine fallsE.*The level of urea and kreatinine grows

775. What changes in haemogram inherent for haemorrhagic fever with kidneys syndrome?

A.Normochromic anaemia, leucocytosis with atypical mononucleosis, thrombocytopenia enhanceable ESR

B.erythrocytosis, lymphocytosis,ESR is enhanceableC.Normochromic anaemia, leucopenia with neutrophylosis, thrombocytopenia

enhanceable ESRD.*Hypochromic anaemia, leucocytosis with neutrophylosis, thrombocytopenia

enhanceable ESRE.Hyperchromic anaemia, leucocytosis with neutrophylosis, thrombocytopenia

mionectic ESR776. The period of polyuria at haemorrhagic fever with kidneys syndrome is a sign of:

A.*RecoveringB.Chronic processC.Unfavorable flow of illness

Page 108: intranet.tdmu.edu.uaintranet.tdmu.edu.ua/data/kafedra/internal/infect_desease... · Web viewBest method to treat diarrhoea in child is: intra venous fluide *ORS antibiotics bowel

D.Development of complicationsE.Complete convalescence

777. In most patients with Congo hemorrhagic fever temperature curve is:A.Wunderlich typeB.Botkin typeC.UndulatingD.IntermittentE.*Two-humped

778. With appearance of hemorrhagic syndrome at Congo fever temperature of body always:

A.NormalizeB.Grows criticallyC.*Goes downD.Does not changeE.Grows gradually

779. What changes in haemogram inherent Congo hemorrhagic fever?A.Normochomic anaemia, leucocytosis mononuclearB.Erythrocytosis, lymphocytosisC.*Hypochromic anemia, erythrofilosisD.Hypochromic anemia, neutrofilosisE.Hyperchromic anemia, neutrofilosis

780. What is typical for the Lassa hemorrhagic fever:A.Effect of cardiovascular systemB.Development of acute hepatic insufficiencyC.Hundred-per-cent lethalityD.*Defeat of breathing organsE.Development of paresis and paralysis

781. Confirm diagnosis of haemorrhagic fever with kidneys syndrome by a way of:A.Only virological methodsB.Only bacteriological methodsC.Bacteriological and serum methodsD.Proper epidemiological informationE.*Virologic and serum methods

782. Confirm the diagnosis of Lassa hemorrhagic fever by a way of:A.Only virological methodsB.Only bacteriological methodsC.Bacteriological and serum methodsD.Proper epidemiological informationE.*Virologic and serum methods

783. Confirm the diagnosis of Congo hemorrhagic fever by a way of:A.Only virological methodsB.Only bacteriological methodsC.Bacteriological and serum methodsD.Proper epidemiological informationE.*Virologic and serum methods

784. Confirm the diagnosis of Ebola fever by a way of:

Page 109: intranet.tdmu.edu.uaintranet.tdmu.edu.ua/data/kafedra/internal/infect_desease... · Web viewBest method to treat diarrhoea in child is: intra venous fluide *ORS antibiotics bowel

A.Growth of viruses on chicken embryonsB.Only bacteriological methodsC.Bacteriological and serum methodsD.Proper epidemiological informationE.*Selection of virus on the Vero culture

785. Confirm the diagnosis of Omsk fever by a way of:A.Growth of virus on chicken embryonsB.Only bacteriological methodsC.Bacteriological and serum methodsD.Proper epidemiological informationE.*Selection of virus on the Vero culture

786. Confirm the diagnosis of Marburg fever by a way of:A.Growth on chicken embryosB.Only bacteriological methodsC.Bacteriological and serum methodsD.Proper epidemiologys informationE.*Selection of virus on the Vero culture

787. What etiothropic means use at treatment of haemorrhagic fever with kidneys syndrome:

A.BenzylpenicillinB.DopamineC.*VirolexD.DexamethazoneE.Etamsylatum

788. What etiothropic means use at treatment of patients with Lassa fever:A.BenzylpenicillinB.DopamineC.*RibavirinD.DexamethazoleE.Etamsylatum

789. What etiothropic means use at treatment of patients with Omsk fever:A.BenzylpenicillinB.DopamineC.*RibavirinD.DexamethazoneE.Etamsylatum

790. What etiothropic means use at treatment of patients with Marburg fever:A.BenzypenicillinB.DopamineC.*RibavirinD.DexamethazoneE.Etamsylatum

791. What etiothropic means use at treatment of patients with Congo fever:A.BenzylpenicillinB.DopamineC.*Ribavirin

Page 110: intranet.tdmu.edu.uaintranet.tdmu.edu.ua/data/kafedra/internal/infect_desease... · Web viewBest method to treat diarrhoea in child is: intra venous fluide *ORS antibiotics bowel

D.DexamethazoneE.Etamsylatum

792. What etiothropic means use at treatment of patients with Ebola fever:A.BenzylpenicillinB.DopamineC.*VirolexD.DexamethazoneE.Etamsylatum

793. What etiothropic means use at treatment of patients with Crimea fever:A.BenzylpenicillinB.DopamineC.*RibavirinD.DexamethazoneE.Etamsylatum

794. Specific prevention of hemorrhagic fevers:A.The live vaccineB.Killed vaccineC.The specific immunoglobulinD.*Do not developedE.Polivalent vaccine

795. Who is the source of the causal agent in the Crimean-Congo haemorrhagic fever?A.Rodents, cattle, birdsB.Iksod and gamazov mitesC.*Rodents, cattle, birds, sick peopleD.The sick man, reconvalenc, bacteriocarriesE.Rodents, cattle, birds, sick people, bacteriocarries

796. The source of infection of Omsk‘s hemorrhagic fever are muskrat, water rats and other rodents. Who are the carriers?

A.Bee and fleaB.*Pliers and fleaC.MosquitoesD.FlyE.Pliers and mosquitoes

797. Specific prevention of Crimean-Congo haemorrhagic fever are:A.*Vaccine and human immunoglobulinB.SerumC.Serum and human immunoglobulinD.Do not developedE.Antibacterial drugs

798. Those who have been in contact with sick haemorrhagic fevers, as well as those who had bite by the ticks in endemic areas are introducing:

A.Specific vaccineB.The specific immunoglobulin in doses of 10-15 ml vaccineC.The specific immunoglobulin in doses of 10-15 mlD.*The specific immunoglobulin in doses 5-7,5 mlE.Nothing

Page 111: intranet.tdmu.edu.uaintranet.tdmu.edu.ua/data/kafedra/internal/infect_desease... · Web viewBest method to treat diarrhoea in child is: intra venous fluide *ORS antibiotics bowel

799. Hemograme in the second period of yellow fever:A.Leukocytosis B.Normal global analysis of bloodC.*Leukopenia, neutropeniaD.Leukopenia, neutrophilosisE.Leukocytosis, lymphomonocytosis

800. What ever complication meets at the yellow fever:A.*Liver insufficiencyB.Kidney insufficiencyC.Infectious-toxic shockD.MyocarditisE.Edema of lungs

801. In the initial period of hemorrhagic fever with a kidney syndrome a characteristic sign is:

A.High temperaturesB.Pains in gastrocnemius muscles and positive Pasternatsky symptomC.*Pains in the joints and positive Pasternatsky symptom D.Hemorragic syndromeE.Dyspepsia phenomena

802. For treatment of patients with the hemorrhagic fever with a kidney syndrome do not use:

A.GlucocorticoidsB.Anabolic steroidC.Disintoxication facilitiesD.*Dihydration facilitiesE.Antihistamins

803. For the initial period of the Congo hemorrhagic fever not characteristically:A.FeverB.Pains in joints and musclesC.Severe pain of headD.*OliguriaE.Dizziness

804. The most characteristic symptom in the climax period of the Congo hemorrhagic fever is:

A.*Hemorrhagic syndromeB.Hepatic insufficiencyC.Dyspepsia phenomenaD.Sharp kidney insufficiencyE.Мeningeal syndrome

805. In the global analysis of blood in case of Congo hemorrhagic fever not characteristically:

A.LeukocytosisB.*LeukopeniaC.NeutropeniaD.ThrombocytopeniaE. Increasing of ESR

Page 112: intranet.tdmu.edu.uaintranet.tdmu.edu.ua/data/kafedra/internal/infect_desease... · Web viewBest method to treat diarrhoea in child is: intra venous fluide *ORS antibiotics bowel

806. What rashes in case of haemorrhagic fevers with kidneys syndrome?A.RoseolaB.Maculo-papularC.PunctuateD.*PetechialE.Rashes is not characteristic

807. What does change in biochemical blood test in the patient with haemorrhagic fever with kidneys syndrome?

A.Increase level of urea and bilirubinB.The level of urea and kreatinine fallsC.The level of kreatinine grows and urea fallsD.The level of urea grows and kreatinine fallsE.*The level of urea and kreatinine grows

808. What does change in haemogram in the patient with haemorrhagic fever with kidneys syndrome?

A.Normochromic anaemia, leucocytosis with atypical mononucleosis, thrombocytopenia enhanceable ESR

B.erythrocytosis, lymphocytosis,ESR is enhanceableC.Normochromic anaemia, leucopenia with neutrophylosis, thrombocytopenia

enhanceable ESRD.*Hypochromic anaemia, leucocytosis with neutrophylosis, thrombocytopenia

enhanceable ESRE.Hyperchromic anaemia, leucocytosis with neutrophylosis, thrombocytopenia

mionectic ESR809. The temperature curve in most patients with Congo hemorrhagic fever is:

A.Wunderlich typeB.Botkin typeC.UndulatingD.IntermittentE.*Two-humped

810. What is typical for the Lassa hemorrhagic fever:A.Effect of cardiovascular systemB.Development of acute hepatic insufficiencyC.Hundred-per-cent lethalityD.*Defeat of breathing organsE.Development of paresis and paralysis

811. What etiothropic means use at treatment of patients with Lassa fever:A.BenzylpenicillinB.DopamineC.*RibavirinD.DexamethazoleE.Etamsylatum

812. What etiothropic means use at treatment of haemorrhagic fever with kidneys syndrome:

A.BenzylpenicillinB.Dopamine

Page 113: intranet.tdmu.edu.uaintranet.tdmu.edu.ua/data/kafedra/internal/infect_desease... · Web viewBest method to treat diarrhoea in child is: intra venous fluide *ORS antibiotics bowel

C.*VirolexD.DexamethazoneE.Etamsylatum

813. Specific prevention of hemorrhagic fevers:A.The live vaccineB.Killed vaccineC.The specific immunoglobulinD.*Do not developedE.Polivalent vaccine

814. Who is the source of the causal agent in the Crimean-Congo haemorrhagic fever?A.Rodents, cattle, birdsB.Iksod and gamazov mitesC.*Rodents, cattle, birds, sick peopleD.The sick man, reconvalenc, bacteriocarriesE.Rodents, cattle, birds, sick people, bacteriocarries

815. Phage symptom in case of yellow fever is:A.Pain in right iliac areaB.Enanthema on a soft palateC.*Replacement of tachicardia on expressed bradicardiaD.Hemorrhages in a conjunctivaE.Yellow hands

816. General view of patient with the hemorrhagic fever with a kidneys syndrome:A.Skinning coversB.*Pallor of nasolabial triangle, hyperemia of neck and overhead half of trunkC.Hyperemia of person, scleritis, conjunctivitisD.Grayish color of personE. Icteric color of skin

817. For confirmation of diagnosis of hemorrhagic fever with a kidney syndrome use:A.Bacteriological methodB.Virological methodC.*Reaction of immunofluorescenceD.Reaction of braking of hemagglutinationE.Research of blood drop under a microscope

818. For treatment of patients with the hemorrhagic fever with a kidney syndrome does not use:

A.CorticosteroidsB.Anabolic steroidsC.Disintoxication facilitiesD.*Dehydration facilitiesE.Antihistaminics

819. For the initial period of the Congo hemorrhagic fever not characteristic:A.FeverB.Pains in joints and musclesC.Severe pain of headD.*OliguriaE.Dizziness

Page 114: intranet.tdmu.edu.uaintranet.tdmu.edu.ua/data/kafedra/internal/infect_desease... · Web viewBest method to treat diarrhoea in child is: intra venous fluide *ORS antibiotics bowel

820. At an objective examination for the Congo hemorrhagic fever character:A.*Mucosal hyperemia of personB.Pallor of personC.Puffiness of personD.Ochrodermia of personE.Exanthema on face

821. The most characteristic symptom in the climax period of the Congo hemorrhagic fever is:

A.*Hemorrhagic syndromeB.Hepatic insufficiencyC.Dyspepsia phenomenaD.Sharp kidney insufficiencyE.Мeningeal syndrome

822. In the global analysis of blood in case of Congo hemorrhagic fever not characteristic:

A.LeukocytosisB.*LeukopeniaC.NeutropeniaD.ThrombocytopeniaE. Increasing of ESR

823. What rashes in case of haemorrhagic fevers with kidneys syndrome?A.RoseolaB.Maculo-papularC.PunctuateD.*PetechialE.Rashes is not characteristic

824. What rashes present in case of Congo hemorrhagic fever?A.RoseolaB.Maculo-papularC.PunctulateD.*PetechialE.Rashes not is characteristic

825. What rashes present in case of Crimea hemorrhagic fever?A.RoseolaB.Maculo-papularC.PunctulateD.*PetechialE.Rashes not is characteristic

826. How long the rash is present in case of hemorrhagic fever with kidneys syndrome?

A.*During all feverish periodB.Before the convalescenceC.Before development of clinical features of kidneys insufficiencyD.During whole diseaseE.Appears yet in a latent period and disappears in the period of early reconvalescense

Page 115: intranet.tdmu.edu.uaintranet.tdmu.edu.ua/data/kafedra/internal/infect_desease... · Web viewBest method to treat diarrhoea in child is: intra venous fluide *ORS antibiotics bowel

827. What changes in biochemical blood test inherent for hemorrhagic fever with kidneys syndrome?

A.Increase level of urea and bilirubinB.The level of urea and kreatinine fallsC.The level of kreatinine grows and urea fallsD.The level of urea grows and kreatinine fallsE.*The level of urea and kreatinine increase

828. What changes in blood analysis inherent for hemorrhagic fever with kidneys syndrome?

A.Normochromic anaemia, leucocytosis with atypical mononucleosis, thrombocytopenia increased ESR

B.erythrocytosis, lymphocytosis,ESR is increased C.Normochromic anaemia, leucopenia with neutrophylosis, thrombocytopenia

increased ESRD.*Hypochromic anaemia, leucocytosis with neutrophylosis, thrombocytopenia

increased ESRE.Hyperchromic anaemia, leucocytosis with neutrophylosis, thrombocytopenia

mionectic ESR829. The period of polyuria at haemorrhagic fever with kidneys syndrome is a sign of:

A.*RecoveringB.Chronic processC.Unfavorable flow of illnessD.Development of complicationsE.Complete convalescence

830. With appearance of hemorrhagic syndrome at Congo fever temperature of body always:

A.NormalB.Grows criticallyC.*Goes downD.Does not changeE.Grows gradually

831. What changes in blood analysis inherent at Congo hemorrhagic fever?A.Normochromic anaemia, leucocytosis mononuclearB.Erythrocytosis, lymphocytosisC.*Hypochromic anemia, erythrophilosisD.Hypochromic anemia, neutrophilosisE.Hyperchromic anemia, neutrophilosis

832. What is typical for the Lassa hemorrhagic fever:A.Effect of cardiovascular systemB.Development of acute hepatic insufficiencyC.Hundred-per cent lethalityD.*Defeat of breathing organsE.Development of paresis and paralysis

833. Confirm the diagnosis of hemorrhagic fever with kidneys syndrome by a way of:A.Only virological methodsB.Only bacteriological methods

Page 116: intranet.tdmu.edu.uaintranet.tdmu.edu.ua/data/kafedra/internal/infect_desease... · Web viewBest method to treat diarrhoea in child is: intra venous fluide *ORS antibiotics bowel

C.Bacteriological and serum methodsD.Proper epidemiological informationE.*Virologic and serum methods

834. Confirm the diagnosis of Lassa hemorrhagic fever by a way of:A.Only virological methodsB.Only bacteriological methodsC.Bacteriological and serum methodsD.Proper epidemiological informationE.*Virologic and serum methods

835. Confirm the diagnosis of Congo hemorrhagic fever by a way of:A.Only virological methodsB.Only bacteriological methodsC.Bacteriological and serum methodsD.Proper epidemiological informationE.*Virologic and serum methods

836. Confirm the diagnosis of Ebola fever by a way of:A.Growth of viruses on chicken embryosB.Only bacteriological methodsC.Bacteriological and serum methodsD.Proper epidemiological informationE.*Selection of virus on the Vero culture

837. Confirm the diagnosis of Omsk fever by a way of:A.Growth of virus on chicken embryonsB.Only bacteriological methodsC.Bacteriological and serum methodsD.Proper epidemiological informationE.*Selection of virus on the Vero culture

838. The measures of urgent prophylaxis of plague.A.Administration of human immunoglobulinB.Chlorochin (delagil) 0,25 g 2 times in weekC.*6-day’s prophylaxis with streptomycin or tetracyclineD.In first 5 days intake antibiotics of penicillin or tetracycline originE. Іnterferon

839. The rules of hospitalization of patients with plague:A.To separate wardB.To ward for respiratory infectionsC.*To ward cubicleD.Patient’s are not hospitalizedE.To ward for intestinal infections

840. Patient T., drives in a country unhappy on a plague. Conduct measures on a specific prophylaxis.

A.Human immunoglobulinB.Interferon C.BacteriophageD.*Dry living vaccineE.Live measles vaccine

Page 117: intranet.tdmu.edu.uaintranet.tdmu.edu.ua/data/kafedra/internal/infect_desease... · Web viewBest method to treat diarrhoea in child is: intra venous fluide *ORS antibiotics bowel

841. Preparations for urgent prophylaxis of plague:A.Injection of human immunoglobulinB.*Streptomycin or tetracycline C.Human immunoglobulinD.Dry living vaccine or tetracycline generations.E. Interferon

842. Y. pestis is transmitted more frequently by:A.*FleaB.WaterC.AirD.Food storageE.Tick

843. The duration of incubation period of plague is:A.3 to 8 days;B.2 to 12 days;C.2 to 10 days;D.1 to 8 days.E.*2 to 6 days;

844. What is the main feature of septicemic plague?A.*Massive bacteriemiaB.HeadacheC.Pain in the abdominalD.Throat acheE.Bleeding

845. What drug is first step of choice for the treatment of plague?A.AmoxicillinB.*StreptomycinC.PenicillinD.BiseptolE.5-NOK

846. What is the treatment of patients with a plague:A.Immediately after hospitalizationB.*Immediately after hospitalization, carrying out only material for researchC.After raising of final diagnosisD.After laboratory and instrumental diagnosticsE.All answers are faithful

847. How many pandemics of plague was in history of mankind? A.*ThreeB.FourC.OneD.TwoE.Five

848. Especially dangerous for surroundings are patients with:A.Skin form of plagueB.*Pulmonary form of plagueC.Skin-bubonic form of plague

Page 118: intranet.tdmu.edu.uaintranet.tdmu.edu.ua/data/kafedra/internal/infect_desease... · Web viewBest method to treat diarrhoea in child is: intra venous fluide *ORS antibiotics bowel

D.Bubonic form of plagueE.Septic form of plague

849. Risk group of plague infection the most frequent is:A.DoctorsB.*HuntersC.AlcoholicD.Drug usersE.Prostitutes

850. Who is the reservoir of causative agent of plague in nature?A.BirdsB.InsectsC.Fresh-water fish D.*Rodents E.Cattle

851. What is the susceptibility of human to plague?A.Non susceptibleB.50 %C.*Almost 100 %D.10 %E.70 %

852. The causative agent of plague is:A.*Yersinia pestisB.Yersinia enterocoliticaC.Yersinia pseudotuberculosisD.Bac. anthracisE.Pseudomonas mallei

853. The duration of incubation period at plague is:A.2-6 hB.*2-6 dC.10-15 dD.17-21 dE.1-6 w

854. To the localized forms of plague belong:A.Secondary-septicB.Primary-septic C.*SkinD.Primary-pulmonaryE. Intestinal

855. To the localized forms of plague belong:A.Intestinal B.Primary-septic C.Secondary-septic D.Primary-pulmonaryE.*Skin-bubonic

856. To the localized forms of plague belong:A.Intestinal

Page 119: intranet.tdmu.edu.uaintranet.tdmu.edu.ua/data/kafedra/internal/infect_desease... · Web viewBest method to treat diarrhoea in child is: intra venous fluide *ORS antibiotics bowel

B.Primary-septic C.Secondary-septic D.Primary-pulmonaryE.*Bubonic

857. To the internal-disseminated forms of plague belong:A.*Primary-septicB.BubonicC.Secondary-pulmonary D.Primary-pulmonaryE. Intestinal

858. Choose the specific treatment of tetanus.A.AntibioticsB.*Serum C.Anticonvulsant medicineD.Cardiac preparationsE.Desintoxication therapy

859. The exciter of tetanus is:A.*ClostridiaB.EscherichiaC.Candida albicansD.NeisseriaE.Gonococcus

860. For the exciter of tetanus characteristic such properties, except:A.Formation of exotoxinsB.Ability to propagate in anaerobic conditionsC.Formation of sporesD.*Formation of gametes

861. The best terms of tetanus exciter cultivation:A.*Anaerobic conditionsB.Oxygen supplyC.Presence of animal albumen in nutritive mediumD.Low temperatureE.1 % peptone water

862. Vegetative form of exciter of tetanus is destroyed in such terms, except for:A.At a temperature of 100 °CB.*At room temperatureC.Under action of carbolic acid D.Under the action of oxygen

863. Who is the source of tetanus?A.Sick personB.RodentsC.*SoilD.Insects E.Cattle

864. The spores of tetanus are saved:A.After boiling during 1 hour

Page 120: intranet.tdmu.edu.uaintranet.tdmu.edu.ua/data/kafedra/internal/infect_desease... · Web viewBest method to treat diarrhoea in child is: intra venous fluide *ORS antibiotics bowel

B.Under act of dry air at the temperature of 115 degrees CC.*In soil during many yearsD.In 1 % solution of formalin during 6 hours

865. Tetanus toxin consists of all units among the listed below, except:A.TetanospasminB.TetanolysinC.ExotoxinD.Low-molecular fractionE.*Enterotoxin

866. What is the receptivity of population to the tetanus?A.0 %B.50 %C.*Almost 100 %D.10 %

867. Causing of tetanus are: A.*C. tetani B.E. coli C.CandidaD.Epstein-Barr virus E.Hemolytic streptococcus group A

868. Duration of the latent period in case of tetanus:A.1-6 hoursB.1-4 daysC.5-14 daysD.*1-6 weeks.E.1-6 months

869. How long does the incubation period of tetanus last?A.1-5 daysB.5-10 daysC.3-5 daysD.*5-14 daysE.15-20 days

870. Tetanus might appear in case of:A.*TraumaB.Mosquito biteC.Usage of stranger clothesD.Contact with the sick peopleE.Drink the water with poor quality

871. Tetanus might appear in case of:A.*Dog biteB.Mosquito biteC.Usage of stranger clothesD.Contact with the sick peopleE.Drink the water with poor quality

872. What is the medical tactic development of the severe tetanus after criminal abortion?

Page 121: intranet.tdmu.edu.uaintranet.tdmu.edu.ua/data/kafedra/internal/infect_desease... · Web viewBest method to treat diarrhoea in child is: intra venous fluide *ORS antibiotics bowel

A.Anticonvulsant preparationsB.Revision of the uterus cavityC.Analgesic therapyD.AntibioticsE.*All answers are correct

873. Choose dose of the specific treatment for patients with tetanus.A.600 units/kg of antytetanus serumB.900 units/kg of antytetanus IgC.500 units/kg of antytetanus IgD.900 units/kg of antytetanus serumE.*500 units/kg of antytetanus serum

874. What is the first aid preparation for the patient with tetanus?A.GlucocorticoidsB.AnalgeticsC.*Anticonvulsant medicineD.Surgical treatment of the woundE.Oxygen therapy

875. Among the listed below choose the complication of the tetanus, which is not early:

A.TracheobronchitisB.*Contracture of muscles and jointsC.AsphyxiaD.MyocarditisE.Pneumonia

876. Among the listed below choose the complication of the tetanus, which is not early:

A.TracheobronchitisB.*Compressive deformation of the spineC.AsphyxiaD.MyocarditisE.Pneumonia

877. Specify the measures of urgent prophylaxis of anthrax.A.Anti-anthrax immunoglobulinB.*Penicillinum or tetracyclinum during 5 daysC.VaccinationD.Medical supervisionE.Biseptolum 5 days

878. What specific test is used for anthrax diagnostic?A.Compliment fixation testB.Indirect hemaglutination testC.*Coetaneous test with antraxinD.Hemaglutination testE.RIFA with anthrax antigen

879. What anthrax prophylactic measures are entertained by farm workers?A.Vitamin therapyB.Immunization by inactivated vaccine

Page 122: intranet.tdmu.edu.uaintranet.tdmu.edu.ua/data/kafedra/internal/infect_desease... · Web viewBest method to treat diarrhoea in child is: intra venous fluide *ORS antibiotics bowel

C.Formulated vaccineD.*Immunization by live vaccineE.Antibiotic therapy

880. The etiological factor of anthrax is:A.Salmonella thyphiB.Erysipelothrix rhysiopothiacC.*Bacillus anthracisD.Rickettsiosis sibiricaE.Toxocara canis

881. The source of infection of anthrax is more frequent than all:A.PeopleB.BirdsC.*Home animalsD.RodentsE.Fly

882. Mechanism of transmission of anthrax are:A.ContactB.AlimentaryC.Air-droplets’D.TransmissivE.*All above it

883. What organ damaged more frequent than all in patients with anthrax?A.*SkinB.LightsC.Gastrointestinal tractD.Lymphatic systemE.Nervous system

884. The basic clinical display of a skin form of anthrax is:A.Hyperemic of skinsB.VesiculsC.*UlcerD.PhlegmonE.Abscess

885. For anthrax most characteristically:A.Change of stoolB.Icterus of skinC.Catarrhal phenomenaD.Meningeal phenomenaE.*Change of skin

886. For a skin form of anthrax the most characteristically:A.HyperemiaB.Painful carbuncleC.*Not painful carbuncleD.Painful noodlesE.Vesicles and bulls

887. For anthrax carbuncle the most characteristically:

Page 123: intranet.tdmu.edu.uaintranet.tdmu.edu.ua/data/kafedra/internal/infect_desease... · Web viewBest method to treat diarrhoea in child is: intra venous fluide *ORS antibiotics bowel

A.Ulcer with a festering bottom, roller on periphery and insignificant area of edemaB.Ulcer with hyperemia on periphery without an edemaC.*Ulcer with a black scab, black color, second vesicles and area of edema around of

ulcerD.Ulcer with a festering bottom, roller on periphery, second vesicles and area of

edemaE.Ulcer with serosis-hemorrhagic exudates, painful, with the area of edema around of

ulcer888. Symptom of Stefansky – it is:

A.Enantema on a soft palateB.Enantema on a conjunctivaC.Shaking of tongue at an attempt to put out a tongueD.*Shaking of edema like to jelly at pattering a hammer in the area of edemaE.Painful of stomach in a right iliac area

889. For the pulmonary form of anthrax characteristically:A.*Foamy sputum with bloodB.Glassy sputum with bloodC.Foamy sputum without bloodD.Foamy green sputumE.Like to «ferruginous» sputum

890. With what diseases it is necessary to differentiate anthrax:A.LeptospirozisB.Typhoid feverC.DermatitisD.*CarbuncleE.Meningococcal infection

891. What material is necessary take for diagnosis of anthrax:A.Spinal liquidB.UrineC.SalivaD.*Content of carbuncleE.Nose swab

892. The diagnostic reaction of anthrax is:A.Rayt‘s reactionB.Vidal‘s reactionC.*Reaction of term precipitation of AskolyD.Paul-Bunnel‘s reactionE.Reaction of agglutination-lysis

893. The diagnostic endermic reaction of anthrax take:A.*AntraksinB.DizenterinC.OrnitinD.MaleinE.Brucellin

894. For treatment of anthrax is:A.Sulfanilamids

Page 124: intranet.tdmu.edu.uaintranet.tdmu.edu.ua/data/kafedra/internal/infect_desease... · Web viewBest method to treat diarrhoea in child is: intra venous fluide *ORS antibiotics bowel

B.NitrofuransC.HormonesD.Antiviral facilitiesE.*Antibiotics

895. It is necessary to appoint for successful treatment of anthrax:A.*Antyanthrax immunoglobulin and penicillinB.Antyanthrax immunoglobulin and prednizolonC.Antyanthrax immunoglobulin and vyrolexD.Antyanthrax immunoglobulin and vermox

896. To what group of infections does the rabies belong?A.*ZoonosisB.AnthroponosisC.CapronosisD.AnthropozoonosisE.Caprozoonosis

897. The basic reservoir of rhabdovirus is:A.PiscesB.ReptilesC.BirdsD.Weed-eatersE.*Carnivores

898. Rhabdovirus from an organism of the patient or animal is revealed to the flow:A.Last 20 days of latent period and during all the illnessB.*Last 7-10 days of latent period and during all the illnessC.Last 7-10 days of latent period D.Last 7-10 days of latent period and at the beginning of illnessE.During all the illness

899. You may be infected a rhabdovirus in case of:A.*Bite +salivation to the skin by an animalB.Infected mealC.Infected waterD.Contact with the infected airE.Bite with the infected insect

900. What is the sensitivity to the rabies?A.45 %B.25 %C.85 %D.*100 %E.10 %

901. What is the mechanism of transmission of rabies?A.TransmissiveB.Fecally-oral C.Air dropD.*Wound E.Domestic contact

902. What is the main mechanism of transmission of rabies?

Page 125: intranet.tdmu.edu.uaintranet.tdmu.edu.ua/data/kafedra/internal/infect_desease... · Web viewBest method to treat diarrhoea in child is: intra venous fluide *ORS antibiotics bowel

A.*AirborneB.AlimentaryC.ContactD.TransmisivE.Vertical

903. Mechanism of transmission of rabies are often: A.*Air B.Contact C.TransmissivD.Fecal-oral E.Transplacental

904. What is the entrance for the rabies?A.*Damaged skin and mucous tissuesB.Respiratory tractsC.Family waysD.Gastrointestinal tractE.Blood

905. Causing of rabies are: A.C. tetani B.E. coli C.CandidaD.Epstein-Barr virus E.*Rabdovirus

906. Duration of the latent period in case of tetanus:A.1-6 hoursB.7-14 daysC.*7 days – 1 earD.1-6 weeks.E.1-6 months

907. What periods of rabies do you know?A.Incubation, depressions, excitationB.*Incubation, depressions, excitation, paralyticC.Depression, excitation, paralyticD.Incubation, excitation, paralyticE. Incubation, depressions, paralytic

908. For rabies the source of infection can be a dog bite in all the cases, except for: A.With rabiesB.Suspicion on rabiesC.VagrantD.*Month prior to the diseaseE.Last 10 days before the disease

909. In rabies the source of the virus can be: A.Wild animalsB.Home animalsC.BatsD.Rodents

Page 126: intranet.tdmu.edu.uaintranet.tdmu.edu.ua/data/kafedra/internal/infect_desease... · Web viewBest method to treat diarrhoea in child is: intra venous fluide *ORS antibiotics bowel

E.*All the answers are correct 910. What is the duration of the prodromal period for the rabies?

A.*1-3 daysB.Up to 1 dayC.4-7 daysD.3-4 daysE.3-5 days

911. The first symptom of prodromal period of rabies is:A.CoughB.NauseaC.VomitingD.DiarrheaE.*Slight swelling and erethema of the scar

912. The first symptom of prodromal period of rabies is:A.CoughB.NauseaC.VomitingD.*Neurological pains in motion nervous barrels, the nearest to the place of biteE.Diarrhea

913. The first symptom of prodromal period of rabies is:A.CoughB.NauseaC.Vomiting D.*Apathy and depression E.Diarrhea

914. Most characteristic symptoms of the rabies are:A.*Paroxysm of hydrophobiaB.Apathy and depressionC.Neuralgic pains on motion nervous barrels, the nearest to the place of biteD.Dyspepsia disordersE.Catarrhal phenomen

915. Most characteristic symptoms of the rabies are:A.*Paroxysm of hydrophobiaB.Paroxysm of aerophobiaC.Paroxysm of fotophobiaD.Paroxysm of akuzophobiaE.Paroxysm of soilphobia

916. What is the duration of excitation period of rabies?A.7-10 daysB.24 hoursC.*2-3 days, sometimes to 6 daysD.Not more than 2 daysE.Up to 6 hours

917. What temperature of the body is typical for the paralytic period?A.*HyperpyrexiaB.Hypothermia

Page 127: intranet.tdmu.edu.uaintranet.tdmu.edu.ua/data/kafedra/internal/infect_desease... · Web viewBest method to treat diarrhoea in child is: intra venous fluide *ORS antibiotics bowel

C.HighD.Normal E.Subfebril

918. Who of the listed below persons must take the conditional course of inoculations against rabies?

A.*A teenager bitten by a dog which is on a leash, not instilledB.Man bitten by a fox which perishedC.A child, scratched by a squirrel which disappeared in-fieldD.A woman, bitten by a cat ill with rabiesE.Man, who had a meal of undercooked of animal with rabies

919. When from the beginning of vaccination an antibodies to the rhabdovirus appear?

A.In a weekB.*In 2 weeksC.In a monthD.After half of yearE.Don’t produced

920. For what infectious pathology is characterized Babesh-Negri‘ bodies?A.Poisoning mushroomsB.Meningo-encefalitC.PoliomyelitisD.*RabiesE.Tetanus

921. Dog bite man on foot. What kind of specific prophylaxis should be conducted for this patient?

A.Human rabies immunoglobulin 6 doses of antirabies vaccineB.12 doses of antirabies vaccineC.Human rabies immunoglobulin and 6 doses of antirabies vaccineD.Human rabies immunoglobulin and 21 dose of antirabies vaccineE.*6 doses of antirabies vaccine

922. What kind of specific prophylaxis should be conducted for patient with bitten foot?

A.Human rabies immunoglobulinB.*Vaccine antirabiesC.Vaccine antirabies and rabies immunoglobulinD.Human rabies immunoglobulin and serumE.Human rabies serum

923. How are the little bodies named (in patients with rabies)?A.Lorin-EpshteynB.BlyumbergC.MursonD.RozenbergE.*Babesh-Negri

924. Where are the little bodies of Babesh-Negri?A.In lungsB.In liver

Page 128: intranet.tdmu.edu.uaintranet.tdmu.edu.ua/data/kafedra/internal/infect_desease... · Web viewBest method to treat diarrhoea in child is: intra venous fluide *ORS antibiotics bowel

C.In a spinal cordD.*In neuronsE. In blood

925. Rabies might appear in case of:A.*Dog biteB.Mosquito biteC.Usage of stranger clothesD.Contact with the sick peopleE.Drink the water with poor quality

926. Select the correct rabies vaccination scheme:A.0, 3, 6, 14, 20, 90 daysB.0, 4, 7, 14, 50, 90 daysC.0, 5, 8, 14, 30, 100 daysD.*0, 3, 7, 14, 30, 90 daysE.0, 1, 5, 13, 60, 90 days

927. The exciter of tetanus is:A.*ClostridiaB.EscherichiaC.Candida albicansD.NeisseriaE.Gonococcus

928. For the exciter of tetanus characteristic such properties, except:A.Formation of exotoxinsB.Ability to propagate in anaerobic conditionsC.Formation of sporesD.*Formation of gametesE.Gram positive

929. The best terms of tetanus exciter cultivation:A.*Anaerobic conditionsB.Oxygen supplyC.Presence of animal albumen in nutritive mediumD.Low temperatureE.1 % peptone water

930. Vegetative form of exciter of tetanus is destroyed in such terms, except for:A.At a temperature of 100 °CB.*At room temperatureC.Under action of carbolic acid D.Under the action of oxygenE.Under action of antibiotics

931. Who is the source of tetanus?A.Sick personB.RodentsC.*SoilD.BacteriocarrierE.Sick person and bacteriocarrier

932. The spores of tetanus are saved:

Page 129: intranet.tdmu.edu.uaintranet.tdmu.edu.ua/data/kafedra/internal/infect_desease... · Web viewBest method to treat diarrhoea in child is: intra venous fluide *ORS antibiotics bowel

A.After boiling during 1 hourB.Under act of dry air at the temperature of 115 degrees CC.*In soil during many yearsD.In 1 % solution of formalin during 6 hoursE.All answers are correct

933. Tetanus toxin consists of all units among the listed below, except:A.TetanospasminB.TetanolysinC.ExotoxinD.Low-molecular fractionE.*Enterotoxin

934. Mechanism of transmission in case of tetanus are:A.Intra muscular conductionB.*ContactC.Insect conductionD.Faecally-oralE.Vertical conduction

935. What is the receptivity of population to the tetanus?A.0 %B.50 %C.*Almost 100 %D.10 %E.70 %

936. Causing of tetanus are: A.*C. tetani B.E. coli C.CandidaD.Epstein-Barr virus E.Hemolytic streptococcus group A

937. Duration of the latent period in case of tetanus:A.1-6 hoursB.1-4 daysC.*5-14 daysD.1-6 weeks.E.1-6 months

938. How long does the incubation period of tetanus last?A.1-5 daysB.5-10 daysC.3-5 daysD.*5-14 daysE.15-20 days

939. Tetanus might appear in case of:A.*TraumaB.Mosquito biteC.Usage of stranger clothesD.Contact with the sick people

Page 130: intranet.tdmu.edu.uaintranet.tdmu.edu.ua/data/kafedra/internal/infect_desease... · Web viewBest method to treat diarrhoea in child is: intra venous fluide *ORS antibiotics bowel

E.Drink the water with poor quality940. Tetanus might appear in case of:

A.*Dog biteB.Mosquito biteC.Usage of stranger clothesD.Contact with the sick peopleE.Drink the water with poor quality

941. What is the medical tactic development of the severe tetanus after criminal abortion?

A.Anticonvulsant preparationsB.Revision of the uterus cavityC.Analgesic therapyD.AntibioticsE.*All answers are correct

942. What measures should be taken in relation to contact persons in case of tetanus?A.VaccinationB.Isolation of contactsC.ChemoprophylaxisD.Laboratory inspectionE.*They need no measures

943. Among the listed below what preparations are not etiological for tetanus?A.AC-anatoxinB.Medical horse serumC.Human immunoproteinD.*Anticonvulsant preparationsE.Penicillin

944. Choose dose of the specific treatment for patients with tetanus.A.500 international units of antytetanus IgB.500 international units of antytetanus serum C.*900 international units of antytetanus IgD.900 international units of antytetanus serumE.900 units/kg of antytetanus serum

945. Which early complications occurs in tetanus?A.TracheobronchitisB.AsphyxiaC.MyocarditisD.PneumoniaE.*All the above

946. Which late complications occurs in tetanus?A.Contracture of muscles and jointsB.Compressive deformation of the spineC.Asthenic syndromeD.Chronic heterospecific diseases of lungsE.*All the above

947. What is the duration of outpatient supervision for patients, recovered of tetanus?A.*2 years

Page 131: intranet.tdmu.edu.uaintranet.tdmu.edu.ua/data/kafedra/internal/infect_desease... · Web viewBest method to treat diarrhoea in child is: intra venous fluide *ORS antibiotics bowel

B.3 monthsC.1 month D.For the decreed groups of population for life timeE.There is no such supervision at all

948. Urgent immuno prophylactic of tetanus in the case of trauma should be conducted in such period:

A.25 days from the moment of traumaB.30 days from the moment of traumaC.In the first 10 days from the moment of trauma D.*At once after the traumaE.Not mentioned

949. Among the listed below people who should receive an immediate prophylactic of the tetanus in form of AC-anatoxin and AC IP injections after trauma?

A.Man of 40 years, in anamnesis with 1 inoculation one year agoB.Pregnant woman of 30 years, in the second half of pregnancyC.Child, 7 months, instilled according to a calendarD.*Retire man of 57 years, who is not instilledE.Child of 6 years, instilled according to a calendar

950. In case of tetanus the epidemiological measures are directed on:A.Elimination of the source of tetanusB.Treatment of the source of tetanusC.*Specific prophylaxisD.Medicines prophylacticsE.Nothing should be performed

951. At what infectious disease does conduct the spasm almost always commences in the muscles of the neck and jaw. causing closure of the jaws?

A.Poisoning mushroomsB.MeningoencefalitisC.PoliomyelitisD.RabiesE.*Tetanus

952. For what disease is characterized this symptom (the generalized spasm of soft muscles, flexion of the arms and extension of the legs)?

A.PoliomyelitisB.BrucellosisC.Pseudo tuberculosisD.*TetanusE.Hydrophobia

953. For what disease is characterized opistotonus?A.PoliomyelitisB.BrucellosisC.Pseudo tuberculosisD.*TetanusE.Hydrophobia

954. For what disease is characterized rizos sardonicus?A.Poliomyelitis

Page 132: intranet.tdmu.edu.uaintranet.tdmu.edu.ua/data/kafedra/internal/infect_desease... · Web viewBest method to treat diarrhoea in child is: intra venous fluide *ORS antibiotics bowel

B.BrucellosisC.Pseudo tuberculosisD.*TetanusE.Hydrophobia

955. For what disease is characterized lockjaw?A.PoliomyelitisB.BrucellosisC.Pseudo tuberculosisD.*TetanusE.Hydrophobia

956. what is name of symptom characterized to the patient with tetanus?A.*Lorin-EpshteynB.BlyumbergC.MursonD.Rozenberg

957. Name the HIV infection high risk groups : A.Homo- and bisexual, prostitutes and other persons who conduct disorderly sexual

life B.Drug addicts who enter drugs parenterally C.Only recipeint of blood, its preparations, sperm and organs D.Only patients with venereal diseases and parenteral viral hepatitis and from the

HIV infected mothers E.*All the above

958. What is the most effective methods of HIV prevention: A.Vaccination and immunoprotein B.Chemoprophylactic C.solation of patients D.*Safe sex and prevention of drug addiction E.Disinfection

959. How many types of HIV are known? A.One B.*Two C.Three D.Four E.Five

960. When HIV/AIDS agent was discovered? A.1981 B.1982 C.*1983 D.2002 E.2003

961. Name the main specific method of HIV diagnosis? A.RPGA B.PLR C.*IFA and ELISA D.Bioassey

Page 133: intranet.tdmu.edu.uaintranet.tdmu.edu.ua/data/kafedra/internal/infect_desease... · Web viewBest method to treat diarrhoea in child is: intra venous fluide *ORS antibiotics bowel

E.RIA 962. Name the most dangerous parenteral way of infection of HIV/AIDS?

A.*Infusion of donor blood and its preparations B.Transplantation of organs C.Injections of medications D.Diagnostic manipulations E.Cosmetic manipulations

963. Name the source of exciter of HIV infection/AIDS? A.*Human B.Animals C.Poultries D.Amphibious E.Fish

964. What is the basic way of transmission of HIV infection: A.Air-born B.Alimentary C.*Parententeral D.TransmissiveE.Water

965. What humans cell of body is a target of HIV? A.Erythrocytes B.Neutrophyl leucocytes C.Monocytes D.T-killer-lymphocytesE.*T-cell helpers

966. What cellular receptors of human attract HIV? A.*CD4 B.CD8 C.CD95 D.CD40 E.CD3

967. What clinical features of sarcoma Kaposhi in patients with AIDS? A.Strike the persons of young and middle age B.Primary elements appear on a head and trunk C.Elements with necrosis and ulceration D.Metastasis in internal organs and high lethality E.*All the above

968. What family of viruses the exciter of HIV/AIDS belong to? A.Orto- and paramyxovirus B.Rabdovirus C.*Retrovirus D.Herpesvirus E.Reovirus

969. What group of infectious diseases an exciter of HIV infection/AIDS belong to according to L. Gromashevsky classification?

A.Intestinal infection

Page 134: intranet.tdmu.edu.uaintranet.tdmu.edu.ua/data/kafedra/internal/infect_desease... · Web viewBest method to treat diarrhoea in child is: intra venous fluide *ORS antibiotics bowel

B.Infections of respiratory tract C.Blood infection D.*Infection of external covers E.Transmissive

970. What group of infectious diseases the exciter of HIV/AIDS belong to? A.*Antroponozis B.Zoonosis C.Sapronosis D.Saprozoonosis E.Zooantroponosis

971. What sexual contact are the most dangerous in relation to an infection with HIV? A.Vaginal B.*Anal C.Oral D.Lesbian E.Artificial impregnation

972. Intravenous introduction of drugs, transfusion of blood or blood products, because of the risk of transmitting BICH should be conducted, except for:

A. For health reasonsB.By decision of the consiliumC. Agreement of the patient or his relativesD. A careful selection of donorsE.* There are no restrictions

973. What dose of antiretroviral drugs for HIV prevention after contact of person with blood and body fluids?

A.600-800 mg / dayB.700-800 mg / dayC.* 800-1000 mg / dayD.. 1000-1100 mg / dayE. . 1100-1200 mg / day

974. How long held antiretroviral prophylaxis regimen after contact with blood and other body fluids?

A.1 WeekB.B. 2 weeksC.* 1 monthD. 3 monthsE. 6 months

975. HIV-infected person is dangerous for others:A. Only in symptomatic periodB. Only in the stage of acute infectionC. Only in the stage of asymptomatic infectionD. Only in the terminal stageE. . * During Lifetime

976. Select an indication for post-exposure prophylaxis of HIV:A. * Medical accident with HIV-infected patientB. HIV-infected pregnant

Page 135: intranet.tdmu.edu.uaintranet.tdmu.edu.ua/data/kafedra/internal/infect_desease... · Web viewBest method to treat diarrhoea in child is: intra venous fluide *ORS antibiotics bowel

C. HIV infectionD. AIDSE. All the above listed

977. Select an indication for post-exposure prophylaxis of HIV:A. AIDSB. HIV-infected pregnantC. HIV infectionD. * Childbirth by HIV-infected motherE. All the above listed

978. Select an indication for post-exposure prophylaxis of HIV:A. Medical accident with HIV-infected patientB. Delivery a baby by HIV-infected motherC. RapeD. TransfusedE. * All the above listed

979. Select an indication for post-exposure prophylaxis of HIV:A. AIDSB. HIV-infected pregnantC. HIV infectionD. * RapeE. All the above listed

980. Select an indication for post-exposure prophylaxis of HIV:A.. * Medical accident with HIV-infected patientsB. HIV-infected pregnantC. HIV infectionD. AIDSE.All the above listed

981. Select an indication for post-exposure prophylaxis of HIV:A.AIDSB.HIV-infected pregnantC.HIV infectionD.* Childbirth from HIV-infected motherE.All the above listed

982. Select an HAART drugs used in the treatment of patients with AIDS except:A.. * Selection of the drug according to the antibiogramB.. Nucleoside reverse transcriptase inhibitors HIVC.. NNRTIs D.. protease InhibitorsE. . integrase inhibitors

983. Terms of prophylaxis with antiretroviral drugs after exposure with blood and body fluids?

A. * No later than 72 hoursB. During a weekC. If the infection is confirmedD.. Dont performE. . To seropositive persons

Page 136: intranet.tdmu.edu.uaintranet.tdmu.edu.ua/data/kafedra/internal/infect_desease... · Web viewBest method to treat diarrhoea in child is: intra venous fluide *ORS antibiotics bowel

984. If the specific markers HIV identified the victim as a result of an accident during the medical examination in the first 5 days after the accident, it means:

A. Occupational exposureB. The patient is in the incubation period of HIV infectionC. * The patient was HIV-positive before the accidentD. Seroconversion after crashE. Does not mean anything

985. Increased risk of infection HIV, except:A. GaysB. AddictsC.. Patients after transplantation of organs and tissuesD. ParamedicsE. * Donors of blood and organs

986. Increased risk of HIV infection , except:A. HomosexualsB. AddictsC.. sex partners of HIV infectedD. paramedicsE.* Persons living in the same apartment

987. Increased risk of infection HIV, except:A. Persons entering into promiscuityB. AddictsC. HemophiliacsD. Children from HIV-infected mothersE. * Donors of blood and organs

988. Increased risk of infection HIV, except:A. HomosexualsB. AddictsC. sex partners of HIV infectedD. paramedicsE. * Persons living in the same apartment

989. Increased risk of infection BICH:A. HomosexualsB. AddictsC. HemophiliacsD. Children from HIV iinfected mothersE. * all the above listed

990. For the diagnosis of Kaposi's sarcoma in AIDS should be:A. The presence of tumor formation or brown cherry color of the skin or mucous

membranesB. Affects young menC. Frequent ulceration, bleedingD. histologic confirmationE. * all the above listed

991. To perform highly active antiretroviral therapy, select the correct combination of drugs:

Page 137: intranet.tdmu.edu.uaintranet.tdmu.edu.ua/data/kafedra/internal/infect_desease... · Web viewBest method to treat diarrhoea in child is: intra venous fluide *ORS antibiotics bowel

A. 2 NRTIs 2 SPB. * 2 NRTIs + 1 NNRTIC. 1 + 2 NRTIs, NNRTIsD. 1 NRTI + 1 PI 2 NNRTIE. 1 2 NRTI PI

992. To perform highly active antiretroviral therapy, select the combination of drugs:A. 1 2 NRTI PIB. 1 + 2 NRTIs, NNRTIsC. * 3 NRTIsD. 1 NRTI + 1 PI 2 NNRTIE. 2 NNRTI 2 SP

993. For post-exposure prophylaxis of HIV infection select the correct combination of drugs:

A. 1 2 NRTI PIB. 1 + 2 NRTIs, NNRTIsC. * 2 NRTIs + 1 NNRTID. 1 NRTI + 1 PI 2 NNRTIE. 2 NNRTI 2 SP

994. For post-exposure prophylaxis of HIV iinfection select the correct combination of drugs:

A. 1 2 NRTI PIB.1 + 2 NRTIs, NNRTIsC. * 2 NRTIs + 1 NNRTID. 1 NRTI + 1 PI 2 NNRTIE. 2 NNRTI 2 SP

995. What is characteristic of Kaposhi's sarcoma in AIDS except:A. common malignancyB. Affects young menC. frequent bleedingD.* Localization only on legs and feetE. frequently ulcerate

996. What is characteristic of Kaposhi's sarcoma in AIDS except:A. Affects young peopleB. * It affects only older menC. widespread localization D.malignancyE. frequently ulcerate

997. What is characteristic of Kaposhi's sarcoma in AIDS , except:A. * The current benignB. Affects young menC. frequent bleedingD. VisceralE. frequently ulcerate

998. HIV contamination may be considered professional if specific markers of the virus identified the victim as a result of an accident during the medical examination at such times, except:

Page 138: intranet.tdmu.edu.uaintranet.tdmu.edu.ua/data/kafedra/internal/infect_desease... · Web viewBest method to treat diarrhoea in child is: intra venous fluide *ORS antibiotics bowel

A. * The first 5 days after the accidentB. 1 monthC. 3 monthsD. after 6 monthsE. 1 year

999. The most important clinical criteria of AIDS should include:A. Significant weight loss (10 %) if it continues over a monthB. Duration of fever, if it continues over a monthC. Pillar, diarrhea, if it lasts more than a monthD. Lymphadenopathy than 3 monthsE.*All the above

1000. The most important clinical criteria of AIDS must include the following, except:A. Significant weight loss (10 %) if it continues over a monthB.Duration of fever, if it continues over a monthC. Pillar, diarrhea, if it lasts more than a monthD.* Rush on the skin, if it lasts more than a monthE.Lymphadenopathy longer than 3 months.

1001. . The most important clinical criteria of AIDS should include:A.Significant weight loss (10 %) if it continues over a monthB.Duration of fever, if it continues over a monthC.Pillar, diarrhea, if it lasts more than a monthD. * Lymphadenopathy than 3 monthsE. All the above

1002. How to decide the child's immunizations in case of AIDS ?A.Vaccination against diphtheria and tetanusB.. Vaccination against polioC. Vaccination against tuberculosisD. Vaccination against polio and tuberculosisE. * Vaccination do not conduct

1003. How to decide on the child's immunizations with established HIV infection?A.* Vaccination against diphtheria and tetanusB.Vaccination against polioC. Vaccination against tuberculosisD. Vaccination against polio and tuberculosisE. Vaccination not conduct

1004. What antiretroviral drugs may prescribed for HIV-infected pregnant women in order to reduce the risk of infection of the fetus?

A. Zidovudine and lopinavirB. * Zidovudine and ViramuneC.. Timazid and RetrovirD.AZT and RetrovirE. . Zidovudine, Retrovir and timazid

1005. What antiretroviral drugs may prescribed for HIV-infected pregnant women in order to reduce the risk of infection of the fetus?

A. * Zidovudine and lopinavirB. Retrovir and Viramune

Page 139: intranet.tdmu.edu.uaintranet.tdmu.edu.ua/data/kafedra/internal/infect_desease... · Web viewBest method to treat diarrhoea in child is: intra venous fluide *ORS antibiotics bowel

C. Timazid and RetrovirD.AZT and RetrovirE.Zidovudine, Retrovir and timazid

1006. What antiretroviral drugs may prescribed for HIV-infected pregnant women in order to reduce the risk of infection of the fetus?

A.Zidovudine and lopinavirB.* Zidovudine and NevirapineC.Timazid and RetrovirD.AZT and RetrovirE.Zidovudine, Retrovir and timazid

1007. What additional testing it is necessary to patients with infectious mononucleosis?A.* IFA for BICH infection, bakobsledovanie of diphtheriaB. IFA for BICH infection, tularemia on bakobsledovanieC. Bacteriological tests of diphtheria and typhoid feverD. Reaction Burne and Wright-HeddlsonaE. The reaction of Paul Bunnelya and lymph node puncture

1008. What immunological changes are identified in patients with HIV infection?A. Is a polyclonal B-cell inhibition, increases the formation of autoantibodies and

immune complexesB.Is a polyclonal B-cell activation, reduced the formation of autoantibodies and

immune complexesC. Is a polyclonal B-cell inhibition, decreases the formation of autoantibodies and

immune complexesD.* There is a polyclonal B-cell activation, increases the formation of autoantibodies

and immune complexesE. Is a polyclonal B-cell activation, increases the formation of autoantibodies and

decreases the formation of immune complexes1009. What laboratory findings are not typical for AIDS-related complex?

A.* Decreasing of the level of circulating immune complexesB.The aspect ratio of 1.0 CD4/CD8C.AnemiaD.LeukopeniaE.Thrombocytopenia

1010. What laboratory findings are not typical for AIDS-related complex?A.Increasing levels of circulating immune complexesB.The aspect ratio of 1.0 CD4/CD8C.AnemiaD.* LeukocytosisE.Thrombocytopenia

1011. What laboratory findings are not typical for AIDS-related complex?A. * Decrease of immunoglobulins A is the G.B.Decreasing CD4/CD8 ratio below 1.0C.AnemiyaD.LeukopeniaE. Increase levels of circulating immune complexes

1012. What laboratory findings are not typical for AIDS-related complex

Page 140: intranet.tdmu.edu.uaintranet.tdmu.edu.ua/data/kafedra/internal/infect_desease... · Web viewBest method to treat diarrhoea in child is: intra venous fluide *ORS antibiotics bowel

A.* Decrease of immunoglobulins A is the G.B.. Decreasing CD4/CD8 ratio below 1.0C. AnemiyaD. LeukopeniaE. Increased levels of circulating immune complexes

1013. What laboratory findings are not typical for AIDS-related complex?A.Increased levels of circulating immune complexesB.The aspect ratio of 1.0 CD4/CD8C.AnemiyaD. * LeukocytosisE. Thrombocytopenia

1014. What laboratory findings are not typical for AIDS-related complex?A. * Decrease in the level of circulating immune complexesB. The aspect ratio of 1.0 CD4/CD8C. AnemiyaD. leukopeniaE. thrombocytopenia

1015. What antiretroviral drug exposure prophylaxis is carried out after contact with blood and other body fluids?

A.azidothymidineB.nevirapineC.* Azidothymidine + nevirapineD.AZT + saquinavirE. Іfavіrents

1016. What term is usually defined by quantitative HIV RNA levels in blood plasma?A. viral strainB. the number of viralC. * Viral loadD. viral factorE. Viral RNA levels

1017. What is the most common term of seroconversion in patients with HIV infection?

A. 1 WeekB. 1 monthC. * 3 monthsD. 1 yearE. indefinitely

1018. What is the maximum duration of the incubation period in HIV infection?A. 1 monthB. 3 monthsC. 1 yearD. * 5-6 yearsE. indefinitely

1019. What is the minimum duration of the incubation period in HIV infection?A. * 1 weekB. 1 month

Page 141: intranet.tdmu.edu.uaintranet.tdmu.edu.ua/data/kafedra/internal/infect_desease... · Web viewBest method to treat diarrhoea in child is: intra venous fluide *ORS antibiotics bowel

C. 3 monthsD. 1 yearE. indefinitely

1020. What stage of the life cycle of the virus suppressed antiretroviral drugs, called protease inhibitors (PI)?

A. Entry of HIV into cellsB. reverse TranscriptionC. integrationD.TranscriptionE.* Build and branch

1021. Clinical manifestations of acute retroviral syndrome, except:A.FeverB. various rashesC. Influenzalike syndromeD.DiarrheaE. . Bleeding

1022. Clinical manifestations of acute retroviral syndrome, except:A. meningeal syndromeB. various rashesC. Influenzalike syndromeD. diarrheaE. * Jaundice

1023. When AIDS can be diagnose?A. Only when the number of CD4-lymphocytes less than 500 in 1 ml of bloodB. Only when the number of CD4-l lymphocytes less than 400 in 1 ml of bloodC.Only when the number of CD4-lymphocytes less than 300 in 1 ml of bloodD.* Only when the number of CD4- lymphocytes less than 200 in 1 ml of bloodE. Only when the number of CD4- lymphocytes less than 100 in 1 ml of blood

1024. When prescribed antiretrovirals to a HIV-infected pregnant women in need of ART for their own health in order to reduce the risk of infection of the fetus?

A.* Regardless of the duration of pregnancy and childbirthB. Only during childbirthC. C 20 weeks of pregnancy and during laborD. C 14 weeksE. C 24 weeks of pregnancy and during labor

1025. When prescribed antiretrovirals HIV-infected pregnant women that do not need ART for their own health in order to reduce the risk of infection of the fetus?

A. throughout pregnancy and during laborB. Only during childbirthC.20 weeks of pregnancy and during laborD.14 weeksE.* C 24 weeks of pregnancy and during labor

1026. When prescribed antiretrovirals HIV-infected pregnant women in need of ART for their own health in order to reduce the risk of infection of the fetus?

A.. * Regardless of the duration of pregnancy and childbirthB. Only during childbirth

Page 142: intranet.tdmu.edu.uaintranet.tdmu.edu.ua/data/kafedra/internal/infect_desease... · Web viewBest method to treat diarrhoea in child is: intra venous fluide *ORS antibiotics bowel

C. C 20 weeks of pregnancy and during laborD. C 14 weeksE. C 24 weeks of pregnancy and during labor

1027. When administered antiretroviral drugs to HIV-infected pregnant women to reduce the risk of infection of the fetus?

A.Throughout pregnancy and during laborB. Only during childbirthC. From 25 weeks of pregnancy and during laborD. With 14 weeks of pregnancyE.* From 28 weeks of pregnancy and during labor

1028. When antiviral therapy for children born from HIV-infected women should starting?

A.* In the first hour after birth, 12.8B.On the second dayC.Do not spendD.After a monthE.After diagnosis SPIDA.

1029. When the planned caesarean section for HIV-infected pregnant women reduce the risk of infection of the fetus?

A. In 36 weeksB. In 37 weeksC.* In 38 weeksD. In 39 weeksE. In 40 weeks

1030. What cells is the main target for HIV?A.T-suppressorB.T-killersC.* T-helper cellsD.D-cellsE.0-cells

1031. What stage of HIV life cycle nucleoside reverse transcriptase inhibitors perform action (NRTIs)?

A.HIV penetration into cellsB.reverse TranscriptionC.integrationD.transcriptionE.* Replacement of the nucleoside

1032. What stage of the life cycle is HIV zidovudine may destroy?A.HIV penetration into cellsB.Reverse transcriptionC.integrationD.transcriptionE.* Replacement of the nucleoside

1033. What stage of the life cycle is HIV non-nucleoside reverse transcriptase inhibitors (NNRTIs) may destroy?

A.HIV penetration into cells

Page 143: intranet.tdmu.edu.uaintranet.tdmu.edu.ua/data/kafedra/internal/infect_desease... · Web viewBest method to treat diarrhoea in child is: intra venous fluide *ORS antibiotics bowel

B.* Reverse TranscriptionC.IntegrationD.TranscriptionE.Broadcasting

1034. What stage of the life cycle is HIV zidovudine nucleoside reverse transcriptase inhibitors (NRTIs) may destroy?

A.HIV penetration into cellsB.Reverse TranscriptionC.IntegrationD.TranscriptionE.* Replacement of the nucleoside

1035. What is acute retroviral syndrome clinically:A.FeverB.LymphadenopathyC.Enlargement of the liverD.Meningeal syndromeE.* All the above

1036. What cells of the immune system, which contain CD4 molecules on their surfaces may infected by HIV.

A.Monocytes, macrophagesB.Macrophages, T-lymphocytesC.T-lymphocytes, macrophages, cells of the central nervous systemD.* T-lymphocytes, cells of the CNS, monocytes, macrophagesE.Monocytes, macrophages, T-lymphocytes

1037. What cells of the immune system, which contain CD4 molecules on their surfaces may infected by HIV.

A.MonocytesB.MacrophagesC.T-lymphocyteD.MicrogliaE.* all the above

1038. What cells of the immune system, which contain CD4 molecules on their surfaces may infected by HIV.

A.MonocytesB.MacrophagesC.T-lymphocyteD.MicrogliaE.* all the above

1039. HIV-positive, practical healthy baby of first year was allowed of such preventive vaccines, except:

A.Diphtheria ToxoidB.Tetany toxoidC.* oral polioD.inactivated polioE.pertussis

Page 144: intranet.tdmu.edu.uaintranet.tdmu.edu.ua/data/kafedra/internal/infect_desease... · Web viewBest method to treat diarrhoea in child is: intra venous fluide *ORS antibiotics bowel

1040. HIV-positive, practical healthy baby of first year was allowed of such preventive vaccines, except:

A.Diphtheria ToxoidB.Tetany toxoidC.* tuberculosisD.inactivated polioE.pertussis

1041. HIV-positive, practical healthy baby of first year was allowed of such preventive vaccines, except:

A.Diphtheria ToxoidB.Tetany toxoidC.* Triple vaccine against measles, mumps, rubellaD.Inactivated polioE.pertussis

1042. HIV-positive, practical healthy baby of first year was allowed of such preventive vaccines, except:

A.Diphtheria ToxoidB.Tetany toxoidC.* measlesD.inactivated polioE.pertussis

1043. HIV-positive, practical healthy baby of first year was allowed of such preventive vaccines, except:

A.Diphtheria ToxoidB.Tetany toxoidC.* rubellaD.inactivated polioE.pertussis

1044. HIV-positive, practical healthy baby of first year was allowed of such preventive vaccines, except:

A.Diphtheria ToxoidB.Tetany toxoidC.* mumpsD.inactivated polioE.pertussis

1045. What is not typical for the IV clinical stage of AIDS, according to clinical stage classification, developed by experts of WHO(2006)?

A.CachexiaB.PCPC.Cerebral toxoplasmosisD.Extrapulmonary cryptococcosisE.* Cryptosporidiosis with diarrhea less than 1 month

1046. According to clinical stage classification developed by experts of WHO (2006) what is not typical for the IV clinical stage of AIDS?

A.Cachexia associated with HIV infectionB.PCP

Page 145: intranet.tdmu.edu.uaintranet.tdmu.edu.ua/data/kafedra/internal/infect_desease... · Web viewBest method to treat diarrhoea in child is: intra venous fluide *ORS antibiotics bowel

C.Cerebral toxoplasmosisD.Extrapulmonary cryptococcosisE.* Diarrhea lasting less than 1 month

1047. According to clinical stage classification developed by experts of WHO (2006) what is not typical for the II clinical stage of AIDS?

A.Loss of body weight less than 10 % from the initialB.Minimal damage of the skin and mucous membranes (seborrheic dermatitis,

pruritus, fungal nail infections)C.Episode of herpes zoster during last five yearsD.Recurrent upper respiratory tract infections (bacterial sinusitis)E.* Tuberculosis of the lungs, which has evolved over the year preceding the

examination1048. According to clinical stage classification developed by experts of WHO (2006)

what is not typical for the II clinical stage of AIDS? A.* Weight loss of more than 10 % from the initialB.Persistent generalized lymphadenopathyC.Episode of herpes zoster during last five yearsD.Recurrent upper respiratory tract infectionsE.Minimal mucosal lesions (recurrent ulcers of the oral mucosa, angular cheilitis)

1049. According to clinical stage classification developed by experts of WHO (2006) what is not typical for the II clinical stage of AIDS?

A.Loss of body weight less than 10 % from the initialB.* Unmotivated chronic diarrhea that lasts more than 1 monthC.Episode of herpes zoster during last five yearsD.Recurrent upper respiratory tract infectionsE.Minimal mucosal lesions (recurrent ulcers of the oral mucosa, angular cheilitis)

1050. According to clinical stage classification developed by experts of WHO (2006) what is not typical for the IV clinical stage of AIDS?

A.Loss of body weight less than 10 % from the initialB.* Multiple lesions of the skin and mucous membranesC.Episode of herpes zoster D.Recurrent upper respiratory tract infectionsE.Minimal mucosal lesions (recurrent ulcers of the oral mucosa, angular cheilitis)

1051. According to clinical stage classification developed by experts of WHO (2006) what is not typical for the IV clinical stage of AIDS?

A.Loss of body weight less than 10 % of the initialB.* Sarcoma KaposhiC.Episode of herpes zoster in the past five yearsD.Recurrent upper respiratory tract infectionsE.Minimal mucosal lesions (recurrent ulcers of the oral mucosa, angular cheilitis)

1052. According to clinical stage classification developed by experts of WHO (2006) what is not typical for the II clinical stage of AIDS?

A.Loss of body weight less than 10 % from the originalB.Minimal damage to the skin and mucous membranes (seborrheic dermatitis,

pruritus, fungal nail infections)C.Episode of herpes zoster

Page 146: intranet.tdmu.edu.uaintranet.tdmu.edu.ua/data/kafedra/internal/infect_desease... · Web viewBest method to treat diarrhoea in child is: intra venous fluide *ORS antibiotics bowel

D.Recurrent upper respiratory tract infections (eg, bacterial sinusitis)E.* Tuberculosis of lungs, which has evolved over the year preceding the

examination1053. According to clinical stage classification developed by experts of WHO (2006)

what is not typical for the II clinical stage of AIDS?A.Loss of body weight less than 10 % from the initialB.* Multiple lesions of the skin and mucous membranesC.Episode of herpes zoster in the past five yearsD.Recurrent upper respiratory tract infectionsE.Minimal mucosal lesions (recurrent ulcers of the oral mucosa, angular cheilitis)

1054. According to clinical stage classification developed by experts of WHO (2006) what is not typical for the II clinical stage of AIDS?

A.* Weight loss of more than 10 % from the initialB.Persistent generalized lymphadenopathyC.Episode of herpes zoster in the past five yearsD.Recurrent upper respiratory tract infectionsE.Minimal mucosal lesions (recurrent ulcers of the oral mucosa, angular cheilitis)

1055. According to clinical stage classification developed by experts of WHO (2006) what is not typical for the II clinical stage of AIDS?

A.Loss of body weight less than 10 % from the initialB.* Unmotivated chronic diarrhea that lasts more than 1 monthC.Episode of herpes zoster in the past five yearsD.Recurrent upper respiratory tract infectionsE.Minimal mucosal lesions (recurrent ulcers of the oral mucosa, angular cheilitis)

1056. According to clinical stage classification developed by experts of WHO (2006) what is not typical for the II clinical stage of AIDS?

A.Loss of body weight less than 10 % of the initialB.* sarcoma KaposhiC.Episode of herpes zoster in the past five yearsD.Recurrent upper respiratory tract infectionsE.Minimal mucosal lesions (recurrent ulcers of the oral mucosa, angular cheilitis)

1057. According to clinical stage classification developed by experts of WHO (2006) what is not typical for the III clinical stage of AIDS?

A.Weight loss of more than 10 % B.Unmotivated chronic diarrhea that lasts more than 1 monthC.Unexplained fever that lasts more than 1 month (permanently or temporarily)D.* Episods of upper respiratory tract infection ( bacterial sinusitis)E.Leukoplakia of the oral mucosa

1058. According to clinical stage classification developed by experts of WHO (2006) what is not typical for the III clinical stage of AIDS?

A.Lymphadenopathy more than 3 monthsB.Unmotivated chronic diarrhea that lasts more than 1 monthC.Unexplained fever that lasts more than 1 month (permanently or temporarily)D.* CachexiaE.Cytomegalovirus chorioretinitis

Page 147: intranet.tdmu.edu.uaintranet.tdmu.edu.ua/data/kafedra/internal/infect_desease... · Web viewBest method to treat diarrhoea in child is: intra venous fluide *ORS antibiotics bowel

1059. According to clinical stage classification developed by experts of WHO (2006) what is not typical for the IV clinical stage of AIDS?

A.Weight loss of more than 10 % of the initialB.Unmotivated chronic diarrhea that lasts more than 1 monthC.Unexplained fever that lasts more than 1 month (permanently or temporarily)D.* Single lymph nodes in one anatomical region, acute painE.Cytomegalovirus chorioretinitis

1060. According to clinical stage classification developed by experts of WHO (2006) what is not typical for the III clinical stage of AIDS?

A.Weight loss of more than 10 % of the initialB.Unmotivated chronic diarrhea that lasts more than 1 monthC.Unexplained fever that lasts more than 1 month (permanently or temporarily)D.* Single upper respiratory tract infection (eg, bacterial sinusitis)E.Hairy leukoplakia of the oral mucos

1061. According to clinical stage classification developed by experts of WHO (2006) what is not typical for the IV clinical stage of AIDS?

A.Cachexia B.PCPC.Cerebral toxoplasmosisD.Carkoma sarcomaE.* All the above

1062. Experts of WHO believe suspicious according to AIDS lymph nodes:A.3 or more nodes in more than two anatomical and topographical groups (except

inguinal), measuring more than 2 cm in diameter, which extends more than 1 month.

B.3 or more nodes in more than two anatomical and topographical groups (except inguinal), larger than 1 cm in diameter, for more than 2 months.

C.* 2 or more nodes in more than two anatomical and topographical groups (except inguinal), larger than 1 cm in diameter, which lasts more than 3 months.

D.2 or more units in more than two anatomical topographic groups (except inguinal), larger than 2 cm in diameter, which extends over 2 months.

E.2 or more units in more than two anatomical topographic groups (except inguinal), larger than 1 cm in diameter, which continued for over 2 months.

1063. Immediately after contact with blood and other biological liquids it is necessary to wash the dirty areas of skin with water and soap and to begin a postcontact prophylaxis, antiretroviral preparations not later than:

A.* 24-36 hrs B.36-48 hrs C.48-60 hrs D.60-72 hrs E.72-86 hrs

1064. What antiretroviral preparation is taken as post contact prophylaxis after a contact with blood and other biological fluids?

A.* Azidotimidin B.Nevirapin C.Indinavir

Page 148: intranet.tdmu.edu.uaintranet.tdmu.edu.ua/data/kafedra/internal/infect_desease... · Web viewBest method to treat diarrhoea in child is: intra venous fluide *ORS antibiotics bowel

D.Saqvinavir E. Ifavirent

1065. When do the plan caesarian section will conduct to HIV infected pregnant with the purpose for decreasing of the risk of infecting the fetus?

A.In 36 weeks B.In 37 weeks C.* In 38 weeks D.In 39 weeks E. In 40 weeks

1066. After a car accident patient has been taken to hospital in critical condition, caused by shock, bleeding. Before blood transfusion the doctor should consider:

A.Health reasonsB.in order to prevent HIV transmission The conclusion of doctors consiliumC.Agree of the patient (or his family) D.The results of a survey of emergency donor about HIV (IHA)E.* All the above

1067. Immediately after contact with blood or other body fluids person should wash exposed skin with soap and water, and contaminated mucous membranes - with clean water. When is the post-exposure prophylaxis with antiretroviral drugs should start?

A.*No later than 72 hoursB.B for a weekC.If the infection is confirmedD.Not availableE.Seropositive persons

1068. Immediately after contact with blood or other body fluids person should wash exposed skin with soap and water, and contaminated mucous membranes - with clean water. How long is conducted post-exposure prophylaxis?

A.During a weekB.*4 weeksC.3 monthsD.Until the end of the observation periodE.Not available

1069. At what dose of antiretroviral drug prophylaxis is carried out after contact with blood and other body fluids?

A.600-800 mg B.700-800 mg C.* 800-1000 mg D.1000-1100 mg E.1100-1200 mg

1070. How long is the antiretroviral prophylaxis after contact with blood and other body fluids perfomed?

A.1 WeekB.2 weeksC.* 1 monthD.3 monthsE.6 months

Page 149: intranet.tdmu.edu.uaintranet.tdmu.edu.ua/data/kafedra/internal/infect_desease... · Web viewBest method to treat diarrhoea in child is: intra venous fluide *ORS antibiotics bowel

1071. The epidemic outbreak rationally organize inspection of the immune system. Using the IHA to identify nonimmune individuals to diphtheria in a few hours. What is the minimum protective titer?

A.1:10B.1:20C.* 1:40D.1:80E.1:160

1072. The most important clinical criteria for AIDS must include the following, except:A.Significant weight loss (10%) if it continues over a monthB.Duration of fever, if it continues over a monthC.Persistent diarrhea, if it lasts more than a monthD.* A rash on the skin, if it continues over a monthE.Lymphadenopathy than 3 mesyatseB.

1073. How to decide on the child's immunizations with unclear HIV status to HIV-infected mothers?

A.Vaccination against diphtheria and tetanusB.Polio vaccinationC.Vaccination against tuberculosisD.Vaccination against polio and tuberculosisE.* Vaccination is not performed

1074. What additional testing is necessary to patients with infectious mononucleosis?A.*ELISA for HIV, bacteriology of diphtheriaB.ELISA for HIV, bacteriology for tularemiaC.bacteriology of diphtheria and typhoid feverD.Reaction Burne and Wright-HeddlsonaE.The reaction of Paul Bunnelya and lymph node puncture

1075. What immunological changes are identified in a patient with HIV infection?A.Is a polyclonal B-cell inhibition, increases the formation of autoantibodies and

immune complexesB.Is a polyclonal B-cell activation and decreases the formation of autoantibodies and

immune complexesC.Is a polyclonal B-cell inhibition, decreases the formation of autoantibodies and

immune complexesD.* There is a polyclonal B-cell activation, increases the formation of autoantibodies

and immune complexesE. Is a polyclonal B-cell activation, increases the formation of autoantibodies and

decreases the formation of immune complexes1076. What laboratory findings are not typical of Sneed-related complex?

A.Decreasing the amount of immunoglobulin A is the G.B.* Reducing the ratio below 1.0 CD4/CD8C.AnemiaD.LeukopeniaE. Increased levels of circulating immune complexes

1077. Which antiretroviral drug prophylaxis is carried out after contact with blood and other body fluids?

Page 150: intranet.tdmu.edu.uaintranet.tdmu.edu.ua/data/kafedra/internal/infect_desease... · Web viewBest method to treat diarrhoea in child is: intra venous fluide *ORS antibiotics bowel

A.AzidothymidineB.NevirapineC.* Azidothymidine + nevirapineD.AZT + saquinavirE. Іfavіrents

1078. What term is usually defined by quantitative HIV RNA levels in blood plasma?A.Viral strainB.The number of viralC.* Viral loadD.Viral factorE.Viral RNA levels

1079. What are the characteristics of plaque in diphtheria?A.Single-sided, gray-white, on the surface ulcer craterB.* The gray-white, dense, with sharp edges and glossy surfaceC.Yellow-white, brittle, located perilakunarnoD.Sided, yellow-white, in the gapsE.White, brittle, easily removed with a spatula

1080. What stage of the life cycle of the virus are suppressed by antiretroviral drugs, called protease inhibitors (PI)?

A.Entry of HIV into cellsB.reverse TranscriptionC.integrationD.transcriptionE.* Build and branch

1081. When you can make a diagnosis of AIDS?A.Only when the content of CD4-lymphocytes less than 500 in 1 ml of bloodB.Only when the content of CD4- lymphocytes less than 400 in 1 ml of bloodC.Only when the content of CD4- lymphocytes less than 300 in 1 ml of bloodD.* Only when the content of CD4- lymphocytes less than 200 in 1 ml of bloodE.Only when the content of CD4- lymphocytes less than 100 in 1 ml of blood

1082. When are antiretroviral drugs administered to HIV-infected pregnant women to reduce the risk of infection of the fetus?

A.Throughout pregnancy and during laborB.Only during childbirthC.From 25 weeks of pregnancy and during laborD.With 14 weeks of pregnancyE.* From 28 weeks of pregnancy and during labor

1083. When is antiviral therapy started for children born to HIV-infected women?A.* In the first hour after birth, 12.8B.On the second dayC.Do not spendD.After a monthE.After diagnosis SPIDA.

1084. When is the planned caesarean section for HIV-infected pregnant women to reduce the risk of infection of the fetus?

A.In 36 weeks

Page 151: intranet.tdmu.edu.uaintranet.tdmu.edu.ua/data/kafedra/internal/infect_desease... · Web viewBest method to treat diarrhoea in child is: intra venous fluide *ORS antibiotics bowel

B.In 37 weeksC.* At 38 weeksD.In 39 weeksE. In 40 weeks

1085. Who is the primary target for HIV?A.T-suppressorB.T-killersC.* T-helper cellsD.D-cellsE.0-cells

1086. Violation of which stage of the life cycle are caused by nucleoside reverse transcriptase inhibitors (NRTIs)?

A.Entry of HIV into cellsB.reverse TranscriptionC.integrationD.transcriptionE.* Replacement of the nucleoside

1087. Violation of which stage of the life cycle of HIV is zidovudine?A.Entry of HIV into cellsB.reverse TranscriptionC.integrationD.transcriptionE.* Replacement of the nucleoside

1088. Violation of which stage of the life cycle are caused by non-nucleoside reverse transcriptase inhibitors (NNRTIs)?

A.Entry of HIV into cellsB.* Reverse TranscriptionC.integrationD.transcriptionE.Translation

1089. Complication in the 4-5th week of diphtheria:A.EncephalitisB.Bulbar disorders, pancreatitis, hepatitisC.* Polyradiculitis, myocarditisD.NephrosonephritisE.Stenosing laryngotracheitis

1090. Complication that often develops in the first week of diphtheria oropharynx:A.PolyradiculitisB.AsphyxiaC.Failure of adrenal glandsD.HepatosplenomegalyE.* Paresis of the soft palate

1091. Particularly high titre diphtheria antitoxic antibodies in moderate titer of anti-tetanus antibodies indicates:

A.TetanusB.Diphtheria

Page 152: intranet.tdmu.edu.uaintranet.tdmu.edu.ua/data/kafedra/internal/infect_desease... · Web viewBest method to treat diarrhoea in child is: intra venous fluide *ORS antibiotics bowel

C.Carriering of Corynebacterium diphtheriaD.Immunity to diphtheria and to the formation of bacteriaE.* Nothing

1092. Before revaccination against diphtheria in adults, they recommended:A.* Identify the antibody titerB.Preventive antibioticsC.Proactively assign antihistaminesD.Five years after the last boosterE.10 years after vaccination

1093. List all the cells of the immune system, which contain CD4 molecules on their surfaces that are infected with HIV.

A.Monocytes, macrophagesB.Macrophages, T-lymphocytesC.T-lymphocytes, macrophages, cells of the central nervous systemD.* T-lymphocytes, cells of the CNS, monocytes, macrophages

1094. How to prevent occupational HIV infection?A.Local wound treatmentB.* Post eccident HAARTC.Laboratory testing for HIVD.Register the fact of the accident in a special registerE.All the above

1095. What preparations are necessary for prevention of occupational HIV infection?A.Local wound treatmentB.* Antiretrovirus drugsC.AntibioticsD.Specific immunoglobulinE.All the above

1096. During assistance nurse accidentally pricked her finger with a contaminated needle. What is prevention of disease?

A.The combination of NRTIs + PIB.* Combination NRTI + NNRTIC.InterferonsD.Specific immunoglobulinE. Initial debridement

1097. During assistance nurse accidentally pricked her finger with a contaminated needle . For the recognition of occupational HIV infection?

A.Seroconversion after crashB.HIV asymptomatic infection in accidentC.Occupational exposureD.* Infection occurred before the accidentE.All the above

1098. During assistance nurse accidentally pricked her finger with a contaminated needle . For the recognition of occupational HIV infection :

A.Confirmation of HIVB.Accounting the fact of the accident in a special registerC.Negative results of laboratory examination in the first 5 days after the accident

Page 153: intranet.tdmu.edu.uaintranet.tdmu.edu.ua/data/kafedra/internal/infect_desease... · Web viewBest method to treat diarrhoea in child is: intra venous fluide *ORS antibiotics bowel

D.Positive results of laboratory testing for HIV at 1, 3 or 6 months after the accidentE.* All the above

1099. During assistance nurse accidentally pricked her finger with a contaminated needle . For the recognition of occupational HIV infection :

A.Confirmation of HIVB.Statement of victimC.* Negative results of laboratory examination in the first 5 days after the accident

and positive 1, 3 or 6 months after the accidentD.Availability of health book in the affectedE.Full-time work regimen

1100. A child of HIV-positive without clinical and laboratory signs of the disease. How to solve the problem of vaccination against polio?

A.* Conduct, as it is provided in the Calendar routine immunizationB.Vaccination dont performC.Hold until the results of a survey on child markers HIVD.Not available as prohibit the use of live vaccinesE.Show all of the above is true

1101. A child of HIV-positive without clinical and laboratory signs of the disease. How to solve the problem of vaccination against polio?

A.Not available as prohibit the use of live vaccinesB.Hold, as it is provided according the Calendar routine immunizationC.Postponed until the results of child markers HIVD.* Replace the live vaccine to inactivatedE.Vaccination dont perform

1102. A child of HIV-infected mother, was born apparently healthy. When you can confirm HIV-negative status of the child?

A.Immediately after birth if there are no clinical manifestationsB.After 3 months in the absence of HIV markersC.After 6 months in the absence of the child HIV markersD.* After 18 months in the absence of the child markers HIVE.Never, since the HIV transmission from mother occurs in 100% of cases

1103. A child of HIV-infected mother, was born apparently healthy. How to solve the problem of vaccination against tuberculosis?

A.Hold, as it is provided in the Calendar routine immunizationB.Performed if there is no clinical and laboratory signs of diseaseC.Postponed until the results of a survey on child markers HIVD.* Not available as prohibit the use of live vaccinesE.Vaccination dont perform

1104. A child of HIV-infected mother, was born prematurely, with clinical signs of AIDS. How to solve the question of vaccination (tuberculosis, hepatitis B)?

A.Hold, as it is provided in the Calendar routine immunizationB.Performed if there is no clinical and laboratory signs of diseaseC.Postponed until the results of a survey on child markers HIVD.Not available as prohibit the use of live vaccinesE.* Vaccination dont perform

Page 154: intranet.tdmu.edu.uaintranet.tdmu.edu.ua/data/kafedra/internal/infect_desease... · Web viewBest method to treat diarrhoea in child is: intra venous fluide *ORS antibiotics bowel

1105. Immediately after contact with blood and other body fluids should be washed exposed skin with soap and water, and contaminated mucous membranes - with clean water. When is the post-exposure prophylaxis with antiretroviral drugs?

A.* No later than 72 hoursB.During the weekC.If the infection is confirmedD.Not availableE.Seropositive persons

1106. In a different places of settlement a few cases of cholera was found. Who in the focus of cholera will be send in a hospital?

A.CarriersB.Persons contact with the patientC.*Patients with choleraD.Persons with dysfunction of alimentary tractE.Persons with hyperthermia

1107. Diagnosed a patient: chronic hepatitis in the stage of integration. What markers will be in patient in this stage disease?

A.HBeAgB.Antibodies to HBeAgC.DNA OF HBVD.Viral DNA-polimeraseE.*HBsAg, anti-НBе

1108. As etiotropic therapy of acute and chronic viral hepatitis B utillize:A.CorticosteroidB.ImmunomodulatorsC.CytostaticsD.AntibioticsE.*Antiviral

1109. Direct bilirubin is increased , in urine there is significant increase of bilirubin and urobilin, increasing of stercobilin of excrements. What is the type of icterus?

A.HaemoliticB.*ParenhimatousC.TransportD.ExtraliverE.Mechanical

1110. On the average 15 to 30 % of all population of the planet suffer from some pathology of liver. Prevalence of hepatitis and cirrhosis in the European countries is about 1 % of adults. Annually in the world there are about 2 million people with acute viral hepatitis. What % of all cases will develop chronic form.

A.100 %B.50 %C.25 %D.*10 %E.1 %

1111. When sick people get epidemic typhus infection, which period affects more?

Page 155: intranet.tdmu.edu.uaintranet.tdmu.edu.ua/data/kafedra/internal/infect_desease... · Web viewBest method to treat diarrhoea in child is: intra venous fluide *ORS antibiotics bowel

A. Over the past 2 days, the incubation period and 2-3 days after lowering temperature

B. All hectic period and 2-3 days after lowering temperatureC. 2-3 days after lowering temperatureD. *Over the past 2 days, the incubation period, all febrile period and 2-3 days after

lowering temperatureE. Over the past 2 days, the incubation period and the hectic period

1112. In which period the maximal growth of infection occurs during epidemic typhus?A. At the incubation periodB. *At the 1th week of illnessC. At the 2nd week of illnessD. At the 3rd week of illnessE. At the time of recovery

1113. Often, in patient with epidemic typhus arise transition petehies in the conjunctivA. What term is used to describe this?

A. Symptom of HellerB. ConjunctivitisC. Symptom of Govorova-GodeleD. *Symptom of Zorohovich-Chiari-AvtsynaE. Enantema Rosenberg‘s

1114. In the family of the patient with epidemic typhus were lice in the children. With the help of any of these measures could prevent the subsequent spread of the disease?

A. *Monitoring and complete sanitation of contact in the centreB. The use of chemoprophylaxisC. The use of antibioticsD. Isolation contactE. Check-up

1115. When can you stop the etiotropic medication treatment of the patient with epidemic typhus?

A. Immediately after the normalization of body temperatureB. After the normalization of the liver and spleenC. *After a 2-day normal body temperatureD. After the disappearance of roseolaE. Within 10 days after the disappearance of roseola

1116. Often, in patient with epidemic typhus arise transition petehies in the conjunctivA. What term did it call?

A.Symptom of HellerB.ConjunctivitisC.Symptom of Govorov-GodeleD.*Symptom of Zorohovich-Chiari-AvtsynaE.Enantema Rosenberg‘s

1117. Often, in patient with epidemic typhus arises petehies on mucosal soft palate. What term did it call?

A.Symptom of HellerB.ConjunctivitisC.Symptom of Govorova-Godele

Page 156: intranet.tdmu.edu.uaintranet.tdmu.edu.ua/data/kafedra/internal/infect_desease... · Web viewBest method to treat diarrhoea in child is: intra venous fluide *ORS antibiotics bowel

D.Symptom of Zorohovich-ChiariE.*Enantema Rosenberg‘s

1118. Often, in patient with epidemic typhus is tongue‘s tremor when protrusion that sticked on the lower teeth. What term did it call?

A.Symptom of HellerB.ConjunctivitisC.*Symptom of Govorova-GodeleD.Symptom of Zorohovich-KiariE.Enantema Rosenberg‘s

1119. In the typical form of typhoid fever, the body temperature rises progressively from day to day 39-40 °C at the end of the 1st week, and during the next 10-14 days it gets back approximately to this level, and then becomes remittent and, gradually goes down to the norm. What is such temperature curve called?

A.*The temperature curve as VunderlihsB.The temperature curve as BotkinC.Temperature curve as KildushevskyD.Temperature curve as EllersE. Intermedium temperature curve

1120. In the typical form of typhoid fever the body temperature rises to 39-40 °C. The temperature curve looks like 2 waves (during 3-4 weeks of disease). What is such temperature curve called?

A.The temperature curve as VunderlihsB.*The temperature curve as BotkinC.Temperature curve as KildushevskyD.Temperature curve as EllersE. Intermedium temperature curve

1121. In the typical form of typhoid fever, the body temperature rises pregressively from day to day to 39-40 °C at the end of the 1st week, and then gradually during 2-3 week it goes down to the norm. What is such temperature curve called?

A.The temperature curve as VunderlihsB.The temperature curve as BotkinC.*Temperature curve as KildushevskyD.Temperature curve as EllersE. Intermedium temperature curve

1122. One methods of diagnostics of typhoid fever and paratyphoid fever is the selection of hemoculture. This is done during the fever period blood is taken from the vein on bilious bulione or Rappaport‘s media in correlation 1:10 is made. What volume of blood must be taken on the 2nd week of disease?

A.5 ml of bloodB.10 ml of bloodC.*15 ml of bloodD.20 ml of bloodE.25 ml of blood

1123. One methods of diagnostics of typhoid fever and paratyphoid fever is the selection of hemoculture. This is done during the fever period blood is taken from the vein

Page 157: intranet.tdmu.edu.uaintranet.tdmu.edu.ua/data/kafedra/internal/infect_desease... · Web viewBest method to treat diarrhoea in child is: intra venous fluide *ORS antibiotics bowel

bilious bulione or Rappaport‘s media in correlation 1:10 is made. What volume of blood must be taken on the 1st week of disease?

A.0,5 ml of bloodB.*10 ml of bloodC.15 ml of bloodD.20 ml of bloodE.25 ml of blood

1124. One methods of diagnostics of typhoid fever and paratyphoid fever is the selection of hemoculture. This is done during the fever period blood is taken from the vein bilious bulione or Rappaport‘s media in correlation 1:10 is made. What volume of blood must be taken on the 3nd week of disease?

A.5 ml of bloodB.10 ml of bloodC.15 ml of bloodD.*20 ml of bloodE.25 ml of blood

1125. When sick people gets epidemic typhus infection, which period affects more?A.Over the past 2 days, the incubation period and 2-3 days after lowering temperatureB.All hectic period and 2-3 days after lowering temperatureC.2-3 days after lowering temperatureD.*Over the past 2 days, the incubation period, all febrile period and 2-3 days after

lowering temperatureE.Over the past 2 days, the incubation period and the hectic period

1126. On which period the maximal growth of infection occurs during epidemic typhus disease?

A.At the incubation periodB.*At the 1th week of illnessC.At the 2nd week of illnessD.At the 3rd week of illnessE.At the time of recovery

1127. One of methods of diagnostics of typhoid fever and paratyphoid fever is the selection of hemoculture. For that in a fever period sowing of blood from a vein on bilious bulione or Rappaport‘s media in correlation 1:10 is made. What volume of blood must be taken on the 2nd week of disease?

A.5 ml of bloodB.10 ml of bloodC.*15 ml of bloodD.20 ml of bloodE.25 ml of blood

1128. One of methods of diagnostics of typhoid fever and paratyphoid fever is the selection of hemoculture. For that in a fever period sowing of blood from a vein on bilious bulione or Rappaport‘s media in correlation 1:10 is made. What volume of blood must be taken on the 1st week of disease?

A.0,5 ml of bloodB.*10 ml of bloodC.15 ml of blood

Page 158: intranet.tdmu.edu.uaintranet.tdmu.edu.ua/data/kafedra/internal/infect_desease... · Web viewBest method to treat diarrhoea in child is: intra venous fluide *ORS antibiotics bowel

D.20 ml of bloodE.25 ml of blood

1129. One of methods of diagnostics of typhoid fever and paratyphoid fever is the selection of hemoculture. For that in a fever period sowing of blood from a vein on bilious bulione or Rappaport‘s media in correlation 1:10 is made. What volume of blood must be taken on the 3nd week of disease?

A.5 ml of bloodB.10 ml of bloodC.15 ml of bloodD.*20 ml of bloodE.25 ml of blood

1130. What is etiology of acute primary tonsillitis?A.Influenza virusB.*β-hemolitic streptococciC.N. meningitidisD.M. tuberculosisE.K. pneumonia

1131. What group of lymph nodes more often change in acute tonsillitis?A.Anterior cervicalB.*SubmandibularC.Posterior cervicalD.RetroauricularE.Occipital

1132. What is most usual complication of acute tonsillitis?A.OtitisB.SinusitisC.*AbscessD.FrontitisE.Pneumonia

1133. Differential diagnosis of acute tonsillitis should perform with?A.Infectious mononucleosisB.DiphtheriaC.Scarlet feverD.Adenoviral infectionE.*All the above

1134. Which group of antibiotics is most effective in treatment of tonsillitis?A.*PenicillinB.TetracyclineC.CephalexinD. SulfalenE.Ampicillin

1135. What is the duration of incubation period of tonsillitis?A.*Few hours – two daysB.Few days – one weekC. Few weeks – one monthD. 1 – 10 hours

Page 159: intranet.tdmu.edu.uaintranet.tdmu.edu.ua/data/kafedra/internal/infect_desease... · Web viewBest method to treat diarrhoea in child is: intra venous fluide *ORS antibiotics bowel

E. 1- 10 days1136. In tonsillitis streptococcus usually cultivated from:

A.BloodB.UrineC.FecesD.*MucousE.Saliva

1137. What group of infectious diseases diphtheria belong to according to L. Gromashevsky classification?

A.Intestinal infection B.*Infection of respiratory tract C.Blood infection D. Infection of external covers E.Transmissive

1138. What is the agent of diphtheria?A.Influenza virusB.β-hemolitic streptococciC.N. meningitidisD. *B. LeffleriE.K. pneumonia

1139. Diphtheria is transmitted by:A.FoodB.WaterC.*AirD.BloodE.Milk

1140. What rear form of diphtheria localization do you know?A. LaryngB.Trachea C.BronchD.*EyeE.Nose

1141. What early complication more often may occur in diphtheria?A.PneumoniaB.*MyocarditisC.GastritisD.OtitisE.Frontitis

1142. What is the agent of infectious mononucleosis?A.*Epstein-Barr virusB.β-hemolitic streptococciC.N. meningitidisD. B. LeffleriE.K. pneumonia

1143. Which of clinical symptoms are more typical for infectious mononucleosis?A.Fever, hepatomegaly, abdominal pain

Page 160: intranet.tdmu.edu.uaintranet.tdmu.edu.ua/data/kafedra/internal/infect_desease... · Web viewBest method to treat diarrhoea in child is: intra venous fluide *ORS antibiotics bowel

B.*Polyadenitis, hepatomegaly, splenomegaly, fever, tonsillitisC.Fever, adenitis, abdominal painD. Dry cough, pain in the throat, high temperatureE.Dysphagia, dyspnoe, wet cough

1144. Specific blood changes in influenza:A.Erythrocytopenia, lymphopenia, monocytosisB.Decreasing of hemoglobin, leucocytosisC.Decreasing of ESR, monocytosis, leucocytosisD.Leucocytosis, increasing of ESR, thrombocytopeniaE.*Leucopenia, lymphocytosis, increasing of ESR

1145. What is forms of adenoviral infection, except:A. Acute respiratory diseaseB.PneumoniaC.Pharyngoconjuctival feverD.ConjuctivatisE.*Adenitis

1146. What group of infectious diseases adenoviral infection belong to according to L. Gromashevsky classification?

A.Intestinal infection B.Infection of respiratory tract C.Blood infection D.*Infection of external covers E.Transmissive

1147. Duration of incubation period of adenoviral infection:A.2-5 daysB.5-7 daysC.1-2 daysD.7-10 daysE.10-15 days

1148. Differential diagnosis of adenoviral infection should perform with?A.Infectious mononucleosisB.DiphtheriaC.Scarlet feverD.Tonsillitis E.*All the above

1149. What is the agent of meningococcal disease?A.Epstein-Barr virusB.β-hemolitic streptococciC.*N. meningitidisD. B. LeffleriE.K. pneumonia

1150. What group of infectious diseases meningococcal disease belong to according to L. Gromashevsky classification?

A.Intestinal infection B.Infection of respiratory tract C.Blood infection

Page 161: intranet.tdmu.edu.uaintranet.tdmu.edu.ua/data/kafedra/internal/infect_desease... · Web viewBest method to treat diarrhoea in child is: intra venous fluide *ORS antibiotics bowel

D.*Infection of external covers E.Transmissive

1151. Which of this symptoms are often present in patients with meningococcol disease?

A. Algor, high temperatura, headacheB.Profuse watery diarrhea, vomiting, dehydration, muscular crampsC.Abdominal pain, diarrhea, constipation, flatulanceD.Headache, dry cough, algorE.*Sore throat, fever, headache, stiff neck, vomiting, confusion

1152. What laboratory test is most helpful in diagnosis of meningococcal disease?A.SerologicalB.BacteriologicalC.*Lumbar punctureD.BiopsyE.X-rays examination

1153. Who is the source of meningococcal disease?A.BirdsB.CattleC.FishD.*HumanE.Pets

1154. Name the agent of epidemic typhus?A. EntamoebaB.ShigellaC.SalmonellaD.*RickettsiaE.Esherihia

1155. What group of infectious diseases epidemic typhus belong to according to L. Gromashevsky classification?

A.Intestinal infection B.Infection of respiratory tract C.*Blood infection D.Infection of external covers E.Transmissive

1156. In what days of epidemic typhus the rash appears?A.2-3B.*4-5C.5-7D.7-10E.10-15

1157. Since what period the serological investigation for epidemic typhus diagnosis can be used?

A.3-5B.*5-7C.7-10D.10-15

Page 162: intranet.tdmu.edu.uaintranet.tdmu.edu.ua/data/kafedra/internal/infect_desease... · Web viewBest method to treat diarrhoea in child is: intra venous fluide *ORS antibiotics bowel

E.15-201158. Most effective etiotropical drug in treatment of epidemic typhus?

A. PenicillinB.*TetracyclineC.CephalexinD. SulfalenE.Ampicillin

1159. What is the agent of malaria?A. PlasmodiumB.ShigellaC.SalmonellaD.*RickettsiaE.Esherihia

1160. What group of infectious diseases malaria belong to according to L. Gromashevsky classification?

A.Intestinal infection B.Infections of respiratory tract C.*Blood infection D.Infection of external covers E.Transmissive

1161. What is specific prophylaxis and supression therapy in malaria:A.*Chloroquine B.CephalexinC.VormilD.DecametoxinE.Laferon

1162. What material should be taken for revealing of Plasmodium malaria?A.UrineB.FecesC.SalivaD.*BloodE.Sputum

1163. Transmissive factor of malaria is:A.WaterB.AirC.*BloodD.FoodE.Saliva

1164. Y. pestis is transmitted by:A. FlyB.*FleaC.TickD.LiceE.Bug

1165. What is the duration of incubation period of plague?A. 3 – 7 days

Page 163: intranet.tdmu.edu.uaintranet.tdmu.edu.ua/data/kafedra/internal/infect_desease... · Web viewBest method to treat diarrhoea in child is: intra venous fluide *ORS antibiotics bowel

B. 2 – 12 daysC.*2 – 10 daysD. 1 – 8 daysE. 3 -15 days

1166. What form of plague is highly fatal?A.SkinB.BubonicC.IntestinalD.*PneumonicE.Septic

1167. What form of plague is most contagious?A.SkinB.BubonicC.IntestinalD.*PneumonicE.Septic

1168. What is the main etiotropical drug of plague treatment?A.PenicillinB.AmoxicillinC.*StreptomycinD.BiseptolE.Cefazolin

1169. What group of infectious diseases plague belong to according to L. Gromashevsky classification?

A.Intestinal B.Respiratory tract C.*Blood D.Infection of external covers E.Transmissive

1170. Drugs of choice for the treatment of amoebae cyst carrier:

A.Monomycinum

B.Delagilum

C.Tetracyclin

D.* Furamid

E.Ursosan

1171. Principles of treatment of patients with shigellosis.

A.Diet

B.Antibacterial drugs

C.Correction of dysbacteriosis

D.Detoxication therapy

Page 164: intranet.tdmu.edu.uaintranet.tdmu.edu.ua/data/kafedra/internal/infect_desease... · Web viewBest method to treat diarrhoea in child is: intra venous fluide *ORS antibiotics bowel

E.*All the above

1172. Largest nematode parasite is:A.Ascaris lumbricoidesB.Necator americanusC.Ancilostoma duodenumD.*Dracunculus medinensisE.Trichinella spiralis

1173. Which of the following nematodes is ovoviviparous?A.Ascaris lumbricoidesB.*Strongiloides stercoralisC.Ancilostoma duodenumD.Dracunculus medinensisE.Trichinella spiralis

1174. Which of the following nematodeslays unsegmented eggs?A.Ascaris lumbricoidesB.Strongiloides stercoralisC.Ancilostoma duodenumD.*Trichuris trichiuraE.Trichinella spiralis

1175. Which of the following nematodes is ovoviviparous?A.Ascaris lumbricoidesB.*Strongiloides stercoralisC.Ancilostoma duodenumD.Dracunculus medinensisE.Trichinella spiralis

1176. Best site for taking biopsy for diagnosis of trichinellosis is:A.*Deltoid muscleB.DiaphragmC.Pectoralis majorD.LiverE.Spleen

1177. Rectal prolapse is seen in infection with:A.Ascaris lumbricoidesB.Strongiloides stercoralisC.Ancilostoma duodenumD.Dracunculus medinensisE.*Tricuris trichiura

1178. Disseminated systemic infection in AIDS patients is seen with:A.Ascaris lumbricoidesB.*Strongiloides stercoralisC.Ancilostoma duodenumD.Dracunculus medinensisE.Trichinella spiralis

1179. “Larva currens” is the name given to the migranting larvae of:

Page 165: intranet.tdmu.edu.uaintranet.tdmu.edu.ua/data/kafedra/internal/infect_desease... · Web viewBest method to treat diarrhoea in child is: intra venous fluide *ORS antibiotics bowel

A.Ascaris lumbricoidesB.*Strongiloides stercoralisC.Ancilostoma duodenumD.Dracunculus medinensisE.Trichinella spiralis

1180. Which of the following nematodes does not pass through lungs during its life cycle?

A.Ascaris lumbricoidesB.Strongiloides stercoralisC.*Tricuris trichiura D.Dracunculus medinensisE.Trichinella spiralis

1181. All the following parasites may cause B12 deficiency anaemia:A.*Diphyllobothrium latumB.Strongiloides stercoralisC.Tricuris trichiura D.Dracunculus medinensisE.Trichinella spiralis

1182. Eggs are passed in sputum in case of infection with: A.Clonorchis sinensis B.*Paragonimus westermaniC.Ascaris lumbricoidesD.Strongiloides stercoralisE.Trichinella spiralis

1183. Which of the following is not a hepatic trematode?A.Fasciola hepaticaB.*Fasciolopsis buskiC.Clonorchis sinensisD.Schistosoma haematobiumE.All of the above

1184. Largest trematode is:A.Fasciola hepaticaB.*Fasciolopsis buskiC.Clonorchis sinensisD.Schistosoma haematobiumE.Opisthorchis felineus

1185. Gynaecophoric canal is seen in case of male worm of:A.Ascaris lumbricoidesB.Fasciolopsis buskiC.Clonorchis sinensisD.*Schistosoma mansoniE.Opisthorchis felineus

1186. Pseudotubercurcles may be formed around the eggs of:A.*Schistosoma haematobiumB.Fasciolopsis buskiC.Clonorchis sinensis

Page 166: intranet.tdmu.edu.uaintranet.tdmu.edu.ua/data/kafedra/internal/infect_desease... · Web viewBest method to treat diarrhoea in child is: intra venous fluide *ORS antibiotics bowel

D.Schistosoma mansoniE.Opisthorchis felineus

1187. Carcinoma of urinary bladder is associated with which of the following parasites?

A.Schistosoma japonicum B.*Schistosoma haematobiumC.Clonorchis sinensisD.Schistosoma mansoniE.Opisthorchis felineus

1188. Flame cells are the organ of excretion in:A.*TrematodesB.NematodesC.CestodesD.PectodesE.All of the above

1189. Convulsive seizures may be seen in infection with:A.Trypanosoma brucei gambienseB.Negleria fowleriC.*Paragonimus westermaniD.Schistosoma mansoniE.Opisthorchis felineus

1190. Cercaria is the infective stage of:A.Schistosoma japonicum B.*Schistosoma haematobiumC.Clonorchis sinensisD.Schistosoma mansoniE.Opisthorchis felineus

1191. Which of the following acts as the main reservoir of Balantidium coli infection in human beings?

A.ManB.MonkeyC.* PigD.CowE.Dog

1192. Which of the following is the largest protozoal parasite inhabiting the large intestine of man?

A.Entamoeba histoliticaB.Entamoeba coliC.*Balantidium coliD.Giardia lambliaE.All of the above

1193. Which of the following is characteristic of Balantidium coli trophozoite?A.Body covered with short ciliaB.Two nucleiC.Two contractive vacuolesD.Numerous food vacuoles

Page 167: intranet.tdmu.edu.uaintranet.tdmu.edu.ua/data/kafedra/internal/infect_desease... · Web viewBest method to treat diarrhoea in child is: intra venous fluide *ORS antibiotics bowel

E.All of the above1194. Toxoplasma gondii lives inside the:

A.Lumen of small intestineB.Lumen of large intestineC.*Reticuloendothelial cells D.Red blood cellsE.White blood cells

1195. During which trimestr pregnancy infection with Toxoplasma gondii is more severy?

A.*FirstB.SecondC.ThirdD.Second and thirdE.First and second

1196. Commonest manifestation of postnatally acquired infection with Toxoplasma gondii is:

A.* LymphadenopathyB.PneumonitisC.MyocarditisD.MeningoencephalitisE.Bronchitis

1197. Drug used for the treatment of toxoplasmosis is:A.PyrimethamineB.SulphadiazineC.SpiramycinD.ClindamycinE.*All of the above

1198. Which is the infective form of the malaria parasite?A.TrophosoiteB.SchizontC.MerozoiteD.*SporozoiteE.All of the above

1199. Which of the following phases of malaria parasites brings on a clinical attack of malaria?

A.Primary exoerythrocytic schizogonyB.*Erythrocytic schizogonyC.GametogonyD.SporogonyE.Secondary exoerythrocytic schizogony

1200. Resting stage of the malaria parasite is known as:A.TrophosoiteB.SchizontC.*HypnozoiteD.MerozoiteE.Sporozoite

Page 168: intranet.tdmu.edu.uaintranet.tdmu.edu.ua/data/kafedra/internal/infect_desease... · Web viewBest method to treat diarrhoea in child is: intra venous fluide *ORS antibiotics bowel
Page 169: intranet.tdmu.edu.uaintranet.tdmu.edu.ua/data/kafedra/internal/infect_desease... · Web viewBest method to treat diarrhoea in child is: intra venous fluide *ORS antibiotics bowel

Ситуаційні Задачі1. For a man 25 years old, half-year ago there was a positive reaction on protien. Last 3

months are complaint with general weakness, fatigueability, somnolence, chest pain, during last 2 weeks developed anxiety, fear, depression, disorders of memory and aphasia, untidiness appeared 5 days ago. Set a diagnosis.

a. Dementia of AIDS b. * Patient has еncephalopathy (AIDS-related complex) c. Organic psychosis d. Anxiously depressed syndrome for HIV infection e. Somatoform depression

2. For a man 30 years old, it was discovered positive reaction on HIV half-year ago. Last 3 months complaints of general weakness, fatigueability, somnolence, chest pain. Last 2 weeks with anxiety, fear, depression, disorders of memory and aphasia, untidiness appeared 5 days ago. What is the treatment?

a. Acylovir b. Cerebroprotector c. Antidepressants d. Neuroleptic e. * Zidovudin, didanosin

3. For a patient 35 years old, after 4-months of treatment by isoniaside - аdynamia, icterus, pain in right hypochondric appeared. The liver is megascopic. In blood activity of enzymes of AlAT is increased in three times, AsAT in two times. Bilirubin of blood - 122 mcmol/l (conjugated - 82, unconjugated - 40). НBs-аntigen is not found out. What is the diagnos?

a. Calculary cholecystitis b. Hepatocirrhosis c. Acute viral hepatitis d. Chronic active hepatitis e. * Toxic hepatitis

4. HIV positive patient, 28 years old, drug addict, complaints of dyspnea, unproductive cough, fever with 37,5 °C for 2 months.Objectively: skin is pale, hyposthenic breathing in lungs, especially in lower lobe, short breathing 24 per min. Roentgenologic chromophilic bilateral infiltrations as “wings of butterfly”. What is the most effective treatment:

a. Antibacterial preparations. Inhibitors of transcriptase b. Inhibitors of proteases. Inhibitors of transcriptase c. Transplantation of marrow. Inhibitors of transcriptase d. Antiviral gamma-globulin. Vitamins of A, C groups. Inhibitors of proteases e. * Inhibitors of proteases. Inhibitors of transcriptase. Antibacterial drugs

5. In 2 months after returning from India, where she often drank unboiled water; the 23-year-old pregnant woman felt nauseated, strong general weakness, head acke appeared. Later the temperature of body rose to 38,6 °C, which stuck to within a week. icterus appeared on the 6th day, the general condition continued to worsen. On the 12th day of illness the general condition became severe. EuphoriA. Vomiting at night. Complete fastidium. Bright icterus, signs of hemorragic syndrome, tachycardiA. BP 110/60 mm Hg, body temperature 37,8 °C. The liver is insignificantly megascopic, soft, painfull, spleen +2 cm

Page 170: intranet.tdmu.edu.uaintranet.tdmu.edu.ua/data/kafedra/internal/infect_desease... · Web viewBest method to treat diarrhoea in child is: intra venous fluide *ORS antibiotics bowel

There is neutrophilic leykocytosis in general blood analysis. General bilirubinum of blood up to 570, to the line - 300, activity of ALAT is enhanceable in 100 times, timol test of 26 units., urea - 2,1 mmol/l. With most probability for a patient:

6. Man 30 years old, drug addict, takes drugs intravenously. Has been taking drugs for 12 years Complaints of weakness, moderate icterus, weight in right subcostal region. The state became gradually worse. Biochemical indexes: general bilirubin 28,2 mcmol/l; ALAT 1,0, ASAT 0,8 . Will you define a diagnostic method which is expedient to conduct for establishment of etiologic diagnosis?

a. Biopsy of liver b. * Polymerase chain reaction (PCR) c. Enzymes of liver d. Computer tomography e. Immunological tests

7. On a background of prolonged treatment for an HIV patient, appeared ulcer on the mucous of mouth. During objective examination, the doctor found erosions on the mucous, hyperemic and filling out mucous of oral cavity, tongue without raid, with a smooth surface. About what complication is possible to think?

a. Leptospirosis b. Acute herpes c. Stevens-Johnson syndrome d. * Candidos stomatitis e. Layel syndrome

8. Patient P., 21 years old, complaints of diarhhea that lasts for one and a half months, changes sometimes, on emptying there are admixtures of blood and mucus, 13 kg loss of body mass, weakness, subfebrile temperature of body, recurrent herpes. It is discovered generalised lymphadenopathy, increase of liver size by 2 cm. Blood test: Er 4,4*1012 g/l, Hb 115 g/l, ESR - 15 mm/hr, L 10,0*109 g/l, е 2 %, b 6 %, n 61 %, s 17 %, m 3 %, atypical mononuclears 6 %. What is most credible diagnosis?

a. Shigellosis b. * AIDS c. Infectious mononucleosis d. Salmonellosis e. Ameobiasis

9. Sick O., 25 years old, complaints of considerable fatigue, fever up to 39 °C profuse sweating, lethargy. Objectively: all groups of peripheral lymphatic node are enlarged, candidiasis of oral cavity, herpetic pouring out with megascopic on lips. In the global analysis of blood: leucocytes 3,3*109 g/l, ESR – 15 mm/hr, B-lymphocyte 12 %, a decline of CD4 to 600/mm. What is most credible diagnosis is?

a. Herpetic infection b. Malaria c. Urogenital chlamydia d. * HIV/AIDS e. Acute brucellosis

10. Student L, 20 years old, complaints of considerable fatigue, fever up to 39 °C profuse sweating, lethargy. Objectively: all groups of peripheral lymphatic node are enlarged, candidiasis of oral cavity, herpetic pouring out with megascopic on lips. In the global

Page 171: intranet.tdmu.edu.uaintranet.tdmu.edu.ua/data/kafedra/internal/infect_desease... · Web viewBest method to treat diarrhoea in child is: intra venous fluide *ORS antibiotics bowel

analysis of blood: leucocytes of 6,2*109 g/l, ESR 15 mm/hr, B-lymp 12 %. What is most credible diagnosis is?

a. Megakaryoblastoma b. Malaria c. * HIV/AIDS d. Urogenital chlamydiasis e. Brucellosis

11. Patient T., 35 years old, appeared to the doctor on the 8th day of gradual development of illness with complaints of general weakness, rapid fatigue, dark color of urine. In the morning noticed an icterus. On examination temperature of body 36,8 °C. Found out the increasing of liver +3 sm. The changes of what laboratory index most informing at this illness?

a. *AlAT b. Hemodiastases c. Protrombin index d. Cholesterol e. Alkaline phosphatase

12. The sick complaints of a prolonged cough, more than half-year, rising temperature of body to 38 °C, enlargement of peripheral lymphnode, frequent herpetic wide-spread pouring out with considerable lowering of body mass. In іmmunogram correlation of T-helper to T-suppression is 0,3. It takes place because the infestant:

a. Infects cells with the receptors of CD22 b. Induces proliferation of Т-helpers c. Induces proliferation of T-suppressors d. Infects cells from receptor CD8 e. * Infects cells from receptor CD4

13. HIV-positive patients, 28 years old, a former injecting drug user, complained of shortness of breath, nonproductive cough, temperature rise up to 37,5 °C for 2 months. Objective: pale skin, lungs auscultated respiratory depression, especially in lowback, dyspnea 24 for 1 min. Radiographically bilateral infiltrates in the form of “butterfly wings”. Choose the most effective approaches to treatment:

a. Antbacterial drugs. Transcriptase inhibitorsb. Protease inhibitors. Transcriptase inhibitorc. Bone marrow transplant. Transcriptase inhibitorsd. Antiviral gamma globulin. Vitamins B, C. Protease Inhibitors

E.* Protease inhibitors. Transcriptase inhibitors. Antibacterial drugs14. During influenza epidemic, a patient in the hospital arrived 43 years with complaints of

fever above 38 °C, malaise, in the analysis of blood on the outpatient phase of the survey neutrophilic leukocytosis 12.0 * 109 g/L, a shift to the left, ESR 50 mm / hour. As it turned out, a fever with a few interruptions lasted for about 3 months, then lost 13 kg in body weight with normal appetite. Catarrhal symptoms are minor. Liver to 2 cm below the costal arch. Other abnormalities are not detected. What kind of disease should think of first?

a. Flub. Pneumoniac. Chronic hepatitis in the acute stage

Page 172: intranet.tdmu.edu.uaintranet.tdmu.edu.ua/data/kafedra/internal/infect_desease... · Web viewBest method to treat diarrhoea in child is: intra venous fluide *ORS antibiotics bowel

d. HIV infectione. *AIDS

15. Patient was taken to a hospital after car accident in critical condition, caused by shock, bleeding. Before emergency blood transfusion in order to prevent HIV transmission everybody should consider:

a. Health reasonsb. The conclusion of the panel of doctorsc. Agree of the patient (or his family)d. The results of a survey of emergency donor HIV (IHA)e. *All the above listed

16. 33 years patient comes in with complaints of fever above 38 °C, and general malaise. As it turned out, a fever with a few interruptions lasted for about 2 months, then lost of body weight to 16 kg. Periodically bowel dysfunction. Palpable enlarged lymph nodes in the axillary regions on both sides of the neck. Other abnormalities are not detected. What changes would expect to find in the study of blood?

a. Anemiab. Leukopeniac. Hypolymphemiad. Thrombocytopeniae. *All the above listed

17. 33 years patient with complaints of fever above 38 °C, and general malaise. As it turned out, a fever with a few interruptions lasted for about 2 months, then lost of body weight to 16 kg. Periodically bowel dysfunction. Palpable enlarged lymph nodes in the axillary regions on both sides of the neck. Other abnormalities are not detected. What changes would expect to find in the study of blood?

a. Leukopeniab. Decrease the number of CD4c. Decrease immunoregulatory indexd. Increase ESRe. *All the above listed

18. 40 years sick with complaints of general weakness, sweating, wasting a 12 kg, often long-term diarrhea and recurrent respiratory infections. Objectively: generalized lymphadenopathy, oral candidiasis, fecal mucus and blood. B blood reduced the number of T-helper cells, T4/T8 less than 0.5. What is the most likely diagnosis?

a. Dysenteryb. Amebiazc. *AIDS

Page 173: intranet.tdmu.edu.uaintranet.tdmu.edu.ua/data/kafedra/internal/infect_desease... · Web viewBest method to treat diarrhoea in child is: intra venous fluide *ORS antibiotics bowel

d. Tuberculosise. Cytomegalovirus infection

19. Student 20 years during the month has been concerned a significant fatigue, increased body temperature 39 °C, profuse perspiration, emaciation. Objective: increasing of all peripheral lymph nodes, oral candidiasis, herpes on lips. General blood analysis: leukocytes 3.3 x 109 / l, ESR 15 mm/hour, B-lymphocytes 12%. What additional test is helpful in diagnosis?

a. Biological assay in guinea pigsb. Determination of the number of T-lymphocytesc. *Determination by IFA antibodies in response to antigens or RIA HIVd. Burne teste. Leukocyte migration inhibition test

20. Patient 18 years intravenous drugs user, complaints of a cough lasting more than 4 months, Increasing of body temperature to 38 °C, treatment of “interstitial pneumonia” is not effective. There have been several episodes of widespread herpes, decreasing of body weight. The study will identify all immunograms authentic:

a. Increasing of T4-helper lymphocytesb. Increase immunoregulatory index T4/T8c. *Decrease the number of T4-helper lymphocytesd. Increase the absolute number of T lymphocytese. Improving performance of delayed-type hypersensitivity

21. Patient 18 years intravenous drugs user, complaints of a cough lasting more than 4 months, Increasing of body temperature to 38 °C, treatment of “interstitial pneumonia” is not effective. There have been several episodes of widespread herpes, decreasing of body weight. The study will identify all immunograms authentic:

a. Increasing of T4-helper lymphocytesb. Increase immunoregulatory index T4/T8c. *Reducing the number of T4-helper lymphocytesd. Increase the absolute number of T lymphocytese. Improving performance of delayed-type hypersensitivity

22. Patient 23 years old, fell ill at the end of summer, when the temperature rose to 37.2 °C, mild headache, and weakness. Up to 7 days of illness treated at the outpatient ARI. Condition worsened, admitted to hospital. The body temperature of 40 °C, a pale, severe weakness. Pulse 96 per 1 min, blood pressure 110 and 70 mm Hg. Tongue with imprints of teeth, the abdomen is soft, swollen, hepatosplenomegaly. Constipation. There is congestion and hyperplasia of the tonsils, on the surface of the right - the ulcer. What disease we may think about?

a. Vincent-Simanovsky tonsillitisb. Herpangina

Page 174: intranet.tdmu.edu.uaintranet.tdmu.edu.ua/data/kafedra/internal/infect_desease... · Web viewBest method to treat diarrhoea in child is: intra venous fluide *ORS antibiotics bowel

c. *Necrotizing tonsillitisd. Kissing diseasee. Angina Duge

23. Patient 30 years after exposure to the earnings to consult a dermatologist complaining of rashes around the skin surface. Over the last three months have seen a sharp emaciation, weakness, constant low-grade fever. Presens what disease we will exam the patient?

a. Syphilisb. TB infectionc. *AIDSd. CMV infectione. Malignant neoplasms of the skin

24. Patient B., 28, a prostitute, 2 years living in the Middle East. Admitted to hospital with complaints of weakness, cough, sputum gray, burning in the chest, shortness of breath, night chills, fever, sweating, pain in the joints. Patients lost of weight near 12 kg during 5 months, decreased appetite. On examination: the soft palate, temples, back of the throat covered by white raids. Lymphadenopathy. Hepatosplenomegaly. In the blood: anemia, leukocytosis, shift formula to myelocytes, lymphopenia, ESR 60 mm/hour. Your diagnosis?

a. Visceral leishmaniasisb. *HIV infection. Pneumocyst pneumoniA. Esophageal candidiasisc. Brucellosisd. Lymphogranulomatosise. Esophageal carcinoma

25. Patient K., 29 years old, not vaccinated, complaints of the increasing temperature to 37,2 °C, headache, weakness, pain in the throat which increase with swallowing, acute disease began two days ago. Objectively: pale skin. Pulse 110 for 1 min. Heart sounds are muffled, especially the first one. Gentle systolic murmur. AP 100 and 65 mm Hg. Tonsils, soft palate, uvula swollen. On the surface of the tonsils, more to the right, pale-gray film, which spreads their borders, removed hard, dense. The regional lymph nodes are enlarged. Subcutaneous edema, which reaches to the middle of the neck. Specify the most likely diagnosis:

a. *Diphtheria oropharynxb. Anginac. Kissing diseased. Paratonsillar abscesse. Acute leukemia

26. A patient, 16 y.o., complaints of general weakness, fever, sore throat. Objectively: oropharyngeal mucosa bright red, the tonsils - white raids are removed easily, an increasing of all groups of lymph nodes, 1-3 cm in diameter, firm, elastic maloboleznennye, not welded together. The liver is enlarged by 3 cm, the spleen - 1 cm in blood leukocytosis, atypical mononuclear cells – 20 %. What is previous diagnosis?

a. *Infectious mononucleosisb. Acute lymphocytic leukemiac. Acute streptococcal tonsillitisd. Diphtheriae. Adenovirul infection

Page 175: intranet.tdmu.edu.uaintranet.tdmu.edu.ua/data/kafedra/internal/infect_desease... · Web viewBest method to treat diarrhoea in child is: intra venous fluide *ORS antibiotics bowel

27. Patient G., 24, complaines of sore throat, fatigue. Temperature 38 °C. Pulse 96 per minute, flushing of the skin, hyperemia of mucous membranes of oropharynx. Enlargment of tonsills, friable. Palpable enlargement submandibular lymph nodes. Spleen + 1 cm. What method can confirm the diagnosis?

a. Paul-Bunnel, Wright-Heddlson, Hoff-Bauer reactionsb. *Paul-Bunnel, Lovrika-Volner, Hoff-Bauer reactionsc. Paul-Bunnel, Wright-Heddlson, Lovrika-Volner reactionsd. Paul-Bunnel, Burne, Hoff-Bauer reactionse. Paul-Bunnel, Hoff-Bauer, Wright-Heddlson, Lovrika-Volner reactions

28. A patient 15 years old, was admitted to the hospital on the third day of illness with complaints of pain in the throat when swallowing, fever. The disease is linked to the use of raw molokA. On examination: body temperature of 38 °C, hyperemic pharynx, enlargement of right tonsill, swollen, with necrotising layering grayish-white color. Right neck lymph node on the size like a hen’s egg, mild painful, with clear contours, the skin over it is not changed. What is preliminary diagnosis?

a. Lacunar tonsillitisb. Diphtheria of oropharynxc. Vincent-Simanovsky tonsillitisd. Duge-Strumpell tonsillitise. *Tonsilar-bubonic form of tularemia

29. The patient became ill 17 years after acute hypothermia: 39,5 ° C fever, headache and muscle pain, sharp pain in the throat when swallowing, aching pain. Hypertrophy of tonsils, bright red, in the gaps of the purulent deposit. Diagnosed as lacunar tonsillitis. What is most effective laboratory method of investigation in this case?

a. Biochemical bloodb. Microscopic examination of the pus gapsc. Immunological study of bloodd. Biological research methode. *Bacteriological study of the content of gaps

30. 28 years old patient complaints of increasing of lymph nodes during six months, progressive weakness, sweating, recurrent fevers to 38 °C. During several years has used drugs intravenously. Malnutrition, widespread seborrheic dermatitis, painless palpable enlarged cervical, axillary and inguinal lymph nodes of up to 2-2.5 cm What research is to assign at first?

a. Lymph node biopsyb. Blood culturesc. *Blood test for antibodies to HIVd. Sternum puncturee. Examination of blood for antibodies to the Epstein-Barr virus

31. A patient 30 years old, was taken to the infectious department by ambulance in serious condition. On examination: the temperature of 38,9 °C, the patient is exhausted, lethargic, pale skin, in the peripheral vein - injection marks. In the mouth - a lot of carious teeth, white attacks on the mucosA. By an increase in the axillary, inguinal, supraclavicular lymph nodes, not painful, not welded to the underlying fiber. In the lungs - breathing weakened, respiratory rate 30 per minute., Pulse 92 / Min., AD 100 and 65 mm Hg. Heart

Page 176: intranet.tdmu.edu.uaintranet.tdmu.edu.ua/data/kafedra/internal/infect_desease... · Web viewBest method to treat diarrhoea in child is: intra venous fluide *ORS antibiotics bowel

sound muffled, rhythmic, hepatosplenomegaly. In history - injecting drug user for 3 years. What is your diagnosis?

a. Toxic hepatitisb. Sepsisc. Lymphogranulomatosisd. Brucellosise. *AIDS

32. A patient B., 18 years old, was admitted to hospital with complaints of headache, weakness, fever up to 37,5-38,2 °C for 6 days, the pain in the throat. Objective: increasing of all lymph nodes, 1-3 cm in diameter, flexible, not painfull, not welded together. The liver is enlarged by 3 cm, the spleen - 1 cm in blood leukocytosis, virotsity - 15%. Likely diagnosis?

a. Tonsillitisb. Adenovirus infectionc. *Infectious mononucleosisd. Diphtheriae. Acute lymphocytic leukemia

33. Patient B., 22 years old, desperately ill, mild pain in the throat during swallowing, nasal voice. Objectively: the left tonsil and the arc is filmy coating. Diagnosed with lacunar tonsillitis, appointed penicillin and rinse 2% soda solution. Till the second day covers spread on the soft palate and uvulA. Swelling of the neck on to the collar bone, nasal voice. Muffled heart sounds, pulse 90 per minute. AP 95 and 65 mm Hg. What is the most expedient treatment in this case?

a. Penicillin and diphtheria toxoidb. Rinse 2% soda solution in combination with erythromycin peroral c. Autohaemotherapy, hot compress and quartz tubed. Disclosure and drainage paratonsillar fibere. *Antitoxic diphtheria serum and benzylpenicillin

34. Patient B., 34 years old, complained of fever, headache, aching joints, fever, sore throat, worse when swallowing. Objectively: flushing of the oropharynx, tonsils hyperemic, hypertrophied, on both - the necrotic areas of dark gray color, which was formed after the delamination defect deep mucosa with bumpy bottom, enlarged and painful submandibular lymph nodes. Put the diagnosis?

a. Diphtheriab. Tonsillitisc. Lacunar tonsillitisd. Vincent's anginae. *Necrotizing tonsillitis

35. C. become sick gradually. There was general weakness, fatigue, sore throat, abdominal pain, and nauseA. Hospitalized on the 5th day of illness. Examination: body temperature 38 ° C, increased cervical, and submandibular lymph nodes. Skin or eyes subikterichny. There have been isolated maculo-papular rash all over body. The deposits on the tonsils, white-yellow, tongue coated with white covering, moderately distended abdomen, hepatosplenomegaly. In blood leukocytosis, neutrophilic left shift, atypical mononuclear cells 10 %, 10 % plasma cells. What is the preliminary diagnosis?

a. Viral hepatitis A

Page 177: intranet.tdmu.edu.uaintranet.tdmu.edu.ua/data/kafedra/internal/infect_desease... · Web viewBest method to treat diarrhoea in child is: intra venous fluide *ORS antibiotics bowel

b. *Infectious mononucleosisc. Typhus, paratyphoid diseasesd. Yersiniosise. Lacunar tonsillitis

36. The patient, age 20, complained of a sore throat when swallowing, weakness and headache on the 2nd day of the disease. OBJECTIVE: Temperature 39,0 ° C, the pulse 110 for 1 min, oropharyngeal mucosa hyperemic, tonsils enlarged in size, loose, covered with plaque that is easily removed, the surface after removal of the plaque does not bleed. What kind of disease is necessary to think about?

a. Tularemiab. Diphtheria oropharynxc. -Vincent Angina Simanovskyd. Kissing diseasee. * Acute streptococcal tonsillitis

37. In infectious hospital, patients who present with acute onset of disease, temperature of 39,9 °C, mild sore throat, swelling, and redness with slight cyanosis of the mucous membranes of the oropharynx, the tonsils are dense, shiny, grayish deposits in the form of a continuous film, which hard to remove, exposing a bleeding surface. Submandibular lymph nodes were moderately enlarged. The patient should have what immediately performed:

a. Swabs from the tonsils, nose or other areas to identify diphtheria bacilli (culture)

b. ELISAc. * Microscopy (Neisser staining)d. Blood culturee. Serology (RPHA with diphtheria diagnostic tools)

38. HIV-positive patient, 28 years old, a former injecting drug user, complained of shortness of breath, nonproductive cough, temperature rise to 37.5 °C for 2 months. OBJECTIVE: pale skin, lungs auscultated respiratory depression, especially in the lower parts of the posterolateral, dyspnea 24 for 1 min. Radiographically bilateral infiltrates in the form of "butterfly wings." Choose the most effective approaches to treatment:

a. Antibacterials. Transcriptase inhibitorsb. Protease inhibitors. Transcriptase inhibitorsc. Bone marrow transplant. Transcriptase inhibitorsd. Antiviral gamma globulin. Vitamins B, C. Protease inhibitorse. * Protease inhibitors. Transcriptase inhibitors. Antibacterials

39. The 40 year-old driver is sick for two months - general weakness, sweating, lost 12 kg, often long-term diarrhea and recurrent respiratory infections. OBJECTIVE: generalized lymphadenopathy, oral candidiasis, fecal mucus and blood. In the blood, reducing the number of T-helper cells, T4/T8 less than 0.5. What is the most likely diagnosis?

a. Dysenteryb. Amebiasisc. * AIDSd. Tuberculosise. Cytomegalovirus infection

Page 178: intranet.tdmu.edu.uaintranet.tdmu.edu.ua/data/kafedra/internal/infect_desease... · Web viewBest method to treat diarrhoea in child is: intra venous fluide *ORS antibiotics bowel

40. Gynecologist, during the operation emergency of a young woman accidentally hurt himself with a scalpel. After 3 days, a reply came from the laboratory with positive antibodies to HIV in the blood of a patient taken before surgery. Priority actions in respect of the operated physician:

a. Debridementb. Immediate laboratory testing for HIVc. Post-exposure prophylaxis with antiretroviral drugs by the results of

laboratory testing for HIVd. * post-exposure prophylaxis with antiretroviral drugse. Observation

41. Girl 8 years old, was admitted to hospital with complaints of general weakness, fever, sore throat. OBJECTIVE: oropharyngeal mucosa is bright red, the tonsils have white plaques which are removed easily, shows an increase in all groups of lymph nodes, 1-3 cm in diameter, firm, elastic slightly painful, not grouped together. The liver is increased by 3 cm, the spleen 1 cm in the blood - leukocytosis, plasma cells 20%. Likely diagnosis?

a. Acute lymphocytic leukemiab. * Infectious mononucleosisc. Anginad. Diphtheriae. Adenovirus infection

42. Female 24 years old, went to the doctor due to prolonged fever, night sweats. Over the past three months lost 7 kg. Was promiscuity. An objective study found an increase in all groups of lymph nodes, hepatoileal syndrome. Leukocytes-. 2.2 *109 / l. What disease should be suspected?

a. Hroniosepsisb. Lymphogranulomatosisc. Tuberculosisd. Kissing diseasee. * HIV infection

43. Woman, 28 years old, complaining of general weakness, headache, fever up to 39-40 ° C, sore throat for 3-days. Her husband has strep throat. OBJECTIVE: pale skin, lips cyanotic. Hyperemia of oropharynx with cyanosis, swelling of tongue, palatine arches, tonsils. On the surface of the tonsils - solid thick white with pearl shade plaque, which are difficult to remove with a spatula, after removing plaque, mucosa bleeds. Enlarged submandibular lymph nodes. Swelling of the neck. TachycardiA. AP 105 and 65 mm Hg.. What is the most likely diagnosis?

a. Acute leukemiab. Anginac. Kissing diseased. * Diphtheria oropharynxe. Adenovirus infection

44. Patient N., 23, was hospitalized in infectious department and diagnosed with oropharyngeal diphtheriA. On examination, the tonsils of the throat have solid thick white with pearl shade plaque, which are difficult to remove with a spatula, after removing plaque, mucosa bleeds. When can the patient be discharged?

a. After the disappearance of clinical symptoms

Page 179: intranet.tdmu.edu.uaintranet.tdmu.edu.ua/data/kafedra/internal/infect_desease... · Web viewBest method to treat diarrhoea in child is: intra venous fluide *ORS antibiotics bowel

b. After the disappearance of clinical symptoms and get a negative culture results mucus tonsils and nasal diphtheria at two-day intervals

c. After the disappearance of clinical symptoms and get negative results of two bacteriology mucus tonsils and nasal diphtheria at a three-day intervals

d. After the disappearance of clinical symptoms of diphtheria and get three negative results of bacteriological studies of mucus from the nose to the tonsils and diphtheria three-day intervals

e. * After the disappearance of clinical symptoms of diphtheria and get negative results of two bacteriology mucus tonsils and nasal diphtheria at two-day intervals

45. Patient K., 40 years old, complained of fever, night sweats, rapid weight loss, muscle aches, sore throat, joint pain, photophobia, recurrent diarrheA. On examination revealed generalized lymphadenopathy. Most likely the following diseases:

a. Lymphogranulomatosisb. Tumor of the digestive systemc. * AIDSd. Tuberculosise. Salmonellosis

46. Patient received a blood transfusion from an unverified voluntary donor. Remains of blood sent to the station of blood transfusion, where the examination of the donor antibodies to HIV. Your actions for recipient:

a. Observationb. laboratory testing for HIVc. Emergency antibioticd. * Post-exposure prophylaxis with antiretroviral drugse. all the above

47. After months of unsuccessful treatment for pneumonia in a 7-month-old baby antibodies to HIV was revealed. During pregnancy, the mother of the child was examined twice by HIV with negative results, the last at 24 weeks of pregnancy. Childbirth was without complications. Baby breastfeeding. What is the most likely route of HIV infection of the child:

a. HIV-infected motherb. Vaginall birthc. Breast-feedingd. Parental interventions after birthe. * All the above

48. After months of unsuccessful treatment for pneumonia in a 7-month-old baby antibodies to HIV was revealed. During pregnancy, the mother of the child was examined twice by HIV with negative results, the last at 24 weeks of pregnancy. Childbirth was without complications. Baby breastfeeding. What is the most likely route of HIV infection of the child:

a. * HIV infection of the motherb. Laboratory examination of the childc. Parental interventions during treatment of pneumoniad. AIDS of the father

Page 180: intranet.tdmu.edu.uaintranet.tdmu.edu.ua/data/kafedra/internal/infect_desease... · Web viewBest method to treat diarrhoea in child is: intra venous fluide *ORS antibiotics bowel

e. All variants are possible49. . 7 -month old child with pneumonia had antibodies to HIV. During pregnancy, the mother

of the child was examined twice by HIV with negative results, the last at 24 weeks of pregnancy. What is possible reasons of HIV of the mother:

a. * HIV infection during pregnancyb. Presence of AIDS of the fatherc. Infection during care of a sick childd. Contamination during breast-feedinge. All variants are possible

50. 7-month old child with pneumonia had antibodies to HIV. During pregnancy, the mother of the child was examined twice by HIV with negative results, the last at 24 weeks of pregnancy. What terms of final investigation of mother for presence of HIV infection:

a. Negative screening result in the first trimesterb. Negative screening result in the II trimesterc. * A negative screening result after 28 weeksd. Negative screening result in I and II trimesterse. All variants are possible

51. . Patient 37 years old, slower muscle and motor responses during 3 monthes have been developed, impaired memory for names, addresses, disrupting cognitive function, there was stiffness, drowsiness, sloppiness, indifference to others and to his state. Loss of weight 12 kg. Intermittent fever. Generalized lymphadenopathy. What is clinical diagnosis.

a. Tuberculosisb. Herpes encephalitisc. * AIDSd. Syphilise. Lymphocytic Leukemia

52. 12 years old boy with catarrhal symptoms observed an increase of all groups of lymph nodes, injection of sclera, hyperplasia of the tonsills with white loose build-up on them in the form of islands, there are single roseolous-papular rash, increasing of liver and spleen. What additional research should be appointed?

a. Ultrasound of the abdomenb. Puncture of the lymph nodes followed by microscopyc. * Blood on atypical mononucleard. Blood culturee. RPHA with influenza viruses

53. 96. 30 years old person, six months ago, was found positive for HIV. 3 months complaints of general weakness, fatigue, drowsiness, headache. 2 Weeks - anxiety, fearfulness, depression. 5 days ago, there were memory disorders and aphasia, tremor, impaired fine movements, myoclonus, sloppiness. What additional tests need to be carried out for the diagnosis of the nervous system?

a. Ganciclovirb. Cerebroprotectorc. Antidepressantsd. Antipsychoticse. * Zidovudine, didanosine

Page 181: intranet.tdmu.edu.uaintranet.tdmu.edu.ua/data/kafedra/internal/infect_desease... · Web viewBest method to treat diarrhoea in child is: intra venous fluide *ORS antibiotics bowel

54. 97. 30 years old person, six months ago, was found positive for HIV. 3 months complaints of general weakness, fatigue, drowsiness, headache. 2 Weeks - anxiety, fearfulness, depression. 5 days ago, there were memory disorders and aphasia, tremor, impaired fine movements, myoclonus, sloppiness. What additional tests need to be carried out for the diagnosis of the nervous system?

a. Ultrasoundb. * Computer Tomographyc. Dopplerd. Blood analysis for HIVe. Pathophisiology study

55. . The patient has been diagnosed with herpes zoster. A patient suffering from childhood diabetes, father and grandfather - asthmA. A few days ago, suffered food poisoning. 7 days took analgin due to headaches. Antibodies against HIVwas find. What factor has the greatest pathogenetic significance?

a. *HIV-relative immune deficiencyb. Food poisoningc. An allergic disease in relativesd. Diabetese. Analgesics

56. Sick C., 20 years old, was admitted to the hospital with complaints of the frequent emptying without stomach-ache, vomiting without nausea, pain in calf muscle. Objectively: Temperature of body is 36,2 oC. Skin is pale, cold, tongue is dry, voice is hoarsed. The stomach is pulled is not painful at palpation. Emptying resemble with “rice-water”. Are there what terms of discharge of patients from the hospital at such illness?

a. A.*Triple negative results of bacteriological examination of excrementsb. Double negative results of bacteriological examination of excrementsc. Single negative result of bacteriological examination of excrementsd. Single negative results of bacteriological examination of excrements

and urinee. Double negative results of bacteriological examination of excrements and

urine57. A sick 18 years, hospitalized in an infectious department with diagnosis of cholera,

dehydration of IV degree. What measures are possible primarily?a. Oral rehydration by glucose solutionsb. Tetracyclinec. *Intravenous stream introduction of salt solutionsd. Proceeding in the normal microflora of intestinee. Intravenous stream introduction of sodium chloride solution

58. Patient B., 20 years, became ill acutely. The unexpected severe diarrhea appeared with frequent vomiting, with plenty of vomits. The patient arrived from a South-east Asia countries. He has temperature 36,1 oC during 3 weeks. Abdomen is pulled, not painful. Stool is a rice-water. What most reliable changes in blood will be present?

a. Increasing amount of erytrocytes, leucocytes, relative closeness of plasma of blood, hyperkalemia, metabolic acidosis

b. Decreasing of amount of erytrocytes, leucocytes, relative closeness of plasma of blood, hypokalemia, metabolic acidosis

Page 182: intranet.tdmu.edu.uaintranet.tdmu.edu.ua/data/kafedra/internal/infect_desease... · Web viewBest method to treat diarrhoea in child is: intra venous fluide *ORS antibiotics bowel

c. Decreasing of amount of erytrocytes, leucocytes, increase of relative closeness of plasma of blood, hyperkalemia, metabolic acidosis

d. *Decreasing amounts of erytrocytes, leucocytes, relative closeness of plasma of blood, metabolic hypokalemic acidosis

e. Increasing amount of erytrocytes, leucocytes, relative closeness of plasma of blood, hypokalemia, metabolic alkalosis

59. A sick, 29 years, emptying watery stool repeatedly, frequent vomiting. Objectively: total cyanosis, dryness of mucous membrane, turgor of skin is decreased Temperature of body 35,2 oC. Pulse in radial artery is not determined. Tachypnea, musle spasm, urination is absent for 6 hours. What is the state of the patient?

a. Dehydration of IV degreeb. Dehydration of I degreec. Infectious-toxic shockd. Anaphylactic shock e. *Uncompensated hypovolemic shock

60. Patient with complaints of a general weakness, diarrhoea appeared to the doctor. Objectively: temperature of body 36,4 oC, skin covers clean, acrocyanosis, tongue is dry, stomach is soft, not painful, emptying is abundant, watery, with the supernatant flakes of white color, odourless and admixtures. For clarification of diagnosis culture of stool was made on ResselyA. What is the color of culture chang?

a. From blue to greenb. From yellow to greenc. From green to yellowd. From yellow to bluee. *From blue to yellow

61. For a patient E., 37 years old, a disease began rapidly, 6 hours the frequent liquid emptying appeared onto the plain rise of normal temperature of body, vomit joined then. At an inspection: aphonia, eyes are hollow, pulse frequent, threadlike, low blood pressure cramps appeared in lower extremities. Liver and spleen are not enlarged. Choose preparations for starting etiotropic therapy.

a. Tetracyclin, erythromycin, levomycetin, ciprofloxacin or imodiumb. Erythromycin, levomycetin, benzylpenicillin or imodiumc. Tetracyclin, erythromycin, levomycetin, benzylpenicillin or ofloxacind. Tetracyclin, erythromycin, levomycetin, or bifi-forme. *Levomycetin, erythromycin or ciprofloxacin

62. Citizen of Pakistan, 30 years became ill rapidly with frequent watery stool which appears like a rice water. Objectively: temperature of body 35.4 oC, skin is cold, acrocynosis, elasticity of skin and turgor is decreased. By what method may estimate the degree of dehydration?

a. To examine an eyeballsb. To check central venous pressurec. To define the level of urea and creatinine in bloodd. *To check the loss of blood plasma e. To check the pressure of blood

63. For a patient E., 37 years, body weight of 70 kg, frequent liquid emptying appeared with rise of body temperature, frequent vomiting joined in later. At an inspection: aphonia,

Page 183: intranet.tdmu.edu.uaintranet.tdmu.edu.ua/data/kafedra/internal/infect_desease... · Web viewBest method to treat diarrhoea in child is: intra venous fluide *ORS antibiotics bowel

eyes are hollow, pulse with frequent threadlike, blood pressure is not determined, tachypnea, total cramps. Liver and spleen are not enlarged. What volume of salt solutions must be entered for primary rehydration?

a. *7 Lb. 3,5 Lc. 5 Ld. 10 Le. 2 L

64. Sick C., 23 years, ill from 3 days after returning from IndiA. The disease has begun with the liquid emptying which looks like a watery, after wards joined with multiple vomiting, expressed weakness and cramps. The state progressively got worse and was admitted within 12 hour in an infectious hospital in a grave condition. The eyes of person is sharped, skin is cold, acrocynosis temperature of body 35,5 oC. Aphonia, cramps of hands and feet. The fold of skin falls out through abdomen. stomach is pulled in, no pain at palpation. Blood pressure is 70 and 30 mm of Hg. Pulse on radial artery is not determined, urination and emptying are not good. What is the most appropriate diagnosis?

a. Intestinal echerihiosisb. Salmonellosisc. Shigellosisd. *Cholerae. Amebiasis, intestinal form

65. Patient with cholera on a background treatment has signs of hyperkalemiA. What solution must be applied for futher rehydration therapy?

a. Neohemodezb. *Disaltc. Chlosaltd. Polyhybride. Lactosalt

66. Sick I., 25 years, returned from rest in Egypt. Early in the morning diarrhea began. Emptying each 1-1,5 hrs, abundant watery, without mucus and blood. Abundant vomiting also appeared. The temperature of body at first rise to 37,3 oC, afterwards became 35,5 oC, stomach-aches negative. Delivered in an infectious department. What is first aid?

a. Treatment with sorbentsb. Polyhybrid solution intravenousc. 5 % solution of glucose intravenousd. Fresh-frozen plasma intravenouse. *Salt solutions intravenous

67. Sick, 20 year old, apeared to the hospital with complaints offrequent emptying without stomach-ache, vomit without nausea, pain in calve. Objectively: temperature of body – 36,2 oC. A skin is pale and cold, a tongue is dry, voice getting hoarse. A stomach is pulled in, painless, emptying like a “rice-water”. What are the criterias for discharging of the patient from the hospital?

a. Double negative bacteriological examination of excrementsb. Single negative bacteriological examination of excrementsc. *Triple negative bacteriological examination of excrementsd. Negative bacteriological examination of excrement and urine

Page 184: intranet.tdmu.edu.uaintranet.tdmu.edu.ua/data/kafedra/internal/infect_desease... · Web viewBest method to treat diarrhoea in child is: intra venous fluide *ORS antibiotics bowel

e. Double negative bacteriological examination of excrement and urine68. For a patient, the disease begun rapidly with a chill, increase of temperature to 39 oC,

vomitting, pain in epigastrium, diarrhea with the watery stinking emptying. 6 hours before the disease he ate a raw egg, potato with the braised meat and drink juice. What exciter did cause the similar state probably?

a. Shigellab. Collibacillusc. Campylobacterd. *Salmonellae. Citrobacter

69. Patient C., 17 years old, worker in vegetable garden. Became sick 2 days ago. The disease is related to use of meal of dirty root crop. Disease begun with chill, fever of 38.1 oC, had pain in abdomen and in the muscles of all groups and joints, weakness, nauseA. Emptying is liquid, viscid, with a strong smell, ordinary color. At objective examination: sclera, conjunctivitis, hyperemia of soft palate, “raspberry” tongue. At palpation of abdomen – moderate pain in epigastrium and in right iliac areA. What will be the diagnosis?

a. Cholerab. Dysenteryc. Salmonellosisd. *Scarlet fevere. Viral hepatitis

70. Patient with cholera has bradycardia, low blood pressure, weakness. What is most important factor in the given clinical situation?

a. . Hypocalcemiab. Hypopotassiumc. Hyponatremiad. Hypernatremiae. *Hyperpotassium

71. Patient E, 47 years, became ill in 4 days after returning from Egypt. A disease begun with the liquid emptying which look like a watery kind afterwards, joined with frequent vomiting, expressed weakness and fatigue. The state progressively got worse and within 18 hrs was admitted in an infectious department in a very grave condition. The lines of person are strained, skin is cold, cyanosis, temperature of body 35.5 oC. Aphonia, cramps of hands and feet. The skin fold falls out through the abdomen. A stomach is pulled; no pain at palpation. Blood pressure is 70 and 30 mm of Hg. The pulse on radial artery is not determined, urination and emptying normal. Weight of patient at admission to the hospital was 60 kg. What is the degree of dehydration of the patient?

a. Ib. IIc. *IVd. IIIe. It is not

72. Sick X., which returned from vacations from Turkey within 5 hrs in the morning, diarrhea began. Emptying each 1-1.5 hrs, with abundant watery without mucus and blood. In future bloodless watery, abundant vomitting appeared through 12 hrs. The temperature of body

Page 185: intranet.tdmu.edu.uaintranet.tdmu.edu.ua/data/kafedra/internal/infect_desease... · Web viewBest method to treat diarrhoea in child is: intra venous fluide *ORS antibiotics bowel

at first rise to 37.5 oC afterwards became 35.7 oC is delivered in an infectious department. What disease did you suspect?

a. Echeriosisb. Food poisoning|c. *Cholerad. Salmonellosise. Balantidiasis

73. Sick P., 25 years old, presents with frequent vomitting. Objectively: dryness of skin and mucous membranes, brief cramps in gastrocnemius muscles, the temperature of body is normal, voice is hoarsed, moderate tachycardia and hypotension. The compensated metabolic acidosis is marked. About what degree of dehydration is it possible to think?

a. IVb. IIc. *IIId. Ie. There is no dehydration

74. An unconscious patient is delivered in the intensive department. Pale dark circles around eyes. Skin is cold with sticky sweat. Temperature of body 35,6 oC. Pulse 140/min and weak. Blood pressure is 40 /0 mm of Hg. Tongue is dry. Emptying is involuntary and “watery”, vomited twice. What infusion must be given as intensive therapy.

a. Albuminb. Rheopoliglykinc. 5 % glucose solutiond. *Polyionic salt solutionse. 10 % glucose solution

75. A farmer O., 50 years old, hospitalized in a moderate condition with complaints of dryness in mouth, multiple vomitings, pain in the epigastriums and frequent watery stool. First aid to the patient is?

a. Hypersaturated oxygenb. Transfusion of fresh-frozen plasmac. Tetracyclind. *Intravenous introduction of salt solutionse. Introduction of polyhybrid

76. Patient P, 35 years old, became ill within 2 days after returning from IndiA. A disease begun with appearance of liquid diarrhea which resembled a watery appearance, afterwards joined with multiple vomit, expressed weakness and fatigue. State progressively got worse and within 12 hrs delivered in an infectious department in a grave condition. The skin is cold, cyanosis, temperature of body 35.5 oC. Aphonia, cramps of hands and feet. A skin fold falls out from abdomen. The stomach is pulled in, no pain at palpation. Blood pressure is 70 and 30 mm of Hg. A pulse on a radial artery is not determined, urination and emptying are normal. Weight of patient at admission to the hospital was 80 kg. What volume of infusion solutions is needed for primary rehydration?

a. 10 litersb. *8 litersc. 4 litersd. 6 liters

Page 186: intranet.tdmu.edu.uaintranet.tdmu.edu.ua/data/kafedra/internal/infect_desease... · Web viewBest method to treat diarrhoea in child is: intra venous fluide *ORS antibiotics bowel

e. 2 liters77. A patient L., who returned from Crimea, developed diarrhea at 5AM. Bowel movements

are each 1-1.5 hrs, watery, without mucus and blood. In 12 hrs a single episode of vomiting developed. The temperature of body at first rise to 37.3 oC, stomach-aches is present. he was examined by the doctor of first-aid and delivered to an infectious isolation with the diagnosis of acute intestinal infection.Which disease is most probable for the patient?

a. Intestinal echerihiosisb. . Salmonellosisc. Echeriosisd. Food poisoninge. *Cholera

78. The ill patient in severe state was delivered to infectious department with no consciousness. Pale dark circles around eyes. Skin is cold, covered with sticky sweat. Temperature of body 35.6 oC. Pulse 140/min and weak.Blood pressure is 40 and 0 m of Hg. Tongue is dry. Emptying is involuntary and watery. Three times vomited like “fountain”.What is the state of the patient?

a. Collapseb. Infectious toxic shockc. *Dehydrational shockd. Cereblral edemae. Intoxicated encephalopathy

79. Patient T., 22 years old, appealed to the hospital with complaints of the frequent stool without the abdominal pain, vomits without nausea, pain in calves. Objectively: Т-36,2 °C. A skin is pale, cold, tongue dry, voice getting hoarse. An abdomen is pulled in, painless. Emptying remind a “rice-water”. What terms of extract of patients from a hospital at this illness?

a. Non-permanent negative bacteriological research of excrementb. Double negative bacteriological research of excrementc. Double negative bacteriological research of excrement and urined. Non-permanent negative bacteriological research of excrement and urinee. *Triple negative bacteriological research of excrement

80. Patient N, 13 years old, hospitalized in the infectious department with a diagnosis of cholera, severe dehydration of Ш-ІV stage. What measures are primary?

a. Setting of etiotropic antibacterial preparationsb. Stream intravenous introduction of salts solutionsc. Peroral rehydratation with glucose-salts solutionsd. Renewal of normal microflora of intestinee. *Setting of enzyme preparations

81. A patient J., 23 years old, became ill suddenly. Profuse diarrhea with frequent and large amount vomits. A patient arrived from one of countries of south-east Asia, where was near 3 weeks. T 36,1 °C. An abdomen is pulled in, painless. The stool has the appearance of rice-water. What most reliable changes will be in a blood?

a. Decrease amount of erythrocytes, leucocytes, relative tightness of plasma of blood, hypopotassemia, metabolic acidosis.

Page 187: intranet.tdmu.edu.uaintranet.tdmu.edu.ua/data/kafedra/internal/infect_desease... · Web viewBest method to treat diarrhoea in child is: intra venous fluide *ORS antibiotics bowel

b. *Increase amount of erythrocytes, leucocytes, relative tightness of plasma of blood, hypopotassemia, metabolic acidosis.

c. Decrease amount of erythrocytes, leucocytes, increase of relative tightness of plasma of blood, hyperpotassemia, metabolic acidosis.

d. Increase amount of erythrocytes, leucocytes, relative tightness of plasma of blood, hyperpotassemia, metabolic acidosis.

e. Increase amount of erythrocytes, leucocytes, relative tightness of plasma of blood, hipopotassemia, metabolic alkalosis.

82. Patient C, 17 years old, became ill suddenly. Profuse diarrhea with frequent vomits without nosier. A patient arrived from south-east AsiA. Temperature – 36,1 °C. An abdomen is pulled in, painless. The stool has the appearance of rice-water. What diagnosis is most reliable?

a. *Cholerab. Dysenteryc. Salmonellosisd. Esheryhiosise. Rotavirus gastroenteritis

83. To the internist appealed patient with complaints of weakness, diarrheA. Rested on a south, where the cases of diarrhea were present. Objectively: t-36,4 °C, skin covers are clean, acrocyanosis, tongue is dry, abdomen is soft, painless, emptying abundant, watery, with the flakes of white color floating on a surface, without odor and admixtures. For clarification of diagnosis sowing on the Ressels medium was made. How will the color of medium change?

a. From yellow to blueb. From yellow to greenc. From green to yellowd. *From blue to yellowe. From blue to green

84. The disease began sharply, 6 hours ago at a normal temperature appeared frequent liquid emptying, then vomit joined. At the inspection: voice is soundless, eyes reddish, pulse frequent, arterial pressure low, urine is not present, cramps appeared in lower extremities. The heart and lungs without changes. A liver and spleen are not enlarged. Choose preparations of ethiotropic therapy which can be used:

a. Tetracycline, erythromycin, levomycetine, gentamycine, ofloxacineb. Erythromycine, evomycetine, gentamycine, ofloxacine, ciprofloxacin,

imodiumc. Tetracycline, erythromycine, levomycetine, benzyl-penicillin sodium salt,

ofloxacined. *Erythromycine, levomycetine, gentamycine, ofloxacine, ciprofloxacine. Erythromycine, gentamycine, ofloxacine, ciprofloxacine, imodium

85. The disease began sharply from diarrhea, that was accompanied by an abdominal pain. The act of defecation brought facilitation. Emptying of green color, abundant, foamy, with a strong unpleasant smell. Temperature of body is subfebrile. It is found out hyperemia and graininess of soft palate. In blood: leucopenia, eozinophiliA. Violations of water-electrolyte balance are moderate.Your diagnosis:

a. Cholera

Page 188: intranet.tdmu.edu.uaintranet.tdmu.edu.ua/data/kafedra/internal/infect_desease... · Web viewBest method to treat diarrhoea in child is: intra venous fluide *ORS antibiotics bowel

b. Toxic food-borne infectionc. Salmonelliosisd. Dysenterye. *Rotaviral gastroenteritis

86. The seventh pandemic of cholera is caused by V. cholera El Tor. It begans in 1961 on the Sulawesi island. However, to the epidemic of cholera arose up only in the countries of the third world. It is known that cholera is classic bacterial infection with the fecal-oral mechanism of transmission with the certain infectious dose of exciter. What is the basic factor of risk, that is instrumental in the such uneven division of morbidity on countries?

a. Biological properties of exciterb. Climate of countryc. *Social-economic conditions of population of countryd. Immune status of populatione. High development of industry and contamination of surrounding environment

87. Patient C., 36 years old, periodically goes on a business trip in Egypt, desperately ill. Complaints of the increasing body temperature up to 39,4 °C with chills and sweating, pain in the right under rib, emaciation. A slight jaundice. Increased liver density, painful. In blood neutrophils leukocytosis, increased ESR. When USD revealed multiple liver abscesses. What kind of illness need to think.

a. Legionellosisb. Echinococcosisc. . Ascariasisd. Liver cancer with metastasese. *Amoebiasis

88. Student A., 22 years old, ill for 3 weeks, a month after returning from Ethiopia: appeared abdominal pain, liquid excrement, abundant, up to 10 times per day, in the form of “raspberry jelly”, joined cramping in the abdomen, more right, growing in the bowel. Over time the disease has lost 6 kg body weight. Your diagnosis.

a. *Intestinal amoebiasisb. Shigellosisc. Salmonellosisd. Tumor bowele. Intestinal yersiniosis

89. Patient K., 40 years old, had complained of fever, night sweating, a significant decrease in body weight, pain in muscles, throat, joints, photophobia, recurrent diarrheA. When inspection revealed a generalized lymphadenopathy. What is the most reliable diagnosis.

a. *AIDSb. Lymphogranulomatosc. The tumor of the digestive systemd. Tuberculosise. Salmonellosis

90. Patient C., 35 years old, the disease has started severe chills, raising the temperature to 39 °C, vomiting, pain in epigastric region, diarrhea with water-smelly bowel movement. Over 6 hours after eating raw eggs, potatoes with braised meat, drank juice. What kind of agent is likely to cause this condition.

a. *Salmonella

Page 189: intranet.tdmu.edu.uaintranet.tdmu.edu.ua/data/kafedra/internal/infect_desease... · Web viewBest method to treat diarrhoea in child is: intra venous fluide *ORS antibiotics bowel

b. Vibrio cholerac. Enteric stickd. Campylobactere. Shigella

91. Patient C., 17 years old, working vegetable base. Acutely ill, 2 days ago. His illness is linked to eating dirty roots. The disease started with chills, body temperature increase to 38,1 °C, headache, pain in muscles and joints of all groups, weakness, nausea, Cramping in the stomach around the navel, in epigastrium, diarrhea up to 5 times. Excrements liquid, viscous, bed smell, normal colour. Objective inspection: scleritis, conjunctivitis, hyperemia of soft palate, “raspberry” tongue. In palpation of abdomen - the moderate morbidity in epigastrium. What is diagnosis.

a. *Yersiniosisb. Dysenteryc. Salmonellosisd. Cholerae. Viral hepatitis

92. Patient 25 years, received complaints of double vision of the eyes, a decline of view, shortness of breath. Before the disease eat mushrooms home preservation. Objective: pallor, wide pupils with a weak reaction to light, dry mouth, a violation of swallowing, flatulence, delay of stool. What is preliminary diagnosis.

a. Leptospirosisb. Yersiniosis c. *Botulismd. Giardiasise. Salmonellosis

93. Patient C., 22 years old, hospitalized in an infectious department complaining of chills, temperature increase to 38,5 °C, vomiting, pain in epigastrii, frequent stool. 7 hours before the disease, ate raw eggs, potatoes with braised meat, drink tomato juice. What kind of organism most likely to cause disease.

a. Enterovirusb. *Salmonellac. Enteric stickd. Staphylococcuse. perfringens

94. Emergency ambulance delivered in infectious hospital girl N., 17 years old, complaining of headaches, expressed general weakness, pain in epigastrium, repeated vomiting, diarrhea to 8 times a day greenish color. She was used 2 raw chicken eggs. Body temperature 39 °C, the tongue covered with white coat, moderately cramps, pain in abdomen and sigmoid areA. The most likely diagnosis.

a. Acute appendicitisb. Crohn’s diseasec. Shigellosisd. Viral gastroenteritise. *Salmonellosis

95. Patient L., 32 years old, was admitted to the infectious hospital complaining of chills, headache, expressed general weakness, cutting pain in epigastric, diarrheA. The

Page 190: intranet.tdmu.edu.uaintranet.tdmu.edu.ua/data/kafedra/internal/infect_desease... · Web viewBest method to treat diarrhoea in child is: intra venous fluide *ORS antibiotics bowel

temperature of 39 °C, repeated vomiting. Excrement abundant, greenish color, without pathological impurities. Over 4 hours to illnesses used the meat salad in the factory canteen. Which drugs will be most effective for the treatment of this patient.

a. Antibotulism serumb. Salt and colloid solutionsc. Sulfanilamidesd. *Antibioticse. Enterosorbent

96. Patient B., 55 years old, was hospitalized in an infectious hospital with complaints of frequent vomiting, liquid excrements, abdominal pain, mainly in the right iliac area and epigastrium. Pale skin, dry tongue with a touch of gray, hypotension, tachycardiA. The doctor was suspected salmonellA. What laboratory test is need to perform.

a. General blood analysisb. Parasytoscopy of blood smearc. Microscopy of stoold. *Coproculturee. Biologic test

97. Patient C., 30 years old, was hospitalized with a preliminary diagnosis of salmonellosis. Body temperature – 40 °C, repeated vomiting, profused diarrheA. Hypotension (BP 55/30 mm Hg), expressed tachycardia (pulse to 120/min), face pale, pain mainly in right iliac area and epigastrium, hepatosplenomegaly were found. What treatment tactic will be most effective in this case.

a. Glucocorticoids, crystalloid solutions, antibiotics, diureticsb. Cardiac glycosides, colloid solutions, antibioticsc. Sorbents, diuretics, desintoxicationd. Dopamine, colloid solutions, antibacteriale. *Glucocorticoids, crystalloid solutions, antibiotics

98. Patient F., 25 years old, was hospitalized in the infectious hospital with complaints of frequent vomiting, liquid stool, abdominal pain in epigastrium. Pale skin, dry tongue with a grey cover, hypotension, tachycardiA. The doctor was suspected salmonellA. What serologic study is need to perform.

a. Microprecipitation reactionb. RA (Widal)c. *RIGA with Salmonella diagnosticumd. RIGA with Shigella diagnosticum and RA (Widal) with paired serume. RKC

99. Patient N., 22 years old, was admitted to the infectious office complaining of chills, headache, expressed general weakness, cutting pain in sigmoid area, diarrheA. The temperature of the body of 39,3 °C, vomiting. Faces watery. Patient was used the salad with sausage and eggs. For further diagnosis doctor was prescribed RIGA with salmonella diagnosticums. When it’s necessary to take blood for the investigation.

a. At the 1st and 2nd days of illnessb. Once in recovering periodc. *At the end of the 1st week and after 7-10 daysd. At the end of the 1st week and after 3-4 dayse. Once in acute period

Page 191: intranet.tdmu.edu.uaintranet.tdmu.edu.ua/data/kafedra/internal/infect_desease... · Web viewBest method to treat diarrhoea in child is: intra venous fluide *ORS antibiotics bowel

100. Patient B., 32 years old, an employ of poultry was hospitalized with complaints of pain in abdomen, mostly in sigmoid area, fever up to 38,8 °C, nausea, vomiting, diarrhea with dark-green colour of stool, without any pathological changes. Skin and mucous membranes are dry, moderate tension in epigastric region and pain in the right iliac areA. The most likely diagnosis.

a. Acute appendicitisb. Cholerac. Disbacteriosisd. *Salmonellosise. Rotavirus gastroenteritis

101. Three patients were delivered with symptoms of fever, diarrhea and vomiting. Excrements dark-green, without any pathological changes. All patients together took part in the preparation of food and have used eggs, meat salad. The most likely diagnosis.

a. Cholerab. Botulismc. Dysbacteriosisd. *Salmonellosise. Shigellosis

102. Patient L., 33 years old, was admitted to the hospital with the complaints of untolarable pain in the abdomen, mostly in sigmoid region, fever up to 38,8 ?C, nausea, vomiting , diarrhea of dark-green colour in every1-1,5 hour, without admixture of blood and mucus. The condition progressively deteriorated, increasing hypotension, paleness, weak cardiac sounds, decreased intestinal peristalsis with moderate tension in epigastric region and pain in the right iliac region. What tactics of treatment?

a. Glucocorticoids, infusion of salt solutions, antibiotics intravenously, diureticb. Cardiac glycosides, infusion colloidal solution, antibacterial drugsc. Enterosorbents, diuretic, desintoxication therapyd. Mezaton 1% / in drops, infusion of salt solutions, antibacterial drugs orallye. *Glucocorticoids, infusion of salt solutions, antibiotics intravenously

103. Patient 45 years old, became ill within 10 hours after consumption of grinded meat. Complaints of vomiting, pain in epigastric region, diarrhea of green color, increasing of temperature up to to 39 °C. During objective examination revealed that: patients general condition is sever, pain in the muscles of the lower limbs, pale face, acrocyanosis. Pulse 130 per min,. B.P 70/40 mm Hg. weak cardiac sounds, dry tongue, brown-whitecovering . abdomen cramps, painful iliac and epigastric regions. What is the most likely diagnosis.

a. Shigellosisb. Cholerac. *Salmonellosisd. Viral gastroenteritise. Eshericiosis

104. Five patients during 15 hours were admited In a hospital. All of them have similiar complaints of mild diarrhea and vomiting developed ,diplopia, midriasis, visual disturbance, difficult swallowing, dry mouth, difficulty in breathing. All patients together joined in a dinner party, used different dishes, including meat, salad, canned mushroom. The most likely diagnosis.

Page 192: intranet.tdmu.edu.uaintranet.tdmu.edu.ua/data/kafedra/internal/infect_desease... · Web viewBest method to treat diarrhoea in child is: intra venous fluide *ORS antibiotics bowel

a. Cholerab. *Botulismc. Thypusd. Salmonellosise. Rotaviral gastroenteritis

105. Patient G., 24 years old, 6 hours after taking his breakfast with boiled eggs, a cup of coffee appeared chill, fever up to 38,8 °C, pain in left inguinal area, nausea,vomiting, rumbling in the stomach, then frequent diarrhea, feces copious, watery, mudy-green. What is the most likely diagnosis.

a. Cholerab. Botulismc. *Salmonellosisd. Abdominal thypoide. Shigellosis

106. Patient, who was arrived from the Crimea, diarrhea appeared in an intervel of 1-1.5 hour. with copious watery increments, without admixtures of mucous and blood. He was admtted in an infectious dipartment with the diagnosis of “acute intestinal infection”. What laboratory tests are necessary to confirm the etoilogical diagnosis.

a. General blood analysisb. Bacteriological blood analysisc. Microscopical analysis of stoold. *Coprograme. The biological examination in mice

107. Patient C. with complaints of diarrhea with mucous, stabing pain in epigastric area , rumbling in the stomach, fever. According to his anamnesis he took preserved milk, eggs, salad of fresh vegetables. What is the disease most likely.

a. Shigellosisb. Yersiniosisc. *Salmonellosisd. Cholerae. Botulism

108. Patient B., 38 years old, came to the admission department with the complaints of fever up to 38 °C, vomiting 4 times in a day, diarrhea up to 6 times in a day. Durin objective examination revealed that B.P 125/75 mm Hg, pulse 80 per min, abdomen is soft,painful in the right iliac and epigastric area .According to anamnesis he took salad of boiled eggs and sausage. What treatment should be started first.

a. Antibotulism serumb. Intravenous antibioticsc. Washing of the stomach and intestine, rehydration therapy , glucocorticoidsd. *Gastric lavage and washing of intestine, rehydraton therapy enterosorbentse. Treatment after getting of the laboratory test results

109. Patient 27 years old, complaints of headaches, weakness, pain in epigastric area, vomiting, diarrhea (9 times in a day; huge excrement, greenish colour). Before the appearance of disease he consumed raw eggs. Objectively: body temperature 38,8 °C. Tongue is white, pain in epigastric and umbilical region. The most likely diagnosis.

a. *Salmonellosis

Page 193: intranet.tdmu.edu.uaintranet.tdmu.edu.ua/data/kafedra/internal/infect_desease... · Web viewBest method to treat diarrhoea in child is: intra venous fluide *ORS antibiotics bowel

b. Cholerac. Dysenteryd. Viral gastroenteritise. Acute appendicitis

110. Patient K., 30 years old, came with complaints of nausea, vomiting, pain in epigastrium and paraumbilical areA. High body temperature, pale skin, dry. tongue covered with whitish-gray coating. quick pulse, low blood pressure. 2 hours before the onset of symptoms of the disease he ate in cafe. What is the diagnosis most likely.

a. Cholerab. Shigellosisc. Thypoid feverd. *Salmonellosise. Meningitis

111. Disease started acutely with the complaints of watery diarrhea, vomiting, cramps in the lower extremities. Objectively: slow voice, shunken eyes, quick pulse, low blood pressure, decrease of urination, weak cardiac sounds. In liver and spleen no changes. Put the correct diagnosis.

a. Salmonellosisb. Dysenteryc. Food poisoningd. *Cholerae. Typhoid fever

112. A Pakistani, 30 years old, severly ill: with the complaints of frequent diarrhea like rice water. Objectively: body temperature 35,4 °C, skin of peripheral parts of the body cold to the touch, acrocyanosis, xerostomia, sharply decreased skin elasticity and turgor. What should be done to assess the degree of dehydration.

a. Measurement of central venous pressureb. Determination of urea and creatinine in bloodc. Funduscopyd. Plain X-ray film of abdomene. *Determination of specific gravity of blood plasma

113. Patient N., 30 years old, works in a restaurant, admitted to the hospital complaining of unbearable pain in the abdomen, increase of temperature 38,8 °C, nausea, vomiting, diarrhea of dark-green colour, He took eggs, mushrooms, canned food. Objective state: general condition is moderately sever, skin and mucous membranes are dry, pain in epigastric region. Name the disease.

a. Shigellosisb. Yersiniosisc. *Salmonellosisd. Cholerae. Rotaviral infection

114. Patient 35 years old with complaints of increasing of temperature to 39 °C with chill, vomiting, pain in epigastric region, diarrhea with foul smell, abdominal cramps. 6 hours before onset of the disease ate raw eggs, fried potatoes with grinded meat, drank juice. To clarify the diagnosis of serological diagnostic methods often used RNGA with

Page 194: intranet.tdmu.edu.uaintranet.tdmu.edu.ua/data/kafedra/internal/infect_desease... · Web viewBest method to treat diarrhoea in child is: intra venous fluide *ORS antibiotics bowel

the salmonella groupdiagnostic tools and RA (Vidal test) . When blood should take for the diagnostic procedures?

a. In the first day of illnessb. At the end of the first monthc. In 1st week in 3-4 daysd. *At the end of the 1st week from 7-10 dayse. During admission to the hospital

115. A patient was hospitalised after 4 hours of consumption of raw eggs, complaining frequent vomiting, abdominal pain, mostly in the right iliac area and umbilical areA. Objectively pale skin, dry tongue with gray coating, hypotension, tachycardiA. What tests should be carried out to confirm the diagnosis.

a. General blood analysisb. Parasitological examinationsc. Microscopic examination of stoold. Biological teste. *Coproculture

116. Patient G., 22 years old, was hospitalised in an infectious department complaining of chills, stabing pain in epigastrium, diarrhea, temperature increased up to 39,6 °C, repeated vomiting. Feces copious, watery, without pathological changes. 4 hours before the onset of symptoms ate the meat, salad. What is the most appropriate drug to be prescribe.

a. Antidiarrheal drugsb. Flouroquinolone group of antibioticsc. Sulfanilamide group of antibioticsd. Desintoxication therapye. *Enterosorbents

117. Patient K., 40 years old, was hospitalised with the diagnosis of intestinal infection. Complaints of general weakness, headache and diarrheA. Symptomes appeared suddenly 2 hours after consumption of cake with custard. Suddenly there were chills, nausea, profuse repeated vomiting, frequent diarrhea, abdominal pain and short-term loss of consciousness, temperature increased to 38,6 °C. The preliminary dianosis?

a. Salmonellosis, localized form, gastroenteritis typeb. *Salmonellosis, localized form gastroenterocolitis typec. Salmonellosis, localized form enterocolitis typed. Salmonellosis, generalized forme. Salmonellosis, nosoparasitic

118. Patient K., 40 years old, hospitalised with the diagnosis of salmonellosis, with symptoms of rise of body temperature to 40 °C, repeated vomiting, diarrheA. Objectively: AP 60/20 mm Hg, pulse 120/minute, pale face, increased intestinal peristalsis, enlarged liver and spleen. What is the best treatment.

a. *Glucocorticoids, desintoxication therapy, antibacterial drugsb. Diuretic, desintoxication therapy, antibacterial drugsc. Enterorsorbents, diuretic, detsintoxication therapyd. Glucocorticoids, diuretic, desintoxication therapy, antibacterial drugse. Glucocorticoids, diuretic, antibacterial drugs

Page 195: intranet.tdmu.edu.uaintranet.tdmu.edu.ua/data/kafedra/internal/infect_desease... · Web viewBest method to treat diarrhoea in child is: intra venous fluide *ORS antibiotics bowel

119. Patient P., 36 years old, a farmer is seriously ill, with high temperature, having chills, headache, flatulence, nauseA. At present he is complaining of severe abdominal pain, diarrhea up to 15 times a day stool is liquid, smelly, with impurities mucous and blood. His eyes are sunken. The tongue is covered with white coating. On palpation- painful colon, increased liver. In blood – neutropil leukocytosis with a shift formula the left anemiA. On rectomanoscopy deep ulcers with irregular edges are observed, including along the folds. What is the diagnosis.

a. Shigellosisb. *Balantidiasisc. Nonspecific ulcerous colitis d. Amoebiasise. Colon tumor

120. A 29 years old patient T. works on a pig farm. Shi was hospitalized with symptoms of balantidiasis. What would be the drug of choice.

a. Benzylpenicillinb. Gentamycinc. Chloramphenicold. *Monomycine. Timogen

121. A 63 years old villager,is gravely ill: increased body temperature, headache, nausea, vomiting, abdominal pain, diarrheA. The patient was not seeking mediacal attention, and was taking levomycetin. The condition had not improved, the frequency of bowel movements was 8-12 times a day. At present he is complaining of severe abdominal pain, diarrhea, general weakness. The stool is liquid, smelly, with mixture of puss and blood. The patient is exosted his eyes are sunken. His tongue is covered with white coating. In blood analys is leukocytosis and anemia observed. On rectomanoscopy detected ulcers with jagged edges. What is the most likely diagnosis.

a. *Balantiasisb. Shigellosisc. Nonspecific ulcerative colitisd. Rotavirus gastroenteritise. Colon tumor

122. Patient C., 35 years old, the disease has started from severe chills, raising the temperature to 39 oC, vomiting, pain in epigastric region, diarrhea with water-smelly bowel movement. Over 6 hours after eating raw eggs, potatoes with braised meat, drank juice. What kind of agent is likely to cause this condition.

a. *Salmonellab. Vibrio cholerac. Enteric stickd. Campylobactere. Shigella

123. Patient C., 17 years old, working at vegetable base. Acutely ill, 2 days ago. His illness is linked to eating dirty roots. The disease started with chills, body temperature increase to 38,1 oC, headache, pain in muscles and joints of all groups, weakness, nausea, Cramping in the stomach around the navel, in epigastrium, diarrhea up to 5 times. Excrements liquid, viscous, bed smell, normal colour. Objective inspection: scleritis,

Page 196: intranet.tdmu.edu.uaintranet.tdmu.edu.ua/data/kafedra/internal/infect_desease... · Web viewBest method to treat diarrhoea in child is: intra venous fluide *ORS antibiotics bowel

conjunctivitis, hyperemia of soft palate, “raspberry” tongue. In palpation of abdomen - the moderate morbidity in epigastrium. What is diagnosis.

a. *Yersiniosisb. Dysenteryc. Salmonellosisd. Cholerae. Viral hepatitis

124. Patient 25 years, received complaints of double vision of the eyes, a decline of view, shortness of breath. Before the disease eat mushrooms home preservation. Objective: pallor, wide pupils with a weak reaction to light, dry mouth, a violation of swallowing, flatulence, delay of stool. What is preliminary diagnosis.

a. Leptospirosisb. Yersiniosis c. *Botulismd. Giardiasise. Salmonellosis

125. Patient C., 22 years old, hospitalized in an infectious department complaining of chills, temperature increase to 38,5 oC, vomiting, pain in epigastrii, frequent stool. 7 hours before the disease, ate raw eggs, potatoes with braised meat, drink tomato juice. What kind of organism most likely to cause disease.

a. Enterovirusb. *Salmonellac. Enteric stickd. Staphylococcuse. Cl. perfringens

126. Emergency ambulance delivered in infectious hospital girl N., 17 years old, complaining of headaches, expressed general weakness, pain in epigastrium, repeated vomiting, diarrhea to 8 times a day greenish color. She was used 2 raw chicken eggs. Body temperature 39 °C, the tongue covered with white coat, moderately cramps, pain in abdomen and sigmoid areA. What is the most likely diagnosis.

a. Acute appendicitisb. Crohn’s diseasec. Shigellosisd. Viral gastroenteritise. *Salmonellosis

127. Patient L., 32 years old, was admitted to the infectious office complaining of chills, headache, expressed general weakness, cutting pain in epigastrii, diarrheA. The temperature of 39 °C, repeated vomiting. Excrement abundant, greenish color, without pathological impurities. Over 4 hours to illnesses used the meat salad in the factory canteen. Which drugs will be most effective for the treatment of this patient.

a. Antibotulism serumb. Salt and colloid solutionsc. Sulfanilamidesd. *Antibioticse. Enterosorbent

Page 197: intranet.tdmu.edu.uaintranet.tdmu.edu.ua/data/kafedra/internal/infect_desease... · Web viewBest method to treat diarrhoea in child is: intra venous fluide *ORS antibiotics bowel

128. Patient B., 55 years old, was hospitalized in an infectious hospital with complaints on frequent vomiting, liquid excrements, abdominal pain, mainly in the right iliac area and epigastrium. Pale skin, dry tongue with a touch of gray, hypotension, tachycardiA. The doctor was suspected salmonellA. What laboratory test is need to perform.

a. General blood analysisb. Parasytoscopy of blood smearc. Microscopy of stoold. *Coproculturee. Biologic test

129. Patient C., 30 years old, was hospitalized with a preliminary diagnosis of salmonellosis. Body temperature – 40 °C, repeated vomiting, profused diarrheA. Hypotension (BP 55/30 mm Hg), expressed tachycardia (pulse to 120/min), face pale, pain mainly in right iliac area and epigastrium, hepatosplenomegaly were found. What treatment tactic will be most effective in this case.

a. Glucocorticoids, crystalloid solutions, antibiotics, diureticsb. Cardiac glycosides, colloid solutions, antibioticsc. Sorbents, diuretics, desintoxicationd. Dopamine, colloid solutions, antibacteriale. *Glucocorticoids, crystalloid solutions, antibiotics

130. Patient F., 25 years old, was hospitalized in the infectious hospital with complaints of frequent vomiting, liquid stool, abdominal pain in epigastrium. Pale skin, dry tongue with a grey cover, hypotension, tachycardiA. The doctor was suspected salmonellA. What serologic study is need to perform.

a. Microprecipitation reactionb. RA (Widal)c. *RIGA with Salmonella diagnosticumd. RIGA with Shigella diagnosticum and RA (Widal) with paired serume. RKC

131. Patient N., 22 years old, was admitted to the infectious office complaining of chills, headache, expressed general weakness, cutting pain in sigmoid area, diarrheA. The temperature of the body of 39,3 °C, vomiting. Faces watery. Patient was used the salad with sausage and eggs. For further diagnosis doctor was prescribed RIGA with salmonella diagnosticums. When it’s necessary to take blood for the investigation.

a. At the 1st and 2nd days of illnessb. Once in recovering periodc. *At the end of the 1st week and after 7-10 daysd. At the end of the 1st week and after 3-4 dayse. Once in acute period

132. Patient B., 32 years old, an employ of poultry was hospitalized with complaints of pain in abdomen, mostly in sigmoid area, fever up to 38,8 oC, nausea, vomiting, diarrhea with dark-green colour of stool, without any pathological changes. Skin and mucous membranes are dry, moderate tension in epigastric region and pain in the right iliac areA. The most likely diagnosis.

a. Acute appendicitisb. Cholerac. Disbacteriosis

Page 198: intranet.tdmu.edu.uaintranet.tdmu.edu.ua/data/kafedra/internal/infect_desease... · Web viewBest method to treat diarrhoea in child is: intra venous fluide *ORS antibiotics bowel

d. *Salmonellosise. Rotavirus gastroenteritis

133. Patient L., 33 years old, was admitted to the hospital with the complaints of untolarable pain in the abdomen, mostly in sigmoid region, fever up to 38,8 oC, nausea, vomiting , diarrhea of dark-green colour in every1-1,5 hour, without admixture of blood and mucus. The condition progressively deteriorated, increasing hypotension, paleness, weak cardiac sounds, decreased intestinal peristalsis with moderate tension in epigastric region and pain in the right iliac region. What tactics of treatment?

a. Glucocorticoids, infusion of salt solutions, antibiotics intravenously, diureticb. Cardiac glycosides, infusion colloidal solution, antibacterial drugsc. Enterosorbents, diuretic, desintoxication therapyd. Mezaton 1% / in drops, infusion of salt solutions, antibacterial drugs orallye. *Glucocorticoids, infusion of salt solutions, antibiotics intravenously

134. Patient 45 years old, became ill within 10 hours after consumption of grinded meat. Complaints of vomiting, pain in epigastric region, diarrhea of green color, increasing of temperature up to to 39 °C. During objective examination revealed that: patients general condition is sever, pain in the muscles of the lower limbs, pale face, acrocyanosis. Pulse 130 per min,. B.P 70/40 mm Hg. weak cardiac sounds, dry tongue, brown-whitecovering . abdomen cramps, painful iliac and epigastric regions. What is the most likely diagnosis.

a. Shigellosisb. Cholerac. *Salmonellosisd. Viral gastroenteritise. Eshericiosis

135. Patient U., who was arrived from the Crimea, diarrhea appeared in an intervel of 1-1.5 hour. with copious watery increments, without admixtures of mucous and blood. He was admtted in an infectious dipartment with the diagnosis of “acute intestinal infection”. What laboratory tests are necessary to confirm the etoilogical diagnosis.

a. General blood analysisb. Bacteriological blood analysisc. Microscopical analysis of stoold. *Coprograme. The biological examination in mice

136. Patient C with complaints of diarrhea with mucous, stabing pain in epigastric area , rumbling in the stomach, fever. According to his anamnesis he took preserved milk, eggs, salad of fresh vegetables. What is the disease most likely.

a. Shigellosisb. Yersiniosisc. *Salmonellosisd. Cholerae. Botulism

137. Patient B., 38 years old, came to the admission department with the complaints of fever up to 38 °C, vomiting 4 times in a day, diarrhea up to 6 times in a day. Durin objective examination revealed that B.P 125/75 mm Hg, pulse 80 per min, abdomen is soft,painful in the right iliac and epigastric area .According to anamnesis he took salad of boiled eggs and sausage. What treatment should be started first.

Page 199: intranet.tdmu.edu.uaintranet.tdmu.edu.ua/data/kafedra/internal/infect_desease... · Web viewBest method to treat diarrhoea in child is: intra venous fluide *ORS antibiotics bowel

a. Antibotulism serumb. Intravenous antibioticsc. Washing of the stomach and intestine, rehydration therapy , glucocorticoidsd. *Gastric lavage and washing of intestine, rehydraton therapy enterosorbentse. Treatment after getting of the laboratory test results

138. Patient 27 years old, complaints of headaches, weakness, pain in epigastric area, vomiting, diarrhea (9 times in a day; huge excrement, greenish colour). Before the appearance of disease he consumed raw eggs. Objectively: body temperature 38,8 °C. Tongue is white, pain in epigastric and umbilical region. The most likely diagnosis.

a. *Salmonellosisb. Cholerac. Dysenteryd. Viral gastroenteritise. Acute appendicitis

139. Patient K., 30 years old, came with complaints of nausea, vomiting, pain in epigastrium and paraumbilical areA. High body temperature, pale skin, dry. tongue covered with whitish-gray coating. quick pulse, low blood pressure. 2 hours before the onset of symptoms of the disease he ate in cafe. What is the diagnosis most likely.

a. Cholerab. Shigellosisc. Thypoid feverd. *Salmonellosise. Meningitis

140. Disease started acutely with the complaints of watery diarrhea, vomiting, cramps in the lower extremities. Objectively: slow voice, shunken eyes, quick pulse, low blood pressure, decrease of urination, weak cardiac sounds. In liver and spleen no changes. Put the correct diagnosis.

a. Salmonellosisb. Dysenteryc. Food poisoningd. *Cholerae. Typhoid fever

141. Patient 35 years old with complaints of increasing of temperature to 39 °C with chill, vomiting, pain in epigastric region, diarrhea with foul smell, abdominal cramps. 6 hours before onset of the disease ate raw eggs, fried potatoes with grinded meat, drank juice. To clarify the diagnosis serological diagnostic methods often used. When blood should be taken for the diagnostic procedures?

a. In the first day of illnessb. At the end of the first monthc. In 1st week in 3-4 daysd. *At the end of the 1st week from 7-10 dayse. During admission to the hospital

142. A patient was hospitalised after 4 hours of consumption of raw eggs, complaining frequent vomiting, abdominal pain, mostly in the right iliac area and umbilical areA. Objectively pale skin, dry tongue with gray coating, hypotension, tachycardiA. What tests should be carried out to confirm the diagnosis.

Page 200: intranet.tdmu.edu.uaintranet.tdmu.edu.ua/data/kafedra/internal/infect_desease... · Web viewBest method to treat diarrhoea in child is: intra venous fluide *ORS antibiotics bowel

a. General blood analysisb. Parasitological examinationsc. Microscopic examination of stoold. Biological teste. *Coproculture

143. Patient G., 22 years old, was hospitalised in an infectious department complaining of chills, stabing pain in epigastrium, diarrhea, temperature increased up to 39,6 °C, repeated vomiting. Feces copious, watery, without pathological changes. 4 hours before the onset of symptoms ate the meat, salad. What is the most appropriate drug to be prescribe.

a. Antidiarrheal drugsb. Flouroquinolone group of antibioticsc. Sulfanilamide group of antibioticsd. Desintoxication therapye. *Enterosorbents

144. Patient K., 40 years old, was hospitalised with the diagnosis of intestinal infection. Complaints of general weakness, headache and diarrheA. Symptomes appeared suddenly 2 hours after consumption of cake with custard. Suddenly there were chills, nausea, profuse repeated vomiting, frequent diarrhea, abdominal pain and short-term loss of consciousness, temperature increased to 38,6 °C. The preliminary dianosis?

a. Salmonellosis, localized form, gastroenteritis typeb. *Salmonellosis, localized form gastroenterocolitis typec. Salmonellosis, localized form enterocolitis typed. Salmonellosis, generalized forme. Salmonellosis, nosoparasitic

145. Patient K., 40 years old, hospitalised with the diagnosis of salmonellosis, with symptoms of rise of body temperature to 40 °C, repeated vomiting, diarrheA. Objectively: AP 60/20 mm Hg, pulse 120/minute, pale face, increased intestinal peristalsis, enlarged liver and spleen. What is the best treatment.

a. *Glucocorticoids, desintoxication therapy, antibacterial drugsb. Diuretic, desintoxication therapy, antibacterial drugsc. Enterorsorbents, diuretic, detsintoxication therapyd. Glucocorticoids, diuretic, desintoxication therapy, antibacterial drugs

146. Young farmer was diagnosed with balantidiasis. Drugs of choice would be all except:

a. Monomycinb. Ampicillinc. Aminarsond. *Gentamicine. Metronidazole

147. A patient complaints of severe abdominal pain, smelly watery diarrheA. with content of blood, . What kind of disease might be considered in the first place.

a. Amoebiasisb. Rotaviral gastroenteritisc. Giardiasisd. *Balantidiasis

Page 201: intranet.tdmu.edu.uaintranet.tdmu.edu.ua/data/kafedra/internal/infect_desease... · Web viewBest method to treat diarrhoea in child is: intra venous fluide *ORS antibiotics bowel

e. Cholera148. Patient P., 36 years old, a farmer is seriously ill, with high temperature, having

chills, headache, flatulence, nauseA. At present he is complaining of severe abdominal pain, diarrhea up to 15 times a day stool is liquid, smelly, with impurities mucous and blood. His eyes are sunken. The tongue is covered with white coating. On palpation- painful colon, increased liver. In blood – neutropil leukocytosis with a shift formula the left anemiA. On rectomanoscopy deep ulcers with irregular edges are observed, including along the folds. What is the diagnosis.

a. Shigellosisb. *Balantidiasisc. Nonspecific ulcerous colitis d. Amoebiasise. Colon tumor

149. A 29 years old patient T. works on a pig farm. She was hospitalized with symptoms of balantidiasis. What would be the drug of choice.

a. Benzylpenicillinb. Gentamycinc. Chloramphenicold. *Monomycine. Timogen

150. A 63 years old villager,is gravely ill: increased body temperature, headache, nausea, vomiting, abdominal pain, diarrheA. The patient was not seeking mediacal attention, and was taking levomycetin. The condition had not improved, the frequency of bowel movements was 8-12 times a day. At present he is complaining of severe abdominal pain, diarrhea, general weakness. The stool is liquid, smelly, with mixture of puss and blood. The patient is exosted his eyes are sunken. His tongue is covered with white coating. In blood analys is leukocytosis and anemia observed. On rectomanoscopy detected ulcers with jagged edges. What is the most likely diagnosis.

a. *Balantidiasisb. Shigellosisc. Nonspecific ulcerative colitisd. Rotavirus gastroenteritise. Colon tumor

151. Patient admitted in hospital with complaints of abdominal pain, increased body temperature, diarrheA. Stool is liquid, smelly, with mixture of mucus and blood. What kind of illness need to think about in the first place?

a. *Balantidiasisb. Rotavirus gastroenteritisc. Food poisoningd. Amoebiasise. Giardiasis

152. Patient S., 53, a resident of the village, heavelly ill – common symptoms of intoxication, intestinal dysfunction excrements are smelly, with mucous and blood. Abdomen on palpation is soft, but painful along spazmotic sygmoid bovell. On rectoromanoscopy - ulcers in diameter up to 10-20 mm along the folds of mucous

Page 202: intranet.tdmu.edu.uaintranet.tdmu.edu.ua/data/kafedra/internal/infect_desease... · Web viewBest method to treat diarrhoea in child is: intra venous fluide *ORS antibiotics bowel

membrane, swelling, moist edge, surrounded by a zone of flushing, the bottom is covered hyperemia and necrotic masses. What kind of illness need to think about?

a. *Balantidiasisb. Shigellosisc. Nonspecific ulcerous colitisd. Amoebiasise. Colon tumor

153. Student U. from India, 22, a month after returning home develops – liquid stool, weak abdominal pain. Over time of the disease has lost 6 kg. The next day heavy diarrhea 10 times a day, stool is like «raspberry jelly». Cramping in the abdomen growing in the bowel. The most likely diagnosis is:

a. Bowel tumorb. Salmonellosisc. Shigellosisd. *Amoebiasise. HIV/AIDS

154. Patient F., 25 years old, was hospitalized complaining of diarrhea – 10-15 times per day. A month ago was returned from Central AsiA. The general condition of relatively satisfactory. The temperature of the body subfebrile. Abdomen soft. Stool is liquid, high-level clear scanty mucus and blood. What is the most likely diagnosis:

a. *Amoebiasisb. Shigellosisc. Salmonellosisd. Tumor of bowele. Enterohemorrhagic escherihiosis

155. Patients with аmebiasis has been hospitalized. For specific therapy are used so-called, indirect and аmebiostic tissue, and preparations with universal action. What group does delagin belong to?

a. *Tissue аmoebiosticb. Indirect аmoebiosticc. Lines of аmoebiosticd. Preparations with universal actione. Does not belong to anyone

156. A student from Africa, 22 years old, liquid stool and weak stomach-aches appeared a month after returning from home. Lost of weight is 6 kg. Diarrhoe is abundant 10 times on a day, like “raspberry jelly”. Stomach-aches increase during defecation. What is most probable diagnosis?

a. Tumor of intestineb. Salmonellosisc. Shigellosisd. *Amoebiasise. HIV/AIDS

157. Patient O., 25 years old, hospitalized with complaints off diarrhea up to 10-15 times a day. Month ago he returned from Middle AsiA. The general condition is relatively satisfactory. The temperature of body is subfebrile.A stomach is soft, pain after motion of

Page 203: intranet.tdmu.edu.uaintranet.tdmu.edu.ua/data/kafedra/internal/infect_desease... · Web viewBest method to treat diarrhoea in child is: intra venous fluide *ORS antibiotics bowel

colon. Emptying is liquid with the considerable admixtures of glassy mucous and blood. What is most probable diagnosis?

a. *Amoebiasisb. Shigellosisc. Salmonellosisd. Tumor of intestinee. Enterohaemorrhagic esherihiosis

158. Patient U., 36 years old, farmer, became ill gradually. Indispositions, tormina, diarrhea (stool was 8-12 times per day). He was in one of countries with a hot climate during last 2 month. Temperature – 36,9 °C. At palpation – pain in iliac areas. Stool is liquid, with the admixtures of glassy mucus, look like to „raspberry jelly”. What is primery diagnosis?

a. *Amoebiasisb. Shigellosisc. Salmonellosisd. Tumor of intestinee. Enterohaemorrhagic esherihiosis

159. Patient C., 46 years old, periodically visited Egypt for a business purpose, became acutely ill. Complaints of a fever of 38 °C, with chills and shevering, pain in right hypochorium. Insignificant icterus.Liver is enlarged,some hyperechogeninic area . In blood neutrophilic leucocytosis, increased ESR. Purulent abscesses of liver were found out during ultrasound examination . What is previous diagnosis?

a. Legionellosisb. Echinnococcosisc. Ascaridosisd. Cancer of liver with metastasese. *Amebiasis

160. Student, 22 years old fell ill within a month after return from EthiopiA.Dull stomach-ache appeared and liquid emptying. Emptying is abundant up to 10 times a day, as “raspberry jelly” excrement, stomach-ache that increase during defecation. In times of illness lost 6 kg of body weight. Your diagnosis will be?

a. *Intestinal аmoebiasisb. Shigellosisc. Salmonellosisd. Tumor of intestinee. Intestinal esheriosis

161. Patient O., 47 years old, became ill acutely. Periodically visits Uzbekistan for a business purpose. Complaints of a fever of 39,4 °C with chills and shivering, and pain in right hypochorium, cough with sputum. Skin with an earthy tint. Signs of rightsided pneumoniA.The liver and spleen are enlarged.In blood neutrophilic leucocytosis. On X-ray infiltration of pulmonary tissues is found, at ultrasound abdominal examination numerous abscesses are present in a liver. What disease is it necessary to think about?

a. Echeriosisb. Alveolitisc. *Amebiasisd. Cancer of liver is with metastases in lungs

Page 204: intranet.tdmu.edu.uaintranet.tdmu.edu.ua/data/kafedra/internal/infect_desease... · Web viewBest method to treat diarrhoea in child is: intra venous fluide *ORS antibiotics bowel

e. Legionellosis162. Sick E., 21 years old, complaints of nausea, insignificant dull or rarely aggressive

pain in a stomach. Appetite is decreased. Periodically presence of flatulence, grumbling in a stomach and semi-fluid emptying. Sick during about 6 month. The diagnosis of chronic cholecystitis was proposed. In examination pallor of skin, lowering of mass of body and signs of vegetative dystonia are found.On palpation pain in еpigastric area also found. What disease you will think about?

a. *Giardiasisb. Salmonellosisc. Amebiasisd. Balantidiasise. Intestinal esherihiosis

163. Diagnosis of giardiasis is based on the discovering of cyst in fresh incandescence and vegetative forms in duodenal content. At diarhoea in incandescence can appear trophozoite on faecal microscopic examination and preparations with solution of Lugola and еоsin. Examination is more expedient to conduct 3-5 times at intervals 1-2 days. How to increase frequency of findings?

a. *By application of the formalin-еpiniphrine enriching methodsb. Cultivation in thermostatc. Cultivation in an anaerobic chamberd. By the method of floatation in bilious clear soupe. To sow on a nutrient medium

164. A patient complaints of severe abdominal pain, smelly watery diarrheA. with content of blood. What kind of disease might be considered in the first place.

a. Amoebiasisb. Rotaviral gastroenteritisc. Giardiasisd. *Balantidiasise. Cholera

165. Patient P., 36 years old, a farmer is seriously ill, with high temperature, having chills, headache, flatulence, nauseA. At present he is complaining of severe abdominal pain, diarrhea up to 15 times a day stool is liquid, smelly, with impurities mucous and blood. His eyes are sunken. The tongue is covered with white coating. On palpation- painful colon, increased liver. In blood – neutrophil leukocytosis with a shift formula the left with anemiA. On rectomanoscopy deep ulcers with irregular edges are observed, including along the folds. What is the diagnosis.

a. Shigellosisb. *Balantidiasisc. Nonspecific ulcerous colitis d. Amoebiasise. Colon tumor

166. A 63 years old villager, is gravely ill: increased body temperature, headache, nausea, vomiting, abdominal pain, diarrheA. The patient was taking levomycetin. The condition had not improved, the frequency of bowel movements was 8-12 times a day. At present he is complaining of severe abdominal pain, diarrhea, general weakness. The stool is liquid, smelly, with mixture of puss and blood. The patient is exosted his eyes are

Page 205: intranet.tdmu.edu.uaintranet.tdmu.edu.ua/data/kafedra/internal/infect_desease... · Web viewBest method to treat diarrhoea in child is: intra venous fluide *ORS antibiotics bowel

sunken. His tongue is covered with white coating. In blood analys is leukocytosis and anemia observed. On rectomanoscopy detected ulcers with jagged edges. What is the most likely diagnosis.

a. Colon tumor b. Shigellosisc. Nonspecific ulcerative colitisd. Rotavirus gastroenteritise. *Balantidiasis

167. A patient was admitted in hospital with complaints of abdominal pain, increased body temperature, diarrheA. Stool is liquid, smelly, with mixture of mucus and blood. What kind of illness need to think about in the first place?

a. *Balantidiasisb. Rotavirus gastroenteritisc. Food poisoningd. Amoebiasise. Giardiasis

168. A patient was admited in permanent establishment with complaints of stomach-ache, fever and diarrheA. Stool is liquid, stinking, with the admixtures of pus and blood. About what disease it is necessary to think about?

a. Food poisoningb. Rotaviral gastroenteritisc. *Balantidiasisd. Amebiosise. Giardiasis

169. Patient F, 53 years, habitant of a village, diseased acutely,the symptoms of general intoxication, dysfunction of bowels appeared, emptying became semi-fluid with the admixtures of pus and blood. The stomach at palpation is soft, morbid along belly and ascending colon with spasm. At rectoscopy are found ulcers with diameter upto 10-20 mm, located along the folds of mucus membrane, have fillings out, sharp edges places, surrounded by the area of hyperemia, bottom is covered by pus and necrotizing masses. What disease it is needed to think about?

a. *Balantidiasisb. Shigellosisc. Idiopatic ulcerous colitisd. Amebiasise. Intestinal cancer

170. A villager, was admited with complaints of severe pain in a stomach, and diarrheA. The stool is liquid with mixtures of blood. About what disease it is possible to think about?

a. Amebiosisb. Rotavirus gastroenteritisc. Gisrdiasisd. *Balantidiasise. Cholera

171. Sick P, 36 years, farmer, became acutely ill when the temperature of body rised. There were a chills, pain in chest, flatulence and nauseA. In the moment of admition

Page 206: intranet.tdmu.edu.uaintranet.tdmu.edu.ua/data/kafedra/internal/infect_desease... · Web viewBest method to treat diarrhoea in child is: intra venous fluide *ORS antibiotics bowel

grumbles about terminal diarrhoea with 15 times day. The stool is liquid, sticky with the admixtures of pus and blood. Tongue is covered with whitecovering. Stomach is distended, painfu colon, enlarged liver is palpable. In blood neutrophilic leucocytosis with the shift of formula to the left and anemiA. At a rectoscopy, deep ulcers with unequal, sharped areas which are placed along the folds was found. What disease will you think about?

a. Shigellosisb. Ulcerous colitisc. *Balantidiasisd. Amebiasise. Cancer of colon

172. Patient O., 29 years old, farmer came to a doctor with the signs of balantidiasis and was hospitalized. What is the best etiotropic drug:

a. Benzylpenicillinb. Gentamycinc. Laevomycitind. *Monomycine. Timogen

173. 63 years old patient became ill suddenly. The temperature of body raised, the stomach-ache with nausea and vomiting and diarrhea were observed. The patient’s condition did not improve even after giving him Levomycitin for long time. The diarrhea had proceeded again 8-12 times a day and became liquid. On admition the patient complaints of diarrhea and general weakness.The excrement is liquid stinking with the admixtures of pus and blood. The persons face is emaciated, eyes are hollow. Tongue is covered by white stratification. The stomach is distended, peristalsis after bowel movments. In blood leucocytosis and anaemic. At a rectoscopy ulcers with sharped, uneven edges was found. What is the most probable diagnosis?

a. *Amebiasisb. Shigellosisc. Idiopatic ulcerative colitisd. Rotavirus gastroenteritise. Cancer of colon

174. Sick E., 21 years old, complaints of nausea, insignificant dull or rarely aggressive pain in a stomach. Appetite is decreased. Periodically presence of flatulence, grumbling in a stomach and semi-fluid emptying. Sick during about 6 month. The diagnosis of chronic cholecystitis was proposed. In examination pallor of skin, lowering of mass of body and signs of vegetative dystonia are found.On palpation pain in еpigastric area also found. What disease you will think about?

a. *Giardiasisb. Salmonellosisc. Amebiasisd. Balantidiasise. Intestinal єcheriosis

175. Diagnosis of giardiasis is based on the discovering of cyst in fresh incandescence and vegetative forms in duodenal content. Trophozoites can be find during faecal

Page 207: intranet.tdmu.edu.uaintranet.tdmu.edu.ua/data/kafedra/internal/infect_desease... · Web viewBest method to treat diarrhoea in child is: intra venous fluide *ORS antibiotics bowel

microscopic examination. Examination is more informative if it conducts 3-5 times with 1-2 days intervals. How is possible to increase the frequency of findings?

a. *By application of the formalin-еpiniphrine enriching methodb. Cultivation in thermostatc. Cultivation in an anaerobic chamberd. By the method of floatation in bilious clear soupe. To sow on a nutrient medium

176. Veterinarian 57 years old, on the 3rd day of disease appealed to the doctor with complaints of headache, high temperature, pain in gastrocnemius muscles, icterus, dark urine and diminishing of its amount. Objectively: temperature of body is 38 °C, sclerotic big peteachia on overhead part of thorax, hepatospleenomegaly. Most reliable preliminary diagnosis will be:

a. *Leptospirosisb. Brucellosisc. Viral hepatitisd. Pseudotuberculosise. Trichinosis

177. A sick P., 40 in 2 weeks after eating of uncooked pork, purchased at the elemental market, has sudden rise of temperature upto 40 °C, myalgias appeared, oedematous, papular rash on extremities and trunk and dry cough. Accepted aspirin. General blood test: leucocytes– 12*109 /l , eosinophilia– 40 %. What disease it is possible to think about?

a. *Trichinosisb. Ascaridosisc. Leptospirosisd. Teniosise. Allergic reaction

178. Patient M., 32 years old, during a month has disturbing cough, increasing of temperature up to 38.0 °C. In anamnesis ascaridosis was found. On x-ray was found separate homogeneous infiltration without clear contour of bronchus. In blood test – еоsinophils 55 %. What is most probable diagnosis?

a. *Eosinophil infiltration (Lefler syndrome)b. Plaguec. Chronic bronchitis, acute phased. Acquired pneumoniae. Tumour of lungs

179. Woman 24, complaints of heaviness in the area of liver during 3 days, with an irradiation in the back. Signs of icterus are not present. Last menstruation – 5 weeks ago. Appointed ultrasonic research of abdominal region and small pelvis was, found marked progress uterogestation. Expansion of common biliary tract, inflammation of gall-bladder, bulge of its its wall, presence of crystal sediment, extended loops of small intestines are visualized. Investigation of colonoscopy found additional exogenic structures as a “ribbon”, which is displaced in intestine and changes form without distal acoustic shade. Most probable diagnosis is:

a. *Ascaridosisb. Partial intestinal uncommunicatingc. Acute calculary cholecystitis

Page 208: intranet.tdmu.edu.uaintranet.tdmu.edu.ua/data/kafedra/internal/infect_desease... · Web viewBest method to treat diarrhoea in child is: intra venous fluide *ORS antibiotics bowel

d. Choledocholithiasise. Cholecystitis

180. Sick, 37 years, acted on stationary treatment with complaints of bad feel, headache, increase of temperature upto to 39.5 °С, edema, pains in muscles of eyes, tongues, lower extremities. 2 weeks prior to beginning of disease used raw pork fat. Objectively: edema present. In a blood test: Leucocytes – 18.5*109 L, eosynophils – 22 %, basophils – 2 %, neutrophils – 56 %, stab neutrophils – 16 %, monocytes – 4 %. It is no pathological changes in urine. What is most probable diagnosis?

a. Leptospirosisb. Acute nephritisc. *Trichinosisd. Quinke’s edemae. Dermatomyositis

181. Patient 35 years old, was entered to the clinic with complaints of grumbling in a stomach, propensity to diarrhea, that are already present during 2 months. After the last two weeks at the act of defecation noticed moving snatches of flat struture. Has habit to use in a meal the raw stuffing from meat of home animal with spices. At colonoscopy there are numerous lateral folds which fills almost all segment.What is the drug of choice for treatment?

a. Mebendazolb. Combantrinc. Albendazold. *Phenasalume. Medamin

182. Patient A., 17 years old, complaints of headache, stomachache, nausea, enhanceable fatigue ability, insignificant pain and sometimes diarrheA. He was sick during 4 months. Day prior he was addressed to the hospital in incandescence noticed a round worm length 15cm. What will be the best treatment for him?

a. Levomycitinb. *Decarisc. Akrikhind. Ftalazole. Furazolidon

183. Patient, 27 years old, complaints of itching in perianal area, which is present in evening before sleep. It continues for 1-3 days and disappears independently, but reconvolense afterwards. Most probable diagnose will be:

a. Trichinosisb. *Enterobiosisc. Ascaridosisd. Helminthosise. Cystitis

184. Patient, 27 years, complaints of weakness, irritation, pain of muscles, fever, dry cough, pain in thorax, shortness of breath. In lung - dry and moist rales and wheezes are determined without dulling of percutory sound. In sputum are еosinophil, crystals of Charkot-Leyden with admixtures of blood. On the repeated X-ray of lungs shows plural

Page 209: intranet.tdmu.edu.uaintranet.tdmu.edu.ua/data/kafedra/internal/infect_desease... · Web viewBest method to treat diarrhoea in child is: intra venous fluide *ORS antibiotics bowel

еosinophilic infiltrations of different sizes which change a form and localization (Leffler syndrome).

a. Tuberculosis of lungsb. Acute pneumoniac. Bronchitisd. AIDSe. *Ascaridosis

185. Patient A, 29 years old, complaints of the decline of appetite, nausea, vomiting, stomach-ache, diarrhea, pain in the chest, weakness, dizziness, parahypnosis.There is present hypochromic anaemiA. Periodically used raw and half cooked raw pork. What most probable diagnosis?

a. Teniarinhosisb. Difilobotriosisc. *Teniosisd. Trichinellosise. Ascaridosis

186. Patient, 37 years old, complaints of pain in the chest, nausea, vomitting, periodic attacks of Grand-mal epilepsy and memory became worse. Papule rash in a skin. What most probable diagnosis?

a. Teniarinhosisb. Difilobotriosisc. Teniosisd. Trichinellosise. *Ascaridosis

187. Patient, 27 years, complaints of nausea, heaviness in epigastrium, stomach-ache, diarhoea and general weakness. An increase in appetite, mass of body is decreased. In blood moderate anaemia and high eosinophiliA. Noticed creeping out of segments with excrement. She likes to use raw meat. What is the most probable diagnosis?

a. *Teniarinhosisb. Difilobotriosisc. Teniosisd. Trichinellosise. Ascaridosis

188. Patient 30 years old, complaints of weakness, increasing of temperature up to 39.2 °C, pain in muscles, fatigue, edema present. 2 weeks ago he was used pork with the flab of meat. Edema is present,an enlarged liver is palpable. The BP is 90/60 mm of Hg. In blood analysis: leucocytes – 12,4*109, eos – 19 %, young neutrophills – 2 %, neutrophills – 59 %, lymphocytes– 16 %, mon – 4 %. Specify the most probable diagnosis.

a. Leptospirosisb. Fluc. *Trichinosisd. Typhoid fevere. Spotted fever

189. Sick, 45 years old was sick for 17days, lives in a village. The sick grumbles about strong stomach-ache, frequent 15 times and more on a day emptying, nausea, vomit, headache and fever. Objectively: person is pale, eyes are hollow, the turgor of skin is

Page 210: intranet.tdmu.edu.uaintranet.tdmu.edu.ua/data/kafedra/internal/infect_desease... · Web viewBest method to treat diarrhoea in child is: intra venous fluide *ORS antibiotics bowel

sharply decreased.Tongue is dry assessed white coating, stomach is sharply blown away, at palpation pain takes place in the area of colons. Liver is moderately enlarged. At hemanalysis leucocytosis 16*109 L, eosinophills 1 %, ESR – 7mm/hr. Change of leucocyte formula takes place. Emptying with addition of bloods, has a pungel smell. At rectoscopy at the area sigmoid and bowels was found deep ulcers with undetermined edges. What most probable pathology which predetermines such picture?

a. Amebiasisb. *Balantidiasisc. Nonspecific ulcerous colitisd. Tumour of rectume. Food poisoning

190. A sick child 8 years old complaints of itching in the perinium areA. Child creaks teeth during sleeping, sleeps bad. Objectively: sufficient nourishment, pallor of skin, tongue is insignificantly covered with white patches, stomach is soft, accessible for palpationis painless. The lowering of haemoglobin level takes place. What is most probable pathology which predetermines such picture?

a. *Enterobiasisb. Salmonellosisc. Ascaridosisd. Giardiasise. Trichinosis

191. A sick 26 years appeared to the therapeutic department with complaints of the itching of skin, weakness, liquid emptying diarhoea 1-2 times a day. Objectively: sufficient nourishment. On the skin has pouring out red color as “hives” which has linear character. In blood eosinophilic reaction of blood – 28 %. Lives in rural area What is the most probable pathology which predetermines such picture?

a. Salmonellosisb. *Strongyloidosisc. Food poisoning d. Giardiasise. Trihocephallosis

192. A sick 20 years arrived from Western Siberia complaint of pain in the area of liver and gall-bladder, bitter taste in mouth. Objectively: abdomen is soft, accessible for palpation, at palpation pain takes place in an area of bilious system. At blood analysis еosinophills – 23 %. She had eaten fish. What is the most probable pathology which predetermines such picture?

a. *Opistorhosisb. Ascaridosisc. Trihocephallosisd. Giardiasise. Trichinosis

193. Patient 28 years old, complaints of an increased appetite, weakness, decreased physical activity tolerance, pain in stomach. Objectively: colour of skin is pale, signs of meteorism,and glossitis. In blood thereis diminished amount of red cells, leucocytes, thrombocyte and hypochromic anaemiA. It is known from epideminological anamnesis,

Page 211: intranet.tdmu.edu.uaintranet.tdmu.edu.ua/data/kafedra/internal/infect_desease... · Web viewBest method to treat diarrhoea in child is: intra venous fluide *ORS antibiotics bowel

that the patient had spent 4th months on Volga river and had eaten the salted fish and caviar. What is the most probable pathology which predetermines such picture?

a. Teniosisb. Anaemiac. *Difilobotriosisd. Opisthorchosise. Helminthosis

194. A 5 years old girl complaints of headache, decreased appetite, weakness, nausea, vomiting, bitter taste in mouth, stomach-ache and periodic diarrheA. Objectively: decreased nourishment status, on her tongue there is white coating observed. The stomach is soft, accessible for palpation, and painful in the area of gall-bladder. No Change in blood analysis was found. According to the mother the same symptoms were observed with her son a month ago. What is the most probable pathology which predetermines such picture?

a. Amebiasisb. Balantidiasisc. Intestinal trichomonosisd. *Giardiasise. Dysbacteriosis

195. A 23 years old patient complaints of weakness, nausea, periodic presence of segments of helminth in the stool. In anamnesis the patient had eaten undercooked meat. Faeces were sent for microscopic examination. The bovin solitaire was found in the stool. What would be the drug of choice?

a. *Biltritsidb. Pyrantelumc. Piperazinumd. Decarise. Fazizhin

196. A 24 years old engineer from Donetsk, has spent one month in India where he drunk unboiled water. After arriving home he has become ill. He appeared to the doctor with complaints of fever, weakness, pain in stomach, diarrhea - 12-15 times a day with mucous and blood (like raspberry jelly stool). Objectively: the state is relatively satisfactory, appetite became worse, tongue is coated with white patches.On deep palpation of abdomen patient complaints of pain especially in his right half and hypochodrium areA. Liver and spleen are not changed. No change was found in blood analysis. On rectoscopy clear mucous and hyperemia of mucous membrane in rectum with ulceration in sigmoid colon were found. The stool test gave the growth of pathogenic florA. What most probable pathology which predetermines such picture?

a. Strongyloidosisb. *Amebiasisc. Ulcerative colitisd. Balantidiasise. Food poisoning

197. Patient K., 30 years old, came with complaints of nausea, vomiting, pain in epigastrium and paraumbilical areA. High body temperature, pale skin, dry. tongue

Page 212: intranet.tdmu.edu.uaintranet.tdmu.edu.ua/data/kafedra/internal/infect_desease... · Web viewBest method to treat diarrhoea in child is: intra venous fluide *ORS antibiotics bowel

covered with whitish-gray coating. Quick pulse, low blood pressure. 2 hours before the onset of symptoms of the disease he ate in cafe. What is the diagnosis most likely.

a. Cholerab. Shigellosisc. Thypoid feverd. *Salmonellosise. Meningitis

198. Disease started acutely with the complaints of watery diarrhea, vomiting, cramps in the lower extremities. Objectively: slow voice, shunken eyes, quick pulse, low blood pressure, decrease of urination, weak cardiac sounds. In liver and spleen no changes. Put the correct diagnosis.

a. Salmonellosisb. Dysenteryc. Food poisoningd. *Cholerae. Typhoid fever

199. A Pakistani, 30 years old, severly ill: with the complaints of frequent diarrhea like rice water. Objectively: body temperature 35,4 °C, skin of peripheral parts of the body cold to the touch, acrocyanosis, xerostomia, sharply decreased skin elasticity and turgor. What should be done to assess the degree of dehydration.

a. Measurement of central venous pressureb. Determination of urea and creatinine in bloodc. *Funduscopyd. Plain X-ray film of abdomene. Determination of specific gravity of blood plasma

200. Patient N., 30 years old, works in a restaurant, admitted to the hospital complaining of unbearable pain in the abdomen, increase of temperature 38,8 °C, nausea, vomiting, diarrhea of dark-green colour, He took eggs, mushrooms, canned food. Objective state: general condition is moderately sever, skin and mucous membranes are dry, pain in epigastric region. Name the disease.

a. Shigellosisb. Yersiniosisc. *Salmonellosisd. Cholerae. Rotaviral infection

201. Patient 35 years old with complaints of increasing of temperature to 39 °C with chill, vomiting, pain in epigastric region, diarrhea with foul smell, abdominal cramps. 6 hours before onset of the disease ate raw eggs, fried potatoes with grinded meat, drank juice. To clarify the diagnosis of serological diagnostic methods often used RNGA with the salmonella groupdiagnostic tools and RA (Vidal test) . When blood should take for the diagnostic procedures?

a. In the first day of illnessb. At the end of the first monthc. In 1st week in 3-4 daysd. *At the end of the 1st week from 7-10 dayse. During admission to the hospital

Page 213: intranet.tdmu.edu.uaintranet.tdmu.edu.ua/data/kafedra/internal/infect_desease... · Web viewBest method to treat diarrhoea in child is: intra venous fluide *ORS antibiotics bowel

202. A patient was hospitalised after 4 hours of consumption of raw eggs, complaining frequent vomiting, abdominal pain, mostly in the right iliac area and umbilical areA. Objectively pale skin, dry tongue with gray coating, hypotension, tachycardiA. What tests should be carried out to confirm the diagnosis.

a. General blood analysisb. Parasitological examinationsc. Microscopic examination of stoold. Biological teste. *Coproculture

203. Patient G., 22 years old, was hospitalised in an infectious department complaining of chills, stabing pain in epigastrium, diarrhea, temperature increased up to 39,6 °C, repeated vomiting. Feces copious, watery, without pathological changes. 4 hours before the onset of symptoms ate the meat, salad. What is the most appropriate drug to be prescribe.

a. Antidiarrheal drugsb. Flouroquinolone group of antibioticsc. Sulfanilamide group of antibioticsd. Desintoxication therapye. *Enterosorbents

204. Patient K., 40 years old, was hospitalised with the diagnosis of intestinal infection. Complaints of general weakness, headache and diarrheA. Symptomes appeared suddenly 2 hours after consumption of cake with custard. Suddenly there were chills, nausea, profuse repeated vomiting, frequent diarrhea, abdominal pain and short-term loss of consciousness, temperature increased to 38,6 °C. The preliminary dianosis?

a. Salmonellosis, localized form, gastroenteritis typeb. *Salmonellosis, localized form gastroenterocolitis typec. Salmonellosis, localized form enterocolitis typed. Salmonellosis, generalized forme. Salmonellosis, nosoparasitic

205. Patient K., 40 years old, hospitalised with the diagnosis of salmonellosis, with symptoms of rise of body temperature to 40 °C, repeated vomiting, diarrheA. Objectively: AP 60/20 mm Hg, pulse 120/minute, pale face, increased intestinal peristalsis, enlarged liver and spleen. What is the best treatment.

a. *Glucocorticoids, desintoxication, antibacterial therapyb. Diuretic, desintoxication therapy, antibacterial drugsc. Enterorsorbents, diuretic, detsintoxication therapyd. Glucocorticoids, diuretic, desintoxication therapy, antibacterial drugse. Glucocorticoids, diuretic, antibacterial drugs

206. Patient P., 36 years old, a farmer is seriously ill, with high temperature, having chills, headache, flatulence, nauseA. At present he is complaining of severe abdominal pain, diarrhea up to 15 times a day stool is liquid, smelly, with impurities mucous and blood. His eyes are sunken. The tongue is covered with white coating. On palpation- painful colon, increased liver. In blood – neutropil leukocytosis with a shift formula the left anemiA. On rectomanoscopy deep ulcers with irregular edges are observed, including along the folds. What is the diagnosis.

a. Shigellosis

Page 214: intranet.tdmu.edu.uaintranet.tdmu.edu.ua/data/kafedra/internal/infect_desease... · Web viewBest method to treat diarrhoea in child is: intra venous fluide *ORS antibiotics bowel

b. *Balantidiasisc. Nonspecific ulcerous colitis d. Amoebiasise. Colon tumor

207. A 29 years old patient T. Has been working on a pig farm. She was hospitalized with symptoms of balantidiasis. What would be the drug of choice.

a. Benzylpenicillinb. Gentamycinc. Chloramphenicold. *Monomycine. Timogen

208. Patient C., 35 years old, the disease has started severe chills, raising the temperature to 39 oC, vomiting, pain in epigastric region, diarrhea with water-smelly bowel movement. Over 6 hours after eating raw eggs, potatoes with braised meat, drank juice. What kind of agent is likely to cause this condition.

a. *Salmonellab. Vibrio cholerac. Enteric stickd. Campylobactere. Shigella

209. Patient 25 years, received complaints of double vision of the eyes, a decline of view, shortness of breath. Before the disease eat mushrooms home preservation. Objective: pallor, wide pupils with a weak reaction to light, dry mouth, a violation of swallowing, flatulence, delay of stool. What is preliminary diagnosis.

a. Leptospirosisb. Yersinioz c. *Botulismd. Giardiasise. Salmonellosis

210. Patient C., 22 years old, hospitalized in an infectious department complaining of chills, temperature increase to 38,5 oC, vomiting, pain in epigastrii, frequent stool. 7 hours before the disease, ate raw eggs, potatoes with braised meat, drink tomato juice. What kind of organism most likely to cause disease.

a. Enterovirusb. *Salmonellac. Enteric stickd. Staphylococcuse. C. perfringens

211. Emergency ambulance delivered in infectious hospital girl N., 17 years old, complaining of headaches, expressed general weakness, pain in epigastrium, repeated vomiting, diarrhea to 8 times a day greenish color. She was used 2 raw chicken eggs. Body temperature 39 °C, the tongue covered with white coat, moderately cramps, pain in abdomen and sigmoid areA. What is the most likely diagnosis.

a. Acute appendicitisb. Crohn’s diseasec. Shigellosis

Page 215: intranet.tdmu.edu.uaintranet.tdmu.edu.ua/data/kafedra/internal/infect_desease... · Web viewBest method to treat diarrhoea in child is: intra venous fluide *ORS antibiotics bowel

d. Viral gastroenteritise. *Salmonellosis

212. Patient L., 32 years old, was admitted to the infectious office complaining of chills, headache, expressed general weakness, cutting pain in epigastrii, diarrheA. The temperature of 39 °C, repeated vomiting. Excrement abundant, greenish color, without pathological impurities. Over 4 hours to illnesses used the meat salad in the factory canteen. Which drugs will be most effective for the treatment of this patient.

a. Antibotulism serumb. Salt and colloid solutionsc. Sulfanilamidesd. *Antibioticse. Enterosorbent

213. Patient B., 55 years old, was hospitalized in an infectious hospital with complaints of frequent vomiting, liquid excrements, abdominal pain, mainly in the right iliac area and epigastrium. Pale skin, dry tongue with a touch of gray, hypotension, tachycardiA. The doctor was suspected salmonellA. What laboratory test is need to perform.

a. General blood analysisb. Parasytoscopy of blood smearc. Microscopy of stoold. *Coproculturee. Biological test

214. Patient C., 30 years old, was hospitalized with a preliminary diagnosis of salmonellosis. Body temperature – 40 °C, repeated vomiting, profused diarrheA. Hypotension (BP 55/30 mm Hg), expressed tachycardia (pulse to 120/min), face pale, pain mainly in right iliac area and epigastrium, hepatosplenomegaly were found. What treatment tactic will be most effective in this case.

a. Glucocorticoids, crystalloid solutions, antibiotics, diureticsb. Cardiac glycosides, colloid solutions, antibioticsc. Sorbents, diuretics, desintoxicationd. Dopamine, colloid solutions, antibacteriale. *Glucocorticoids, crystalloid solutions, antibiotics

215. Patient F., 25 years old, was hospitalized in the infectious hospital with complaints of frequent vomiting, liquid stool, abdominal pain in epigastrium. Pale skin, dry tongue with a grey cover, hypotension, tachycardiA. The doctor was suspected salmonellA. What serologic study is need to perform.

a. Microprecipitation reactionb. RA (Widal)c. *RIGA with Salmonella diagnosticumd. RIGA with Shigella diagnosticum and RA (Widal) with paired serume. RKC

216. Patient N., 22 years old, was admitted to the infectious office complaining of chills, headache, expressed general weakness, cutting pain in sigmoid area, diarrheA. The temperature of the body of 39,3 °C, vomiting. Faces watery. Patient was used the salad with sausage and eggs. For further diagnosis doctor was prescribed RIGA with salmonella diagnosticums. When it’s necessary to take blood for the investigation.

a. At the 1st and 2nd days of illness

Page 216: intranet.tdmu.edu.uaintranet.tdmu.edu.ua/data/kafedra/internal/infect_desease... · Web viewBest method to treat diarrhoea in child is: intra venous fluide *ORS antibiotics bowel

b. Once in recovering periodc. *At the end of the 1st week and after 7-10 daysd. At the end of the 1st week and after 3-4 dayse. Once in acute period

217. Patient B., 32 years old, an employ of poultry was hospitalized with complaints of pain in abdomen, mostly in sigmoid area, fever up to 38,8 oC, nausea, vomiting, diarrhea with dark-green colour of stool, without any pathological changes. Skin and mucous membranes are dry, moderate tension in epigastric region and pain in the right iliac areA. The most likely diagnosis.

a. Acute appendicitisb. Cholerac. Dysbacteriosisd. *Salmonellosise. Rotavirus gastroenteritis

218. 3 patients were delivered with symptoms of fever, diarrhea and vomiting. Excrements dark-green, without any pathological changes. All patients together took part in the preparation of food and have used eggs, meat salad. The most likely diagnosis.

a. Cholerab. Botulismc. Dysbacteriosisd. *Salmonellosise. Shigellosis

219. Patient L., 33 years old, was admitted to the hospital with the complaints of untolarable pain in the abdomen, mostly in sigmoid region, fever up to 38,8 oC, nausea, vomiting , diarrhea of dark-green colour in every1-1,5 hour, without admixture of blood and mucus. The condition progressively deteriorated, increasing hypotension, paleness, weak cardiac sounds, decreased intestinal peristalsis with moderate tension in epigastric region and pain in the right iliac region. What tactics of treatment?

a. Glucocorticoids, infusion of salt solutions, antibiotics intravenously, diureticb. Cardiac glycosides, infusion colloidal solution, antibacterial drugsc. Enterosorbents, diuretic, desintoxication therapyd. Mezaton 1% / in drops, infusion of salt solutions, antibacterial drugs orallye. *Glucocorticoids, infusion of salt solutions, antibiotics intravenously

220. Patient 45 years old, became ill within 10 hours after consumption of grinded meat. Complaints of vomiting, pain in epigastric region, diarrhea of green color, increasing of temperature up to to 39 °C. During objective examination revealed that: patients general condition is sever, pain in the muscles of the lower limbs, pale face, acrocyanosis. Pulse 130 per min,. B.P 70/40 mm Hg. weak cardiac sounds, dry tongue, brown-whitecovering . abdomen cramps, painful iliac and epigastric regions. What is the most likely diagnosis.

a. Shigellosisb. Cholerac. *Salmonellosisd. Viral gastroenteritise. Eshericiosis

221. 5 patients during 15 hours were admited In a hospital. All of them have similiar complaints of mild diarrhea and vomiting developed ,diplopia, midriasis, visual

Page 217: intranet.tdmu.edu.uaintranet.tdmu.edu.ua/data/kafedra/internal/infect_desease... · Web viewBest method to treat diarrhoea in child is: intra venous fluide *ORS antibiotics bowel

disturbance, difficult swallowing, dry mouth, difficulty in breathing. All patients together joined in a dinner party, used different dishes, including meat, salad, canned mushroom. The most likely diagnosis.

a. Cholerab. *Botulismc. Typhoid feverd. Salmonellosise. Rotaviral gastroenteritis

222. Patient G., 24 years old, 6 hours after taking his breakfast with boiled eggs, a cup of coffee appeared chill, fever up to 38,8 °C, pain in left inguinal area, nausea,vomiting, rumbling in the stomach, then frequent diarrhea, feces copious, watery, mudy-green. What is the most likely diagnosis.

a. Cholerab. Botulismc. *Salmonellosisd. Typhoid fevere. Shigellosis

223. Patient U., who was arrived from the Crimea, diarrhea appeared in an intervel of 1-1.5 hour. with copious watery increments, without admixtures of mucous and blood. He was admitted in an infectious dipartment with the diagnosis of “acute intestinal infection”. What laboratory tests are necessary to confirm the etoilogical diagnosis.

a. General blood analysisb. Bacteriological blood analysisc. Microscopical analysis of stoold. *Coprograme. The biological examination in mice

224. Patient C with complaints of diarrhea with mucous, stabing pain in epigastric area, rumbling in the stomach, fever. According to his anamnesis he took preserved milk, eggs, salad of fresh vegetables. What is the disease most likely.

a. Shigellosisb. Yersiniosisc. *Salmonellosisd. Cholerae. Botulism

225. Patient B., 38 years old, came to the admission department with the complaints of fever up to 38 °C, vomiting 4 times in a day, diarrhea up to 6 times in a day. Durin objective examination revealed that BP 125/75 mm Hg, pulse 80 per min, abdomen is soft,painful in the right iliac and epigastric area .According to anamnesis he took salad of boiled eggs and sausage. What treatment should be started first.

a. Antibotulism serumb. Intravenous antibioticsc. Washing of the stomach and intestine, rehydration therapy , glucocorticoidsd. *Gastric lavage and washing of intestine, rehydraton therapy enterosorbentse. Treatment after getting of the laboratory test results

226. Patient 27 years old, complaints of headaches, weakness, pain in epigastric area, vomiting, diarrhea (9 times in a day; huge excrement, greenish colour). Before the

Page 218: intranet.tdmu.edu.uaintranet.tdmu.edu.ua/data/kafedra/internal/infect_desease... · Web viewBest method to treat diarrhoea in child is: intra venous fluide *ORS antibiotics bowel

appearance of disease he consumed raw eggs. Objectively: body temperature 38,8 °C. Tongue is white, pain in epigastric and umbilical region. The most likely diagnosis.

a. *Salmonellosisb. Cholerac. Dysenteryd. Viral gastroenteritise. Acute appendicitis

227. Patient K., 30 years old, came with complaints of nausea, vomiting, pain in epigastrium and paraumbilical areA. High body temperature, pale skin, dry. tongue covered with whitish-gray coating. quick pulse, low blood pressure. 2 hours before the onset of symptoms of the disease he ate in cafe. What is the diagnosis most likely.

a. Cholerab. Shigellosisc. Thypoid feverd. *Salmonellosise. Meningitis

228. Disease started acutely with the complaints of watery diarrhea, vomiting, cramps in the lower extremities. Objectively: slow voice, shunken eyes, quick pulse, low blood pressure, decrease of urination, weak cardiac sounds. In liver and spleen no changes. Put the correct diagnosis.

a. Salmonellosisb. Dysenteryc. Food poisoningd. *Cholerae. Typhoid fever

229. A Pakistani, 30 years old, severly ill: with the complaints of frequent diarrhea like rice water. Objectively: body temperature 35,4 °C, skin of peripheral parts of the body cold to the touch, acrocyanosis, xerostomia, sharply decreased skin elasticity and turgor. What should be done to assess the degree of dehydration.

a. Measurement of central venous pressureb. Determination of urea and creatinine in bloodc. Funduscopyd. X-rays examination of abdomene. *Determination of specific gravity of blood plasma

230. Patient N., 30 years old, works in a restaurant, admitted to the hospital complaining of unbearable pain in the abdomen, increase of temperature 38,8 oC, nausea, vomiting, diarrhea of dark-green colour, He took eggs, mushrooms, canned food. Objective state: general condition is moderately sever, skin and mucous membranes are dry, pain in epigastric region. Name the disease.

a. Shigellosisb. Yersiniosisc. *Salmonellosisd. Cholerae. Rotaviral infection

231. Patient 35 years old with complaints of increasing of temperature to 39 °C with chill, vomiting, pain in epigastric region, diarrhea with foul smell, abdominal cramps. 6

Page 219: intranet.tdmu.edu.uaintranet.tdmu.edu.ua/data/kafedra/internal/infect_desease... · Web viewBest method to treat diarrhoea in child is: intra venous fluide *ORS antibiotics bowel

hours before onset of the disease ate raw eggs, fried potatoes with grinded meat, drank juice. To clarify the diagnosis serological diagnostic methods often used. When blood should be taken for the diagnostic procedures?

a. In the first day of illnessb. At the end of the first monthc. In 1st week in 3-4 daysd. *At the end of the 1st week from 7-10 dayse. During admission to the hospital

232. A patient 20 years old, had treated himself concerning an acute respiratory disease for 5 days, but marked no displays of respirator syndrome. Last 2 days temperature has been normal, appetite disappeared, however, appeared pain in epigastrium with nausea, and urine turned dark. About what illness is it possible to think?

a. Hepatitis Bb. Infectious mononucleosisc. Pseudotuberculosisd. Leptospirosise. *Hepatitis A

233. A patient 28 years old, an injection drug addict, complaints of dull pain in right subcostal region, weakness, decline of appetite, pain in joints. The symptoms have been present for 2 weeks. At examination: the icterus of skin and scleras observed. The liver and spleen are enlarged. Urine is dark, excrements are discoloured. What is preliminary diagnosis?

a. Hepatitis Ab. *Hepatitis Bc. Chronic cholecystitisd. Toxic hepatitise. HIV-infection

234. A patient 25 years old got sick suddenly after chills and temperature increase up to 38,0 °C. There was vomiting 2 times. Moderate pharyngeal pain appeared at swallowing with stomach discomfort. Signs of bursitis observed on knee and elbow joints. The light icterus of sclera and skins of joints on 4th day, on the skin of lateral surfaces of trunk, forehead and lower extremities. The small bright red papular rash appeared, more concentrated in natural folds.Liver is enlarged, the tongue is raspberry like appearence. Preliminary diagnosis would be:

a. Hepatitis Ab. Hepatitis Bc. *Pseudotuberculosisd. Scarlet fevere. Infectious mononucleosis

235. A 19 years old patient was diagnosed with hepatitis B. After violation of diet and nervous stress the condition of patient got worse: increasing of icterus intensity , pulse 110/min, BP 80/50 mm Hg. «Coffee-grounds» vomiting . Decreasing of the liver size, pain during palpation. What complication would you think about?

a. Infectious-toxic shock b. Acute kidneys insufficiency c. Acute adrenal insufficiency

Page 220: intranet.tdmu.edu.uaintranet.tdmu.edu.ua/data/kafedra/internal/infect_desease... · Web viewBest method to treat diarrhoea in child is: intra venous fluide *ORS antibiotics bowel

d. Hemolitic icteruse. *Acute hepatic insufficiency

236. A patient C., 43 years old, has been treated for 5years . During the last hospitalization a liver-biopsy was conducted. Histologicaly there is bulb dystrophy and necrosis of hepatocytes, with leucocyts infiltration, and pericellular fibrosis. What diagnosis is most credible in this case?

a. *Alcohol liver cirrhosisb. Viral hepatitis Bc. Viral hepatitis Cd. Cryptogenic hepatitise. Fatty dystrophy of liver

237. Patient B., 23 years old. Objectively: skin is yellow, icterus of sclerA. Pulse 66/min, BP 120/80. Indexes of AlAT, AsAT, and Tymol tests are normal. General billirubin is 34 mmol/l due to indirect fraction. Specify the most credible diagnosis.

a. Chronic toxic hepatitisb. Chronic cryptogenic hepatitisc. Chronic hepatitis Cd. *Zhilber’s syndromee. Chronic B hepatitis

238. A nurse got sick on 16.09: general weakness, nausea, vomiting, pain in joints appeared. 21.09 urine turned dark, and skin yellow. 21.09 she was hospitalized. Objectively: general status worse, temperature 38,2 °C, expressed skin and mucous membranes jaundice. Liver palpated 3 cm below the costal arc. 26.09 stomach-aches and increased sleepiness with disorientation in time and place were appeared. 27.09 haematomas in the places of injections also appeared. Hepatic breath was noticed. Light edema of feet and lumbar region were observed. The liver was not palpable. Development of what syndrome can be?

a. Meningoencephalitisb. *Hepatic encephalopathy c. Acute nephrosonephritisd. Pancreatitise. IDS syndrom

239. A patient B., 52 years old, for 3 month complaints of nausea, periodic vomiting, swelling of stomach, weakness, loss of weight up to 12 kg, consistency of stool is chainging. During some days consciousness is entangled, somnolence, allolalia, general weakness are expressed. Temperature of body 37,4 °C. Icterusis on the skin. Tremor of brushes and nystagmus is marked. Pupils are narrowed, with a slow reaction on light, increase of tendon reflexes. Ps 112/min, unrhythmical. BP 90/65 mm Hg. A liver is dense +2. Laboratory tests: Hb 86 g/L, Leuc 4,2*109/L, ESR 18 mm/hour, glucose 3,4 mmol/l, general bilirubin 56,7 mm/L, albumen 52 g/L, K+ 3,2 mmol/l, AsAT 0,62; AlAT 0,84; prothrombin index-58 %; рН blood 7,3. What is previous diagnosis?

a. Hepatocirrhosisb. Vilsona-Konovalov diseasec. Cancer of liverd. *Endogenous hepatic encephalopathye. Meygs disease

Page 221: intranet.tdmu.edu.uaintranet.tdmu.edu.ua/data/kafedra/internal/infect_desease... · Web viewBest method to treat diarrhoea in child is: intra venous fluide *ORS antibiotics bowel

240. A patient, 17 years, complaints of a weakness, worsening of appetite, nausea, painfull in right subcostal area, dull ache in the body, dark color of urine, temperature to 37,8 °C. She stood in contact with sick on icterusis 1,5 months ago. Objectively: yellow colour of the skin and visible mucous membranes. The liver is increased to 1,5 cm. Unsignificant enlargment of spleen. What is your preliminary diagnosis?

a. Viral hepatitis Bb. Infectious mononucleosisc. *Viral hepatitis Ad. Leptospirosise. Pseudotuberculosis

241. Before the appearance of jaundice in a patient 16 years old, during 2 days there were an increasing of the body temperature (38,5 °C), headache, dull ache disturbed whole body. Name the variant of pre-icteric period of viral hepatitis for this patient?

a. *Influenza-likeb. Astenovegetativec. Artralgicd. Dyspeptice. Allergic

242. A patient T., 28 years, appilled due to worsening of common status at the seventh day to the infectious department concerning viral hepatitis. Nausea and vomiting were present, hepatic smell appeared, increasing of the liver sizes. What biochemical index does allow to suspect complication in the patient?

a. Increasing activity of AlATb. Increasing of bilirubin, increasing activity of AlATc. Increasing timol testd. Decreasing of protrombin, increasing of bilirubine. *Decreasing of AlAT

243. A sick P., 54 years old, complaints of dull pain in right subcostal area, bad taste in the mouth, itching of the skin, increasing of abdomen. After the liver puncture fatty dystrophy of hepatocytes, eccentric placing of veins in a hepatic lobule were found. What laboratory index are most characteristic in this case?

a. Alkaline phosphotaseb. Hemodiastasec. *Level of transaminasesd. Glucose of bloode. Creatinphosphokinase

244. Direct bilirubin is increased , in urine there is significant increase of bilirubin and urobilin, increasing of stercobilin of excrements. What is the type of icterus?

a. Haemoliticb. *Parenhimatousc. Transportd. Extralivere. Mechanical

245. On the average 15 to 30 % of all population of the planet suffer from some pathology of liver. Prevalence of hepatitis and cirrhosis in the European countries is about

Page 222: intranet.tdmu.edu.uaintranet.tdmu.edu.ua/data/kafedra/internal/infect_desease... · Web viewBest method to treat diarrhoea in child is: intra venous fluide *ORS antibiotics bowel

1 % of adults. Annually in the world there are about 2 million people with acute viral hepatitis. What % of all cases will develop chronic form.

a. 100 %b. 50 %c. 25 %d. *10 %e. 1 %

246. A patient, 24, complaints of pains in right subcostal area, increasing after-meal, nausea, increase temperature of body to 37,7 °C, icterus, pains in large joints. He is ill from 8 months. Suffers a nonspecific ulcerative colitis. Hepatosplenomegaly. ESR 47 mm/hr, general bilirubin level is 86,1mmol/lt, direct-42,3 mmol/lt. In blood found out antibodies against smooth muscles. General albumen of 62 gram/lt, albumin б. 40 %, globulin. 60 %, gamma globulins 38 %. Not found out the markers of viral hepatitis. On USD diameter of portal vein is 1 cm What will be your diagnosis?

a. Primary biliary cirrhosisb. Zhilber syndromec. Viral hepatitisd. Hemochromatosise. *Autoimmune hepatitis

247. Woman of 22 years old, in the 7th month of pregnancy felt ill acutely in 3 weeks after arrival from Turkmenistan. An icterus, untraceable vomiting, pain in right subcostal region, hemmorhaeges on the skin, appeared after three-day fever. Most probable diagnosis will be:

a. Acute fatty hepatosis of pregnancyb. Cholestatic hepatosis of pregnantc. *Viral hepatitis of Ad. Viral hepatitis Be. Acute sepsis

248. Weakness appeared in a patient, decrease in an appetite, began pains in the joints of extremities, felling of weight in right subcostal region, vomiting. Urine became dark in 12 days, and in a day – sclera and skin turned yellow. It was discovered in an immunogram: antuIgG -HAV (+), HBsAg (+), antiIgМ -НBcorAg (+), anti-НCV (–), anti-НDV (–). To the patient 3 months back was extracted a tooth. Most probable diagnosis?

a. *Viral hepatitis Bb. Viral hepatitis Ac. Viral hepatitis Cd. Viral hepatitis De. Viral hepatitis E

249. A patient 42 years, radiologist, entered surgical department with bleeding from the mouth, was admitted to the gastroenterological departments . He considered himself healthy. 20 years ago had haemotransfusion. Fibrogastroduodenoscopy shows venous dilation of the oesophagus of the III degree. Most probable etiologic reason of disease:

a. Autoantibodies against hepatocytesb. Viral hepatitis Ac. X-ray irradiationd. *Viral of hepatitis B

Page 223: intranet.tdmu.edu.uaintranet.tdmu.edu.ua/data/kafedra/internal/infect_desease... · Web viewBest method to treat diarrhoea in child is: intra venous fluide *ORS antibiotics bowel

e. Defficiency of antitrypsin250. A patient during half of the year got frequent parenterally injections concerning

bronchial asthmA. There was decrease in appetite gradually, started to feel weakness, arthralgia, dark coloured urine, the icterus of skin appeared. Objectively: temperature of 37 °C, pulse 68/min BP 115/70 mm of Hg Liver +4 cm, spleen +1 cm, skin and sclera is yellow coloured In the general blood analysis: amount of leucocytes: 3,6 109,among them 52 % lymphocytes, ESR 6 mm/hr, activity of ALAT is increased in 10 times. which information will be more credible than all in blood of patient?

a. Anti-HBs antibodiesb. Anti-HAV IgM antibodiesc. *Anti-HBV antibodiesd. Anti-HCV IgG antibodiese. Anti-HEV IgM antibodies

251. Patient 34 years old complaints of fatigue, decreasing of appetite, nausea, feeling of weight in a epigastric region, bitter taste in to the mouth. Objectively: Pulse is 76/min , temperature 37,2 °C. Skin is pale, with single vascular “asterisks” present on thorax. Liver is enlarged with 2 cm below the costal arch . In blood: bilirubinn :36 mmol/lt , ASAT :2,5 mmol/lt, ALAT :2,8 mmol. HBeAg, anti-HBc (–), HBsAg and anti-HBe (+). What will be the diagnosis of the patient?

a. *Chronic hepatitis, phase of integration, activity is poorly expressedb. Chronic hepatitis, phase of replication, activity is poorly expressedc. Chronic hepatitis, phase of integration, high activityd. Chronic hepatitis, phase of replication, activity is poorly expressede. Chronic hepatitis, phase of replication, moderate activity

252. A patient of 34 years complaints of fatigue, decreasing of appetite, nausea, feeling of weight in a epigastric region, bitter taste in the mouth. Objective examination : Pulse 76/min, temperature 37,2 °C. Skin is pale, single vascular star-like rash on the thorax. Liver is 2 cm below the costal arch. In blood: bilirubin level is 36 mmol/lt, ASAT 2,5, ALAT 2,8 . Anti-HBc (–), HBsAg and HBeAg (+). What therapeutic tactic is expedient in this case?

a. Therapy by hepatoprotectorsb. *Antiviral therapy by lamivudine c. Antiviral therapy by ribavirind. Therapy by corticosteriodse. Therapy by immunostimulators

253. Man 30 years, drug addict, takes drugs intravenously. Has been taking drugs for 12 years Complaints of weakness, moderate icterus, weight in right subcostal region. The state was worsened gradually. Biochemical indexes: general bilirubin 28,2 mmol/lt; ALAT 1,0, ASAT 0,8 . Will you define a diagnostic method which it is expedient to conduct for establishment of etiologic diagnosis?

a. Biopsy of liverb. *Polymerase chain reaction (PCR)c. Enzymes of liverd. Computer tomographye. Immunological tests

Page 224: intranet.tdmu.edu.uaintranet.tdmu.edu.ua/data/kafedra/internal/infect_desease... · Web viewBest method to treat diarrhoea in child is: intra venous fluide *ORS antibiotics bowel

254. Patient B., 51 years, a lot of years used an alcohol. Complaints of nausea, liquid stools, icterus of skin. Objectively: skin and sclera is yellow coloured, atrophy of muscles, subfebrile temperature. Liver 3 cm below the costal arch, painful on palpation What method of diagnostics most informing in this case?

a. Activity of cytolysis (AsAT, AlAT)b. Proteinogramc. *Biopsy of liverd. Markers of viral hepatitise. Violation of cellular immunity (T4,T8)

255. Patient of B., 64 years old, has been sick with chronic hepatitis. Complaints of pain in the liver, growth retardation. Objectively: his skin is dry, erythemic, atrophy of muscles, telangiectasis on shoulders, hyperemia of hands, abdomen is enlarged. The liver below the costal arc on 3 cm, painful, dense, spleen on 1 cm below the costal arch, positive symptom of fluctuation. Laboratory investigation: hypergammaglobulinaemia, increasing of AsAT activity, AlAT is highly increased. What drug of choice for the patient?

a. Lactuloseb. *Hepatoprotectors c. Antibiotics d. Glucocorticoidse. Alpha interferon

256. A patient 48 years old, complaints of attacks pains in right subcostal area after the physical loading. Periodically marks more light excrement, darkening of urine. Objectively: skin and mucous membranes high icteric. General bilirubin 36,8 mm/L, direct fraction - 26,4. Ultrasound of gall-bladder: thickness of wall 4 mm, there is a lot of bile in the ducts. It is necessary to prescribe with a lytolitic purpose:

a. Cholereticsb. *Ursofalkc. Cholekineticsd. Spasmolyticse. Cytostatics

257. During annual inspection increasing of a liver to 4 cm of 23 years old patient was found, increasing of bilirubin level in 2 times, AlAT in 2,5 time. He has been often sick with genital herpes. It was discovered RNA of hepatitis C virus. What is etiologic treatment

a. *Interferonsb. Essencial phosphotidesc. Cholereticsd. Cholekineticse. Ursodesoksihole acids

258. A woman of 22 years old, on the 7th month of pregnancy, fell ill suddenly after 3 weeks arrival from Turkmenistan. An icterus, incessant vomiting, pain in right subcostal area, skin haemorrhages, were appeared after 7-daily fever. What is most credible diagnosis:

a. *Hepatitis Ab. Hepatitis B

Page 225: intranet.tdmu.edu.uaintranet.tdmu.edu.ua/data/kafedra/internal/infect_desease... · Web viewBest method to treat diarrhoea in child is: intra venous fluide *ORS antibiotics bowel

c. Acute fatty hepatosis of pregnantd. Cholestatic hepatosis of pregnante. Cholecystitis

259. A patient 28 years old, injection drug addict, complaints of dull pain in the right subcostal area, weakness, decline of appetite, pain in joints, which disturbs during 2 weeks. At examination: icterus of the skin and sclera, enlargement of the liver and spleen, dark urine, excrement is discoloured. What is preliminary diagnosis?

a. HIV-infecionb. Hepatitis B c. Hepatitis Cd. *Toxic hepatitise. Hepatitis A

260. At inspection of a 8 weeks term pregnant woman HBsAg was found. Level of bilirubin of blood and activity of ALAT were normal. What is necessary to do?

a. *To save pregnancy and conduct the inoculation to newborn against hepatitis B

b. Termination of pregnancyc. Termination of pregnancy and conduct treatment by interferond. To save pregnancy and conduct treatment by lamivudine. To save pregnancy and ultrasonic inspection of the fetus

261. Student 20 years, treated oneself on an occasion of ARVI (increasing of temperature to 38,2 °C during 3 days). He complaints of worsening of appetite, increasing fatigue at a normal temperature and absence of the catarrhal phenomena of upper respiratory tracts. A doctor found out an increasing and moderate sickliness of liver. There were cases of hepatitis A in a student’s group. What method of investigation will allow?

a. Ultrasound scanning of the liverb. Determination of bilirubin level of the bloodc. Determining the amount of beta-lipoproteinsd. *Determination of activity of аminotransferases of the bloode. Immunofluorescent research of the nasal smears

262. A patient I., 25 years old, appealed to the internist with complaints of a general weakness, loss of appetite, pain in the right hypochodrium. Treated himself on an occasion of ARVI ambulatory 3 days. Became acute worsening of fealling: nausea, pain in right hypochondrium, ochrodermia of sclera, darkening of urine, appeared. Objectively: breathing superficial, cardiac tone is rhythmic. Abdomen is soft, painfull in right hypochondrium, a liver is megascopic, a spleen not palpable. What is most credible diagnosis?

a. *Hepatitis Ab. Leptospirosisc. Cholecystitisd. Influenzae. Pseudotuberculosis

263. A patient 42 years old, complaints of dull pain in a right hypochondrium, weakness, decline of appetite, itching of skin, icterus. Disease began 1 month ago from protracted аrthralgia, disgust for a meal, strong weakness. An icterus which now increased considerably appeared three weeks ago. Temperature 36,0 °C. Pulse 56 per 1 min. A

Page 226: intranet.tdmu.edu.uaintranet.tdmu.edu.ua/data/kafedra/internal/infect_desease... · Web viewBest method to treat diarrhoea in child is: intra venous fluide *ORS antibiotics bowel

tongue is assessed with white cover. A stomach is soft, sickly in right hypochondrium. A liver +3 cm, spleen is not megascopic. Common analysis of blood: leukopenia, relative lymphomonocytosis, ESR 3 mm/hour. What diagnosis is most credible?

a. Cancer of head of a pancreasb. *Hepatitis B c. Hepatitis Ad. Cirrhosis of livere. Chronic cholecystitis

264. For a patient 35 years after 4-months of treatment by isoniaside - аdynemia, icterus, pain in right hypochondric were appeared. A liver is megascopic. In blood activity of enzymes of AlAT is enhanceable in three times, AsAT in two times. Bilirubin of blood of 122 gm/ml (conjugated – 82, unconjugated – 40). НBs-аntigen is not found out. What is the diagnos?

a. Calculary cholecystitisb. Hepatocirrhosisc. Acute viral hepatitisd. Chronic active hepatitise. *Toxic hepatitis

265. A patient 75 years old. Complaints of a subfibrile temperature, general weakness, pharyngalgia, conjunctivitis. A child in family an acute adenoviral disease is ill. A patient considers itself a patient the second day. At examination discovered sign of acute pharyngitis. Lymphatic nodes are megascopic: neck front and back, arm-pits and inguinal, to 1 cm in a diameter, soft, not soldered between itself and with a surrounding tissue. Hyperemia of pharynx and tonsills. Wheezes are not present in lungs. Breathing clean. Tones of heart are muffled. BP - 140/80 mm Hg, Ps - 80 per 1 min. Stomach soft. Palpatory- megascopic liver and spleen, 3 cm below the costal arc, soft, painless. Choose the most credible diagnosis:

a. Flu, to middle weightb. Limfogranulomatosisc. *Adenoviral infectiond. Infectious mononucleosise. Hepatitis A

266. A patient 35 years old, complaints of aching pain in right hypocostal area, nausea, decline of appetite. Beginning of disease binds to appendicitis. After it in 2 months an icterus appeared first. Treated oneself in an infectious department. In 1 began to notice aching pain in right hypocostal area, in analyses is an increase of level of bilirubin. Your diagnosis?

a. Calculary cholecystitisb. Zhil'ber diseasec. Acute viral hepatitisd. Chronic cholangitise. *Chronic hepatitis

267. The patient T., 35 years, operating trained nurse, appealed to the doctor on the 8th day of gradual development of illness with complaints of a general weakness, rapid fatigueability, dark color of urine. In the morning noticed the icterus. On examination

Page 227: intranet.tdmu.edu.uaintranet.tdmu.edu.ua/data/kafedra/internal/infect_desease... · Web viewBest method to treat diarrhoea in child is: intra venous fluide *ORS antibiotics bowel

temperature of body 36,8 °C. Found out the increase of liver -+3 sm The changes of what laboratory index most informing at this illness?

a. *AlAT b. Hemodiastasesc. Protrombin indexd. Cholesterole. Alkaline phosphatase

268. 23-years old patient during 6 months gets diabetes mellitus in a policlinic the injections of insulin. A weakness, arthalgia, grew gradually, an appetite disappeared, then dull pain appeared in an epigastrium after-meal. In 2 weeks from the beginning of illness noticed the dark color of urine, and afterwards – icterus on a background which the general state continues to be worsened. Temperature of body 36,5°C, Ps 58 per 1 min Liver +5 sm, spleen +1 sm What from hemanalysis is it needed to appoint for confirmation of diagnosis?

a. Activity of alkaline phosphataseb. Activity of lactatdehydrogenasec. *Activity of ALATd. Activity of creatinphosphokinasee. Activity of amylase

269. 42-years old patient got blood transfusion two months ago. A weakness was gradually increased, an appetite was worsened, arthalgia, small nausea appeared. On a 12th day noticed an insignificant icterus, appealed to the doctor. At examination is the general state fully satisfactory. The temperature of body is normal, Ps 60 per 1 min, BP 100/70 mm Hg.A liver and spleen is moderate megascopic, a stomach at palpatory is not sickly. Anti-HCV IGM is found in blood. What from changes in the global analysis of blood most probably?

a. Neutrocytosisb. *Leykopeniya c. Lymphopeniad. Speed-up ESRe. Aneozinofiliya

270. A 17-years-old patient during 4 days suddenly had headackes, myalgias, fever of permanent type. From a 5th day is a normal temperature of body, the general state was considerably improved, an appetite appeared, nausea, general weakness, disappeared, but an icterus became noticeable. The general state is satisfactory, liver +3 cm, spleen +1 cm, Ps 56 per 1 mins Peripheral lymphatic knots, amygdales are not megascopic, hyperemia of pharynx is not present. At research of global analysis of blood is leykopenia with a relative lymphocytosis, 3 % virocytis, ESR normal. Activity of ALAT of blood is increase in 5 times. What diagnos is most credible?

a. *Hepatitis Ab. Infectious mnonucleosisc. Citomegaloviral infectiond. Hepatitis Be. Hepatitis C

271. On the 15th day of illness in a 17-years-old patient with hepatitis anti-HBs is found in blood. The condition of patient was considerably worsened the day before. There

Page 228: intranet.tdmu.edu.uaintranet.tdmu.edu.ua/data/kafedra/internal/infect_desease... · Web viewBest method to treat diarrhoea in child is: intra venous fluide *ORS antibiotics bowel

was excitation, a nose-bleed, a hemorragic rash appeared on a skin, diminished and became sickly at palpatory liver. Ps 106 per 1 min, BP of 110/ 70 mm Hg, the temperature of body subfebrile. What changes of blood indexes is most credible?

a. Growth of alkalinephosphataseb. Growth of Fibrinogenum c. Growth of indirect bilirubinumd. *Decline of protrombin indexe. Growth of albumin

272. 25-years old woman during a year got numerous injections concerning bronchial asthmA. An appetite disappeared gradually, a weakness, arthalgia grew, urine became dark in two weeks, through three – an icterus on a background which the general condition continues to be worsened appeared. The temperature of body is normal, Ps 62 per 1 min. Moderate sickliness in the area of pancreas, positive symptom of Voskresensky, liver +4 cm, spleen +2 cm. What markers of viral hepatitis more credible will be positive for a patient?

a. *Anti-HBc IGMb. ANTI-HEV IGMc. ANTI-CMV IGMd. HBsAge. ANTI-HAV IGM

273. 26-years old man during 6 days marks gettings up temperatures with a strong chill. 6 months ago there were alike attacks of fever during a trip to Afghanistan, from where returned 4 months ago. Temperature of body 36,4°C, Ps 94 per 1 min. The moderate increasing of spleen and liver is marked. A diuresis suffices, without pathological changes. What disease can be suspected for a patient?

a. Leptospirosisb. Hemorragic feverc. *Malaria d. Viral hepatitise. Sepsis

274. A 16-years-old patient in a month after returning from Crimea had headackes, myalgias appear suddenly, strong weakness with a fever to 39°C during 3 days. The general condition was considerably improved farther, a bad appetite and nausea, dull pain, was saved only in right hypocostal area, became dark urine, an excrements became white. On a 6th day a moderate icterus appeared. Liver +4 cm, spleen +1 cm. In the general blood analysis is leykopenia, relative limphomonocytosis, ESR 4 mm/hour. General bilirubin of blood 89 mmol/L, the direct prevails, activity of ALAT is increase in 4 times, ASAT – in 3 times. What is most credible diagnoses?

a. Hepatitis Bb. *Hepatitis Ac. Opisthorchiasisd. Hemolitic icteruse. Acute cholecystitis

275. Patient A., 19 years, became sick acutely – the temperature of body rose to 39 °C, a weakness, nausea, appeared. Vomiting, a stomach-ache are not present. A temperature reposed on high numbers 2 days, then went down to normal. Urine became dark on the 6th

Page 229: intranet.tdmu.edu.uaintranet.tdmu.edu.ua/data/kafedra/internal/infect_desease... · Web viewBest method to treat diarrhoea in child is: intra venous fluide *ORS antibiotics bowel

day of illness, subicterus noticed on 5th. To this time condition of patient was improved, nausea, weakness, disappeared. Objectively: moderate icterus of skin and sclera, a rash is not present. Pulse 66 per 1 min, BP 110/70 mm Hg. Abdomen is soft, painless in all of parts, liver + 2,5 cm, the spleen was palpitated. What is the reason of changes in the pre-icteric period of the disease?

a. *Toxicemiyab. Bacterialemiyac. Pancreatitisd. Viremiae. Cholecystitis

276. 40-years old patient during 2th days marks absence of appetite, nausea, general weakness. A year ago carried acute hepatitis B, avoided a clinical supervision. Objectively: the temperature of body is normal, skin and sclera are icteric, liver + 3 cm. A spleen is not megascopic. Urine is moderate dark colouring. ALAT of blood 4,0. What is most credible diagnosis?

a. Chronic viral hepatitis, minimum activityb. Chronic cholecystitisc. *Chronic viral hepatitis, moderate activityd. Hepatocirrhosise. Chronic viral hepatitis, high activity

277. A man, 37 years, injection drug addict, alcoholic, complaints of a general weakness, dull pain in right hypocostal area, increasing of abdomen, shortness of breath. He was sick by chronic hepatitis during 10 years. Objectively: temperature of body 37,1°C, an expressed icterus. Liver +6 cm. Free liquor in abdomen. Positive symptom of fluctuation. What complication present?

a. Exacerbation of chronic hepatitis Cb. Acute hepatic insufficiencyc. Hepatocarcinomad. Exacerbation of chronic hepatitis Be. *Hepatocirrhosis

278. A patient is 59 years, suffers from chronic viral hepatitis with development of cirrhosis, hospitalized with a diagnosis: “bleeding from the veins of esophagus”. Acute pallor of skin covers. BP 80/40 mm Hg, pulse 100 per 1 min, the temperature of body is normal. On a front abdominal wall a venous net is extended. What preparation is it necessary to begin therapy from?

a. Albumenb. Plasmac. Neogemodezd. *Bloode. Glucose

279. Patient 20 years, treated oneself on an occasion ARVI during 5 days, but marked no displays of respirator syndrome. Last a temperature is normal 2 days, an appetite disappeared however, pain appeared in an epigastrium, nausea, urine had darked. About what illness is it possible to think?

a. Acute hepatitis B

Page 230: intranet.tdmu.edu.uaintranet.tdmu.edu.ua/data/kafedra/internal/infect_desease... · Web viewBest method to treat diarrhoea in child is: intra venous fluide *ORS antibiotics bowel

b. Infectious mononucleosisзc. Pseudotuberculosisd. Leptospirosise. *Hepatitis A

280. In 2 months after returning from India, where often drank unboiled water, the 23-years-old pregnant nauseated, strong general weakness, head pain, later the temperature of body rose to 38,6°, which stuck to within a week. An icterus appeared on a 6th day, the general state continued to be worsened. On the 12th day of illness the general state heavy. EuphoriA. Vomiting at night. Complete fastidium. Bright icterus, signs of hemorragic syndrome, tachycardiA. BP 110/60 mm Hg, temperature of body of 37,8°C. A liver is insignificantly megascopic, soft, painfull, spleen +2 sm There is neutrophilic leykocytosis in the global analysis of blood. General bilirubinum of blood of 570, to the line – 300, activity of ALAT is enhanceable in 100 times, timol test of 26 units., urea – 2,1 mmol/l. With most probability for a patient:

a. *Hepatitis Ab. Malignant icterus of pregnantc. Typhoidd. Mechanical icterus e. Leptospirosis

281. A patient is 25 years, appealed to the internist with complaints of a general weakness, worsening of appetite, feeling of weight in right hypocostal areA. Treated oneself on an occasion ARVI ambulatory 3 days. A feel was worsened, nausea, pain in right hypocostal arrea, ochrodermia of sclerotica, darkening of urine, appeared. Objectively: breathing normal, cardiac tones are rhythmic. A stomach is soft, sickly in right hypocostal area, a liver is megascopic, a spleen is not megascopic. What is the most credible diagnosis?

a. *Hepatitis Ab. Leptospirosisc. Calculary cholecystitisd. Flue. Pseudotuberculosis

282. A patient 20 years old, had treated himself concerning an acute respiratory disease for 5 days, but marked no displays of respirator syndrome. Last 2 days temperature has been normal, appetite disappeared, however, appeared pain in epigastrium with nausea, and urine turned dark. About what illness is it possible to think?

a. Hepatitis Bb. Infectious mononucleosisc. Pseudotuberculosisd. Leptospirosise. *Hepatitis A

283. A patient 28 years old, an injection drug addict, complaints of dull pain in right subcostal region, weakness, decline of appetite, pain in joints. The symptoms have been present for 2 weeks. At examination: the icterus of skin and scleras observed. The liver and spleen are enlarged. Urine is dark, excrements are discoloured. What is preliminary diagnosis?

a. Hepatitis A

Page 231: intranet.tdmu.edu.uaintranet.tdmu.edu.ua/data/kafedra/internal/infect_desease... · Web viewBest method to treat diarrhoea in child is: intra venous fluide *ORS antibiotics bowel

b. *Hepatitis Bc. Chronic cholecystitisd. Toxic hepatitise. HIV-infection

284. A patient 25 years old got sick suddenly after chills and temperature increase up to 38,0 °C. There was vomiting 2 times. Moderate pharyngeal pain appeared at swallowing. with stomach discomfort. Signs of bursitis observed on knee and elbow joints. The light icterus of sclera and skins of joints on 4th day, on the skin of lateral surfaces of trunk, forehead and lower extremities. The small bright red papular rash appeared, more concentrated in natural folds.Liver is enlarged, the tongue is raspberry like appearence. Preliminary diagnosis would be:

a. Hepatitis Ab. Hepatitis Bc. *Pseudotuberculosisd. Scarlet fevere. Infectious mononucleosis

285. A 19 years old patient was diagnosed with hepatitis B. After violation of diet and nervous stress the state of patient got worse: intensity of ichterus was increased, pulse 110/min, BP 80/50 mm Hg. Also there was vomiting by «coffee-grounds». The size of liver has decreased, the liver on palpation was painful. What complication would you think about?

a. Infectious-toxic shock b. Acute kidneys insufficiency c. Acute adrenal insufficiencyd. Hemolitic icteruse. *Acute hepatic insufficiency

286. A patient C., 43 years old, has been treated for 5years . During the last hospitalization a liver-biopsy was conducted. Histologicaly there is bulb dystrophy and necrosis of hepatocytes, with leucocyts infiltration, and pericellular fibrosis. What diagnosis is most credible in this case?

a. *Alcoholic liver cirrhosisb. Viral hepatitis Bc. Viral hepatitis Cd. Cryptogenic hepatitise. Fatty dystrophy of liver

287. Patient B., 23 years old. Objectively: skin is yellow, icterus of sclerA. Pulse 66/min, BP 120/80 . Indexes of AlAT, AsAT, and Tymol tests are normal. General billirubin is 34 mmol/l due to indirect fraction. Specify the most credible diagnosis.

a. Chronic toxic hepatitisb. Chronic cryptogenic hepatitisc. Chronic hepatitis Cd. *Zhilber’s syndromee. Chronic B hepatitis

288. A trained nurse got sick on 16.09: general weakness, nausea, vomiting, pain in joints appeared. 21.09 urine turned dark, and skin yellow. 21.09 she was hospitalized. Objectively: general status satisfactory, temperature 38,2 °C, expressed skin and mucous

Page 232: intranet.tdmu.edu.uaintranet.tdmu.edu.ua/data/kafedra/internal/infect_desease... · Web viewBest method to treat diarrhoea in child is: intra venous fluide *ORS antibiotics bowel

membranes jaundice. Liver palpated 3 cm below the costal arc. 26.09 stomach-aches and increased sleepiness with disorientation in time and place were appeared. 27.09 haematomas in the places of injections also appeared. Hepatic breath was noticed. Light edema of feet and lumbar region were observed. The liver was not palpable. Development of what syndrome can be forecast?

a. Meningoencephalitisb. Hepatic encephalopathy c. Acute nephrosonephritisd. Pancreatitise. *ID syndrom

289. A patient B., 52 years old, for 3 month complaints of nausea, periodic vomiting, swelling of stomach, weakness, loss of weight up to 12 kg, consistency of stool is chainging. During some days consciousness is entangled, somnolence, allolalia, general weakness are expressed. Temperature of body 37,4 °C. Icterusis on the skin. Tremor of brushes and nystagmus is marked. Pupils are narrowed, with a slow reaction on light, increase of tendon reflexes. Ps 112/min, unrhythmical. BP 90/65 mm Hg. A liver is dense +2. Laboratory tests: Hb 86 g/L, Leuc 4,2*109/L, ESR 18 mm/hour, glucose 3,4 mmol/l, general bilirubin 56,7 mm/L, albumen 52 g/L, K+ 3,2 mmol/l, AsAT 0,62; AlAT 0,84; prothrombin index-58 %; рН blood 7,3. What is previous diagnosis?

a. Hepatocirrhosisb. Vilson-Konovalov diseasec. Cancer of liverd. *Endogenous hepatic encephalopathye. Meygs disease

290. A patient, 17 years, complaints of a weakness, worsening of appetite, nausea, painfull in right subcostal area, dull ache in the body, dark color of urine, temperature to 37,8 °C. She stood in contact with sick on icterusis 1,5 months ago. Objectively: yellow colour of the skin and visible mucous membranes. The liver is increased to 1,5 cm. Unsignificant enlargment of spleen. What is your preliminary diagnosis?

a. Viral hepatitis Bb. Infectious mononucleosisc. *Viral hepatitis Ad. Leptospirosise. Pseudotuberculosis

291. Before the appearance of jaundice in a patient 16 years old, during 2 days there were an increasing of the body temperature (38,5 °C), headache, dull ache disturbed whole body. Name the variant of pre-icteric period of viral hepatitis for this patient?

a. *Influenza-likeb. Astenovegetativec. Artralgicd. Dyspeptice. Allergic

292. A patient T., 28 years, appilled due to worsening of common status at the seventh day to the infectious department concerning viral hepatitis. Nausea and vomiting were present, hepatic smell appeared, increasing of the liver sizes. What biochemical index does allow to suspect complication in the patient?

Page 233: intranet.tdmu.edu.uaintranet.tdmu.edu.ua/data/kafedra/internal/infect_desease... · Web viewBest method to treat diarrhoea in child is: intra venous fluide *ORS antibiotics bowel

a. Increasing activity of AlATb. Increasing of bilirubin, increasing activity of AlATc. Increasing timol testd. *Decreasing of protrombin, increasing of bilirubine. Decreasing of AlAT

293. A sick P., 54 years old, complaints of dull pain in right subcostal area, bad taste in the mouth, itching of the skin, increasing of abdomen. After the liver puncture fatty dystrophy of hepatocytes, eccentric placing of veins in a hepatic lobule were found. What laboratory index are most characteristic in this case?

a. Alkaline phosphotaseb. Hemodiastasec. *Level of transaminasesd. Glucose of bloode. Creatinphosphokinase

294. A patient, 24, complaints of pains in right subcostal area, increasing after-meal, nausea, increase temperature of body to 37,7 °C, icterus, pains in large joints. He is ill from 8 months. Suffers a nonspecific ulcerative colitis. Hepatosplenomegaly. ESR 47 mm/hr, general bilirubin level is 86,1mmol/lt, direct-42,3 mmol/lt. In blood found out antibodies against smooth muscles. General albumen of 62 gram/lt, albumin б. 40 %, globulin. 60 %, gamma globulins 38 %. Not found out the markers of viral hepatitis. On USD diameter of portal vein is 1 cm What will be your diagnosis?

a. Primary biliary cirrhosisb. Zhilber syndromec. Viral hepatitisd. Hemochromatosise. *Autoimmune hepatitis

295. Woman of 22 years old, in the 7th month of pregnancy felt ill acutely in 3 weeks after arrival from Turkmenistan. An icterus, untraceable vomiting, pain in right subcostal region,, hemmorhaeges on the skin, appeared after three-day fever. Most probable diagnosis will be:

a. Acute fatty hepatosis of pregnancyb. Cholestatic hepatosis of pregnantc. *Viral hepatitis of Ad. Viral hepatitis Be. Acute sepsis

296. Weakness appeared in a patient, decrease in an appetite, began pains in the joints of extremities, felling of weight in right subcostal region, vomiting. Urine became dark in 12 days, and in a day – sclera and skin turned yellow. It was discovered in an immunogram: antuIgG -HAV (+), HBsAg (+), antiIgМ -НBcorAg (+), anti-НCV (–), anti-НDV (–). To the patient 3 months back was extracted a tooth. Most probable diagnosis?

a. *Viral hepatitis Bb. Viral hepatitis Ac. Viral hepatitis Cd. Viral hepatitis De. Viral hepatitis E

Page 234: intranet.tdmu.edu.uaintranet.tdmu.edu.ua/data/kafedra/internal/infect_desease... · Web viewBest method to treat diarrhoea in child is: intra venous fluide *ORS antibiotics bowel

297. A patient 42 years, radiologist, entered surgical department with bleeding from the mouth, was admitted to the gastroenterological departments . He considered himself healthy. 20 years ago had haemotransfusion. Fibrogastroduodenoscopy shows venous dilation of the oesophagus of the III degree. Most probable etiologic reason of disease:

a. Autoantibodies against hepatocytesb. Viral hepatitis Ac. X-ray irradiationd. *Viral of hepatitis Be. Defficiency of antitrypsin

298. A patient during half of the year got frequent parenterally injections concerning bronchial asthmA. There was decrease in appetite gradually, started to feel weakness, arthralgia, dark coloured urine, the icterus of skin appeared. Objectively: temperature of 37 °C, pulse 68/min BP 115/70 mm of Hg Liver +4 cm, spleen +1 cm, skin and sclera is yellow coloured In the general blood analysis: amount of leucocytes: 3,6 109,among them 52 % lymphocytes, ESR 6 mm/hr, activity of ALAT is increased in 10 times. which information will be more credible than all in blood of patient?

a. Anti-HBs antibodiesb. Anti-HAV IgM antibodiesc. *Anti-HBV antibodiesd. Anti-HCV IgG antibodiese. Anti-HEV IgM antibodies

299. Patient 34years old complaints , about fatigue, decreasing of appetite, nausea, feeling of weight in a epigastric region, bitter taste in to the mouth. Objectively: Pulse is 76/min , temperature 37,2 -°C. Skin is pale, with single vascular “asterisks” present on thorax. Liver is enlarged with 2 cm below the costal arch . In blood: bilirubinn :36 mmol/lt , ASAT :2,5 mmol/lt, ALAT :2,8 mmol. HBeAg, anti-HBc (–), HBsAg and anti-HBe (+). What will be the diagnosis of the patient?

a. *Chronic hepatitis B, phase of integration, activity is poorly expressedb. Chronic hepatitis B, phase of replication, activity is poorly expressedc. Chronic hepatitis B phase of integration, high activityd. Chronic hepatitis C, phase of replication, activity is poorly expressede. Chronic hepatitis C phase of replication, moderate activity

300. A patient of 34 years complaints of fatigue, decreasing of appetite, nausea, feeling of weight in a epigastric region, bitter taste in the mouth. Objective examination : Pulse 76/min, temperature 37,2 °C. Skin is pale, single vascular star-like rash on the thorax. Liver is 2 cm below the costal arch. In blood: bilirubin level is 36 mmol/lt, ASAT 2,5, ALAT 2,8 . Anti-HBc (–), HBsAg and HBeAg (+). What therapeutic tactic is expedient in this case?

a. Therapy by hepato-protectorsb. *Antiviral therapy by lamivudine c. Antiviral therapy by ribavirind. Therapy by corticosteriods

Page 235: intranet.tdmu.edu.uaintranet.tdmu.edu.ua/data/kafedra/internal/infect_desease... · Web viewBest method to treat diarrhoea in child is: intra venous fluide *ORS antibiotics bowel

e. Therapy by immunostimulators301. Man 30 years, drug addict, takes drugs intravenously. Has been taking drugs for 12

years Complaints of weakness, moderate icterus, pain in right subcostal region. The state was worsened gradually. Biochemical indexes: general bilirubin 28,2 mmol/lt; ALAT 1,0, ASAT 0,8 . Will you define a diagnostic method which it is expedient to conduct for establishment of etiologic diagnosis?

a. Biopsy of liverb. *Polymerase chain reaction (PCR)c. Enzymes of liverd. Computer tomographye. Immunological tests

302. Patient B., 51 years, a lot of years used an alcohol. Complaints of nausea, liquid stools, icterus of skin. Objectively: skin and sclera is yellow coloured, atrophy of muscles, subfebrile temperature. Liver 3 cm below the costal arch, painful on palpation What method of diagnostics most informing in this case?

a. Activity of cytolysis (AsAT, AlAT)b. Proteinogramc. *Biopsy of liverd. Markers of viral hepatitise. Violation of cellular immunity (T4,T8)

303. Patient of B., 64 years old, has been sick with chronic hepatitis. Complaints of pain in the liver, growth retardation. Objectively: his skin is dry, erythemic, atrophy of muscles, telangiectasis on shoulders, hyperemia of hands, abdomen is enlarged, look like “jelly-fish head”. The liver below the costal arc on 3 cm, painful, dense, spleen on 1 cm below the costal arch, positive symptom of fluctuation. Laboratory investigation: hypergammaglobulinaemia, increasing of AsAT activity, AlAT is highly increased. What drug of choice for the patient?

a. Lactuloseb. *Hepatoprotectors c. Antibiotics d. Glucocorticoidse. Alpha interferon

304. A patient 48 years old, complaints of attacks pains in right subcostal area after the physical loading. Periodically marks more light excrement, darkening of urine. Objectively: skin and mucous membranes high icteric. General bilirubin 36,8 mm/L, direct fraction - 26,4. Ultrasound of gall-bladder: thickness of wall 4 mm, there is a lot of bile in the ducts. It is necessary to prescribe with a lytolitic purpose:

a. Cholereticsb. *Ursofalkc. Cholekineticsd. Spasmolyticse. Cytostatics

305. During annual inspection increasing of a liver to 4 cm of 23 years old patient was found, increasing of bilirubin level in 2 times, AlAT in 2,5 time. He has been often sick with genital herpes. It was discovered RNA of hepatitis C virus. What is etiologic treatment

Page 236: intranet.tdmu.edu.uaintranet.tdmu.edu.ua/data/kafedra/internal/infect_desease... · Web viewBest method to treat diarrhoea in child is: intra venous fluide *ORS antibiotics bowel

a. *Interferonsb. Essencial phosphotidesc. Cholereticsd. Cholekineticse. Ursodesoksihole acids

306. A patient, 19 years old, with hepatitis B. After violation of diet and nervous stress the state of the patient became worse: increasing of intensity of jaundice, pulse 110 per 1 min, BP 80/50 mm Hg, vomiting with “coffee-grounds”, decreasing of the liver size. What complication is it possible to think about?

a. *Acute liver insufficiencyb. Infectious-toxic shockc. Acute kidney insufficiency d. Acute extrarenal insufficiencye. Hemolytic

307. A woman of 22 years old, on the 7th month of pregnancy, fell ill suddenly after 3 weeks arrival from Turkmenistan. An icterus, incessant vomiting, pain in right subcostal area, skin haemorrhages, were appeared after 7-daily fever. What is most credible diagnosis:

a. *Hepatitis Ab. Hepatitis Bc. Acute fatty hepatosis of pregnantd. Cholestatic hepatosis of pregnante. Cholecystitis

308. A patient, 25 years old, fell ill suddenly: after a chill temperature rose up to 38,0 °C, 2 times vomiting, moderate pharyngalgias at swallowing, pain in the stomach, knee and elbow joints. Non intensive jaundice of scleras and skin at 4th day of disease were appiared, urine became dark; on the lateral surfaces of trunk, overhead and lower extremities small spot bright red rash appeared. Liver is increased, tongue is assessed, “raspberry”, increased lymphonodes. What is preliminary diagnosis?

a. Hepatitis Ab. *Pseudotuberculosisc. Allergodermiad. Scarlet fevere. Infectious mononucleosis

309. A patient 21 years old, complaints of increasing of temperature to 39,2 °C, weakness, headache, pharyngalgia, pain in the muscles, joints and stomach, nausea, vomiting once. On the 3rd day stomach-ache increased and began to be expressly localized in the right iliac areA. The patient was hospitalised in a surgical department with a diagnosis “appendicitis”. At examination: hyperemia of mucous of nasopharynges, subicteroses, liver +2. At operation changed appendix and packages of mesenterial lymphonodes has been found. What is preliminary diagnosis:

a. Typhoid feverb. Adenoviral infectionc. Hepatitis Ad. *Pseudotuberculosise. Enteroviral infection

Page 237: intranet.tdmu.edu.uaintranet.tdmu.edu.ua/data/kafedra/internal/infect_desease... · Web viewBest method to treat diarrhoea in child is: intra venous fluide *ORS antibiotics bowel

310. Animal technician 57 years, on the 3th day of illness appealed to the doctor with complaints of headacke, high temperature, pain in gastrocnemius muscles, dark urine and diminishing of its amount. Objectively: temperature 38,1 °C, injection of the sclera vessels, petechial rash on upper part of the thorax, hepatosplenomegaly. What is most credible preliminary diagnosis?

a. Pseudotuberculosisb. Brucellosisc. Viral hepatitisd. *Leptospirosis e. Influenza

311. A patient 28 years old, injection drug addict, complaints of dull pain in the right subcostal area, weakness, decline of appetite, pain in joints, which disturbs during 2 weeks. At examination: icterus of the skin and sclera, enlargement of the liver and spleen, dark urine, excrement is discoloured. What is preliminary diagnosis?

a. HIV-infecionb. Hepatitis B c. Hepatitis Cd. *Toxic hepatitise. Hepatitis A

312. At inspection of a 8 weeks term pregnant woman HBsAg was found. Level of bilirubin of blood and activity of ALAT were normal. What is necessary to do?

a. *To save pregnancy and conduct the inoculation to newborn against hepatitis B

b. Termination of pregnancyc. Termination of pregnancy and conduct treatment by interferond. To save pregnancy and conduct treatment by lamivudine. To save pregnancy and ultrasonic inspection of the fetus

313. A patient I., 26 years old, delivered in a hospital on the 4th day of disease with complaints of fever, headache, pain in gastrocnemius muscles. Works as a specialist in land-reclamation. Has a lot of sexual contacts. Objectively: temperature – 39,7 °C. Severe common condition. Expressed icterosis of skins and sclerA. Hemorrhages in conjunctiva and sclerA. There is a hemorragic rash on a skin. A liver increased on 3 see below the costal arc, edge of spleen, the Day's diuresis 300 ml. The etiologic factor of disease most for certain is:

a. Rickettsiab. Virus c. *Leptospirad. Spirochetae. Chlamidia

314. Patient 43, miner, on the 7th day of disease complaints of acute weakness, high temperature, pain in the muscles of feet and back, icterus, dark color of urine, headache. Fell ill sharply from a chill, temperature 40,2 °, there was a nose-bleed. A diuresis 200 ml. What is a credible diagnosis?

a. Sepsisb. Typhoidc. Viral hepatitis

Page 238: intranet.tdmu.edu.uaintranet.tdmu.edu.ua/data/kafedra/internal/infect_desease... · Web viewBest method to treat diarrhoea in child is: intra venous fluide *ORS antibiotics bowel

d. *Leptospirosise. Malaria

315. 36 years old man, during 8 days marks increasing of temperature with a strong chill, which end with abundant sweating and repeat oneself regularly in a day. There were alike attacks of fever in Afghanistan, from where returned 4 months ago. Temperature 39,2 °, pulse 94 after 1 mins A stomach during palpation is not sickly. There is a moderate increase of spleen and liver, insignificant icterus. What diagnosis is most credible?

a. Viral hepatitisb. Typhoidc. Sepsisd. *Malariae. Tuberculosis

316. Patient 40 years old, in a week after returning from Ethiopia on a motherland disturb the periodic attacks of fever. The icterus of sclera and skin covers, hepatospleenomegaly, is marked. What from the transferred diagnoses most credible?

a. Sepsisb. *Malariac. Hepatitis Ad. Hemolitic anaemiae. Leptospirosis

317. Patient 45 years, hospitalized on the 7th day of fever. Objectively: temperature 39,8 °C, somnolence (at night insomnia), dormancy, аdynemia, pallor of skin covers, pulse 78 per 1 min, BP 105/70 mm of Hg. The tangue is thickly assessed with a grey cover with the imprints of a teeth. A stomach is swollen, liver and spleen megascopic, in a right iliac area rumbling and hyperemia of skin. Stool is absent 2 days. What investigation is it neccessary to perform for clarification of diagnosis?

a. *Bacteriologic examination of bloodb. Spinal puncturec. Myelogramd. Colonoscopye. Analysis on the markers of viral hepatitis

318. Patient 52 years old, suffering sharply from a chill and headache, fever to 40,5 °C. On a 3rd day of disease nausea, vomiting, dark urine appeA. On a 4th day temperature falls down to 37,2 °C, but an icterus appeared and the amount of urine decreased to 600 ml. What disease such symptoms develop from?

a. Sepsisb. Hemorrhagic fever with a kidney syndromec. Hepatitis Ad. *Leptospirosis e. Acute glomerulonephritis

319. A patient, 35 years, suffering sharply, complaints of a headache, pain in the muscles of lower extremities, increasing of temperature to 39,3 °C. Objectively on the 4th day of disease: the state is severy, hyperemic face, icterosis of skin and sclerA. Enlargment of liver and spleen. A diuresis is mionectic. What is most credible diagnosis?

a. Trichinosisb. Hepatitis A

Page 239: intranet.tdmu.edu.uaintranet.tdmu.edu.ua/data/kafedra/internal/infect_desease... · Web viewBest method to treat diarrhoea in child is: intra venous fluide *ORS antibiotics bowel

c. Yersiniosisd. Infectious mononucleuse. *Leptospirosis

320. Student 20 years, treated oneself on an occasion of ARVI (increasing of temperature to 38,2 °C during 3 days). He complaints of worsening of appetite, increasing fatigue at a normal temperature and absence of the catarrhal phenomena of upper respiratory tracts. A doctor found out an increasing and moderate sickliness of liver. There were cases of hepatitis A in a student’s group. What method of investigation will allow?

a. Ultrasound scanning of the liverb. Determination of bilirubin level of the bloodc. Determining the amount of beta-lipoproteinsd. *Determination of activity of аminotransferases of the bloode. Immunofluorescent research of the nasal smears

321. For a patient, workwoman of a pig farm, on a background of complete health a chill appeared suddenly, a temperature rose to 39,9 °C, there was a headache, nauseA. On the next day marked pains in the muscles of lower extremities, nose-bleeding began. On the 3rd day of illness, state became more severy. Face is hyperemic, scleritis, hyperemic scleritis. Liver +3 cm. Daily diuresis 700 ml. What is previous diagnosis?

a. Yersiniosisb. Hepatitis Ac. Hemorrhagic fever with a kidney syndromed. Flue. *Leptospirosis

322. A patient Z., 33 years old, miner, entered clinic of infectious diseases on the 7th day of disease with complaints of a acute weakness, high temperature, pain in the muscles of feet and back, icterus, dark color of urine, headache. Became sick sharply from a chill, temperature rises up to 40,1 °C. On a 4th day there is an icterus, nose-bleeding, hemorrhages in sclerA. Duration of fever 6 days. Diuresis is 200 ml. What is credible diagnosis?

a. Typhoid feverb. *Leptospirosisc. Hepatitis Ad. Sepsise. Influenza

323. A patient is disturbed by attacks of fever which repeated every third day. The icterus of sclera and skin, hepatospleenomegaly were marked. What is most credible diagnosis?

a. Viral hepatitisb. Sepsis c. *Malaria d. Influenza e. Leptospirosis

324. A patient Y., 25 years old, entered infectious department on the 3rd day of disease with complaints of headache, pain in the back, gastrocnemius muscles, high fever, chill. State is moderate. Icteruses of the scleras. Mucous membrane of soft palate is hyperemic. Tongue is dry, assessed with brown cover. The abdomen is swollen. Liver +2 cm, spleen

Page 240: intranet.tdmu.edu.uaintranet.tdmu.edu.ua/data/kafedra/internal/infect_desease... · Web viewBest method to treat diarrhoea in child is: intra venous fluide *ORS antibiotics bowel

is not megascopic. Painfull muscles, especially gastrocnemius. Urine is dark, excrements ordinary color. What is the most credible diagnosis?

a. Infectious moneucliousb. Hepatitis Ac. Malariad. *Leptospirosise. Yersiniosis

325. On the 3rd day of illness the sick is delivered in a severy condition with complaints of a suddenly arising up high temperature, headache, repeated nose-bleed, pains in gastrocnemius muscles. Objectively: moderate icterus of sclera and skins, hepatospleenomegaly, оliguriA. What is most credible diagnosis?

a. *Leptospirosisb. Viral hepatitisc. Influenzad. Infectious mononucleosise. Malaria

326. A patient S., 45 years old, suffering suddenly from appearance of chill and increasing of temperature to 39,2 °C. In the evening pain appeared in a stomach and gastrocnemius muscles. In 2 days noticed the ochrodermia of skin and sclerA. Objectively: the state is severy, temperature 39,9 °C. The tongue is covered. Moderate jaundice of the skin and sclerA. There is plural petachiae on a trunk. Superficial breathing 20 times per 1 min, pulse 102 per 1 min, AP 100/60 mm of Hg. A stomach is soft, sickly in epigastrium, a liver on 3 cm comes from a costal arc. Daily diuresis 300 mm, urine is sad-coloured. What is preliminary diagnosis?

a. Sepsisb. *Leptospirosisc. Influenzad. Hepatitis Be. Infectious mononucleus

327. A patient I., 25 years old, appealed to the internist with complaints of a general weakness, loss of appetite, pain in the right hypochodrium. Treated himself on an occasion of ARVI ambulatory 3 days. Became acute worsening of fealling: nausea, pain in right hypochondrium, ochrodermia of sclera, darkening of urine, appeared. Objectively: breathing superficial, cardiac tone is rhythmic. Abdomen is soft, painfull in right hypochondrium, a liver is megascopic, a spleen not palpable. What is most credible diagnosis?

a. *Hepatitis Ab. Leptospirosisc. Cholecystitisd. Influenzae. Pseudotuberculosis

328. A patient 42 years old, complaints of dull pain in a right hypochondrium, weakness, decline of appetite, itching of skin, icterus. Disease began 1 month ago from protracted аrthralgia, disgust for a meal, strong weakness. An icterus which now increased considerably appeared three weeks ago. Temperature 36,0 °C. Pulse 56 per 1 min. A tongue is assessed with white cover. A stomach is soft, sickly in right hypochondrium. A

Page 241: intranet.tdmu.edu.uaintranet.tdmu.edu.ua/data/kafedra/internal/infect_desease... · Web viewBest method to treat diarrhoea in child is: intra venous fluide *ORS antibiotics bowel

liver +3 cm, spleen is not megascopic. Common analysis of blood: leukopenia, relative lymphomonocytosis, ESR 3 mm/hour. What diagnosis is most credible?

a. Cancer of head of a pancreasb. *Hepatitis B c. Hepatitis Ad. Cirrhosis of livere. Chronic cholecystitis

329. For a patient 35 years after 4-months of treatment by isoniaside - аdynemia, icterus, pain in right hypochondric were appeared. A liver is megascopic. In blood activity of enzymes of AlAT is enhanceable in three times, AsAT in two times. Bilirubin of blood of 122 gm/ml (conjugated – 82, unconjugated – 40). НBs-аntigen is not found out. What is the diagnos?

a. Calculary cholecystitisb. Hepatocirrhosisc. Acute viral hepatitisd. Chronic active hepatitise. *Toxic hepatitis

330. A patient is 35 years, grumbles about aching pain in right hypocostal area, nausea, decline of appetite. Beginning of disease binds to appendicitis. After it in 2 months an icterus appeared first. Treated oneself in an infectious department. In 1 began to notice aching pain in right hypocostal area, in analyses is an increase of level of bilirubin. Your diagnosis?

a. Calculary cholecystitisb. Illness of Zhil'berac. Sharp viral hepatitisd. Chronic cholangitise. *Chronic hepatitis

331. The patient T., 35 years, operating trained nurse, appealed to the doctor on the 8th day of gradual development of illness with complaints of a general weakness, rapid fatigueability, dark color of urine. In the morning noticed the icterus. On examination temperature of body 36,8 °C. Found out the increase of liver -+3 sm The changes of what laboratory index most informing at this illness?

a. *AlAt b. Hemodiastasesc. Protrombin indexd. Cholesterole. Alkaline phosphatase

332. Diagnosed a patient: chronic hepatitis in the stage of integration. What markers will be in patient in this stage disease?

a. HBeAgb. Antibodies to HBeAgc. DNA OF HBVd. Viral DNA-polimerasee. *HBsAg, anti-НBе

333. 23-years old patient during 6 months gets diabetes mellitus in a policlinic the injections of insulin. A weakness, arthalgia, grew gradually, an appetite disappeared, then

Page 242: intranet.tdmu.edu.uaintranet.tdmu.edu.ua/data/kafedra/internal/infect_desease... · Web viewBest method to treat diarrhoea in child is: intra venous fluide *ORS antibiotics bowel

dull pain appeared in an epigastrium after-meal. In 2 weeks from the beginning of illness noticed the dark color of urine, and afterwards – icterus on a background which the general state continues to be worsened. Temperature of body 36,5°C, Ps 58 per 1 min Liver +5 sm, spleen +1 sm What from hemanalysis is it needed to appoint for confirmation of diagnosis?

a. Activity of alkaline phosphataseb. Activity of lactatdehydrogenasec. *Activity of ALATd. Activity of creatinphosphokinasee. Activity of amylase

334. 42-years old patient got blood transfusion two months ago. A weakness was gradually increased, an appetite was worsened, arthalgia, small nausea appeared. On a 12th day noticed an insignificant icterus, appealed to the doctor. At examination is the general state fully satisfactory. The temperature of body is normal, Ps 60 per 1 min, BP 100/70 mm Hg.A liver and spleen is moderate megascopic, a stomach at palpatory is not sickly. Anti-HCV IGM is found in blood. What from changes in the global analysis of blood most probably?

a. Neutrocytosisb. *Leykopeniya c. Lymphopeniad. Speed-up ESRe. Aneozinofiliya

335. A 17-years-old patient during 4 days suddenly had head pains, myalgias, fever of permanent type. From a 5th day is a normal temperature of body, the general state was considerably improved, an appetite appeared, nausea, general weakness, disappeared, but an icterus became noticeable. The general state is satisfactory, liver +3 sm, spleen +1 sm, Ps 56 per 1 mins Peripheral lymphatic knots, amygdales are not megascopic, hyperemia of pharynx is not present. At research of global analysis of blood is leykopenia with a relative lymphocytosis, 3 % virocytis, ESR normal. Activity of ALAT of blood is enhanceable in 5 times. What from diagnoses most credible for a patient?

a. *Hepatitis Ab. Infectious mnonucleosisc. Citomegaloviral infectiond. Hepatitis Be. Hepatitis C

336. On the 15th day of illness for a 17-years-old patient by hepatitis In anti-HBs is found in blood. The state of patient was considerably worsened the day before. There was excitation, there was a nose-bleed, a hemorragic rash appeared on a skin, diminished and became sickly at palpatory liver. Ps 106 per 1 min, BP of 110/ 70 mm Hg, the temperature of body subfebrile. What changes of indexes blood will a patient have most credible?

a. Growth of alkalinephosphataseb. Growth of Fibrinogenum c. Growth of indirect bilirubinumd. *Decline of protrombin indexe. Growth of albumin

Page 243: intranet.tdmu.edu.uaintranet.tdmu.edu.ua/data/kafedra/internal/infect_desease... · Web viewBest method to treat diarrhoea in child is: intra venous fluide *ORS antibiotics bowel

337. 25-years old woman during a year got numerous injections concerning bronchial asthmA. An appetite disappeared gradually, a weakness, arthalgia grew, urine became dark in two weeks, through three – an icterus on a background which the general state continues to be worsened appeared. The temperature of body is normal, Ps 62 after 1 mins Moderate sickliness in the area of pancreas, positive Voskresensky symptom, liver +4 cm, spleen +2 cm What from the markers of viral hepatitis more credible will be positive for a patient?

a. *Anti-HBc IGMb. ANTI-HEV IGMc. ANTI-CMV IGMd. HBsAge. ANTI-HAV IGM

338. 26-years old man during 6 days marks the 6-8-sentinel gettings up temperatures with a strong chill, which end with abundant then and repeat oneself regularly in a day. 6 months ago there were alike attacks of fever during a stay in Afghanistan, from where returned 4 months back. Temperature of body 36,4°C, Ps 94 per 1 min. The moderate increase of spleen and liver is marked. A diuresis suffices, wetting without pathological changes. What disease can be suspected for a patient?

a. Leptospirosisb. Hemorragic feverc. *Malaria d. Viral hepatitise. Sepsis

339. 40-years old patient during 2th days marks absence of appetite, nausea, general weakness. A year ago was treated due to acute hepatitis B, avoided a clinical supervision. Objectively: the temperature of body is normal, icterus of skin and sclera, increasing of liver 3 cm. A spleen is not megascopic. Urine moderatory dark colouring. ALAT of blood 4,0. What is most credible diagnosis?

a. Chronic viral hepatitis, minimum activityb. Chronic cholecystitisc. *Chronic viral hepatitis, moderate activityd. Hepatocirrhosise. Chronic viral hepatitis, high activity

340. A man, 37 years, injection drug addict, practises upon an alcohol, grumbles about a general weakness, dull pain in right hypocostal area, increase of stomach, shortness of breath. It is ill chronic hepatitis on an extent 10 years. Objectively: temperature of body 37,1°C, an icterus is expressed. Liver +6 sm. In an abdominal region is a free liquid. Positive symptom of fluctuation. What complication did arise up for a patient?

a. Intensifying of chronic hepatitis Cb. Acutehepatic insufficiencyc. Hepatocarcinomad. Intensifying of chronic hepatitis Be. *Hepatocirrhosis

341. A patient is 59 years, suffers chronic viral hepatitis with development of cirrhosis, hospitalized with a diagnosis: “bleeding from the veins of gullet”. Sharp pallor of skin covers. BP 80/40 mm Hg, pulse 100 per 1 min, the temperature of body is normal. On a

Page 244: intranet.tdmu.edu.uaintranet.tdmu.edu.ua/data/kafedra/internal/infect_desease... · Web viewBest method to treat diarrhoea in child is: intra venous fluide *ORS antibiotics bowel

front abdominal wall a venous net is extended. What preparation is it necessary to begin therapy from?

a. Albumenb. Plasmac. Neogemodezd. *Bloode. Glucose

342. 24 years old patien is bothered by pharyngalgia, general weakness. Objectively: temperature 38,0 °C. pulse 96 per min, hyperemia of otopharynx, enlargment of tonsills. Neck and submandibular lymphatic nodes are palpated, spleenomegaly present. Polymorhyc spots on the skin of trunk. What disease is most probable?

a. Scarlet feverb. Acute respiratory diseasec. Follicle tonsillitisd. Typhoid fevere. *Infectious mononucleosis

343. 17 years old patient has a temperature of 38,2 °C, generalized lymphadenopathy (neck lymphatic nodes which are located along m. sternocleidomastoideus, mild icterus, hepatospleenomegaly. What is the preliminary diagnosis?

a. Tuberculosis of lymphatic nodesb. Bacterial tonsillitisc. Diphtheriad. *Infectious mononucleosise. Megacaryoblastoma

344. Sick, 24 years old, is bothered by pharyngalgia, general weakness. Objectively: temperature – 38 °C. Pulse – 96 per min., increasing and huperemia of tonsills. Also increasing of neck and submandibular lymphatic nodes, spleenomegaly. What disease is most probable?

a. Measlesb. Follicular tonsillitisc. Typhoid feverd. *Infectious mononucleosise. Acute respiratory infection

345. Sick girl 8 years, entered permanent establishment with complaints of a general weakness, increasing of temperature, pharyngalgiA. Objectively: the mucous of otopharynx is bright red, on tonsills are white raids, is taken off easily. Increasing of all groups of lymphonodes, 1-3 cm in a diameter, dense, elastic, littlesickly, not soldered between itself. A liver is megascopic, spleen – on +1 cm. Leycocytosis, plasmatic cells – 20 %. What is the possible diagnosis?

a. Acute lympholeycosisb. *Infectious mononucleosisc. Tonsillitisd. Diphtheriae. Adenoviral infection

Page 245: intranet.tdmu.edu.uaintranet.tdmu.edu.ua/data/kafedra/internal/infect_desease... · Web viewBest method to treat diarrhoea in child is: intra venous fluide *ORS antibiotics bowel

346. At a child with the catarrhal phenomena increasing of submandibular and back neck lymphatic nodes, hyperplasia of tonsills, presence of the single roseol-papular rash, increasing of liver it was found. What is the possible diagnosis.

a. *Infectious mononucleosisb. Adenoviral infectionc. Scarlet feverd. Measlese. Flu

347. In a policlinic 18 years old patient complaints of a moderate pharyngalgia, headacke, general weakness, high temperature of 38,9 °C. He was sick during three days. Objectively it was found: increasing of back neck, and inguinal lymphatic nodes, hepatospleenomegaly, subicterus of scleras and skin, changes in a throat, characteristic for a quinsy. Lymphomonocytosis in a general blood analysis. What is the possible diagnosis?

a. *Infectious mononucleosisb. Follicular tonsillitisc. Adenoviral infectiond. Flue. Viral hepatitis

348. In a policlinic the patient C., 18 years old, appealed with complaints of a moderate pharyngalgia, head pain, general weakness fervescence, to 38,9 °C. It is ill already three days. Objectively: it is found out megascopic back neck, and inguinal lymphatic knots, hepatospleenomegaly, subicterus scleroticas and skin, changes in a throat, characteristic for a quinsy. At the hemanalysis – lymphomonocytosis. What are the principles of treatment?

a. Antibiotics, hepar protective preparations, antihistaminicb. Antiherpetic preparations, hepar protective preparations, antihistaminicc. *Antibiotics, preparations of interferon, hepar protective preparationsd. Antibiotics, Antiherpetic preparations , antihistaminice. Antibiotics, preparations of interferon, vitamins

349. At a patient, 17 years: quinsy, temperature 38,2 °C, generilized lymphadenopathy (the first multiplied neck lymphatic nodeswhich are located along m. sternocleidomastoideus), small icterus hepatospleenomegaly. What exciter causes this disease?

a. Herpesvirus I typeb. Herpesvirus ІІ typec. Herpesvirus ІІІ typed. *Herpesvirus ІV typee. Herpesvirus V type

350. Sick, in 24, disturbs a pharyngalgia, general weakness. Objectively: temperature – 38 °C. Pulse 96 shots per min, dermahemia mucus of otopharynx bloodshot, tonsills are megascopic in sizes, loose. Palpated megascopic back neck and submandibular lymphatic knots., a spleen comes forward from under the edge of costal arc of to 1 sm. What methods of diagnostics is it possible to confirm the diagnosis?

a. Reaction of Paul-Bynnel, Right-Heddlson, Goffa-Bauerb. *Reaction of Paul-Bynnel, Lovrik-Volner, Goffa-Bauerc. Reaction of Paul-Bynnel, Right-Heddlson, Lovrik-Volner

Page 246: intranet.tdmu.edu.uaintranet.tdmu.edu.ua/data/kafedra/internal/infect_desease... · Web viewBest method to treat diarrhoea in child is: intra venous fluide *ORS antibiotics bowel

d. Reaction of Paul-Bynnel, Goffa-Bauere. Reaction of Paul-Bynnel, Goffa-Bauer, Right-Heddlson, Lovrik-Volner

351. Patient P, 17 years old, has got ill gradually. General weakness, fatigue, painful throat, and nausea were marked. He was hospitalized on the 5th day of illness. Objectively: temperature is 38 °C. Objectively: skin and sclera are yellowish, neck, anticubital and subclavian lymph nodes are enlarged. Not numerous mculo-papular elements of rash on a trunk are found. The tongue is covered with white coat. The tonsils covered with white yellowish patches.On palpation of stomach enlarged spleen and liver are found. In blood there is leucocytosis with neutophylic shift, atypical mononuclears-10 %, plasmatic cells- 10 %. What would be the diagnosis?

a. Scarlet feverb. *Infectious mononucleosisc. Typhoid feverd. Iersiniosis e. Lacunar tonsilitis

352. The boy 12 years old, with catarrhal phenomena seen an increase of all the lymph nodes, sclera, hyperplasia of tonsils white layers on them as islands, existing single maculo-papular rash, increased liver and spleen size. What additional research should be done?

a. Reaction of passive heamaglutination(RPG) with influenza virusesb. Ultrasound of the abdomenc. Puncture of the lymph nodes, with following microscopyd. Inoculation of bloode. *Blood test for atypical mononuclears

353. Sick C., 17 years old fell ill gradually. Disturbed by general weakness, fatigue, sore throat, abdominal pain, nauseA. Hospitalized at 5th day of illness. Examination: body temperature 38,0 °C, increasing of the neck, shaped inframaxillary lymph nodes. Skin and eye sclera were subicteric. Macula-papular rash on the trunk. Raids on tonsils thick, white, yellow, white furred tongue stomach is moderately swollen, hepatosplenomegaly. In blood analyses: leukocytosis, microphage with the left shift, atypical mononuclear – 10 %, plasma cells – 10 %. What is the preliminary diagnosis?

a. *Infectious mononucleosisb. Yersyniosisc. Lacunar tonsillitisd. Viral hepatitis Ae. Typho-paratyphoid disease

354. Patient 18 years, entered permanent establishment with complaints of head pain, general weakness increase of temperatures, to 37,5–38,2 °C during 6 days, pharyngalgiA. Objectively: all groups of lymphonoduses, 1-3 cm in a diameter, elastic, are megascopic, little sickly, not soldered between itself. A liver is megascopic on 3 sm, spleen – on 1 sm. In a blood is leycocytosis, plasmatic mews – 15 %. What group of herpes does the exciter of this illness belong to?

a. Alphab. Betac. *Gammad. Delta

Page 247: intranet.tdmu.edu.uaintranet.tdmu.edu.ua/data/kafedra/internal/infect_desease... · Web viewBest method to treat diarrhoea in child is: intra venous fluide *ORS antibiotics bowel

e. Teta355. Patient A., 18 years old, is complaining of headache, weakness, high temperature,

sore throat. Objectively: general lymphadenopaty,with lymph nodes 1-3 cm in diameter, dense, elastic not painful and not combined between themselves. Hepatospleenomegaly was noticed. In blood there is leucocytosis, monocytes – 15 %. What is the diagnosis?

a. *Infectious mononucleosisb. Adenoviral infectionc. Tonsillitisd. Diphtheriae. Acute leukemia

356. An 18 years old patient, entered the hospital with complaints of headache, general weakness, raising the temperature to 37,5-38,0 °C for 6 days, a sore throat. Objective examination: increasing of all lymph nodes, 1-3 cm in diameter, flexible, megaloblastic not soliter together. Increasing of the liver size till 3 cm, spleen – 1 cm. Common blood analysis: leukocytosis, plasma cells – 15 %. What is the preliminary diagnosis?

a. Diphtheriab. Adenovirul infectionc. Lacunar tonsillitisd. *Infectious mononucleosise. Acute lympholucosis

357. Patient D., 20 years old, fell ill and went to infectious polyclinic cabinet with complaints of moderate pain in the throat, headache, general weakness, increased body temperature of 38.9 °C. Fells bed during three days. At examination: increasing of inguinal lymph nodes, hepatosplenomegaly, subicteric sclera and skin, changes in the throat are typical for sore throats. In the blood – lymphogranulomatosis. The principles of treatment are?

a. *Antibiotics, interferons, hepatoprotectorsb. Antihypertensives hepatoprotectors, antihistaminesc. Antibiotics, hepatoprotectors, antihistaminesd. Antibiotics, antihypertensives, antihistaminee. Antibiotics, interferones, vitamins

358. A patient D., 20 years old came to the infectious polyclinic with complaints of moderate pain in the throat, headache, general weakness, increasing of body temperature to 38.9 °C. He fells sick during 3 days. Objective examination: increasing inguinal lymph nodes, hepatosplenomegaly, subicteric sclera and skin, changes in the throat are typical for sore throats were found. Blood analysis – lymphogranulomatosis. What is the probable diagnosis?

a. *Infectious mononucleosisb. Tonsillitisc. Adenovirus infectiond. Influenzae. Viral hepatitis

359. The patients, 20 years old, with 5 days fever, pain in the throat during swallowing, pain in muscles. During review – there is thick, congested tonsils with purulent layers that are easily removed, enlarged neck, and inframaxillary lymph glands, liver + 1.5 cm possible palpation of edge of spleen. What additional research should be done?

Page 248: intranet.tdmu.edu.uaintranet.tdmu.edu.ua/data/kafedra/internal/infect_desease... · Web viewBest method to treat diarrhoea in child is: intra venous fluide *ORS antibiotics bowel

a. Puncture of the lymph nodes, followed by microscopyb. Ultrasound of the abdominal cavityc. *Blood test for atypical mononucleard. Inoculation of bloode. Reaction of passive heamoglutination(RPH) influenza A viruses

360. In children with catarrhal phenomena revealed an increase of inframaxillary lymph nodes, tonsil hyperplasia with whitish soft touch on them in the form of points and islands, the presence of rare maculo-papular bulging out, increased liver and spleen. Identify the likely diagnosis?

a. *Infectious mononucleosisb. Acute leukemiac. Measlesd. Scarlet fevere. Chicken pox

361. Patient A., 35 years old, came to the clinic on the third day of disease with complaints of drowsiness, sweat, headache, fever up to 38.5 °C. Reviewing physician found sore throat, splenomegaly, enlarged lymph nodes. Liver not enlarged. Blood analysis – leukocytosis, lymphomonocytosis. What is the treatment for the patient?

a. *Antibiotic, drugs interferon, hepatoprotectorsb. Antihistamin, antiherpetic preparations hepatoprotectorsc. Antibiotic, hepatoprotector, antihistamined. Antibiotics, antihistamine, antiherpatic drugse. Vitamins, antibiotics, drugs interferon

362. 35 years old patient A., came to the clinic on the third day of illness with complaints of drowsiness, sweat, headache, fever up to 38.5 °C. Reviewing physician found sore throat, splenomegaly, enlarged lymph nodes. Liver is not enlarged. Blood analysis – leukocytosis, lymphocytosis. What additional test should hold the patient for infectious mononucleosis?

a. Byurne and Wright-Hadulson’s reaction b. ELISA-test, bacteriological test for tularemiac. Bacteriological test for diphtheria and typhoid feverd. *ELISA-test, bacteriological test for diphtheriae. Paul-Bunnel’s reaction and lymph node puncture

363. Patient L., 38 years old, complaints of recurrent attacks of fever with severe chills and heavy then. Epemiologically found that he had recently returned from IndiA. When parazitoscopii blood found erythrocytic schizonts. Vector of the disease is:

a. Flyb. *Mosquitoesc. Flead. Beee. Pliers

364. Patient G., 28 years old, sailor, joined the 18-day illness, complaining of high fever, headache, weakness. Disease began as a chills during an hour, then the temperature had risen to 39,8 °C. In next 5-6 days, chills repeated every day in the middle of the day, after which the temperature rose to 40-41 °C, such attacks fever occur every other day. Face is hyperemic, herpes, vascular injection of sclerA. Tons of the heart muted,

Page 249: intranet.tdmu.edu.uaintranet.tdmu.edu.ua/data/kafedra/internal/infect_desease... · Web viewBest method to treat diarrhoea in child is: intra venous fluide *ORS antibiotics bowel

rhythmic, pulse 140 for 1 min, AD 140 and 60 mm RT. Art. Liver is normal, spleen +2 sm. Which method of diagnosing is the most valuable in this situation?

a. *Smear and thick drop of blood for malariab. An overall analysis of bloodc. Bilirubin in the blood, ALaTd. culture of bloode. Immunological Methods

365. Patient G., 28 years old, sailor, has been ill in 18-day illness, complaining of high fever, headache, weakness. Disease began as a chills during an hour, then the temperature had risen to 39,8 °C. In next 5-6 days, chills repeated every day in the middle of the day, after which the temperature rose to 40-41 °C, such attacks fever occur every other day. Face is hyperemic, herpes, vascular injection of sclerA. Tons of the heart muted, rhythmic, pulse 140 for 1 min, AD 140 and 60 mm RT. Art. Liver is normal, spleen +2 sm. What you need to start treatment?

a. *Primaquineb. Fansidarc. Metakelfind. Quininee. Tetracycline

366. A citizen of Nigeria, 19 years old, came to study in Ukraine. After 10 days of appeared a strong chill, which lasted about 1 hour and changed the feeling of heat. There has been a headache, slice, nagging pain in the muscles. Attack of such clinical symptoms rapid again after 2 days. OBJECTIVE: body temperature 39,6 °C, sclera subicterich, appeared lips herpes. Spleen significantly increased, dense, the liver is a normal. What investigation needs to be done?

a. *Microscopy of peripheral bloodb. Microscopy of centrifuge of urinec. Lumbar punctured. Observation of oculie. USD abdomen

367. The patient concerned about attacks of fever, which are repeated every third day. There are jaundice of sclera and skin, hepatosplenomegali. Which of the diagnoses most likely?

a. Sepsisb. *Malariac. viral hepatitisd. Hemolytic anemiae. Leptospirosis

368. Patient G., 20 years old, six months ago returned from AfricA. A month later he introduced the three-day clinical symptoms of malariA. Is it necessary to carry out epidemic contacts that the patient had?

a. You do not need because there is no vectorb. You need because there is vectorc. *You do not need because reconvalence not caused. It should be, because rekonvalence allocates causee. You do not need because epidemic activities inefficient

Page 250: intranet.tdmu.edu.uaintranet.tdmu.edu.ua/data/kafedra/internal/infect_desease... · Web viewBest method to treat diarrhoea in child is: intra venous fluide *ORS antibiotics bowel

369. A patient G., 42 years old, for a week every 48 hours had attacks of chills, which followed by heat. Body temperature rises to 40 °C, in 3-4 hours was a severe sweat. Worsened appetite, a common weakness. Skin pale. Hepatosplenomegaly. What are the most effective method for verification of the diagnosis.

a. *Microscopy and a thick smear of blood dropsb. Microscopy hanging dropsc. An overall analysis of bloodd. Bacteriological methode. Enzyme multiplied immunoassay

370. Citizen B. travelling in the summer as a tourist to resort, disadvantaged malariA. How to prevent malaria infection?

a. Follow the rules of personal hygieneb. Do not drink raw waterc. Make vaccined. *Hold chemoprophylaxise. Conduct heat treatment of food

371. Patient A., 25, treated at a hospital about malariA. The latest attack was the day before. Assign etiotropic treatment.

a. Antibioticsb. Serum transferc. Delagild. Primaquinee. *Delagil + Primaquine

372. A patient 20 years old, had treated himself concerning an acute respiratory disease for 5 days, but marked no displays of respirator syndrome. Last 2 days temperature has been normal, appetite disappeared, however, appeared pain in epigastrium with nausea, and urine turned dark. About what illness is it possible to think?

a. Hepatitis Bb. Infectious mononucleosisc. Pseudotuberculosisd. Leptospirosise. *Hepatitis A

373. A patient 28 years old, an injection drug addict, complaints of dull pain in right subcostal region, weakness, decline of appetite, pain in joints. The symptoms have been present for 2 weeks. At examination: the icterus of skin and scleras observed. The liver and spleen are enlarged. Urine is dark, excrements are discoloured. What is preliminary diagnosis?

a. Hepatitis Ab. *Hepatitis Bc. Chronic cholecystitisd. Toxic hepatitise. HIV-infection

374. A patient 25 years old got sick suddenly after chills and temperature increase up to 38,0 °C. There was vomiting 2 times. Moderate pharyngeal pain appeared at swallowing. with stomach discomfort. Signs of bursitis observed on knee and elbow joints. The light icterus of sclera and skins of joints on 4th day, on the skin of lateral surfaces of trunk,

Page 251: intranet.tdmu.edu.uaintranet.tdmu.edu.ua/data/kafedra/internal/infect_desease... · Web viewBest method to treat diarrhoea in child is: intra venous fluide *ORS antibiotics bowel

forehead and lower extremities. The small bright red papular rash appeared, more concentrated in natural folds.Liver is enlarged, the tongue is raspberry like appearence. Preliminary diagnosis would be:

a. Hepatitis Ab. . Hepatitis Bc. *Pseudotuberculosisd. Scarlet fevere. Infectious mononucleosis

375. A 19 years old patient was diagnosed with hepatitis B. After violation of diet and nervous stress the state of patient got worse: intensity of ichterus was increased, pulse 110/min, BP 80/50 mm Hg. Also there was vomiting by «coffee-grounds». The size of liver has decreased, the liver on palpation was painful. What complication would you think about?

a. Infectious-toxic shock b. Acute kidneys insufficiency c. Acute adrenal insufficiencyd. Hemolitic icteruse. *Acute hepatic insufficiency

376. A patient C., 43 years old, has been treated for 5years . During the last hospitalization a liver-biopsy was conducted. Histologicaly there is bulb dystrophy and necrosis of hepatocytes, with leucocyts infiltration, and pericellular fibrosis. What diagnosis is most credible in this case?

a. *Alcoholic liver cirrhosisb. Viral hepatitis Bc. Viral hepatitis Cd. Cryptogenic hepatitise. Fatty dystrophy of liver

377. Patient B., 23 years old. Objectively: skin is yellow, icterus of sclerA. Pulse 66/min, BP 120/80 . Indexes of AlAT, AsAT, and Tymol tests are normal. General billirubin is 34 mmol/l due to indirect fraction. Specify the most credible diagnosis.

a. Chronic toxic hepatitisb. Chronic cryptogenic hepatitisc. Chronic hepatitis Cd. *Gilbert’s syndromee. Chronic B hepatitis

378. A trained nurse got sick on 16.09: general weakness, nausea, vomiting, pain in joints appeared. 21.09 urine turned dark, and skin yellow. 21.09 she was hospitalized. Objectively: general status satisfactory, temperature 38,2 °C, expressed skin and mucous membranes jaundice. Liver palpated 3 cm below the costal arc. 26.09 stomach-aches and increased sleepiness with disorientation in time and place were appeared. 27.09 haematomas in the places of injections also appeared. Hepatic breath was noticed. Light edema of feet and lumbar region were observed. The liver was not palpable. Development of what syndrome can be forecast?

a. Meningoencephalitisb. *Hepatic encephalopathy

Page 252: intranet.tdmu.edu.uaintranet.tdmu.edu.ua/data/kafedra/internal/infect_desease... · Web viewBest method to treat diarrhoea in child is: intra venous fluide *ORS antibiotics bowel

c. Acute nephrosonephritisd. Pancreatitise. IDS syndrom

379. A patient B., 52 years old, for 3 month complaints of nausea, periodic vomiting, swelling of stomach, weakness, loss of weight up to 12 kg, consistency of stool is chainging. During some days consciousness is entangled, somnolence, allolalia, general weakness are expressed. Temperature of body 37,4 °C. Icterusis on the skin. Tremor of brushes and nystagmus is marked. Pupils are narrowed, with a slow reaction on light, increase of tendon reflexes. Ps 112/min, unrhythmical. BP 90/65 mm Hg. A liver is dense +2. Laboratory tests: Hb 86 g/L, Leuc 4,2*109/L, ESR 18 mm/hour, glucose 3,4 mmol/l, general bilirubin 56,7 mm/L, albumen 52 g/L, K+ 3,2 mmol/l, AsAT 0,62; AlAT 0,84; prothrombin index-58 %; рН blood 7,3. What is previous diagnosis?

a. Hepatocirrhosisb. Vilson-Konovalov diseasec. Cancer of liverd. *Endogenous hepatic encephalopathye. Meygs disease

380. A patient, 17 years, complaints of a weakness, worsening of appetite, nausea, painfull in right subcostal area, dull ache in the body, dark color of urine, temperature to 37,8 °C. She stood in contact with sick on icterusis 1,5 months ago. Objectively: yellow colour of the skin and visible mucous membranes. The liver is increased to 1,5 cm. Unsignificant enlargment of spleen. What is your preliminary diagnosis?

a. Viral hepatitis Bb. Infectious mononucleosisc. *Viral hepatitis Ad. Leptospirosise. Pseudotuberculosis

381. Before the appearance of jaundice in a patient 16 years old, during 2 days there were an increasing of the body temperature (38,5 °C), headache, dull ache disturbed whole body. Name the variant of pre-icteric period of viral hepatitis for this patient?

a. *Influenza-likeb. Astenovegetativec. Artralgicd. Dyspeptice. Allergic

382. A patient T., 28 years, appilled due to worsening of common status at the seventh day to the infectious department concerning viral hepatitis. Nausea and vomiting were present, hepatic smell appeared, increasing of the liver sizes. What biochemical index does allow to suspect complication in the patient?

a. Increasing activity of AlATb. Increasing of bilirubin, increasing activity of AlATc. Increasing timol testd. *Decreasing of protrombin, increasing of bilirubine. Decreasing of AlAT

383. A sick P., 54 years old, complaints of dull pain in right subcostal area, bad taste in the mouth, itching of the skin, increasing of abdomen. After the liver puncture fatty

Page 253: intranet.tdmu.edu.uaintranet.tdmu.edu.ua/data/kafedra/internal/infect_desease... · Web viewBest method to treat diarrhoea in child is: intra venous fluide *ORS antibiotics bowel

dystrophy of hepatocytes, eccentric placing of veins in a hepatic lobule were found. What laboratory index are most characteristic in this case?

a. Alkaline phosphotaseb. Hemodiastasec. *Level of transaminasesd. Glucose of bloode. Creatinphosphokinase

384. Direct bilirubin is increased , in urine there is significant increase of bilirubin and urobilin, increasing of stercobilin of excrements. What is the type of icterus?

a. Haemoliticb. *Parenchimatousc. Transportd. Extralivere. Mechanical

385. On the average 15 to 30 % of all population of the planet suffer from some pathology of liver. Prevalence of hepatitis and cirrhosis in the European countries is about 1 % of adults. Annually in the world there are about 2 million people with acute viral hepatitis. What % of all cases will develop chronic form.

a. 100 %b. 50 %c. 25 %d. *10 %e. 1 %

386. A patient, 24, complaints of pains in right subcostal area, increasing after-meal, nausea, increase temperature of body to 37,7 °C, icterus, pains in large joints. He is ill from 8 months. Suffers a nonspecific ulcerative colitis. Hepatosplenomegaly. ESR 47 mm/hr, general bilirubin level is 86,1mmol/lt, direct-42,3 mmol/lt. In blood found out antibodies against smooth muscles. General albumen of 62 gram/lt, albumin б. 40 %, globulin. 60 %, gamma globulins 38 %. Not found out the markers of viral hepatitis. On USD diameter of portal vein is 1 cm What will be your diagnosis?

a. Primary biliary cirrhosisb. Gilbert syndromec. Viral hepatitisd. Hemochromatosise. *Autoimmune hepatitis

387. Woman of 22 years old, in the 7th month of pregnancy felt ill acutely in 3 weeks after arrival from Turkmenistan. An icterus, untraceable vomiting, pain in right subcostal region,, hemmorhaeges on the skin, appeared after three-day fever. Most probable diagnosis will be:

a. Acute fatty hepatosis of pregnancyb. Cholestatic hepatosis of pregnantc. *Viral hepatitis of Ad. Viral hepatitis Be. Acute sepsis

388. Weakness appeared in a patient, decrease in an appetite, began pains in the joints of extremities, felling of weight in right subcostal region, vomiting. Urine became dark in

Page 254: intranet.tdmu.edu.uaintranet.tdmu.edu.ua/data/kafedra/internal/infect_desease... · Web viewBest method to treat diarrhoea in child is: intra venous fluide *ORS antibiotics bowel

12 days, and in a day – sclera and skin turned yellow. It was discovered in an immunogram: antuIgG -HAV (+), HBsAg (+), antiIgМ -НBcorAg (+), anti-НCV (–), anti-НDV (–). To the patient 3 months back was extracted a tooth. Most probable diagnosis?

a. *Viral hepatitis Bb. Viral hepatitis Ac. Viral hepatitis Cd. Viral hepatitis De. Viral hepatitis E

389. A patient 42 years, radiologist, entered surgical department with bleeding from the mouth ,was admitted to the gastroenterological departments . He considered himself healthy. 20 years ago had haemotransfusion. Fibrogastroduodenoscopy shows venous dilation of the oesophagus of the III degree. Most probable etiologic reason of disease:

a. Autoantibodies against hepatocytesb. Viral hepatitis Ac. X-ray irradiationd. *Viral hepatitis Be. Defficiency of antitrypsin

390. A patient during half of the year got frequent parenterally injections concerning bronchial asthmA. There was decrease in appetite gradually, started to feel weakness, arthralgia, dark coloured urine, the icterus of skin appeared. Objectively: temperature of 37 °C, pulse 68/min BP 115/70 mm of Hg Liver +4 cm, spleen +1 cm, skin and sclera is yellow coloured In the general blood analysis: amount of leucocytes: 3,6 109,among them 52 % lymphocytes, ESR 6 mm/hr, activity of ALAT is increased in 10 times. which information will be more credible than all in blood of patient?

a. Anti-HBs antibodiesb. Anti-HAV IgM antibodiesc. *Anti-HBV antibodiesd. Anti-HCV IgG antibodiese. Anti-HEV IgM antibodies

391. A patient of 34 years complaints of fatigue, decreasing of appetite, nausea, feeling of weight in a epigastric region, bitter taste in the mouth. Objective examination : Pulse 76/min, temperature 37,2 °C. Skin is pale, single vascular star-like rash on the thorax. Liver is 2 cm below the costal arch. In blood: bilirubin level is 36 mmol/lt, ASAT 2,5, ALAT 2,8 . Anti-HBc (–), HBsAg and HBeAg (+). What therapeutic tactic is expedient in this case?

a. Therapy by hepato-protectorsb. *Antiviral therapy by lamivudine c. Antiviral therapy by ribavirind. Therapy by corticosteriodse. Therapy by immunostimulators

392. Man 30 years, drug addict, takes drugs intravenously. Has been taking drugs for 12 years Complaints of weakness, moderate icterus, weight in right subcostal region. The state was worsened gradually. Biochemical indexes: general bilirubin 28,2 mmol/lt; ALAT 1,0, ASAT 0,8 . Will you define a diagnostic method which it is expedient to conduct for establishment of etiologic diagnosis?

a. Biopsy of liver

Page 255: intranet.tdmu.edu.uaintranet.tdmu.edu.ua/data/kafedra/internal/infect_desease... · Web viewBest method to treat diarrhoea in child is: intra venous fluide *ORS antibiotics bowel

b. *Polymerase chain reaction (PCR)c. Enzymes of liverd. Computer tomographye. Immunological tests

393. Patient B., 51 years, a lot of years used an alcohol. Complaints of nausea, liquid stools, icterus of skin. Objectively: skin and sclera is yellow coloured, atrophy of muscles, subfebrile temperature. Liver 3 cm below the costal arch, painful on palpation What method of diagnostics most informing in this case?

a. Activity of cytolysis (AsAT, AlAT)b. Proteinogramc. *Biopsy of liverd. Markers of viral hepatitise. Violation of cellular immunity (T4,T8)

394. Patient of B., 64 years old, has been sick with chronic hepatitis. Complaints of pain in the liver, growth retardation. Objectively: his skin is dry, erythemic, atrophy of muscles, telangiectasis on shoulders, hyperemia of hands, abdomen is enlarged, look like “jelly-fish head”. The liver below the costal arc on 3 cm, painful, dense, spleen on 1 cm below the costal arch, positive symptom of fluctuation. Laboratory investigation: hypergammaglobulinaemia, increasing of AsAT activity, AlAT is highly increased. What drug of choice for the patient?

a. Lactuloseb. *Hepatoprotectors c. Antibiotics d. Glucocorticoidse. Alpha interferon

395. A patient 48 years old, complaints of attacks pains in right subcostal area after the physical loading. Periodically marks more light excrement, darkening of urine. Objectively: skin and mucous membranes high icteric. General bilirubin 36,8 mm/L, direct fraction - 26,4. Ultrasound of gall-bladder: thickness of wall 4 mm, there is a lot of bile in the ducts. It is necessary to prescribe with a lytolitic purpose:

a. Cholereticsb. *Ursofalkc. Cholekineticsd. Spasmolyticse. Cytostatics

396. During annual inspection increasing of a liver to 4 cm of 23 years old patient was found, increasing of bilirubin level in 2 times, AlAT in 2,5 time. He has been often sick with genital herpes. It was discovered RNA of hepatitis C virus. What is etiologic treatment

a. *Interferonsb. Essencial phosphotidesc. Cholereticsd. Cholekineticse. Ursodesoksihole acids

397. A patient, 19 years old, with hepatitis B. After violation of diet and nervous stress the state of the patient became worse: increasing of intensity of jaundice, pulse 110 per 1

Page 256: intranet.tdmu.edu.uaintranet.tdmu.edu.ua/data/kafedra/internal/infect_desease... · Web viewBest method to treat diarrhoea in child is: intra venous fluide *ORS antibiotics bowel

min, BP 80/50 mm Hg, vomiting with “coffee-grounds”, decreasing of the liver size. What complication is it possible to think about?

a. *Acute liver insufficiencyb. Infectious-toxic shockc. Acute kidney insufficiency d. Acute extrarenal insufficiencye. Hemolytic

398. A patient 28 years old, injection drug addict, complaints of dull pain in the right subcostal area, weakness, decline of appetite, pain in joints, which disturbs during 2 weeks. At examination: icterus of the skin and sclera, enlargement of the liver and spleen, dark urine, excrement is discoloured. What is preliminary diagnosis?

a. HIV-infectionb. Hepatitis B c. Hepatitis Cd. *Toxic hepatitise. Hepatitis A

399. At inspection of a 8 weeks term pregnant woman HBsAg was found. Level of bilirubin of blood and activity of ALAT were normal. What is necessary to do?

a. *To save pregnancy and conduct the inoculation to newborn against hepatitis B

b. Termination of pregnancyc. Termination of pregnancy and conduct treatment by interferond. To save pregnancy and conduct treatment by lamivudine. To save pregnancy and ultrasonic inspection of the fetus

400. Student 20 years, treated oneself on an occasion of ARVI (increasing of temperature to 38,2 °C during 3 days). He complaints of worsening of appetite, increasing fatigue at a normal temperature and absence of the catarrhal phenomena of upper respiratory tracts. A doctor found out an increasing and moderate sickliness of liver. There were cases of hepatitis A in a student’s group. What method of investigation will allow?

a. Ultrasound scanning of the liverb. Determination of bilirubin level of the bloodc. Determining the amount of beta-lipoproteinsd. *Determination of activity of аminotransferases of the bloode. Immunofluorescent research of the nasal smears

401. A patient I., 25 years old, appealed to the internist with complaints of a general weakness, loss of appetite, pain in the right hypochodrium. Treated himself on an occasion of ARVI ambulatory 3 days. Became acute worsening of fealling: nausea, pain in right hypochondrium, ochrodermia of sclera, darkening of urine, appeared. Objectively: breathing superficial, cardiac tone is rhythmic. Abdomen is soft, painfull in right hypochondrium, a liver is megascopic, a spleen not palpable. What is most credible diagnosis?

a. *Hepatitis Ab. Leptospirosisc. Cholecystitisd. Influenzae. Pseudotuberculosis

Page 257: intranet.tdmu.edu.uaintranet.tdmu.edu.ua/data/kafedra/internal/infect_desease... · Web viewBest method to treat diarrhoea in child is: intra venous fluide *ORS antibiotics bowel

402. A patient 42 years old, complaints of dull pain in a right hypochondrium, weakness, decline of appetite, itching of skin, icterus. Disease began 1 month ago from protracted аrthralgia, disgust for a meal, strong weakness. An icterus which now increased considerably appeared three weeks ago. Temperature 36,0 °C. Pulse 56 per 1 min. A tongue is assessed with white cover. A stomach is soft, sickly in right hypochondrium. A liver +3 cm, spleen is not megascopic. Common analysis of blood: leukopenia, relative lymphomonocytosis, ESR 3 mm/hour. What diagnosis is most credible?

a. Cancer of head of a pancreasb. *Hepatitis B c. Hepatitis Ad. Cirrhosis of livere. Chronic cholecystitis

403. For a patient 35 years after 4-months of treatment by isoniaside - аdynamia, icterus, pain in right hypochondric were appeared. A liver is megascopic. In blood activity of enzymes of AlAT is enhanceable in three times, AsAT in two times. Bilirubin of blood of 122 gm/ml (conjugated – 82, unconjugated – 40). НBs-аntigen is not found out. What is the diagnos?

a. Calculary cholecystitisb. Hepatocirrhosisc. Acute viral hepatitisd. Chronic active hepatitise. *Toxic hepatitis

404. A patient 75 years old. Complaints of a subfibrile temperature, general weakness, pharyngalgia, conjunctivitis. A child in family an acute adenoviral disease is ill. A patient considers itself a patient the second day. At examination discovered sign of sharp pharyngitis. Lymphatic nodesare megascopic: neck front and back, arm-pits and inguinal, to 1 sm in a diameter, soft, not soldered between itself and with a surrounding cellulose. A pharynx is bloodshot, amygdales are hypertrophied and hyperaemiA. In lights wheezes are not present. Breathing clean. Tones of heart are muffled. BP - 140/80 mm Hg, Ps - 80 per 1 min. Stomach soft. Palpatory- megascopic liver, salient on 3 sm below than costal arc, and spleen, soft, painless. Choose the most credible diagnosis from offered:

a. Flu, to middle weightb. Limfogranulomatosisc. *Adenoviral infectiond. Infectious mononucleosise. Hepatitis A

405. A patient is 35 years, grumbles about aching pain in right hypocostal area, nausea, decline of appetite. Beginning of disease binds to appendicitis. After it in 2 months an icterus appeared first. Treated oneself in an infectious department. In 1 began to notice aching pain in right hypocostal area, in analyses is an increase of level of bilirubin. Your diagnosis?

a. Calculary cholecystitisb. Zhil'ber diseasec. Sharp viral hepatitisd. Chronic cholangitise. *Chronic hepatitis

Page 258: intranet.tdmu.edu.uaintranet.tdmu.edu.ua/data/kafedra/internal/infect_desease... · Web viewBest method to treat diarrhoea in child is: intra venous fluide *ORS antibiotics bowel

406. The patient T., 35 years, operating trained nurse, appealed to the doctor on the 8th day of gradual development of illness with complaints of a general weakness, rapid fatigueability, dark color of urine. In the morning noticed the icterus. On examination temperature of body 36,8 °C. Found out the increase of liver -+3 sm The changes of what laboratory index most informing at this illness?

a. *AlAT b. Hemodiastasesc. Protrombin indexd. Cholesterole. Alkaline phosphatase

407. 23-years old patient during 6 months gets diabetes mellitus in a policlinic the injections of insulin. A weakness, arthalgia, grew gradually, an appetite disappeared, then dull pain appeared in an epigastrium after-meal. In 2 weeks from the beginning of illness noticed the dark color of urine, and afterwards – icterus on a background which the general state continues to be worsened. Temperature of body 36,5°C, Ps 58 per 1 min Liver +5 sm, spleen +1 sm What from hemanalysis is it needed to appoint for confirmation of diagnosis?

a. Activity of alkaline phosphataseb. Activity of lactatdehydrogenasec. *Activity of ALATd. Activity of creatinphosphokinasee. Activity of amylase

408. 42-years old patient got blood transfusion two months ago. A weakness was gradually increased, an appetite was worsened, arthalgia, small nausea appeared. On a 12th day noticed an insignificant icterus, appealed to the doctor. At examination is the general state fully satisfactory. The temperature of body is normal, Ps 60 per 1 min, BP 100/70 mm Hg.A liver and spleen is moderate megascopic, a stomach at palpatory is not sickly. Anti-HCV IGM is found in blood. What from changes in the global analysis of blood most probably?

a. Neutrocytosisb. *Leucopenia c. Lymphopeniad. Increased ESRe. Aneozinophilia

409. A 17-years-old patient during 4 days suddenly had head pains, myalgias, fever of permanent type. From a 5th day is a normal temperature of body, the general state was considerably improved, an appetite appeared, nausea, general weakness, disappeared, but an icterus became noticeable. The general state is satisfactory, liver +3 cm, spleen +1 sm, Ps 56 per 1 mins Peripheral lymphatic knots, amygdales are not megascopic, hyperemia of pharynx is not present. At research of global analysis of blood is leykopenia with a relative lymphocytosis, 3 % virocytis, ESR is normal. Activity of ALAT of blood is enhanceable in 5 times. What from diagnoses most credible for a patient?

a. *Hepatitis Ab. Infectious mnonucleosisc. Cytomegaloviral infectiond. Hepatitis B

Page 259: intranet.tdmu.edu.uaintranet.tdmu.edu.ua/data/kafedra/internal/infect_desease... · Web viewBest method to treat diarrhoea in child is: intra venous fluide *ORS antibiotics bowel

e. Hepatitis C410. In 2 months after returning from India, where often drank unboiled water, a 23-

years-old expectant mother nauseated, strong general weakness, head pain, later the temperature of body rose to 38 °, and stuck to week. An icterus appeared on a 6th day, the general state continued to be worsened. On a 12th day illnesses are the general state heavy. EiforiyA. At night there was vomiting. Complete fastidium. Bright icterus, signs of hemorragic syndrome, tachycardiA. BP 110/60, temperature of body 37,8 °C. A liver is insignificantly megascopic, soft, sickly, spleen +2 cm There is neutrophilic leycocytosis in the global analysis of blood. General bilirubin of blood is 570 to the line 300, ALAT is enhanceable in 100 times, timol test of 26 units., urea – 2,1 mmol/l. What is the most probable diagnosis for this patient:

a. Malignant icterus of pregnantb. Mechanical icterusc. Typhoid feverd. *Acute Hepatitis e. Leptospirosis

411. On the 15th day of illness for a 17-years-old patient by hepatitis In anti-HBs is found in blood. The state of patient was considerably worsened the day before. There was excitation, there was a nose-bleed, a hemorragic rash appeared on a skin, diminished and became sickly at palpatory liver. Ps 106 per 1 min, BP of 110/ 70 mm Hg, the temperature of body subfebrile. What changes of indexes blood will a patient have most credible?

a. Increasing of alkalinephosphataseb. Increasing of fibrinogen c. Increasing of indirect bilirubinumd. *Decline of protrombin indexe. Increasing of albumin

412. 25-years old woman during a semiyear got numerous injections concerning bronchial asthmA. An appetite disappeared gradually, a weakness, arthalgia grew, urine became dark in two weeks, through three – an icterus on a background which the general state continues to be worsened appeared. The temperature of body is normal, Ps 62 per 1 min. Moderate sickliness in the area of pancreas, positive Voskresensky symptom, liver +4 cm, spleen +2 cm What markers of viral hepatitis will be positive in such case?

a. *Anti-HBc IGMb. ANTI-HEV IGMc. ANTI-CMV IGMd. HBsAge. ANTI-HAV IGM

413. 26-years old man during 6 days marks the 6-8-sentinel gettings up temperatures with a strong chill, which end with abundant then and repeat oneself regularly in a day. 6 months ago there were alike attacks of fever during a stay in Afghanistan, from where returned 4 months back. Temperature of body 36,4 °C, Ps 94 per 1 min The moderate increase of spleen and liver is marked. A diuresis suffices, wetting without pathological changes. What disease can be suspected for a patient?

a. Leptospirosisb. Hemorragic feverc. *Malaria

Page 260: intranet.tdmu.edu.uaintranet.tdmu.edu.ua/data/kafedra/internal/infect_desease... · Web viewBest method to treat diarrhoea in child is: intra venous fluide *ORS antibiotics bowel

d. Viral hepatitise. Sepsis

414. For a 16-years-old patient in a month after return from Crimea did head pain, myalgias appear suddenly, strong weakness with a simultaneous fervescence to 39°C, which stuck to 3 days. The general state was considerably improved, a bad appetite and nausea, dull pain, was saved only in right hypocostal area, became dark wetting, white is an excrement. On a 6th day is a moderate icterus, an appetite appeared, a general weakness diminished. Liver +4 cm, spleen +1 cm. Leykopenia in the general blood analysis, relative limphomonocytosis, ESR 4 mm/hour. General bilirubinum of blood of 89, the direct prevails, activity of ALAT is enhanceable in 4 times, ASAT – in 3 times. What from diagnoses most credible?

a. Hepatitis Bb. *Hepatitis Ac. Opisthorchiasisd. Hemolitic icteruse. Sharp calculary cholecystitis

415. Patient of A., 19 years, had ill acutely – the temperature of body rose to 39°C, a weakness, nausea, appeared. Vomiting and stomach-ache was not present. A temperature reposed on high numbers 2 days, then went down to normal. Urine became dark on the 6th day of illness, subicterus noticed on 5th. To this time the feel of patient was improved, nausea, weakness, disappeared. Objectively: moderate icterus of skin and sclerotica, a rash is not present. Pulse 66 per 1 min, BP 110/70 mm Hg.A stomach is soft, painless in all of departments, a liver on 2,5 cm comes forward from under a costal arc, the edge of spleen was palpitated. Is there what reason of fervescence in the pre-icteric period of this disease?

a. *Toxicemiab. Bacteriaemiac. Pancreatitisd. Viremiae. Cholecystitis

416. 40-years old patient during 2th days marks absence of appetite, nausea, general weakness. A year ago carried sharp hepatitis B, avoided a clinical supervision. Objectively: the temperature of body is enhanceable, skin and sclerotica are icterus, the megascopic palpatory, to the moderate closeness, sickly liver, salient from under the edge of costal arc on 3 sm. A spleen is not megascopic. Wetting moderatory dark colouring. The sickliness of joints is marked at motions. ALAT of blood 4,0. What most credible diagnosis?

a. Chronic viral hepatitis, minimum activityb. Chronic cholecystitisc. *Chronic viral hepatitis, moderate activityd. Hepatocirrhosise. Chronic viral hepatitis, high activity

417. A man, 37 years, injection drug addict, practises upon an alcohol, grumbles about a general weakness, dull pain in right hypocostal area, increase of stomach, shortness of breath. It is ill chronic hepatitis on an extent 10 years. Objectively: temperature of body

Page 261: intranet.tdmu.edu.uaintranet.tdmu.edu.ua/data/kafedra/internal/infect_desease... · Web viewBest method to treat diarrhoea in child is: intra venous fluide *ORS antibiotics bowel

37,1°C, an icterus is expressed. Liver +6 cm. In an abdominal region is a free liquid. Positive symptom of fluctuation. What complication did arise up for a patient?

a. Intensifying of chronic hepatitis Cb. Sharp hepatic insufficiencyc. Gepatokarcinomad. Intensifying of chronic hepatitis Be. *Hepatocirrhosis

418. A patient is 59 years, suffers chronic viral hepatitis with development of cirrhosis, hospitalized with a diagnosis: “bleeding from the veins of gullet”. Sharp pallor of skin covers. BP 80/40 mm Hg, pulse 100 per 1 min, the temperature of body is normal. On a front abdominal wall a venous net is extended. What preparation is it necessary to begin therapy from?

a. Albumenb. Plasmac. Neogemodezd. *Bloode. Glucose

419. Patient A., 37 years old, entered to infectious hospital on the third days of disease in the severe condition. He complaints of the high fever with chills and sweat, general weakness, pain in right under a rib. Objectively: temperature of body 41 °С, icterus of skin, liver +2 cm, pain at palpation in abdomen, positive symptoms of Ortner and Mussy, a spleen is normal, tachycardiA. What is the previous diagnosis?

a. Malariab. *Cholangitisc. Viral hepatitisd. Sepsise. Leptospirosis

420. Patient W., 38 years old, entered to infectious hospital on the 5th days of disease in the severe condition. He complaints of the high fever with chills and sweat, general weakness, pain in sacrum. Objectively: temperature of body of 41 °С, tachycardia, positive symptom of Pasternacky, liver and spleen not is normal. Previous diagnosis?

a. Malariab. Cholangitisc. *Pyelonephritisd. Sepsise. Leptospirosis

421. Patient D., 39 years old, entered to infectious hospital on the second week days of disease in the severe condition. Local habitant, nowhere arrived. She is complaints for the high fever with chills and sweat, general weakness. Attacks of fever without correct periodicity. Objectively: the temperature of body – 41 °С, subicterus of sclera, tachycardia, Pasrernacky‘s symptom positive, liver + 2 cm, spleen + 2 cm. What is the previous diagnosis?

a. Malariab. Cholangitisc. Pyelonephritisd. *Sepsis

Page 262: intranet.tdmu.edu.uaintranet.tdmu.edu.ua/data/kafedra/internal/infect_desease... · Web viewBest method to treat diarrhoea in child is: intra venous fluide *ORS antibiotics bowel

e. Leptospirosis422. Patient D., 39 years old, entered to infectious hospital on the second week days of

disease in the severe condition. She is complaints for the high fever with chills and sweat, general weakness, pharyngalgia at swallowing. Objectively: temperature of body 37,7 °С, hyperemia of mucus of pharynx, tonsils are enlarged, loose, festering raid in lakuns, enlarged submandibula, neck, axilars lymphonodules, icterus of sclera and skin, bradycardia, liver + 2 cm, spleen + 1 cm. Urine is color of beer, an excrement is discolored. What is the previous diagnosis?

a. Malariab. Infectious mononucleosisc. Viral hepatitisd. *Sepsise. Leptospirosis

423. Patient of 52 years, fell ill sharply from a chill and head pain, fevers to 40°C. On a 3th day illnesses are nausea and vomiting, dark urine. On a 4th day a temperature went down to 37 °, but an icterus appeared and the amount of urine diminished to 600 ml. What disease is such development of symptoms characteristic for?

a. Hepatitis Ab. Hemorragic fever with a kidney syndromec. *Leptospirosisd. Sepsise. Acute glomerulonephritis

424. Sick, 24 years old, complaints on a pain in a throat, general weakness. Objectively: temperature 38,0 °C. pulse 96 shots per a min., dermahemia mucus of otopharynx bloodshot, tonsils are enlarged in sizes, loose. During palpation-enlarged posterior cervical and submandibular lymph nodes, a spleen comes forward from under the edge of costal arc on 1 cm. About what disease it is necessary to think?

a. Scarlet feverb. Acute respirator diseasec. Follicle tonsillitisd. Typhoid fevere. *Infectious mononucleosis

425. At a patient, 17 years: anginA. Temperature 38,2 °C, generilised lymphadenopathy (the first multiplied neck lymphatic nodeswhich are located along m. sternocleidomastoideus), small icterus, hepatospleenomegaly. Previous diagnosis?

a. Tuberculosis of lymphatic knotsb. Bacterial quinsyc. Diphtheriad. *Infectious mononucleosise. Megacaryoblastoma

426. A girl, 6 years old, admitted to hospital with complaints on general weakness, increase of temperature, pain in a throat. Objectively: observed oropharyngeal mucous membrane is bright red, on tonsils – white covering, can take off easily and enlarged all groups of lymph nodes 1-3 cm in a diameter, dense and elastic in consistancy, not very painfull, are not soldered between themselves. Liver is enlarged till 3 cm, spleen - till 1 cm. In bloods - leykocytosis, plazmocytes – 20 %. What will be the credible diagnosis?

Page 263: intranet.tdmu.edu.uaintranet.tdmu.edu.ua/data/kafedra/internal/infect_desease... · Web viewBest method to treat diarrhoea in child is: intra venous fluide *ORS antibiotics bowel

a. Acute lympholeycosisb. *Infectious mononucleosisc. Anginad. Diphtheriae. Adenoviral infection

427. In children with catarrhal phenomena revealed an increase of inframaxillary lymph nodes, tonsil hyperplasia with whitish soft touch on them in the form of points and islands, the presence of rare maculo-papular bulging out, increased liver and spleen. Identify the likely diagnosis?

a. *Infectious mononucleosisb. Acute leukemiac. Measlesd. Scarlet fevere. Chicken pox

428. At a child with the catarrhal phenomena it is found out the increase of submandibular and back neck lymphatic knots, hyperplasia of tonsils with the magnificent raid of gum-blush on them as points and aits, presence of the single roseol-papular pouring out, increase of liver. Name a credible diagnosis.

a. *Infectious mononucleosisb. Adenoviral infectionc. Scarlet feverd. Measlese. German measles

429. In a policlinic the patient C., 18 years old, appealed with complaints of a moderate pharyngalgia, head pain, general weakness fervescence, to 38,9 °C. It is ill already three days. Objectively: it is found out megascopic back neck, and inguinal lymphatic knots, hepatospleenomegaly, subicterus scleroticas and skin, changes in a throat, characteristic for tonsils. At the hemanalysis – lymphomonocytosis. Credible diagnosis?

a. *Infectious mononucleosisb. Follicle quinsyc. Adenoviral infectiond. Flue. Viral hepatitis

430. In a policlinic the patient C., 18 years old, appealed with complaints of a moderate pharyngalgia, head pain, general weakness fervescence, to 38,9 °C. It is ill already three days. Objectively: it is found out megascopic back neck, and inguinal lymphatic knots, hepatospleenomegaly, subicterus scleroticas and skin, changes in a throat, characteristic for tonsils. In general blood analysis – lymphomonocytosis. Principles of treatment?

a. Antibiotics, hepar protective preparations, antihistaminicb. Antiherpetic preparations, hepar protective preparations, antihistaminicc. *Antibiotics, preparations of interferon, hepar protective preparationsd. Antibiotics, Antiherpetic preparations , antihistaminice. Antibiotics, preparations of interferon, vitamins

431. At a patient, 17 years: tonsils, temperature 38,2 °C, generilized lymphadenopathy (the first multiplied neck lymphatic nodeswhich are located along m.

Page 264: intranet.tdmu.edu.uaintranet.tdmu.edu.ua/data/kafedra/internal/infect_desease... · Web viewBest method to treat diarrhoea in child is: intra venous fluide *ORS antibiotics bowel

sternocleidomastoideus), small icterus hepatospleenomegaly. What exciter causes this disease?

a. Herpesvirus I typeb. Herpesvirus ІІ typec. Herpesvirus ІІІ typed. *Herpesvirus ІV typee. Herpesvirus V type

432. Sick, in 24, disturbs a pharyngalgia, general weakness. Objectively: temperature – 38 °C. Pulse 96 shots per min, dermahemia mucus of otopharynx bloodshot, tonsills are megascopic in sizes, loose. Palpated megascopic back neck and submandibular lymphatic knots, a spleen comes forward from under the edge of costal arc of to 1 sm. What methods of diagnostics is it possible to confirm a diagnosis by?

a. Reaction of Paul-Bynnel, Right-Heddlson, Goffa-Bauerb. *Reaction of Paul-Bynnel, Lovrik-Volner, Goffa-Bauerc. Reaction of Paul-Bynnel, Right-Heddlson, Lovrik-Volnerd. Reaction of Paul-Bynnel, Goffa-Bauere. Reaction of Paul-Bynnel, Goffa-Bauer, Right-Heddlson, Lovrik-Volner

433. Patient P, 17 years old, has got ill gradually. General weakness, fatigue, painful throat, and nausea were marked. He was hospitalized on the 5th day of illness. Objectively: temperature is 38 °С. Objectively: skin and sclera are yellowish, neck, anticubital and subclavian lymph nodes are enlarged. Not numerous maculo-papular elements of rash on a trunk are found. The tongue is covered with white coat. The tonsils covered with white yellowish patches.On palpation of abdomen enlarged spleen and liver are found. In blood there is leucocytosis with neutophylic shift, atypical mononuclears-10 %, plasmatic cells- 10 %. What would be the diagnosis?

a. Scarlet feverb. *Infectious mononucleosisc. Typhoid feverd. Yersiniosis e. Lacunar tonsillitis

434. The boy 12 years old, with catarrhal phenomena seen an increase of all the lymph nodes, sclera, hyperplasia of tonsils white layers on them as islands, existing single maculo-papular rash, increased liver and spleen size. What additional research should appoint?

a. Reaction of passive heamaglutination(RPG) with influenza virusesb. Ultrasound of the abdomenc. Puncture of the lymph nodes, with following microscopyd. Inoculation of bloode. *Blood test for atypical mononuclears

435. Patient A., 17 years old, became ill gradually. A general weakness, fatigue, pain in throat, abdomen-ache, nausea, was marked. He was hospitalized on the 5th day of illness. Objectively: body temperature 38 °C, increased cervical, cubital and axillary lymph nodes. Subecteric skin and sclerA. Observed elements of spot-papul rash on a trunk. Raids on tonsils are loose, yellow, tongue coated by white covering, a abdomen is moderately swollen, hepatospleenomegaly. In general blood – leucocytosis, neutrophyle shift to left, atypical mononuclears – 10 %, plasmatic cells – 10 %. What is preliminary diagnosis?

Page 265: intranet.tdmu.edu.uaintranet.tdmu.edu.ua/data/kafedra/internal/infect_desease... · Web viewBest method to treat diarrhoea in child is: intra venous fluide *ORS antibiotics bowel

a. *Infectious mononucleosisb. Yersyniosisc. Lacunar tonsillitisd. Viral hepatitis Ae. Typho-paratyphoid disease

436. Patient 18 years, entered permanent establishment with complaints of headacke, general weakness increase of temperatures, to 37,5–38,2 °C during 6 days, pharyngalgiA. Objectively: all groups of lymphonoduses, 1-3 cm in a diameter, elastic, are megascopic, little sickly, not soldered between itself. A liver is megascopic on 3 cm, spleen – on 1 cm. In a blood is leycocytosis, placmatic mews – 15 %. What group of herpes does the exciter of this illness belong to?

a. Alphab. Betac. *Gammad. Deltae. Teta

437. Patient A., 18 years old, is complaining about headache, weakness, high temperature, soar throat. Objectively: general lymphadenopaty,wit lymph nodes 1-3 cm in diameter, dense, elastic not painful and not combined between themselves. Hepatospleenomegaly was noticed. In blood there is leucocytosis, monocytes – 15 %. What is the diagnosis?

a. *Infectious mononucleosisb. Adenoviral infectionc. Tonsillitisd. Diphtheriae. Acute leukemia

438. An 18 years old patient, entered the hospital with complaints of headache, general weakness, raising the temperature to 37,5-38,0 °C for 6 days, a sore throat. Objective examination: increasing of all lymph nodes, 1-3 cm in diameter, flexible, megaloblastic not soliter together. Increasing of the liver size till 3 cm, spleen – 1 cm. Common blood analysis: leukocytosis, placma cells – 15 %. What is preliminary diagnosis?

a. Diphtheriab. Adenovirul infectionc. Lacunar tonsillitisd. *Infectious mononucleosise. Acute lympholucosis

439. Patient D., 20 years old, fell ill and went to infectious polyclinic cabinet with complaints of moderate pain in the throat, headache, general weakness, increased body temperature of 38.9 °C. Fells bed during three days. At examination: increasing of inguinal lymph nodes, hepatosplenomegaly, subicteric sclera and skin, changes in the throat are typical for sore throats. In the blood – lymphogranulomatosis. The principles of treatment are?

a. *Antibiotics, interferons, hepatoprotectorsb. Antihypertensives hepatoprotectors, antihistaminesc. Antibiotics, hepatoprotectors, antihistaminesd. Antibiotics, antihypertensives, antihistamine

Page 266: intranet.tdmu.edu.uaintranet.tdmu.edu.ua/data/kafedra/internal/infect_desease... · Web viewBest method to treat diarrhoea in child is: intra venous fluide *ORS antibiotics bowel

e. Antibiotics, interferones, vitamins440. A patient D., 20 years old came to the infectious polyclinic with complaints of

moderate pain in the throat, headache, general weakness, increasing of body temperature to 38.9 °C. He fells sick during 3 days. Objective examination: increasing inguinal lymph nodes, hepatosplenomegaly, subicteric sclera and skin, changes in the throat are typical for sore throats were found. Blood analysis – lymphogranulomatosis. What is probable diagnosis?

a. *Infectious mononucleosisb. Tonsillitisc. Adenovirus infectiond. Influenzae. Viral hepatitis

441. The patients, 20 years old, with 5 days fever, pain in the throat during swallowing, pain in muscles. During review – there is thick, congested tonsils with purulent layers that are easily removed, enlarged neck, and inframaxillary lymph glands, liver + 1.5 cm possible palpation of edge of spleen. What additional research should be done?

a. Puncture of the lymph nodes, followed by microscopyb. Ultrasound of the abdominal cavityc. *Blood test for atypical mononucleard. Inoculation of bloode. Reaction of passive heamoglutination(RPH) influenza A viruses

442. Patient A., 35 years old, came to the clinic on the third day of disease with complaints of drowsiness, sweat, headache, fever up to 38.5 °C. Reviewing physician found sore throat, splenomegaly, enlarged lymph nodes. Liver not enlarged. Blood analysis – leukocytosis, lymphomonocytosis. What is treatment for the patient?

a. *Antibiotic, drugs interferon, hepatoprotectorsb. Antihistamine, antiherpetic preparations hepatoprotectorsc. Antibiotic, hepatoprotector, antihistamined. Antibiotics, antihistamine, antiherpatic drugse. Vitamins, antibiotics, drugs interferon

443. 35 years old patient A., came to the clinic on the third day of illness with complaints of drowsiness, sweat, headache, fever up to 38.5 °C. Reviewing physician found sore throat, splenomegaly, enlarged lymph nodes. Liver is not enlarged. Blood analysis – leukocytosis, lymphocytosis. What additional test should hold the patient for infectious mononucleosis?

a. Byurne and Wright-Hadulson’s reaction b. ELISA-test, bacteriological test for tularemiac. Bacteriological test for diphtheria and typhoid feverd. *ELISA-test, bacteriological test for diphtheriae. Paul-Bunnel’s reaction and lymph node puncture

444. A female M., 24 years old came to the doctor with long-lasting fever, night sweat. Over the past three months, weight reduced by 7 kg. Low intention for sex. Objective examination found an increasing of all lymph nodes, hepatolienal syndrome. In the blood: Leucocyte – 2,2?109. What disease should be suspected?

a. *HIV infection B. Lymphogranulomatosis

Page 267: intranet.tdmu.edu.uaintranet.tdmu.edu.ua/data/kafedra/internal/infect_desease... · Web viewBest method to treat diarrhoea in child is: intra venous fluide *ORS antibiotics bowel

C.Tuberculosis D.Infectious mononucleosisE.Sepsis

445. A patient, 13 years old, complaints on pain in a throat, body temperature rise till 38 °C. Objectively: hyperemia of skin, hyperpigmentation of oropharyngeal mucous , tonsils are enlarged in size, marked suppuration of follicles. During palpation enlarged posterior cervical and submandibular lymph nodes. Pulse 96 times/minute, spleen – under the edge of costal arc on 1 cm. About what disease it is necessary to think?

a. *Infectious mononucleosisb. Follicular tonsillitisc. Typhoid feverd. Measlese. Acute respiratory disease

446. During the examination of a patient, 17 years old, observe tonsilitis, body temperature 38,2 °C, generalized lymphadenopathy (cervical ltmph nodes, located along the m. sternocleidomastoideus), mild jaundice, hepatospleenomegaly. What is preliminary diagnosis?

a. Tuberculosis of lymph nodesb. Bacterial tonsillitisc. Diphtheriad. *Infectious mononucleosise. Lymphogranulematosis

447. Patient C., 16 years old, consulted to the infectious hospital on the 2nd day of diseases with complaints for a pain in throat at swallowing, increase of temperature. Objectively: body temperature 38,6 °C, hyperemia to the pharynx, edematous and loose tonsils, on both sides – purulent covering which can be taken off easily by spatulA. By palpation painful enlarged submandibular lymph nodes. Pulse 114 times/minute. From epidemic anamnesis it is clear that the same symptoms had her boyfriend. Changes in other organs didn’t observe. What preliminary diagnosis can be suspected?

a. *Lacunar tonsillitisb. Diphtheria of pharynx.c. Infectious mononucleosis.d. Influenzae. Scarlet fever

448. A 17 years old boy, got ill suddenly: the temperature rose up-to 40,3 °С, extremely sharp headache, makes him to yell. The patient is agitated and vomits frequently. Tremor of fingers of extremities is observed. The star like hemorrhagic rash of different form and sizes appeared, mainly on buttocks, thighs, shins, and trunk. Meningeal signs are positive. What is the most credible diagnosis?

a. Encephalitis b. Flu with a hemorrhagic syndrome c. * Meningococcal infection d. Measles e. Leptospirosis

449. A 19-years-old patient became ill 5 days ago: subfebrile temperature, acquired voice, dry cough weakness. After hospitalization: t - 37,1 °C, but general condition is

Page 268: intranet.tdmu.edu.uaintranet.tdmu.edu.ua/data/kafedra/internal/infect_desease... · Web viewBest method to treat diarrhoea in child is: intra venous fluide *ORS antibiotics bowel

serious, pallor, expressed weakness, aphonia, noisy stenotic breathing, inciters indrawings of intercostal intervals, pulse 110 per min, frequency of breathings 36 per min., cyanosys of lips and nailes. Tonsills is covered by the greyish dense raid. Define strategy of intensive therapy:

a. Tracheotomy, AVL, antitoxin b. Tracheotomy, AVL, toxoid, desintoxic therapy c. * Sedative, intubation, antitoxin, dexametazon, desintoxic therapy d. intubation, sedative, toxoid, penicillinum, desintoxic therapy e. sedative, oxygenotherapy, antitoxin i/v

450. A girl, 6 years old, admitted to hospital with complaints of general weakness, increase of temperature, pain in a throat. Objectively: observed oropharyngeal mucous membrane is bright red, on tonsils - white covering, can take off easily and enlarged all groups of lymph nodes 1-3 cm in a diameter, dense and elastic in consistancy, not very painfull, are not soldered between themselves. Liver is enlarged till 3 cm, spleen - till 1 cm. In bloods - leykocytosis, plazmocytes - 20 %. What will be the credible diagnosis?

a. Acute lympholeycosis b. *Infectious mononucleosis c. Angina d. Diphtheria e. Adenoviral infection

451. A new born child on 10th day of life became worse: Temperature 39.2 °C, frequent vomiting, generalized cramps, violations of consciousness, spastic paresis of left extremities. Month prior to his birth herpes virus was present in the mother, which she did not treat. What disease is most possible?

a. * Herpetic encephalitis b. Meningococcal meningitis c. Subarechnoid hemorrhage d. Cerebral abscess e. Violation of blood cerebral circulation

452. A patient 17 years old, became suddenly ill: .Temperature rose to 40,3 °C. Severe headache, motive excitation, frequent vomiting, tremor of fingers of extremities. Hemorrhagic spots of round form and different sizes, more frequently as stars, mainly on buttocks and trunk. Meningeal signs are positive. What is the most possible diagnosis?

a. Encephalitis b. Flu with a hemorrhagic syndrome c. * Meningococcal infection d. Measles e. Leptospirosis

453. A patient 21 years old 4th day grumbles about a general weakness, moderate pharyngalgia fervescence, to 39 °C. Did not treat oneself. Objectively: edema and cyanosys of mucous of otopharynx of left tonsill rough fibrinose raid which goes out on a palatal handle and tongue. Bloodstreaks appear at the attempt of removal by his spatulA. Submandibular lymphatic nodesare megascopic, more on the left side. What diagnosis is most credible?

a. Paratonsill abscess b. Lacunar quinsy

Page 269: intranet.tdmu.edu.uaintranet.tdmu.edu.ua/data/kafedra/internal/infect_desease... · Web viewBest method to treat diarrhoea in child is: intra venous fluide *ORS antibiotics bowel

c. Quinsy of Vensan-Symanovskiy d. * Diphtheria of otopharynx e. Infectious mononucleosis

454. A patient 60 years old for 2 days has disturbed severe pain in a right arm. On 3rd day appeared blisters, pouring out as a chain on the skin of shoulder, forearm and brush. Sensitiveness in the area of pouring out is mionectic. What disease can be diagnosed?

a. Dermatitis b. * Herpetic ganglionitis c. Neck-pectoral redicals d. Psoriasis e. Allergy

455. A patient A., 23 years old, hospitalized on the 2th day of disease with complaints of a weakness, pharyngalgias, that increase at swallowing, chill. State is moderate, temperature of body - 38.3 °C, a moderate hyperemia with cyanosys of mucous of the throat, tonsills is megascopic, covered by the pellicles raids which spread on a wall pharynx and tongue. Increasing of neck lymphonodes. Pulse - 88 per 1 min, BP 120/60 mm of Hg. A liver and spleen are not megascopic. What is previous diagnosis:

a. * Diphtheria of oropharynx b. Mycotic tonsillitis c. Simanovsky-Vensent's tonsillitis d. Infectious mononucleosis e. Acute leucosis

456. A patient became ill sharply. Severe pain of head, frequent vomits appeared in the morning. Temperature of body rose to 39,9 °C. Adopted febrifuge, however much the state got worse. Till the evening patient lost consciousness. Excited, sharply expressed meningeal signs. What is most reliable diagnosis?

a. Status typhosus in typhoid fever b. Viral meningoencephalitis c. Sepsis, infectious-toxic shock d. * Meningococcal infection, meningitis e. Status typhosus in epidemic typhus

457. A patient C., 25 years old, fell suddenly ill. Every morning severe headache, frequent vomiting, temperature of the body is 39,9 °C. Adopted fatigue, then state got much worse. In the evening lost of consciousness. Expressed muscles pains of back and head. Positive Кеrning's symptom. Leukocytes - 18,0*109/L. What is the most reliable diagnosis?

a. Flu b. Epidemic typhus, typhus state c. Viral menigoencephalitis d. Sepsis, infectious-toxic shock e. * Bacterial menigoencephalitis

458. A patient came with complaints of erosions of his penis. From anamnesis frequent appearance of similar rashes during a year is found out. Objectively: on a balanus are the grouped blisters and erosions of polycyclic outlines, with clear margin, soft during palpation. What is your diagnosis?

a. * Recurrent congenital herpes

Page 270: intranet.tdmu.edu.uaintranet.tdmu.edu.ua/data/kafedra/internal/infect_desease... · Web viewBest method to treat diarrhoea in child is: intra venous fluide *ORS antibiotics bowel

b. Pemphigus vulgaris c. Primary syphillis d. Pyodermia e. Scabies

459. A patient came with complaints of sickly erosions on his penis. From anamnesis frequent appearance of similar rashes is found out during a year. Objectively: on a balanus are the grouped blisters and erosions, soft on palpation. What is your diagnose?

a. * Recurrent herpes of ІІ type b. Vulvar pemphigus c. Primary syphyllis d. Shankoform pyoderma e. Recurrent herpes of ІІІ type

460. A patient D., 20 years old came to the infectious polyclinic with complaints of moderate pain in the throat, headache, general weakness, increasing of body temperature to 38.9 °C. He fells sick during 3 days. Objective examination: increasing inguinal lymph nodes, hepatosplenomegaly, subicteric sclera and skin, changes in the throat are typical for sore throats were found. Blood analysis - lymphogranulomatosis. What is probable diagnosis?

a. * Infectious mononucleosis b. Tonsillitis c. Adenovirus infection d. Influenza e. Viral hepatitis

461. A patient D., yesterday in the evening delivered in the hospital due to high temperature of body, increasing pharyngalgia, pain lockjaw of masseters, nausea, vomits. At nasopharhyngoscopy by an otolaryngologist the moderate expressed hyperemia, edema, cyanosys of mucous membrane of left palatal tonsil, oral cavity and nasopharynx were found out. The surface of tonsil is covered by the raid of dirty-grey color pellicle which goes beyond his borders (in nasopharynx). Increasing of left submandibular lymphatic node of dense consistency was also found. The edema of hypoderm of neck reaches to the collar-bone. What is the described disease?

a. * Hypertoxic diphtheria of nasopharynx b. Localized diphtheria of nasopharynx c. Subtoxic diphtheria of nasopharynx d. Widespread diphtheria of nasopharynx e. Lacunar tonsillitis

462. A patient has herpetic meningitis. What preparation of specific therapy for viral neuro infection should be given?

a. * Acyclovir b. Cefataxime c. Ceftriaxone d. Gentamycin e. Furazolidon

463. A patient is hospitalized in infectious permanent establishment: sharp beginning of disease, temperature 39,9 °C, moderate pharyngalgia, takes place edema, insignificant hyperemia with cyanosys of mucuses shells of otopharynx, on tonsills dense, brilliant,

Page 271: intranet.tdmu.edu.uaintranet.tdmu.edu.ua/data/kafedra/internal/infect_desease... · Web viewBest method to treat diarrhoea in child is: intra venous fluide *ORS antibiotics bowel

greyish color the raids placed as continuous pellicle, is heavily taken off, bare a surface, that bleeds. Submandibular lymphonoduss are moderatory megascopic. A patient must immediately do:

a. Strokes with tonsills, nose or other areas for the exposure of diphtherial stick b. IFA c. * Microscopy (painting for Neyser) d. Haemoculture e. RDHA with a diphtherial diagnosticum

464. A patient is sick with meningococcal meningitis. He take a massive dose of penicillin. 4 days temperature of body 36,6-36,8 °C. Meningeal signs are negative. When is it possible to stop the antibiotic therapy.

a. * At a cytosis in a CSF 100 and less, lymphocytes prevail b. After 10 days from the beginning antibiotic therapy c. After 7 days from the beginning antibiotic therapy d. At a cytosis 100 and less, neutrophil prevail e. From 6 days from the beginning antibiotic

465. A patient N., 45 years old, complaints of headache, general weakness increasing of temperature, to 37.4 °C. In 2 days pain appeared in the pectoral region of spine with an irradiation in a right between scapular regions. After some time skin in this region turned red as a strip from a spine to the subarmpit line, and in 2 days red knots which through the set time grew into blisters with transparent maintenance appeared in this place. What is your diagnosis?

a. * Herpes zoster b. Thoracal rediculopathy c. Neurology of intercostal nerves d. Neurology of superscapular nerve e. Herpetiform dermatitis

466. A patient sharply had a chill, headacke, vomits, temperature of body up to 38,5 °C. Till evening rigidity of muscles, Kernig symptom appeared. Herpetic blisters are marked on mucouse of lips and nose. Neurological symptoms is not found out expressed. What disease will you suspect?

a. Subarachnoiditis hemorrhage b. Herpetic encephalitis c. Abscess of brain d. Hemorrhage in a brain e. * Meningococcal meningitis

467. A patient was delivered in a hospital on the 2th day of illness with the symptoms of expressed toxicosis. Objectively: found out the severe condition, hemorrhagic rash of wrong form of different size on a skin, hemorrhages in the scleras of both eyes. Extremities are cold. AP 60/20 mm/hg, pulse - 120 per minute. Previous diagnosis?

a. * Meningococcemia b. Hemorrhagic fever with a kidney syndrome c. Leptospirosis d. Typhoid fever e. Pseudotuberculosis

Page 272: intranet.tdmu.edu.uaintranet.tdmu.edu.ua/data/kafedra/internal/infect_desease... · Web viewBest method to treat diarrhoea in child is: intra venous fluide *ORS antibiotics bowel

468. A patient with meningococcal meningitis gets penicillin during 7 days. The temperature of body is normal 4 days. Meningeal signs are negative. When is it possible to stop the antibiotic therapy.

a. In default of leucocytosis displacement in blood b. * At a cytosis in a neurolymph 100 and less, lymphocyte prevail c. At a cytosis in a neurolymph 100 and less, neutrophil prevail d. At a cytosis in a neurolymph 150, lymphocyte prevail e. At once immediately

469. A patient X., 25 years old, was examined by a otolaryngologist on the 4th day of illness. Temperature of body - 38.1 °C. Complaints of indisposition, moderate pain in the throat. Objectively: a mouth opens fully. Mucous of a soft palate, tongue were swollen, insignificant hyperemia with cyanosis. Increasing of tonsills, covered by the grey dense raid. The raid is taken off hard. The edema of neck is not present. Increasing of submandibular lymph nodes. What is most credible diagnosis?

a. Lacunar tonsillitis b. Infectious mononucleosis c. Simanovsky-Vensent's tonsillitis d. * Diphtheria of oropharynx e. Follicular tonsillitis

470. A patient, 13 years old, complaints of pain in a throat, body temperature rise till 38 °C. Objectively: hyperemia of skin, hyperpigmentation of oropharyngeal mucous , tonsils are enlarged in size, marked suppuration of follicles. During palpation enlarged posterior cervical and submandibular lymph nodes. Pulse 96 times/minute, spleen - under the edge of costal arc on 1 cm. About what disease it is necessary to think?

a. * Infectious mononucleosis b. Follicular tonsillitis c. Typhoid fever d. Measles e. Acute respiratory disease

471. A patient, 20 years old, during few days complaints of pharyngalgias. After supercooling the state became worse : sudden chills, increase of temperature to 40,6 °C, headache. On skin of low extremities, trunk and buttocks there are a lot of different sizes hemorragic spots, acrocyanosis. Consiouness is preserved. Meningeal signs are absent. What is the previous diagnosis?

a. * Meningococcal infection b. Flu c. Epidemic typhus d. Hemorrhagic fever e. Leptospirosis

472. A patient, 22 years old, became ill sharply. History showed fever up to 38.2 °C with headache, repeated vomiting, olfactory and tastes hallucinations. Quickly got complex of meningeal symptoms, pyramidal paresis. The general epileptic attack and comatose state also developed. Neurolymph is with mixed lymphocytosis, cytochrome, single red corpuscles. What is previous diagnosis?

a. Brain abscess b. Subdural empyema

Page 273: intranet.tdmu.edu.uaintranet.tdmu.edu.ua/data/kafedra/internal/infect_desease... · Web viewBest method to treat diarrhoea in child is: intra venous fluide *ORS antibiotics bowel

c. * Herpetic encephalitis d. Tumor of brain e. Encephalopathy

473. A pregnant woman, 27 years (pregnancy ІІ, 8-10 weeks.), temperature of body increased. At the inspection on a TORCH-infection antibodies are found to the herpes virus, ІІ types of class IGM. What we must recommend to pregnant women?

a. * To cut pregnancy b. To prolong the supervision c. Treatment with acyclovir d. Symptomatic treatment e. Appoint of alpha-fetoprotein

474. A sick 3 years old child came to the doctor with symptoms of the fever, languor, waiver of meal. A boy is capricious, temperature of body 37.9 °C. On the mucous of soft palate, cheeks are single vesicle, hypersalivation. What is the diagnosis?

a. * Herpetic stomatitis b. Candidosis of oral cavity c. Leucoplacia d. Follicular tonsillitis e. Lacunar tonsillitis

475. A sick person, 65 years old, complaints of rash, pain in a subscapular region. Objectively: on a skin surface of the subscapular region present the arcwise rose-red filling out hearths some infiltrative, with clear scopes. On-the-spot hearths grouped vesicles with transparent maintenance. What preparation he should take?

a. Suprastyn b. Prednisolone c. Biseptolum-480 d. Loratidin e. * Laferon

476. A woman 65 years old the disease had beginning sharply from increase of temperature to 39.0 °C, weakness, and pain in the left part of thorax that increased with breathing motions. On 3rd day of disease vesicular breathing appeared after motion of rib on the left on a hyperemic background. Together with sick a grandchild lives 4 years. What measures of prophylaxis of disease need to be adopted?

a. Vaccination b. Final disinfection c. Reception of specific immunoprotein d. * Isolation of patient e. Acyclovir administration

477. A woman C., during 3 days complaints of a general weakness, headache increasing of temperature to 39-40 °C, insignificant pharyngalgiA. Her husband is sick with tonsillitis. Objectively: a skin is pale, cyanosis of lips. Hyperemia of mucous of oropharynx, increasing of tonsils. On the spot of tonsils there are continuous dense accented mother-of-pearl raids, which are taken off with great effort with bleading. Increasing of submandibular lymphatic nodes. Edema of the neck. AP 105/65 mm Hg. What is most credible diagnosis?

a. Acute leucosis

Page 274: intranet.tdmu.edu.uaintranet.tdmu.edu.ua/data/kafedra/internal/infect_desease... · Web viewBest method to treat diarrhoea in child is: intra venous fluide *ORS antibiotics bowel

b. Lacunar tonsillitis c. Infectious mononucleosis d. * Diphtheria of oropharynx e. Adenoviral infection

478. After the disease which was accompanied by the fever and pharyngalgias, there were an odynophagia, dysarthria, weakness and violation of motions in hands and feet, hyporeflexia, violation of sensitiveness in extremities to the polyneurotic type. What disease does it follow to think about above all things?

a. Neuropathy of hypoglossus b. * Diphtherial polyneuropathy c. Neuropathy of glossopharyngeus nerve d. Trunk encephalitis e. Pseudobulbar syndrome

479. Among the students of PTU 2 cases of generalized form of meningococcal infection are registered. What preparation does it follow to enter to the contact persons with the purpose of urgent prophylaxis?

a. Normal immunoglobulin b. Leukocytic interferon c. * Meningococcal vaccine d. Meningococcal anatoxin e. Bacteriophage

480. An 18 years old patient, entered the hospital with complaints of headache, general weakness, raising the temperature to 37,5-38,0 °C for 6 days, a sore throat. Objective examination: increasing of all lymph nodes, 1-3 cm in diameter, flexible, megaloblastic not soliter together. Increasing of the liver size till 3 cm, spleen - 1 cm. Common blood analysis: leukocytosis, placma cells - 15 %. What is preliminary diagnosis?

a. Diphtheria b. Adenovirul infection c. Lacunar tonsillitis d. * Infectious mononucleosis e. Acute lympholucosis

481. A 17 years old boy, after tick bite got ill suddenly: the temperature rose up-to 40,3 °С, extremely sharp headache, makes him to yell. The patient is agitated and vomits frequently. Tremor of fingers of extremities is observed. Meningeal signs are positive. What is the most credible diagnosis?

a. *Viral meningoencephalitis b. Leptospirosis c. Measles d. Meningococcal infection e. Flu with a hemorrhagic syndrome

482. A citizen of Nigeria, 19 years old, came to study in Ukraine. After 10 days a strong chill appeared. There has been a headache, slice, nagging pain in the muscles. OBJECTIVE: body temperature 39,6 °C, sclera subicteric, herpes of lips. Symptoms of bronchitis were present. Spleen is significantly increased, dense, the liver is normal. What investigation is necessary to do?

a. USI of abdomen

Page 275: intranet.tdmu.edu.uaintranet.tdmu.edu.ua/data/kafedra/internal/infect_desease... · Web viewBest method to treat diarrhoea in child is: intra venous fluide *ORS antibiotics bowel

b. Lumbar puncture c. *Microscopy of tissue slades d. Microscopy of peripheral blood e. Observation of oculi

483. A patient B., 62 years old, became ill suddenly. Severe pain of head, frequent vomiting appeared in the morning. Temperature of body rose to 39,9 °C. Adopted febrifuge, however much the state got worse. Till the evening patient lost consciousness. Excited, expressed meningeal signs present. What is most probably diagnosis?

a. Status typhosus in epidemic typhus b. Status typhosus in typhoid fever c. *Viral meningoencephalitis d. Sepsis, infectious-toxic shock e. Meningococcal infection, meningitis

484. Patient B., 62 years old, became ill suddenly. Severe pain of head, frequent vomiting appeared in the morning. Temperature of body rose to 39,9 °C. Adopted febrifuge, however much the state got worse. Till the evening patient lost consciousness. Excited, expressed meningeal signs present. What is most probably diagnosis?

a. Meningococcal infection, meningitis b. Status typhosus in typhoid fever c. Sepsis, infectious-toxic shock d. Status typhosus in epidemic typhus e. *Viral meningoencephalitis

485. A patient became ill sharply. Severe pain of head, frequent vomits appeared in the morning. Temperature of body rose to 39,9 °C. Adopted febrifuge, however much the state got worse. Till the evening patient lost consciousness. Excited, sharply expressed meningeal signs. What is most reliable diagnosis?

a. *Viral meningoencephalitis b. Meningococcal infection, meningitis c. Status typhosus in epidemic typhus d. Sepsis, infectious-toxic shock e. Status typhosus in typhoid fever

486. A patient became sick 2 days ago suddenly. Temperature of body rose to 41°С, headache, vomit, positive Kernig’s and Brudzinsky’s symptoms appeared. In cerebrospinal liquid: cytosis 7 000 in 1 mkl, 90 % are lymphocytes, some increasing of protein, sugar, chlorides. What is clinical diagnosis?

a. Endocarditis b. *Meningeal form of viral encephalitis c. Infectious-toxic shock d. Meningococcemia e. Meningicm

487. A patient C., 25 years old, fell suddenly ill. Every morning severe headache, frequent vomiting, temperature of the body is 39,9 °C. Adopted fatigue, then state got much worse. In the evening lost of consciousness. Expressed muscles pains of back and head. Positive Кеrning’s symptom. Leukocytes – 8,0*109. What is the most reliable diagnosis?

a. Epidemic typhus, typhus state

Page 276: intranet.tdmu.edu.uaintranet.tdmu.edu.ua/data/kafedra/internal/infect_desease... · Web viewBest method to treat diarrhoea in child is: intra venous fluide *ORS antibiotics bowel

b. Flu c. *Viral menigoencephalitis d. Bacterial menigoencephalitis e. Sepsis, infectious-toxic shock

488. A patient is disturbed by attacks of fever. The icterus of sclera and skins is observed, hepatosplenomegaly is found on palpation. What is possible diagnosis accept?

a. Leishmaniosis b. Leptospirosis c. *Sepsis d. Malaria e. Viral hepatitis

489. A patient sharply had a chill, head pain, vomits, temperature of body rose up to 38,5 °C. Till evening rigidity of muscles, Kernig’s symptom appeared. Herpetic blisters are marked on mucous of lips and nose. Neurological symptoms is not found out expressed. What disease will you suspect?

a. Herpetic encephalitis b. Abscess of brain c. *Meningoencephalitis d. Hemorrhage in a brain e. Subarachnoid hemorrhage

490. A patient became sick 2 days ago suddenly. Temperature of body rose to 41°С, headache, vomit, positive Kernig’s and Brudzinsky’s symptoms appeared. In cerebrospinal liquid: cytosis 7 000 in 1 mkl, 90 % are lymphocytes, some increasing of protein, sugar, chlorides. What is clinical diagnosis?

a. Endocarditisb. Meningococcemiac. Infectious-toxic shockd. * Meningeal form of viral encephalitise. Meningicm

491. A patient C., 25 years old, fell suddenly ill. Every morning severe headache, frequent vomiting, temperature of the body is 39,9 °C. Adopted fatigue, then state got much worse. In the evening lost of consciousness. Expressed muscles pains of back and head. Positive Кеrning’s symptom. Leukocytes – 8,0*109. What is the most reliable diagnosis?

a. Epidemic typhus, typhus state b. Bacterial menigoencephalitis c. Flu d. Sepsis, infectious-toxic shock e. *Viral menigoencephalitis

492. At a patient with meningoencephalitis 44 years old, rose up general clonic-tonic cramps, abundant sweat, hyperemia of the face, bradycardia quickly changed on tachycardia, violation of breathing (Cheyne-Stokes type). What complication develop?

a. Swelling and edema of cerebrum b. *Wedging of cerebrum in the cervical channel c. Hypovolemic shock d. Waterhause-Friedrichsen syndrome

Page 277: intranet.tdmu.edu.uaintranet.tdmu.edu.ua/data/kafedra/internal/infect_desease... · Web viewBest method to treat diarrhoea in child is: intra venous fluide *ORS antibiotics bowel

e. Infectious toxic shock 493. In a patient, 27 y.o., after tick biting, headache which accompanied by nausea,

repeated vomits, hyperesthesia, photophobia appear. At a review: feet are bended to the trunk, expressed rigidity of muscles of the back of head, positive symptoms of Kerning’s, Brodzinsky’s. It is not found out paresises. Select a basic neurological syndrome.

a. Root syndrome b. *Meningeal syndrome c. Syndrome of liquor hypertension d. Vegetative crisis e. Syndrome of liquor hypotension

494. Patient B., 38 years, delivered by emergency doctor, anamnesis is unknown. However, according to neighbours, patient 2 days ago was healthy. At a review: consciousness is absent, motion of left overhead and lower extremities is absent, increasing of muscular tone is marked in the same extremities. Periodically are clonic-tonic cramps in right extremities. Expressed asymmetry of person. Sharply expressed rigidity of muscles of the back of head, positive Кеrnig symptom, positive overhead and lower Brudzinsky symptom. Acrocyanоsis is marked, temperature of body 38,8 °С. Breathning 36/min, unrhythmical. Vesicular breathing during auscultation. Pulse 72/min, weak filling and tension. Heart tones are muffled, AP 80/50 mm/hg. Tongue is dry, assessed by the white raid. Physiology sending in a norm. Your diagnosis?

a. Epidemic typhys b. Hemorrhagic fever c. Typhoid fever d. *Viral meningoencephalitis e. Leptospirosis

495. Patient D., 30 y.o., teacher, have been examined by the doctor of first-aid and delivered to a clinic on the 2nd day of illness. Fell ill acutely, temperature 40 °C, severy headacke, repeated vomits, photophobiA. The general condition is severe, consciousness is darkened. Sharply expressed rigidity of neck muscles, positive Kernig symptom. Pulse 100 min, weak; tones of heart deaf, AP 60/30 mmHg. What is your diagnosis?

a. Crimean hemorrhagic fever b. Disease of blood c. Flu, toxic form d. Hemorrhagic stroke e. *Viral meningoencephalitis

496. Patient D., 53 years, have been examined by doctor on the 2th day of illness. Fell ill suddenly, temperature 40 °С, acute headache, repeated vomiting, photophobiA. The common state is severe, consciousness is darkened, expressed rigidity of cervical muscles, positive Kernig’s symptom. Pulse 110/min, weak. BP 60/30 mm/hg. What is your diagnosis?

a. Hemorrhagic stroke b. Leptospirosis c. Flu, toxic form d. *Viral meningoencephalitis e. Marburg hemorrhagic fever

Page 278: intranet.tdmu.edu.uaintranet.tdmu.edu.ua/data/kafedra/internal/infect_desease... · Web viewBest method to treat diarrhoea in child is: intra venous fluide *ORS antibiotics bowel

497. Patient K., 21 y.o., the disease began from the increasing of temperature to 39,0 °C, headache, chill, repeated vomit. Objectively: temperature of 39,3 °C, pulse 76. Rigidity of muscles of the back of head. The tendon reflexes are expressed. Analysis of liquor: 84 % lymphocytes, 16 % neutrophiles, liquid is turbid, flows out under the promoted pressure. What disease is most reliable?

a. Second festering meningitis b. *Viral meningoencephalitis c. Meningococcal infection: serous meningitis d. Infectious mononucleosis e. Meningococcal infection: purulent meningitis

498. Patient L., 38 years old, complaints of attacks of fever with chill and common sickness. Epemiologically was found that he had recently returned from IndiA. Leishmania donovani was found under a microscopy. What is the vector of the disease:

a. Bee b. Pliers c. Flea d. *Mosquitoes e. Fly

499. Person 16 years entered permanent establishment on 6 day of illness. Illness began after tick bite. Temperature 37,7 °C. The state became worse: severe head pain, frequent vomits unconnected with a meal appeared. The common state is heavy. Pupils are extended, photoharmose is not present. Positive meningeal symptoms. General hyperesthesiA. Tones of heart are deaf, BP 100/50 mm/hg. What reliable diagnosis?

a. Meningococcal meningitis b. Influenza c. *Viral meningoencephalitis d. Typhoid fever e. Infectious mononucleosis

500. Patient, 15 y.o., back from Japane, became ill 3 days ago. Complaints of headache, temperature of 37,4 °C. After next days: headache increased, a temperature rise to 38,3 °C, repeated vomits. Objectively: a skin is pale, without rash. Mucous membrane of posterior wall of the throat is normal. Pulse 86, satisfactory filling. Meningeal signs are positive. What investigation is necessary to perform:

a. Intracutaneous allergic test b. Biological test on white mice c. *Cerebrospinal puncture d. Hemoculture e. Vidal test

501. Patient, 16 y.o., during 3 days stood with high temperature at home. 10 days ago been bited by tick. Next 2 days felt better. A temperature became normal. There was a chill on a 6th day, a temperature rose to 40 °C again, intensive, quickly increased headache, during 3 hours – repeated vomiting. The state is severe, consciousness is absent, psychomotor violations expressed meningeal signs. Meningitis is suspected. What is his etiology?

a. Tubercular b. Meningococcal

Page 279: intranet.tdmu.edu.uaintranet.tdmu.edu.ua/data/kafedra/internal/infect_desease... · Web viewBest method to treat diarrhoea in child is: intra venous fluide *ORS antibiotics bowel

c. Enteroviral d. Postinfluenzal e. *Viral meningoencephalitis

502. Patient, 23 y.o., suddenly experienced intensive headacke, nausea, pain in a neck and lumbar areA. Temperature of body 39,2 °C. Expressed meningeal symptoms. Light, tactile, pain hyperesthesiA. Blood: leucocytes – 17 000 /l, ESR-29 mm/hour. What method of investigation is most informative?

a. Echoencephalography b. Transcranial dopplerography c. Electroencephalography d. *Lumbar puncture e. Computer tomography

503. Sick 15 year old, became ill 3 days ago. Disturbed| headache, temperature of 37,4 °C. During next days: headackeache increased, a temperature had rose to 38,3 °C, repeated vomiting. Objectively: a skin is pale, without rash. Pulse 86/min satisfactory filling. Meningeal signs positive. It is needed to conduct for the selection of exciter.

a. Endermic test b. Bioassay on mice c. *Bacteriological examination of CSF d. Widal test e. Cultivation on bilious clear soup

504. Sick, 54 years old, hospitalized in an infectious department in a severy condition. Complaint about the expressed headache, mainly in frontal and temporal areas, superciliary arcs, origin of vomit on height of pain, pain at motions by eyeballs, in muscles and joints. Objectively: a patient is excited, temperature of body 39 °C, BP 100/60 mm Hg. Bradycardia changed by tachycardiA. There were tonic cramps. Doubtful meningeal signs. From anamnesis it is known that at home a woman is ill. What preparations must be entered?

a. Aspirin, analgin, d, dimedrol b. Lasix, analgin, ampicillin c. Mannitol, acetophene, prednisolone d. *Mannitol, lasix, prednisolone, euphylin, suprastin e. Veroshpiron, euphylin, prednisolone, dimedrol

505. Worker, 22 y.o., became ill sharply: t 39°C, severe pain of head, frequent vomits. Objectively: the condition is severe, psychomotor excitation, moans due to the great pain of head, expressed rigidity of neck muscles. Sharply positive symptoms of Brudzinsky’s and Kernig’s, general hyperesthesiA. What’s necessary to do for clarification of diagnosis?

a. Computer tomography of cerebrum b. *General blood analysis and liquor analysis c. X-ray of skull in two projections d. Analysis of the excrement and urine for a pathological flora e. General blood analysis and blood sterility

506. Patient A., 23 years suddenly experienced intensive head pain, nausea, pain in a neck, and lumbar. Examination is delivered in a clinic. Objectively: temperature 40,2 °C. Expressed meningeal symptoms. Light, haptic, pain hypesthesiA. Blood: leucocytes – 5

Page 280: intranet.tdmu.edu.uaintranet.tdmu.edu.ua/data/kafedra/internal/infect_desease... · Web viewBest method to treat diarrhoea in child is: intra venous fluide *ORS antibiotics bowel

thousand/l, ERS-19 mm/hr. A neurolymph under high pressure, turbid, lymphocytosis, some increasing of protein, chlorides. What is your diagnosis?

a. Meningococcal meningitis b. *Viral meningitis c. Tubercular meningitis d. Staphylococcus meningitis e. Pneumoccocal meningitis

507. In a patient, 35 y.o., came back from China one week ago, sharp beginning of disease have happened. Hyperthermia to 39,5 °C, headache, which is accompanied with vomits, myalgias, scleritis; rigidity of cervical muscles, Kernig’s and Brudzinsky’s symptoms are positive, neutrophilic leukocytosis in common blood analisis. What is most credible diagnosis?

a. Yersiniosis b. Leptospirosis c. *Viral meningoencephalitis d. Thrombocytopenic purpura e. Typhoid fever

508. In permanent establishment on the 2nd day of illness a patient is delivered with the symptoms of expressed toxicosis. Have been visited Japan. Objectively: the general condition is severe, hemorrhages in sclera of both eyes. Extremities are cold. AP is 60\20 mmHg, pulse – 120. What is previous diagnosis?

a. Typhoid fever b. Hemorrhagic fever with a renal syndrome c. Meningococcemia d. *Viral meningoencephalitis e. Crimean hemorrhagic fever

509. At a patient, 35 years, after the trip to Japan sharp beginning of disease happened. Body temperature – 39,5 °С, head ache, which is accompanied with vomits, myalgias. Hemorrhagic rash, that are localized on scleras. Rigidity of cervical muscles, Kernig and Brudzinsky symptoms are expressed. Spinal liquor without changes. What is most credible diagnosis?

a. Meningococcal infection b. Leptospirosis c. Epidemic typhus d. Typhoid fever e. * Viral meningoencephalitis

510. For the treatment of acidosis in viral meningoencephalitis is better to use.a. 10 % chloride solution b. 10-20 % glucose solution c. *4 % sodium bicarbonate solution d. Concentrated dry placma

511. Patient, 35 years was hospitalized with diagnosis localized diphtheria of pharynx. What is the first dose of antitoxic antidyphtherial serum?

a. *30 000 AUb. XX. 50 000 AUc. 80 000 AU

Page 281: intranet.tdmu.edu.uaintranet.tdmu.edu.ua/data/kafedra/internal/infect_desease... · Web viewBest method to treat diarrhoea in child is: intra venous fluide *ORS antibiotics bowel

d. 120 000 AUe. AAA. 150 000 AU

512. What laboratory examination is compulsory to do for the patient with signs of tonsillitis?

a. Isolation of hemolytic streptococcus from the throat mucosab. Biochemical blood analysisc. X-ray examinationd. *Cmear from nose and pharynxe. Immune-enzyme analysis

513. In preschool the registered case of diphtheriA. What from the measures adopted below does not conduct to the contact children?

a. *Introduction of antidiphterial wheyb. Non-permanent is stroke from a pharynx and nose for the bacteriologic

examinationc. Daily is supervision during 7 daysd. Determination of titres of specific antibodiese. At the repeated cases of disease is extraordinary revaccination diphtheria

514. At a patient the dense darkly-grey raid covers tonsils is considerably megascopic and spreads for their scopes. Mucus shell bloodshot accented cyanochroic, was considerably swollen. Immediate medical measure:

a. Ultraviolet irradiation of throatb. Punction of peritonsillar space c. Section of peritonsillar spaced. *Introduction of antidiphterial serume. Compress on a neck

515. A boy 6 years was in the close touch with a patient with diphtheriA. What treatment-prophylactic measures need to be conducted, if vaccine anamnesis is unknown?

a. Introduction of AWDT vaccineb. Antibacterial therapyc. Introduction of ADT-м to the toxoidd. *Antibacterial therapy and double introduction of ADT toxoide. Antibacterial therapy and introduction of immunoprotein

516. At a girl, 22 years old, severy form of diphtheria of otopharynx have happened. Specific treatment begun only on a 5th day from the beginning of disease. What complication of diphtheria is potentially dangerous?

a. Stenotic laryngotracheitisb. Pneumotoraxc. Meningoencephalitisd. Septicopyemiae. *Infectious-toxic shock

517. What remedy is most effective for treatment of dyphtheria should be used immediately?

a. Antibioticsb. Oxygenotherapyc. *Antitoxic antidyphtherial serumd. Antipyretic drugs

Page 282: intranet.tdmu.edu.uaintranet.tdmu.edu.ua/data/kafedra/internal/infect_desease... · Web viewBest method to treat diarrhoea in child is: intra venous fluide *ORS antibiotics bowel

e. Sulfanilamides 518. At sick L, 35 years old, a diagnosis is set is diphtheria of pharynx,

noncommunicative form. What first dose of antitoxic antidiphtheria whey is it necessary to appoint?

a. 120 thousand of AОb. 80 thousand of AОc. *30 thousand of AОd. 50 thousand of AО e. 150 thousand of AО

519. A child 2 years carries a diphtherial croup. There was the stop of breathing on 2nd days of whey therapy. What was the reason of asphyxia?

a. *Mechanical obturation by tapesb. Stenosis of larynxc. Anaphylaxis shockd. Whey illnesse. Paresis of respiratory musculature

520. When patient refered to the doctor with such complaints: prodromal respiratory illness, sore throat, fever, headache, stiff neck, vomiting, confusion, irritability. What is previous diagnosis? What main methods can confirm the diagnosis?

a. Epidemic typhys. ELISA-testb. Hemorrhagic fever. ELISA-testc. Leptospirosis. Lumbar puncture.d. Typhoid fever. ELISA-teste. *Meningococcal infection. Lumbar puncture

521. A patient is sick with meningococcal meningitis. He take a massive dose of penicillin. 4 days temperature of body 36,6-36,8 °C. Meningeal signs are negative. When is it possible to stop the antibiotic therapy.

a. *At a cytosis in a CSF 100 and less, lymphocytes prevailb. After 10 days from the beginning antibiotic therapyc. After 7 days from the beginning antibiotic therapyd. At a cytosis 100 and less, neutrophil prevaile. From 6 days from the beginning antibiotic

522. In patients with intensive head acke, nausea, pain in a neck and lumbar area, expressed meningeal symptoms; light, tactile, pain hyperesthesia what method of inspection is most informing?

a. *Lumbar punctureb. Computer tomographyc. Electroencephalographyd. Transcranial dopplerographye. Echoencephalography

523. Among the students of school 2 cases of generalized form of meningococcal infection are registered. What preparation does it follow to enter to the contact persons with the purpose of urgent prophylaxis?

a. Normal immune globulinb. Leukocytic interferonc. *Meningococcal vaccine

Page 283: intranet.tdmu.edu.uaintranet.tdmu.edu.ua/data/kafedra/internal/infect_desease... · Web viewBest method to treat diarrhoea in child is: intra venous fluide *ORS antibiotics bowel

d. Meningococcal anatoxine. Bacteriophage

524. In the kindergarden a child had a meningococcal infection. She was immediately hospitalized. After clinical convalescence in child sowed meningococcus. To which category can the carrier (child) of pathogen belongs?

a. Convalescent, chronicb. *Convalescent, acutec. Healthyd. Immune in vaccinatede. Immune in those, that had infection

525. A patient with meningococcal meningitis gets penicillin during 7 days. The temperature of body is normal 4 days. Meningeal signs are negative. When is it possible to stop the antibiotic therapy.

a. In default of leucocytosis displacement in bloodb. *At a cytosis in a neurolymph 100 and less, lymphocyte prevailc. At a cytosis in a neurolymph 100 and less, neutrophil prevaild. At a cytosis in a neurolymph 150, lymphocyte prevaile. At once immediately

526. At a patient the dense darkly-grey raid covers tonsills is considerably megascopic and spreads for their scopes. Mucus shell bloodshot accented cyanochroic, was considerably swollen. Immediate medical measure:

a. *Antidiphterial serumb. Punction of peritonsillar spacec. Section of peritonsillar spaced. Microscopic research of stroke from under tapee. Bacteriologic examination of stroke from under pallatum

527. In an epidemic focusl rationally to organize verification of the state of immunity. The Use of RIHA allows to find out persons unimmune to diphtheria during a few hours. What minimum protective titre?

a. 1:10b. 1:20c. *1:40d. 1:80e. 1:160

528. At a child 4 years on the third day of disease the widespread form of diphtheria of nasopharynx is diagnosed. Preparation of specific therapy:

a. Macrolids per osb. Penicillin i/mc. Cortycosteroidd. *Antidiphterial serum i/ve. Antitoxic therapy

529. At maintenance of call on a house a district pediatrician put to the sick 5 years old child diagnosis “Acute lacunar tonsillitis”. Specify, who must carry out the laboratory inspection of patient and in what terms.

a. Worker of SES upon receipt report

Page 284: intranet.tdmu.edu.uaintranet.tdmu.edu.ua/data/kafedra/internal/infect_desease... · Web viewBest method to treat diarrhoea in child is: intra venous fluide *ORS antibiotics bowel

b. A district medical sister is at once after determination of diagnosisc. Doctor pediatrician in 5 hoursd. *Doctor pediatrician at once after determination of diagnosise. District medical sister on a next day

530. At a patient with meningococcal meningitis 44 years old, rose up general clonic-tonic cramps, abundant sweat, hyperemia of person, bradycardia quickly changed on tachycardia, violation of breathing (Cheyne-Stokes type). What complication develop?

a. Infectious toxic shockb. *Wedging of cerebrum in the cervical channelc. Encephalitisd. Swelling and edema of cerebrume. Waterhause-Friedrichsen syndrome

531. At a patient with meningococcal meningitis, in 52, there was the syndrome of cerebral hypotension on the 6th day of illness. Objectively: expressed toxicosis and dehydration of organicm, meningial symptoms disappeared, muscles tones was low and areflexia developed. What complication arose up?

a. * Infectious toxic shockb. Wedging of cerebrum in the cervical channelc. Encephalitisd. D. Swelling and edema of cerebrume. Waterhause-Friedrichsen syndrome

532. Patient D., 30 years, have been examined by emergency doctor and delivered in a clinic on the 2th day of illness. Fell ill sharply, temperature 40 °С, sharp head pain, repeated vomits, photophobiA. The common state is severe, consciousness is darkened. Star-like shape rashes at the skin of abdomen, buttocks and lower extremities appeared. Sharply expressed rigidity of cervical muscles, positive Kernig symptom been present. Pulse 100/min, weak, tones of heart seak, BP 60/30 mm/hg. Your diagnosis?

a. * Meningococcal infectionb. Flu, toxic formc. Marburg hemorragic feverd. Leptospirosise. Hemorrhagic fever

533. Patient Н., 21 year, entered hospital on the 7th day of disease with complaints of the expressed weakness, pains in muscles and joints, head pain, nauseA. The disease began with irritation on the throat, chill, general weakness. After 2 days the state became worse, temperature increased to 39-40°С. Objectively: patient adynamic, consciousness is stored. Temperature of body 37,5 °С, the state as severe, pale skin, lips and nail phalanxes cyantic, hemorrhagic rashes on the skin of trunk and hands, the scopes of heart are extended to the left on 1,5 cm, tones deaf, pulse 130/min, weak filling, BP 80/40 mm/hg. Vesicular breathing. Tongue is assessed, moist, phenomena of pharyngitis present. Inlargement of neck lymphonodes. Meningeal symptoms are not present. OliguriA. What is previous diagnosis?

a. Hemorrhagic feverb. Leptospirosisc. Epidemic typhusd. *Meningococcal infection

Page 285: intranet.tdmu.edu.uaintranet.tdmu.edu.ua/data/kafedra/internal/infect_desease... · Web viewBest method to treat diarrhoea in child is: intra venous fluide *ORS antibiotics bowel

e. Typhoid fever534. Patient B., 38 years, delivered by emergency doctor, anamnesis is unknown.

However, according to neighbours, patient 2 days ago was healthy. At a review: consciousness is absent, motion of left overhead and lower extremities is absent, increasing of muscular tone is marked in the same extremities. Periodically are clonic-tonic cramps in right extremities. Expressed asymmetry of person. Sharply expressed rigidity of muscles of the back of head, positive Кеrnig symptom, positive overhead and lower Brudzinsky symptom. On the skin of overhead and lower extremities, buttocks, trunk abundant hemorrhagic rashes “star shape” character with necrosis in a center. Acrocyanоsis is marked, temperature of body 38,8 °С, hyperemia of pharynx Breathning 36/min, unrhythmical. Vesicular breathing during auscultation. Pulse 72/min, weak filling and tension. Heart tones are muffled, AP 80/50 mm/hg. Tongue is dry, assessed by the white raid. Physiology sending in a norm. Your diagnosis?

a. Epidemic typhysb. Hemorrhagic feverc. Leptospirosisd. Typhoid fevere. *Meningococcal infection

535. At a patient with meningococcal nasopharynsitis in 2 days from the beginning of illness the temperature of body rose to 41°С, head pain, vomit, positive Kernig and Brudzinsky symptoms appeared. In cerebrospinal liquid: cytosis 15 000 in 1 mkl, 90 % are neutrophils. What diagnosis?

a. Chronic meningococcemiab. *Meningitisc. Endocarditisd. Infectious-toxic shocke. Meningicm

536. A patient was delivered in permanent establishment on the 2th day of illness with the symptoms of expressed toxicosis. Objectively: found out the severe condition, hemorrhagic rash of wrong form of different size on a skin, hemorrhages in the scleras of both eyes. Extremities are cold. AP 60/20 mm/hg, pulse – 120 per minute. Previous diagnosis?

a. *Meningococcemiab. Hemorrhagic fever with a kidney syndromec. Leptospirosisd. Typhoid fevere. Pseudotuberculosis

537. At a patient with nasopharyngitis after 2 days from the beginning of illness the temperature of body rose up to 41°С, head pain, vomit, positive Kernig and Brudinsky symptoms appeared. Cerebrospinal liquid: cytosis 15000 in 1 mcl, 90 % are neutrophils. Diagnosis?

a. Chronic meningococcemiab. Endocarditisc. *Meningitisd. Meningicme. Meningococcal nasopharyngitis

Page 286: intranet.tdmu.edu.uaintranet.tdmu.edu.ua/data/kafedra/internal/infect_desease... · Web viewBest method to treat diarrhoea in child is: intra venous fluide *ORS antibiotics bowel

538. A patient sharply had a chill, head pain, vomits, temperature of body rose up to 38,5 °C. Till evening rigidity of muscles, Kernig symptom appeared. Herpetic blisters are marked on mucouse of lips and nose. Neurological symptoms is not found out expressed. What disease will you suspect?

a. Subarachnoiditis hemorrhageb. Herpetic encephalitisc. Abscess of braind. Hemorrhage in a braine. *Meningococcal meningitis

539. Patient A., 23 years suddenly experienced intensive head pain, nausea, pain in a neck, and lumbar. Examination is delivered in a clinic. Objectively: hemorrhagic rashes on a body. Temperature 40,2 °C. Expressed meningeal symptoms. Light, haptic, pain hypesthesiA. Blood: leucocytes – 25 thousand/l, ERS-29 mm/hr. A neurolymph is turbid, neutrophilic erythrocytosis, meningococci. What is your diagnosis?

a. Staphylococcus meningitisb. Tubercular meningitisc. *Meningococcal meningitisd. Viral meningitise. Pneumoccocal meningitis

540. Person 16 years entered permanent establishment on 6 day of illness. Illness began from a cold and cough. Temperature 37,7 °C. The state became worse: severe head pain, frequent vomits unconnected with a meal appeared. The common state is heavy. Pupils are extended, photoharmose is not present. Positive meningeal symptoms. General hyperesthesiA. Tones of heart are deaf, BP 100/50 mm/hg. What reliable diagnosis?

a. Infectious mononucleosisb. *Meningococcal meningitisc. Toxic food-born infectiond. Influenzae. Typhoid fever

541. At a patient, 35 years, acute beginning of disease happened. Symptoms of nasopharyngitis. Body temperature – 39,5 °С, head ache, which is accompanied with vomits, myalgias. Hemorrhagic rash, that are localized on scleras, brushes, feet, buttocks. Rigidity of cervical muscles, Kernig and Brudzinsky symptoms are expressed. Neutrophilic leucocytosis in common blood analisis.What is most credible diagnosis?

a. *Meningococcal infectionb. Typhoid feverc. Yersiniosisd. Leptospirosise. Epidemic typhus

542. A 17 years old boy, got ill suddenly: the temperature rose up-to 40,3 °С, extremely sharp headache, makes him to yell. The patient is agitated and vomits frequently. Tremor of fingers of extremities is observed. The star like hemorrhagic rash of different form and sizes appeared, mainly on buttocks, thighs, shins, and trunk. Meningeal signs are positive. What is the most credible diagnosis?

a. Encephalitis

Page 287: intranet.tdmu.edu.uaintranet.tdmu.edu.ua/data/kafedra/internal/infect_desease... · Web viewBest method to treat diarrhoea in child is: intra venous fluide *ORS antibiotics bowel

b. Flu with a hemorrhagic syndromec. *Meningococcal infection d. Measlese. Leptospirosis

543. Patient S., 20 years old refered to the doctor with such complaints: prodromal respiratory illness, sore throat, fever, headache, stiff neck, vomiting, confusion, irritability. Previous diagnosis. What main methods can confirm the diagnosis?

a. Epidemic typhys. ELISA-testb. Hemorrhagic fever. ELISA-testc. Leptospirosis. Lumbar puncture.d. Typhoid fever. ELISA-teste. *Meningococcal infection. Lumbar puncture

544. Sick, 54, hospitalized in an infectious department in a severy condition. Complaint about the expressed headache, mainly in frontal and temporal areas, superciliary arcs, origin of vomit on height of pain, pain at motions by eyeballs, in muscles and joints. Objectively: a patient is excited, temperature of body 39 °C, BP 100/60 mm Hg. Bradycardia changed by tachycardiA. There were tonic cramps. Doubtful meningeal signs. From anamnesis it is known that at home a woman is ill. What preparations must be entered?

a. *Mannitol, lasix, prednisolone, euphylin, suprastinb. Mannitol, acetophenec. Lasix, analgin, ampicillind. Veroshpiron, euphylin, dimedrole. Aspirin, analgin, dimedrol

545. A patient, 20 years old, during few days complaints of pharyngalgias. After supercooling the state became worse : sudden chills, increase of temperature to 40,6 °C, headache. On skin of lower extremities trunk and buttocks there are a lot of different sizes of hemorragic spots, acrocyanosis. Consiouness is preserved. Meningeal signs are absent. What is the previous diagnosis?

a. *Meningococcal infection b. Fluc. Epidemic typhusd. Hemorrhagic fevere. Leptospirosis

546. A patient C., 25 years old, fell suddenly ill. Every morning severe headache, frequent vomiting, temperature of the body is 39,9 °C. Adopted fatigue, then state got much worse. In the evening lost of consciousness. Expressed muscles pains of back and head. Positive Кеrning’s symptom. Leukocytes – 18,0*109. What is the most reliable diagnosis?

a. Flub. Epidemic typhus, typhus statec. Viral menigoencephalitisd. Sepsis, infectious-toxic shocke. *Bacterial menigoencephalitis

547. A patient 17 years old, became suddenly ill: .Temperature rose to 40,3 °C. Severe headache, motive excitation, frequent vomiting, tremor of fingers of extremities.

Page 288: intranet.tdmu.edu.uaintranet.tdmu.edu.ua/data/kafedra/internal/infect_desease... · Web viewBest method to treat diarrhoea in child is: intra venous fluide *ORS antibiotics bowel

Hemorrhagic spots of round form and different sizes, more frequently as stars, mainly on buttocks and trunk. Meningeal signs are positive. What is the most possible diagnosis?

a. Encephalitisb. Flu with a hemorrhagic syndromec. *Meningococcal infectiond. Measlese. Leptospirosis

548. In permanent establishment on the 2nd day of illness a patient is delivered with the symptoms of expressed toxicosis. Objectively: the general condition is severe, on skin hemorrhagic rash of different form and sizes, hemorrhages in sclera of both eyes. Extremities cold. AP is 60\20 mmHg., pulse – 120. Previous diagnosis?

a. *Meningococcemiab. Crimean hemorrhagic feverc. Hemorrhagic fever with a renal syndromed. Typhoid fevere. ECНО-eczanthema

549. Patient, 16 y.o., during 3 days there was a increase of temperature to 38 °C, першіння in a throat, indisposition. Next 2 days felt better. A temperature was normal. There was a chill on a 6th day, a temperature rose to 40 °C, intensive, quickly increased headache, through 3 hours – repeated vomits. The state is heavy, consciousness is absent, psychomotor violations expressed meningeal signs. Meningitis is suspected. What is his etiology?

a. Tubercularb. Enteroviralc. Postinfluenzald. * Meningococcale. Lymphocytic choreomeningitis

550. Patient D., 30 y.o., teacher, examined by the doctor of first-aid and delivered to a clinic on the 2nd day of illness. Fell ill sharply, temperature 40 °C, sharp head pain, repeated vomits photophobiA. The general condition is severe, consciousness is darkened. On the skin of abdomen, buttocks and lower extremities is зірчаста rash. Sharply expressed rigidity of neck muscles, positive Kernig symptom. Pulse 100 min, weak; tones of heart deaf, AP 60/30 mmHg. Your diagnosis?

a. Flu, toxic formb. Crimean hemorrhagic feverc. Disease of bloodd. Hemorrhagic strokee. *Meningococcal infection, meningococcemia

551. Worker, 22 y.o., became ill acutely: t 39°C, severy headacke, frequent vomits. Objectively: the condition is severe, psychomotor excitation, moans due to the great pain of head, expressed rigidity of neck muscles. Sharply positive symptoms of Brudzinsky’s and Kernig’s, general hyperesthesiA. What needs to be done for clarification of diagnosis?

a. * General blood analysis and liquor analysisb. General blood analysis and blood sterilityc. Computer tomography of cerebrumd. X-ray of skull in two projections

Page 289: intranet.tdmu.edu.uaintranet.tdmu.edu.ua/data/kafedra/internal/infect_desease... · Web viewBest method to treat diarrhoea in child is: intra venous fluide *ORS antibiotics bowel

e. Analysis of the excrement and urine for a pathological flora552. A patient with meningococcal meningitis gets penicillin during 7 days. Last 4 days

temperature of body is normal. Meningeal signs are absent. When is it possible to abolish an antibiotic?

a. *At cytosis in liquor 100 and less, lymphocytes prevailsb. At absence of leukocytosis and stab-nucleus shift in a bloodc. At cytosis in liquor 100 and more less, neutrophils prevailsd. At cytosis in liquor 150, lymphocytes prevailse. At once

553. Among the students of university 2 cases of generalized form of meningococcal infection are registered. What preparation does it follow to enter to the contact persons with the purpose of urgent prophylaxis?

a. Normal immunoglobulinb. Leukocytic interferonc. *Meningococcal vaccined. Meningococcal anatoxine. Bacteriophage

554. Patient K., 21 y.o.., the disease began from the increase of temperature to 39,0 °C, headache, chill, repeated vomit. Objectively: temperature of 39,3 °C, pulse 76. Rigidity of muscles of the back of head. The tendon reflexes are expressed. Analysis of liquor: cytosis 1237 in 1 ml, from them: 84 % neutrophiles, 16 % lymphocytes, reaction to Panda ++, albumen 0,66 g/l, liquid is turbid, flows out under the promoted pressure. Bacterioscopy found out in liquor gram(-) cooks morphologically similar with meningococcus. What disease is most reliable?

a. Serous meningitisb. Infectious mononucleosisc. *Meningococcal meningitisd. Tuberculosis meningitise. Purulent meningitis

555. In the kindergarden a child had a meningococcal infection. She was immediately hospitalized. After clinical convalescence in child sowed meningococcus. To which category can the carrier (child) of pathogen belongs?

a. Convalescent, chronicb. *Convalescent, acutec. Healthyd. Immune in vaccinatede. Immune in those, that had infection

556. In a patient, 27 y.o., on the 5th day of respirator disease, there was sharp head pain, that was accompanied by nausea, repeated vomits, hyperesthesia, photophobiA. At a review: lies with the neglected head, feet are bended to the trunk, expressed rigidity of muscles of the back of head, positive symptoms of Kerning’s, Brodzinsky’s. It is not found out paresises. Select a basic neurological syndrome.

a. Meningeal syndromeb. *Syndrome of liquor hypertensionc. Syndrome of liquor hypotensiond. Root syndrome

Page 290: intranet.tdmu.edu.uaintranet.tdmu.edu.ua/data/kafedra/internal/infect_desease... · Web viewBest method to treat diarrhoea in child is: intra venous fluide *ORS antibiotics bowel

e. Vegetative crisis557. Patient, 23 y.o., suddenly experienced intensive head pain, nausea, pain in a neck

and lumbar areA. He was delivered to the clinic. Objectively: on a body hemorrhagic rash. Temperature of body 39,2°C. Expressed meningeal symptoms. Light, tactile, pain hyperesthesiA. Blood: leucocytes-25x10 9/l, ESR-29 mm/hour. What method of inspection is most informing?

a. *Lumbar punctureb. Computer tomographyc. Electroencephalographyd. Transcranial dopplerographye. Echoencephalography

558. Patient, 15 y.o., became ill 3 days ago. Complaints of headache, running nose, pain in a throat, temperature of 37,4°C. Next days : headache increased, a temperature risen to 38,3°C, repeated vomits. Objectively: a skin is pale, without rash. Mucous membrane of posterior wall of throat is moderately hyperemic, swollen. Pulse 86, satisfactory filling. Meningeal signs are positive. What is needed to conduct the selection of pathogen:

a. * Bacteriological research of liquorb. Biological test on micec. Intracutaneous testd. Reaction of Vidala’se. Cmear of the blood on bilious bullion

559. In a patient, 35 y.o., sharp beginning of disease, preceding nasopharyngitis, t° 39,5°C, headache, which is accompanied with vomits, myalgias, hemorrhagic rash, that is localized on sclera, fists, feet, buttocks; rigidity of cervical muscles, symptoms of Kering’s and Brudzinsky’s are positive, expressed neutrophilic leukocytosis. What is most credible diagnosis?

a. Typhoid feverb. Yersiniosisc. Leptospirosisd. *Meningococcal infectione. Thrombocytopenic purpura

560. Sick C., 8 years, appealed to the infectious hospital on the second day of disease with complaints of a pharyngalgia at swallowing, increase of temperature. Objectively: temperature 38,6 °C, hyperemia of soft palate, tonsills, filling out, loose, on both there are festering stratifications which are taken off by a spatulA. Megascopic sickly submandibular lymphonoduses are palpated. Pulse – 114 per a min. Present roseol-papular pouring out on all body. Pastia‘s symptom is positive. It is known from epidemogical anamnesis, that its comrade had alike symptoms. It is not found out the change from the side of other organs. What previous diagnosis can be suspected?

a. Lacunar quinsyb. Diphtheria of pharynxc. *Scarlet feverd. Flue. Infectious mononucleosis

561. A 4 years old child complaints of: cough, temperature of body 38,1 °C. Skin without rashes. Conjunctiva hyperemic. Exanthema on skin. On mucous of cheeks there

Page 291: intranet.tdmu.edu.uaintranet.tdmu.edu.ua/data/kafedra/internal/infect_desease... · Web viewBest method to treat diarrhoea in child is: intra venous fluide *ORS antibiotics bowel

are points of hyperemia gum blushs. In lungs difficult breathing. What is the most possible diagnosis?

a. Scarlet feverb. Rubellac. *Measlesd. Enteroviral infectione. Flu

562. Sick C., 8 years, appealed to the infectious hospital on the second day of disease with complaints of a pharyngalgia at swallowing, increase of temperature. Objectively: temperature of body – 38,6 °C, sharp hyperemia of soft palate, tonsills, filling out, loose, on both there are festering stratifications which are taken off by a spatulA. Palpated megascopic, sickly submandibular lymphonoduses. Pulse – 114 per min. Present roseol-papular pouring out on all body. Pastia symptom is positive. It is known from epid anamnesis, that at its comrade were alike symptomes. It is not found out the change from the side of other organs. What previous diagnosis can be suspected?

a. Lacunar tonsillitisb. Diphtheria of pharynxc. Infectious mononucleosisd. Flue. *Scarlet fever

563. Patient A. , 27 years old, was admited on the 4th day of illness with the diagnosis of viral ARVI, and allergic dermatitis. The patient felt ill suddenly with increase of temperature up-to 38 °C, headache, sore throat, and intensive cough. On the 3d day rash appeared on the skin of neck and face. Patient was taking aspirin in order to decrease the temperature. Objectively: temperature of body 38,8 °C. The face of patient looks puffy. Signs of conjunctivitis, and renitis were observed. On the skin of neck face and chest there was intensive papular rash, without itch. Mucous membrane of oropharynx was brightly hyperemic. Submandibular and frontal neck lymph nodes were enlarged. The liver and spleen were not palpable. What is the diagnosis?

a. *Measlesb. Allergic dermatitisc. Infectious mononucleosisd. Rubellae. Scarlet fever

564. A child 10 years old presents with temperature 38 °C, renitis, conjunctivitis, moist cough. On the mucous membrane of cheeks, lips, gums there are greyish-white points, reminding a farinA. What is the diagnosis?

a. *Measlesb. Adenoviral infection c. URTI d. Enteroviral infection e. Infectious mononucleosis

565. In a newborn, the septic state was accompanied with the increase of temperature to 40 oC which developed on 5th day, with pneumonia expressed intoxication: pallor, vomiting, shortness of breath, disturbance, cramps. Appeared rash (blisters with hemorrhages) on skin, mucous membrane of mouth cavity, throat and conjuctivA. The

Page 292: intranet.tdmu.edu.uaintranet.tdmu.edu.ua/data/kafedra/internal/infect_desease... · Web viewBest method to treat diarrhoea in child is: intra venous fluide *ORS antibiotics bowel

child suffers with congenital herpes. Lungs: difficult breathing and vesicular wheezing. Mild enlargement of liver. What is the most reliable diagnosis?

a. *Herpetic infectionb. Cytomegaloviral infectionc. Chicken poxd. Rubellae. AIDS

566. A new born child on 10th day of life became worse: Temperature 39.2 °C, no frequent vomiting, generalized cramps, violations of consciousness, spastic paresis of left extremities. Month prior to his birth herpes virus was present in the mother, which she did not treat. What disease is most possible?

a. *Herpetic encephalitisb. Meningococcal meningitisc. Subarechnoid hemorrhaged. Cerebral abscesse. Violation of blood cerebral circulation

567. A patient has herpetic meningitis. What preparation of specific therapy for viral neuro infection should be given?

a. *Acyclovirb. Cefataximec. Ceftriaxoned. Gentamycine. Furazolidon

568. A patient, 22 years old, became ill sharply. History showed fever up to 38.2 °C with headache, repeated vomiting, olfactory and tastes hallucinations. Quickly got complex of meningeal symptoms, pyramidal paresis. The general epileptic attack and comatose state also developed. Neurolymph is with mixed lymphocytosis, cytochrome, single red corpuscles. What is previous diagnosis?

a. Brain abscessb. Subdural empyemac. *Herpetic encephalitisd. Tumor of braine. Encephalopathy

569. A boy 11 years old, complaints of sickness at mastication, increasing of temperature to 37.1 °C, enlargment of parotid salivary glands . At the age of 8 years carried a paraflu infection. Objectively: in the region of right parotid salivary gland tubular sickly at palpation, a skin above it is not changed. A pharynx is moderate hyperemic, tonsils are not coated. What is your previous diagnosis?

a. *Cytomegaloviral syaloadenitisb. Lymphadenitisc. Parotitisd. Infectious mononucleosise. Cholylithiasis

570. At junior nurse, who works in child’s infectious department, herpes simplex was found. What should manager of department must do?

a. *Create a quarantine in the department

Page 293: intranet.tdmu.edu.uaintranet.tdmu.edu.ua/data/kafedra/internal/infect_desease... · Web viewBest method to treat diarrhoea in child is: intra venous fluide *ORS antibiotics bowel

b. To appoint an immunoprotein to the childrenc. Discharge all children from the departmentd. To appoint immunomodulators with a prophylactic purposee. To inspect a junior nurse on a staphylococcus

571. A woman 65 years old the disease had beginning sharply from increase of temperature to 39.0 °C, weakness, and pain in the left part of thorax that increased with breathing motions. On 3rd day of disease vesicular breathing appeared after motion of rib on the left on a hyperemic background. Together with sick a grandchild lives 4 years. What measures of prophylaxis of disease need to be adopted?

a. Vaccinationb. Final disinfection c. Reception of specific immunoproteind. *Isolation of patiente. Acyclovir administration

572. Patient 60 years old complain of pains in right hand which last for 2 days. On the 3rd day appeared vesicular chain-like rash on the skin of arm, forearm and fist. Sensitivity in the rash area is decreased. Which disease can be diagnosed?

a. Dermatitisb. *Herpetic ganglionitisc. Neck-thorax radiculitisd. Psoriasise. Allergy

573. A patient 60 years old for 2 days has disturbed severe pain in a right arm. On 3rd day appeared blisters, pouring out as a chain on the skin of shoulder, forearm and brush. Sensitiveness in the area of pouring out is mionectic. What disease can be diagnosed?

a. Dermatitisb. *Herpetic ganglionitisc. Neck-pectoral redicalsd. Psoriasise. Allergy

574. The patient, 58 years old, was hospitalised in the infectious department with complaints of pain in the left half of thorax, fever. At a review: temperature of body 37.5 °C, in XI-XII intercostal area the grouped shallow blisters on a hyperemic-filling background are filled by transparent maintenance. Preparation of choice for treatment of this patient is:

a. Suprastinb. Prednisolonec. Biseptolum-480d. Semavine. *Laferon

575. Patient C., 60 years old during one year has 4th relapse of Herpes zoster. Recommended treatment and relapses prophylaxis?

a. *Valcyclovirb. Acyclovirc. Herpevird. Proteflazid

Page 294: intranet.tdmu.edu.uaintranet.tdmu.edu.ua/data/kafedra/internal/infect_desease... · Web viewBest method to treat diarrhoea in child is: intra venous fluide *ORS antibiotics bowel

e. Cycloferon576. A pregnant woman, 27 years (pregnancy ІІ, 8-10 weeks.), temperature of body

increased. At the inspection on a TORCH-infection antibodies are found to the herpes virus, ІІ types of class IGM. What we must recommend to pregnant women?

a. *To cut pregnancyb. To prolong the supervisionc. Treatment with acyclovird. Symptomatic treatmente. Appoint of alpha-fetoprotein

577. Patient, 25 years old, came to the physician on the 3rd day of disease with complaints of rash on the mouth, external nose and ears, which are itching and painful. Objectively: T-37,7 °C. On the not changed skin of mouth, nose and ear auricle there are vesicles with size of 1-2 mm grouped localization. About what disease you can think?

a. Erysipelas, bulbous formb. Anthraxc. *Herpetic infectiond. Eczemae. Streptodermic infection

578. Sick patient, 25 years old, was consulted by a doctor on the third day of illness with complaints of pouring out on lips, wings of nose and ears, pain and swelling in the places of these pouring out. Objectively: temperature of body – 37.7 °C, unchanged skin of overhead lip, wings of nose, auricles, there is vesicular scars 1-2 mm with the group location. What disease you may think about?

a. Erysipelas, bullous formb. Anthraxc. *Herpetic infectiond. Anthrax, skin forme. Chicken pox

579. Sick A., complaints of turning red rash and edema on a right cheek. During a review: temperature of body – 38.7 °C, submandibular lymph nodes enlarged and painful, border between turning red and healthy skin is clear, there are blisters with a dark liquid inwardly, palpation is painful. What is your previous diagnosis?

a. *Erysipelas, hemorrhagic formb. Anthrax, skin formc. Herpetic infectiond. Chicken poxe. Phlegmon of cheek

580. In patient R., 34 years old with general intoxication and increased body temperature up to 38 °C, appear pain in region of right auditory canal and next day distortion of face to the left side. Review: softening of frontal and nasal-mouth skin folds, right eyelid cleft is wider than left, mouth cavity turned left, right eyebrow doesn’t move upwards and cheek is strenght. Right xerophthalmia, xerotomia and disordered taste sensation on the surface of right anterior 2/3 half of tongue. Herpetic vesicles in right exterior auditory canal and auricle. What is the most possible diagnosis?

a. Rossolimo-Melkerson-Rozental’s syndrome right sideb. Postherpetic neuralgia of I-st and II-nd branch of right trigeminal nerve

Page 295: intranet.tdmu.edu.uaintranet.tdmu.edu.ua/data/kafedra/internal/infect_desease... · Web viewBest method to treat diarrhoea in child is: intra venous fluide *ORS antibiotics bowel

c. Postherpetic neuralgia of all 3 branches of right trigeminal nerved. Sluder’s syndrome right sidee. *Hant’s syndrome

581. A 32 years old patient appealed to a doctor on a background of fever and increase of temperature to 38.0 °C, pain in the right ear area, left side “deviation” appear during the next day. On examination: right side cmoothed out frontal and occipital skinning folds, right eyelids fissure is wider than left, a mouth is overtighten to the left, a right eyebrow does not rise upwards, and a cheek “hyperemia”, xerostomia and violation of taste receptors on front 2/3 right halves of tongue. Herpetic blisters in right external auditory canal and auricle. What is the most possible diagnosis?

a. *Hunt’s syndromeb. Post herpetic neuralgia of the I- and II- branches of right trifacial nervec. Post herpetic neuralgia of all branches of right trifacial nerved. Sludder’s syndromee. Rosolimo-Меlkerson-Rozantalia syndrome

582. A 40 years old scientist, became ill sharply with chill, temperature of body – 39.8 °C, severe headache, vomiting, pain in muscles. Did not went to the doctor, the state had become worse, hyperemia of sclera, appeared, on lips herpes with hemorrhagic maintenance, the nose-bleeding, skin and sclera, became icteric, urine color is of strong tea, diuresis 200 ml, an anacholia was not present. What is the most reliable source of infection?

a. *Ratsb. Catsc. Infected peopled. Bacillocarriere. Mosquito

583. A patient came with complaints of erosions of his penis. From anamnesis frequent appearance of similar rashes during a year is found out. Objectively: on a balanus are the grouped blisters and erosions of polycyclic outlines, with clear margin, soft during palpation. What is your diagnosis?

a. *Recurrent congenital herpesb. Pemphigus vulgarisc. Primary syphillisd. Pyodermiae. Scabies

584. Sick 65 years old patient, complaints of pain in a subscapular region. Objectively: on a skin surface of subscapular region the placed arcwise rose-red filling out hearths some infiltrative, with clear scopes was present. On-the-spot hearths grouped vesicles with transparent maintenance. What is the diagnosis?

a. Impetigob. Herpes simplexc. Erysipelasd. Allergodermiae. *Herpes zoster

585. A sick person, 65 years old, complaints of rash, pain in a subscapular region. Objectively: on a skin surface of the subscapular region present the arcwise rose-red

Page 296: intranet.tdmu.edu.uaintranet.tdmu.edu.ua/data/kafedra/internal/infect_desease... · Web viewBest method to treat diarrhoea in child is: intra venous fluide *ORS antibiotics bowel

filling out hearths some infiltrative, with clear scopes. On-the-spot hearths grouped vesicles with transparent maintenance. What preparation he should take?

a. Suprastynb. Prednisolonec. Biseptolum-480d. Loratidine. *Laferon

586. A patient N., 45 years old, complaints of headache, general weakness increasing of temperature, to 37.4 °C. In 2 days pain appeared in the pectoral region of spine with an irradiation in a right between scapular regions. After some time skin in this region turned red as a strip from a spine to the subarmpit line, and in 2 days red knots which through the set time grew into blisters with transparent maintenance appeared in this place. What is your diagnosis?

a. *Herpes zosterb. Thoracal rediculopathyc. Neurology of intercostal nervesd. Neurology of superscapular nervee. Herpetiform dermatitis

587. A 37 years old patient, 2 days ago a spot on a hand appeared, which for days grew into pustule with a black bottom, painless at touch, with the crown of daughters vesicles on periphery. There are painless edema on a hand and shoulder. Temperature rise to 39.0 °C, dizziness appeared. Pulse – 100 beats per min, AP – 95/60 mm Hg. BR – 30 per a minute. What is the most possible diagnosis?

a. *Anthraxb. Plaguec. Tularemiad. Brucellosise. Herpes

588. A 70 years old sick person, after supercooling severy pain in the left half of head in the area of forehead and left eye appeared. 3 days ago the temperature of body increased to 37,6 °C, the blister of pouring out at the head and left overhead eyelid appeared. What disease can be diagnosed?

a. *Herpetic ganglionitisb. Encephalitisc. Allergyd. Dermatitise. Trifacial neuritis

589. A sick 3 years old child came to the doctor with symptoms of the fever, languor, waiver of meal. A boy is capricious, temperature of body 37.9 °C. On the mucus shell of soft palate, cheeks are single vesicle hypersalivation. What is the diagnosis?

a. *Herpetic stomatitisb. Candidosis of oral cavityc. Leucoplaciad. Follicular tonsillitise. Lacunar tonsillitis

Page 297: intranet.tdmu.edu.uaintranet.tdmu.edu.ua/data/kafedra/internal/infect_desease... · Web viewBest method to treat diarrhoea in child is: intra venous fluide *ORS antibiotics bowel

590. Sick, 49 years old, came to the doctor with complaints of pain. On the 3rd day he became ill, first marked heartburn and pain in thorax, yesterday is pouring out. At examination: temperature of body 37,8 °C, after motion of the V-VI intercostals intervals on a hyperemic skin group of the blisters filled by transparent maintenance. What is the diagnosis?

a. *Herpes zosterb. Chicken poxc. Erysipelas, erythematous-bulous formd. Allergic dermatitise. Myositis

591. Patient K., 56 years old, during last 5 days has Herpes zoster with localization of the process on the right cheek and paraauricular region. Treating with herpevir. Today pain in the right eye appeared. During examination present of edema of eyelid and hyperemia of conjunctivA. What is the action of a physician?

a. Prescribing eye drops (оphtan, аlbucid and other)b. Prolonged treatment with herpevirc. *Immediately consultation of ophthalmologistd. Intensify treatment with antiviral drugse. Prescribe warm compress

592. To the infectious diseases department was admitted patient M. 58 years old, with complaints of pain in left part of thorax, fever. During a review: body temperature 37,5 °C, on the level of XI–XII intercostals spaces on hyperemic-edemous shadow grouped cmall vesicles with transparent content. Preparation of choice for treatment of this patient?

a. Suprastynb. Prednisolonc. Biseptol-480d. Cymevene. *Laferon

593. A patient appealed to the doctor with complaints of difficulties in opening of the mouth. Two weeks ago fell down and head was hurt, did not have medication. At a review mouth opens on 1.5 cm, moderate expressed pain of muscles at the back of head. The paralysis of muscles of person, eyeballs, are more prominent. In a temporal area dry bloody crusts in the place of traumA. What are your diagnosis?

a. Neuritis of facial nerveb. Throat abscessc. *Facial paralytic stupor of Rоsеd. Bulbar encephalitise. Herpetic ganglionitis of knot of trifacial

594. A patient came with complaints of sickly erosions on his penis. From anamnesis frequent appearance of similar rashes is found out during a year. Objectively: on a balanus are the grouped blisters and erosions, soft on palpation. What is your diagnose?

a. *Recurrent herpes of ІІ typeb. Vulvar pemphigusc. Primary syphyllisd. Shankoform pyodermae. Recurrent herpes of ІІІ type

Page 298: intranet.tdmu.edu.uaintranet.tdmu.edu.ua/data/kafedra/internal/infect_desease... · Web viewBest method to treat diarrhoea in child is: intra venous fluide *ORS antibiotics bowel

595. Patient A., complaints of redness of the skin and edema on the right cheek. During a review: body temperature 38,7 °C, enlarged and painful right submandibular lymphatic nodes, the border between red and normal skin is sharp, present vesicles with dark content, palpation is painful. Your primary diagnosis?

a. *Erysipelas, hemorrhagic formb. Anthrax, skin formc. Herpetic infectiond. Varicella (chickenpox)e. Phlegmone of the cheek

596. At a patient G., 41 years old, was a high temperature during 8 days, severe headache, constipation. Objectively: temperature of body 39,5 °C, pale, languid. Pulse 82 per a min, a tongue is dry, assessed by the brown coat. An abdomen is moderato exaggerated, painful in a right ileac areA. A liver + 2 cm. What is the most credible diagnosis?

a. *Typhoid feverb. Epidemic typhusc. Flud. Appendicitise. Yersiniosis

597. Patient L., 20 years old, had appendectomy on the 5th day of disease. During operation was found an appendix with the signs of catarrhal inflammation and hyperplasia of lymphatic nodes of mesentery. After a day his state became severe: temperature – 40 °C, with hallucinations, hepatosplenomegaly. Single roseols elements appeared on the skin of abdominal on a 10th day. The tongue is great, covered with grey coating, with the imprints of teeth, wound is in normal. In the analysis of blood is leucopenia, relative lympho- and monocytosis. In anamnesis was contact with a patient with typhoid fever. What is the most credible diagnosis?

a. *Typhoid fever, atypical form: appendicotyphoidb. Epidemic typhusc. Yersiniosisd. Pseudotuberculosise. Flu

598. Patient L., 20 years old, had appendectomy on the 5th day of disease. During operation was found an appendix with the signs of catarrhal inflammation and hyperplasia of lymphatic nodes of mesentery. After a day his state became severe: temperature – 40 °C, with hallucinations, hepatosplenomegaly. Single roseols elements appeared on the skin of abdominal on a 10th day. The tongue is great, covered with grey coating, with the imprints of teeth, wound is in normal. In the analysis of blood is leucopenia, relative lympho- and monocytosis. In anamnesis was contact with a patient with typhoid fever. By which method is it possible to diagnostic this disease?

a. Urinoculturab. Coproculturec. Biliculturad. *Hemoculturae. Reaction of Vidal

Page 299: intranet.tdmu.edu.uaintranet.tdmu.edu.ua/data/kafedra/internal/infect_desease... · Web viewBest method to treat diarrhoea in child is: intra venous fluide *ORS antibiotics bowel

599. Pain in abdomen appeared at a patient with typhoid fever on the 19th day of disease, and was during 4 hours. Pulse – 100 per a min, rhythmic. A tongue is dry, assessed by wait coat. The abdomen is tense, does not take part in the act of breathing. Stool and urine was absent. About what complication is it necessary to think?

a. Appendicitisb. Infectiously-toxic shockc. Bleedingd. *Perforatione. Urolithiasis

600. At the patient B., 25 years old, was diagnosed typhoid fever. On the 17th day of disease the temperature of body critically went down to the norm, a pallor color of skin increased. Consciousness is stored. Pulse 120 per a min, rhythmic. On the top of heart is systolic noise. Constipation. About what complication is it necessary to think?

a. *Bleedingb. Perforationc. Infectious-toxic shockd. Infectious-allergic myocarditise. Pneumonia

601. A sick person, 23 years old, appealed to the hospital on 6th day with gradual development of illness and complaints of severe headache, pain at the back of his head, sleep disturbance and fever. Objectively: Т-39,7 °C, Рs-84/min. His face is pale, tongue is dry, near the root covered by a grey covering. The abdomen is flatulent. A percutory sound is heard in the right iliac areA. The liver and spleen are enlarged. Which day does rash appear on the skin in this illness?

a. On 12 dayb. On 4 daysc. On 5 dayd. On 6 daye. *On 8 day

602. In a 42 y.o. annual explorers the temperature of body rose to 39 °C. At reception the patient is pale, tongue is edematous and covered by a dirty-brown raid, with clean edges and tag, and on the sides imprints of teeth, protrusion of the tongue is impaired and he trembles finely, he has an ammonia breath. On anterior surface of the abdomen are found monomorphic single roseollas. In his mouth cavity are hyperplasic lymphatic follicles of soft palate especially in front, appeared symmetric flat superficial oval form of ulcer, diameter up to 5 mm. Such ulcers are on tonsils. In case of such disease the convalescents are discharge after:

a. Triple bacteriological research of blood, excrement, urine, bileb. *Triple bacteriological research of excrement, urines and once of the bilec. Triple bacteriological research of excrement, urines and once of the bloodd. Triple bacteriological research of excrement, urine, bilee. Triple bacteriological research of blood, urine, bile and once excrement

603. To the reception of the infectious hospital a patient came with complaints of high fever – 38-40 °C In 3 weeks, headache, weakness and insomniA. She didn’t get a doctors consultation She took antipyretic drugs, seduxen. Objectively: Т 35,7 °C, Рs – 140/min., BP-80/50 mm Hg. Her general condition is severe. Skin and mucous

Page 300: intranet.tdmu.edu.uaintranet.tdmu.edu.ua/data/kafedra/internal/infect_desease... · Web viewBest method to treat diarrhoea in child is: intra venous fluide *ORS antibiotics bowel

membranes are pale. The tongue is thickened with the imprints of teeth, with a dirty-brown covering, apex and edges of the tongue are clean. her abdomen is flatulent. her liver and spleen are moderately enlarged. Her stool on admission was mixed with fresh blood. Why was there a decrease of temperature and an increase in pulse?

a. Poisoning by the drugsb. Infectious-toxic shockc. Hemorrhoid bleedingd. *Intestinal bleedinge. Endometriosis of colon

604. Patient Н., 28 years old, came to the clinic on the ninth day of illness with complaints of increased Т to 39,0 °C, headache, general weakness, delay of voiding, sleep disturbance. At inspection: the skin of abdomen had single roseollas, the tongue is covered with a brown covering, Ps. 78 beats per min. and rhythmic, the liver is enlarged to 2сm. What changes will be seen in the cardio-vascular system?

a. Bradycardia, dicrotic pulse, muffling of cardiac tones, hypotensionb. Tachycardia, dicrotic pulse, muffling of cardiac tones, hypotensionc. Tachcardia, dicrotic pulse, muffling of cardiac tones, hypertensiond. *Relative bradycardia, dicrotic pulse, muffling of cardiac tones, hypotensione. Dicrotic pulse, muffling of cardiac tones, hypotension

605. A 30 y/o patient is seen on the 9th day of illness. The symptoms of illness has been building –up gradually with increase in temperature and intoxication. Roseolar rash has appeared on his abdomen. His skin is pale, temperature is 40 °C, pulse 80 /1 min., BP 100/65 mmHg. His tongue is covered with sediments and his abdomen is swollen. Spleen and liver are palpable. What symptom will be positive for this patient?

a. Symptom of Botkinb. *Symptom of Padalkac. Symptom of Kyl'dyshevskyd. Symptom of Ortnere. Symptom of Pasternacky

606. A sick woman, 32 years, complaints of diarrhea, headache, severe weakness, insomnia and a dull pain in her right iliac areA. It is the 8th day of her illness. At a review: Т-39,8 °C, Рs – 86/min., AP – 90/60 mm Hg. Pulse is dictoric. Skin is pale. Single roseollas are on the abdomen. Tongue is dry, assessed by the coverings, with the imprints of teeth on a lateral surface. Soft, dulling of percutory sound is observed in the illeocaecal area of her abdomen. Hepatosplenomegaly, positive Blumberg’s symptom, neutrophilic leukocytosis. What changes will be at roentgenologic examination?

a. No changesb. *Presence of air under a diaphragmc. Presence of the exaggerated loops of intestined. Enlarged liver and spleene. Signs of impassability of intestine

607. Patient with the diagnosis “typhoid fever” is hospitalized in an infectious hospital. He lives in an isolated apartment with a woman and two children. What preparations is the mean defense against typhoid fever for people in contact with the patient?

a. Vaccine

Page 301: intranet.tdmu.edu.uaintranet.tdmu.edu.ua/data/kafedra/internal/infect_desease... · Web viewBest method to treat diarrhoea in child is: intra venous fluide *ORS antibiotics bowel

b. Antibioticc. Immunoglobulind. Antitoxine. *Bacteriophage

608. A district doctor suspected typhoid fever in a patient M., 15 y.o. To make the patient safe as the sources of infection, which of the following measures should be taken? except:

a. *Introduction to the patient of antityphoid monovaccineb. Hospitalization in the infectious department during 3-6 hoursc. Introduction etiotropic antibacterial treatmentd. Conducting of controls bacteriological researches of excrement and urine (triply)

and bile before the dischargee. Clinical supervision after the discharge during 3 months

609. Patient 24 y.o., was hospitalized in the infectious department on the 10th day of disease with complaints of general weakness, headache, poor appetite and cough. Objectively: body temperature is 39,5 °C, pallor of skin. Adynamicm. Single roseolas are present on the skin of anterior wall of the abdomen, thorax. Liver is palpable + 1 cm below the rib angle on the midclavicular line, spleen is not significantly palpated. What is the most probable diagnosis?

a. *Typhoid feverb. Fluc. Typhoid rashd. Brucellosise. Pneumonia

610. A patient A., 43 years old, is ill for 2 weeks. The disease started as an increase in temperature to 37,2 °C, headache, decline of appetite and weakness. Then the temperature got to 39-40 °C. Objectively: his condition is severe, he responds slowly to questions. AdynamiA. Pulse 80 in a min. BP is 100/60 mmHg. An abdomen is painless, flatulence, hepatosplenomegaly. Stool is of green color. How long is it necessary to look after people who were in contact with this patient?

a. 35 daysb. 1 monthc. *21 daysd. 1 weeke. 12 days

611. A plumber, 45 y.o., is hospitalized on the 7th day of fever. Objectively: t – 39,8 °C, somnolence (at night insomnia), adynamia, pallor of skin, Ps 78/min, BP 105/70 mmHg. Tongue is thickly assessed by the grey raid with the imprints of teeth. Palpation: the abdomen is distended, the liver is enlarged to 2 cm, and spleen to 1 cm below costal arc, in the right iliac area – the crepitating grumbling and hyperesthesia of skin. Defecation is absent for 2 days. What additional research should be performed for clarification of diagnosis?

a. Spinal punctureb. *Bacteriological research of bloodc. Analysis of myelogramd. Colonoscopy

Page 302: intranet.tdmu.edu.uaintranet.tdmu.edu.ua/data/kafedra/internal/infect_desease... · Web viewBest method to treat diarrhoea in child is: intra venous fluide *ORS antibiotics bowel

e. Research of blood for the markers of viral hepatitis612. Patient S., 23 years old, became ill at the end of the summer, His temperature

rose to 37,2 °C, insignificant headache and weakness appeared. For 7 days of the illness he was treated ambulatorily as ARI (acute respiratory infection). His state became worse, he was hospitalized in the permanent establishment. Temperature – 40 °C, pale, weakness. Pulse 96 per a min, BP 110/70 mm Hg. Tongue with the imprints of teeth, abdomen soft, distended, hepatosplenomegaly. Stool is not present. There is hyperemia and hyperplasia of palatine tonsils, on the surface of right palatine tonsils are ulcers. What disease is comes to your mind?

a. Angina of Simonovskyb. Herpes anginac. Ulcerative-necrotic anginad. Infectious mononucleosise. *Angina of Diuge

613. Patient L., 43 years old, entered to the clinic of infectious diseases with a diagnosis „fever of idiopathic etiology”. He has been ill for 15 days: temperature of body – 39,3 °C. Skin is pale. Pulse – 86 per a min, satisfactory properties. BP is 110/70 mm Hg. Tongue is dry, assessed by the coverings, with the imprints of teeth on a lateral surface. On a abdomen are 10-12 roseolas to 5 mm in a diameter. A liver and spleen is enlarged. What is the most credible diagnosis?

a. Yersiniosisb. *Typhoid feverc. Brucellosisd. Epidemic typhuse. Sepsis

614. A patient K., 26 years old, come to the permanent establishment on the 5th day of the disease with complaints of a high temperature, chill and a dry cough. The disease began suddenly from getting up of temperature to 38,8 °C, chill, then a dry cough. Treated himself as ARI, took analgin and dimedrol. Objectively: moderate severity, on his abdomen single roseollas, hepatosplenomegaly, diarrhea up to 4 times without admixtures. The most informative methods of diagnostics for this disease are:

a. Cmear from a pharynx for a virusb. Passive hemaglutination reaction with О-, Н- and Vi-antigensc. Clinical blood testd. Reaction of Vidale. *Hemoculture

615. Sick person, 18 years old, became ill suddenly, when appeared chill, increased of temperature to 39,6 °C, weakness, headache. On the 2nd day of illness appeared diarrhea up to 10 times per day, without admixtures. A few days ago ate beef. On the 5th day of illness appeared maculous-papulous rash on his body. Pulse was 100 per a min., BP 110/60 mmHg. Abdomen is soft, painful in the epigastrium and mesogastrium. Liver +1 cm. Watery diarrhea 5 time per days. Choose the most reliable diagnosis:

a. Typhoid feverb. Yersiniosisc. *Paratyphoid fever Bd. Sepsis

Page 303: intranet.tdmu.edu.uaintranet.tdmu.edu.ua/data/kafedra/internal/infect_desease... · Web viewBest method to treat diarrhoea in child is: intra venous fluide *ORS antibiotics bowel

e. Paratyphoid fever A616. In a patient 30 years old, on the 9th day of illness disease began gradually, with

slow progression of fever and intoxication, appeared unabundant roseollar rash on the skin of abdomen. Skin was pale, Т-40 °C, pulse-80 per a min, BP-100/65 mm Hg. The tongue is assessed, abdomen is distended, the spleen is enlarged, and the liver is palpable. What symptom will be positive?

a. Botkin’s symptomb. *Padalka’s symptomc. Kildushevsky‘s symptomd. Оrtner’s symptome. Pasternatsky‘s symptom

617. A patient R., 23 years old, was hospitalized on the 4th day of disease, he had contact with a patient with typhoid fever. During hospitalization body temperature – 37,8 °C, severe headache. A tongue is covered wit a yellow coverings. The abdomen is soft, painless, rumbling in an ileocecal corner. Liver +1 cm. Defecation is absent during 3 days. Hospitalized for an inspection on typhoid fever. What examination (laboratory diagnosis) do you suggest for this patient?

a. Urinocultureb. Coproculturec. *Hemocultured. Biliculturee. Positive reaction with a typhoid antigen 1:200

618. In a patient A., 33 years old, with the diagnosis of typhoid fever, on the 5th day of normal temperature appeared tachycardia and roseol rash on the middle and lateral surfaces of the abdomen. About what is it necessary to consider?

a. Measlesb. Bleedingc. Perforationd. *Relapse of typhoid fevere. Infectiously-allergic myocarditis

619. In the induction centre of ian nfectious hospital a patient came in with a three weeks history of high fever 38-40 °C, head pain, weakness, insomniA. He couldn’t speak to the doctor, accepted antipyretics. Objectively: body temperature is 35,7 C, pulse 140 per 1 mines, BP 80/50 mm Hg. General condition is severe. Skin and mucous membranes are pale. The tongue has teeth imprints, with a dirty-brown raid, a tag and edges of the tongue is clean. The abdomen is swollen. The liver and spleen are enlarged. There was defecation on reception, in the excrement was fresh blood. Why the increase in body temperature and decrease in pulse?

a. Perforationb. Acute poisoning medicationsc. Infectiously toxic shockd. Endometriosis of colone. *Bleeding

620. The patient O., 23 years old, appealed to the hospital on the 6th day of gradual development of illness, with complaints of severe headache (pain at the back of head)and fever. Objectively: temperature of body 39,7 °C, pulse 84 per a min The face is pale,

Page 304: intranet.tdmu.edu.uaintranet.tdmu.edu.ua/data/kafedra/internal/infect_desease... · Web viewBest method to treat diarrhoea in child is: intra venous fluide *ORS antibiotics bowel

tongue is dry, near a root covered a grey raid. abdomen is swollen. Dulling of percutic sound is observed in a right iliac areA. His liver and spleen are palpated. What day does rash appear on in this illness?

a. *On a 8th dayb. On a 2tnd dayc. On a 4th dayd. On a 5th daye. On a 14th day

621. A patient, 23 years old, appealed to the hospital on the 6th day of gradual development of illness, with complaints of severe head pain at the back of his head, parahypnosis and fever. Objectively: temperature 39,7 °C, Ps 84 per min. His face is pale, tongue is dry, near the root is covered with a grey raid. A abdomen is swollen. Dulling of percutory sound is observed in a right iliac areA. The liver and spleen are palpated. What day does rash appear in this illness?

a. On a 12th dayb. On a 4th dayc. On a 5th dayd. On a 6th daye. *On a 8th day

622. A woman D., 41 years, complaints on diarrhea, headache, severe weakness, insomnia, dull pain in a right iliac areA. It is a 8th day of illness. At a review: Т-39,8 °C, Рs – 86/min., AP – 90/60 mm Hg. Pulse is dicotic. Skin is pale. Single roseollas are on the abdomen. Tongue is dry, assessed by the coverings, with the imprints of teeth on a lateral surface. Soft, dulling of percutory sound is determined in a illeocaecal area of abdomen. Hepatosplenomegaly, positive Blumberg’s symptom, neutrophilic leukocytosis. What diagnosis is most reliable?

a. *Typhoid feverb. Epidemic typhusc. Sepsisd. Megakaryoblastomae. Tuberculosis

623. Patient R., 35 years old, entered to the clinic on the ninth day of illness with complaints of increased Т to 39,0 °C, headache, general weakness, delay in voiding, sleep disturbance At a review: on the skin of abdomen are single roseollas, a tongue is assessed by the brown covering, Ps. 78 shots in a min., rhythmic, liver is enlarged for 2сm. What is reliable diagnosis?

a. *Typhoid feverb. Leptospirosisc. Brucellosisd. Sepsise. Tuberculosis

624. A vagabond 45 years old is hospitalized on the 7th day of fever. Objectively: t – 39,8 °C. He complaints of headache and insomniA. The patient is excited, talkative. Face is hyperemic. Rash mainly on the lateral surfaces of trunk, abundant, roseollar-patechial. Pulse is rapid. Enlarged liver and spleen. What is the preliminary diagnosis?

a. Typhoid fever

Page 305: intranet.tdmu.edu.uaintranet.tdmu.edu.ua/data/kafedra/internal/infect_desease... · Web viewBest method to treat diarrhoea in child is: intra venous fluide *ORS antibiotics bowel

b. *Epidemic typhusc. Paratyphoid Bd. Leptospirosise. Scarlet fever

625. A patient, 24 y/o, was hospitalized in infectious department on the 10th day of illness with complaints of weakness, headache, bad appetite and cough. Objectively: temperature of body 39,5 °C, pallor of skin. On the abdomen, and chest some roseolas were found. There is hard breathing over the lungs with no wheezes. RR 20 / min. Pulse 80 /1 min. The liver edge is palpable to 1 cm. below the costal arc. The spleen is not enlarged. What is the diagnosis?

a. *Typhoid feverb. Fluc. Spotted fever d. Brucellosis e. Pneumonia

626. Patient P., 45 years old, entered on consultation with a diagnosis „fever of idiopathic etiology”. He has been ill 8 days. The disease developed gradually, then appeared headache, insomnia, adynamiA. Temperature – 39,5 °C, skin is pale. Pulse – 88 per a min. The center of the tongue was covered with a thick grey coat. The abdomen is exaggerated, grumbles in a right iliac region. The liver and spleen are enlarged on 2 cm. What is the most probable diagnosis?

a. *Typhoid feverb. Yersiniosisc. Epidemic typhusd. Adenoviral infectione. Sepsis

627. At the inspection of persons who had contact with a patient with typhoid fever, stick of typhoid fever was found in urine. The reaction of Widal was negative. The patient considers himself healthy. What is your preliminary diagnosis?

a. Typhoid fever, latent periodb. Typhoid fever, relapsec. Transitory bacteriocarriers of stick of typhoid feverd. *Chronic bacteriocarriers of stick of typhoid fevere. Any of the enumerated diagnoses is possible

628. A 39 years old train conductor, is hospitalized on the 4th day of illness with complaints of headache, weakness, dizziness, increased perspiration, insomnia, chills. Hyperemia of face with edema, and conjunctivitis have been observed. On a transitional fold a conjunctiva are not numerous petechias. On the skin of trunk, thorax, abdomen, extremities intensive roseolopetechial rash was found. On exam there was tachycardia BP – 100/60 mm of Hg tremor of tongue were marked. The liver, and spleen, are enlarged. Patient is constipated for 3 days. Most credible diagnosis is:

a. *Epidemic typhusb. Typhoid feverc. Flud. Meningococcemia e. Leptospirosis

Page 306: intranet.tdmu.edu.uaintranet.tdmu.edu.ua/data/kafedra/internal/infect_desease... · Web viewBest method to treat diarrhoea in child is: intra venous fluide *ORS antibiotics bowel

629. A patient V., 23 years old, acted to permanent establishment on the 6th day of the disease with complaints for a high temperature, chill, dry cough. The disease began suddenly with a rise of temperature to 39,7 °C, chill, then dry cough. Treated oneself as ARI, took an analgin, dimedrol. Objectively: state of moderate severity, on an abdomen single roseollas, hepatosplenomegaly, diarrhea up to 4 times without admixtures. What is the preliminary diagnosis?

a. Yersiniosisb. Typhoid feverc. Brill‘s diseased. Meningococcemiae. *Epidemic typhus

630. Patient I., 28 years old, hospitalized on the 9th day of illness with complaints of increase of temperature to 39,2 °C, headache, general weakness and absence of defecation. There are singles roseolas on the abdomen, pulse 78 per a min, a liver + 2 cm. What is probable diagnosis?

a. Sepsisb. Spotted feverc. *Typhoid feverd. Brucellosise. Leptospirosis

631. At sick, 32 years, on the 9th day of illness which began gradually, from slow rise of fever and intoxication, appeared 3 roseols on the skin of abdomen. Objectively: skin is pale, temperature – 40,4 °C, pulse 80 per a min, BP 100/65 mm Hg. The tongue is assessed, an abdomen is swollen, and the spleen and liver are palpable. Inspection of which disease is needed above all the options?

a. Spotted feverb. *Typhoid feverc. Measlesd. Scarlet fevere. Sepsis

632. A child, 8 years old, has been ill for 9 days. Complaints of weakness, headache and insomniA. A temperature to – 38,5-39,5 °C. Skin is pale. There are 3 roseolas on the abdomen. A tongue is assessed. Liver and spleen are enlarged. What disease is the most probable?

a. Sepsisb. Yersiniosisc. Infectious mononucleosisd. *Typhoid fevere. Leptospirosis

633. At sick P., 40 years old, the high temperature of the body is marked during 8 days, severe headache. Objectively: temperature – 39,5 °C, the patient is pale, languid, adynamic. Pulse 82 per a min. Tongue is dry, assessed a brown raid, on the skin of abdomen singles roseolas. A liver + 2 cm. What is the most probable diagnosis?

a. *Typhoid feverb. Spotted feverc. Sepsis

Page 307: intranet.tdmu.edu.uaintranet.tdmu.edu.ua/data/kafedra/internal/infect_desease... · Web viewBest method to treat diarrhoea in child is: intra venous fluide *ORS antibiotics bowel

d. Tuberculosise. Brucellosis

634. Patient B., 36 years old, complaints of a great headache, general weakness, insomnia, fever to 39,7 °C. Fell ill gradually. Objectively: skin is pale, on a abdomen are singles roseolas. A tongue with the imprints of teeth and white raid, edges and tag is clean. Flatulence. A liver and spleen are enlarged. Dulling of percussive sound is in a right iliac areA. Pulse 70 per a min, BP 100/60 mm Hg. In lights there are the dissipated dry wheezes, hard breathing. Blood test: leycocytosis 3,1?109/l, RSE 25 mm/hour, eos. 0 %, n/n 9 %, s/n 51 %, lymphs. 31 %, monocyts 5 %. What is the most credible diagnosis?

a. *Typhoid feverb. Epidemic typhusc. Pneumoniad. Leptospirosise. Sepsis

635. Patient K, 32 years old, of no fixed residence went to see a doctor for 5-days illness with complaints of fever, severe headache, insomniA. On body temperature 40 °C, pulse 110/min. The patient is excited and talkative. Hyperemic, scleritis. At whole over the body there is pink petehial rashes. Positive Govorova-Godele‘s symptom. Enlarged liver and spleen. What do you need to find out the epidemic status?

a. *The existence of head liceb. Use of poor foodc. Availability of parenteral interventionsd. Contact with rodentse. Drinking water from unhygienic sources

636. A patient L., 32 years old, who complaints of severe headache and fever, the 6-day illness positive agglutination test with rickettsia reaction. Vector of the disease is:

a. Fleab. Flyc. Mosquitoesd. Beee. *Lice

637. A patient L, 72 years old, who complaints of severe headache and fever, the 6-day illness positive agglutination test with rickettsia reaction. Past sick sick was typhus. Vector of the disease is:

a. Fleab. Flyc. Mosquitoesd. Licee. *Carrier does not need

638. A patient 29 years, a few days ago, a chill, the temperature for 2-3 days increased to 39-40 °C. There hyperemia and edema person,significant sclera like "drunk" person and "rabbit" eyes. On the third day of illness – on mucuos of soft palate, parenthesis bright red enantema is seen. At 3-4-day patient when trying to protrude the tongue, there was hypermovement, tremor, rejecting it to the side. Diagnosis: epidemic typhus. Which disease is likely in a patient?

Page 308: intranet.tdmu.edu.uaintranet.tdmu.edu.ua/data/kafedra/internal/infect_desease... · Web viewBest method to treat diarrhoea in child is: intra venous fluide *ORS antibiotics bowel

a. *Epidemic typhusb. Typhoid feverc. Brill disease d. Paratyphoid Ae. Paratyphoid B

639. A patient at the 4-day fever has profuse rosy-petehia exanthema predominantly located on the lateral surface of the torso and limbs flexion surfaces. Hyperemic, vascular conjunctivitis, petehii transition of anterior fold conjunctivA. What kind of illness can think?

a. Typhoid feverb. *Epidemic typhusc. Measlesd. Haemorrhagic fever with renal syndromee. Crimean hemorrhagic fever

640. Patient B., complaints of headaches, delirium. When inspection: a body temperature of 39 °C, initiated, a person hyperemic, positive symptom Govorova-Godele, petehia rash on the trunk, limbs, tachycardia, hypotension, hepatosplenomegaly. Drunker, drug user, living in the basement room type. What is your preliminary diagnosis?

a. AIDS infectionb. Influenzac. Leptospirosisd. *Epidemic typhuse. Alcohol psychosis

641. A patient 28 years old, of no fixed abode, hospitalized with a preliminary diagnosis «flu», a 5-day illness appeared rosy-petehia rashes on the body and interior surfaces of the extremities. The temperature of 41 °C, euphoria, hyperemia person, redness sclera, tongue tremor, tachycardia, enlarged spleen; excitement. What is the likely diagnosis?

a. Typhoid feverb. Leptospirosisc. Alcohol deliryd. Measlese. *Epidemic typhus

642. The watchman 42 years old, complaining of?desperately ill. Entered the 6-day illness with fever 39,7 °C, severe headaches, noise in the ears, insomniA. OBJECTIVE: instituted, euphoric, talkative. Face red, his eyes shining, sclera and conjunctiva enhanced vascular pattern. On the inside shoulder and the side surfaces of chest rosy-petehia rash. Found head lice and nits. Reaction Vidal 1:40. What is diagnosis?

a. Brill disease b. Malariac. *Epidemic typhusd. Typhoid fevere. Q-fever

643. A patient 28 years at the 4-day fever has profuse rosy-petehia exanthema predominantly located on the lateral surface of the torso and limbs flexion surfaces. An individual patient hyperemic expressed vessels conjunctival injection, petehies transition

Page 309: intranet.tdmu.edu.uaintranet.tdmu.edu.ua/data/kafedra/internal/infect_desease... · Web viewBest method to treat diarrhoea in child is: intra venous fluide *ORS antibiotics bowel

to fold conjunctivA. Analysis of urine protein single hyaline and granular cylinders. What kind of illness can think?

a. Typhoid feverb. Haemorrhagic fever with renal syndromec. Crimean hemorrhagic feverd. Measlese. *Epidemic typhus

644. For patients 78 years of acute illness began, which is characterized by fever with chilling, rash dominated rosy petehia elements. The condition of the patient violated moderately. Agent and the source of infection is not detected, but it is known that 45 years ago patient had severe typhus. What is your diagnosis?

a. Yersiniozisb. * Brill‘s diseasec. Typhoid feverd. Drug allergye. Food Allergies

645. The patient, a train conductor, 39 years old, hospitalized for a 4-day illness with complaints of headache, weakness, dizziness, sweating, insomnia, fever. Hyperemic, edema, conjunctivitis. At the transition fold conjunctiva – single petehies. At the skin torso, chest, abdomen, limbs abundance rosy-petehia rash. TachycardiA. AD 100 and 60. Tremor of the tough. Palpable liver, spleen. Stool arrested. What is the most likely diagnosis?

a. Leptospirosisb. Typhoid feverc. Influenzad. Meningoccemiae. *Epidemic typhus

646. A patient 30 years on 9th day of illness that began gradually, the slow rise of fever and intoxication, a painless rosy rashes on the skin of the abdomen. OBJECTIVE: pale, temperature 40 ° C, pulse 80/min, AD 100 and 65 mm RT. Art. Language is coached, abdomen bubbles, enlarged spleen and liver. For what disease you can suggest in the first place?

a. *Typhoid feverb. Epidemic typhusc. Measlesd. Scarlet fevere. Sepsis

647. Patient 20 years, complained about the high temperatures of up to 39 ° C, headache in the frontal area, pain in the eyeball, photophobia, pain in muscles, dry cough. Acutely ill the day before. Objective: condition serious, hyperaemic, eyes shining, vascular injection sclerA. Pulse 96/min, rhythmical. Tone heart weakened. In the lungs scattered dry bubbling rale. Faces hyperemic, granular. Meningeal symptoms are not present. Blood tests: Leu. 3?109, eoz. 1 %, yang neu. 6 %, neu. 51 %, lymph. 35 %, mon. 7 %. What is the most likely diagnosis?

a. Measlesb. *Influenza

Page 310: intranet.tdmu.edu.uaintranet.tdmu.edu.ua/data/kafedra/internal/infect_desease... · Web viewBest method to treat diarrhoea in child is: intra venous fluide *ORS antibiotics bowel

c. Meningococcal diseased. Pneumoniae. Typhus

648. Often, in patient with epidemic typhus arise transition petehies in the conjunctivA. What term did it call?

a. Symptom of Hellerb. Conjunctivitisc. Symptom of Govorova-Godeled. *Symptom of Zorohovich-Chiari-Avtsyne. Enantema Rosenberg‘s

649. In the family of the patient with epidemic typhus, were lice in the children. With the help of any of these events could prevent the subsequent spread of the disease?

a. *Monitoring and complete sanitation of contact in the centreb. The use of chemoprophylaxisc. The use of antibioticsd. Isolation contacte. Check-up

650. When you can stopped etiotropic medications treatment of the patient with epidemic typhus?

a. Immediately after the normalization of body temperatureb. After the normalization of the liver and spleenc. *After a 2-day normal body temperatured. After the disappearance of roseolae. Within 10 days after the disappearance of roseola

651. Patient K., 23 years old, hospitalized on the 3rd day of illness, which was accompanied by mild running nose, high fever to 40,2 °C, headache and hemorrhagic rash on the skin. In the 2 hours after the introduction of penicillin blood pressure dropped to 40 and 10 mm Hg. Distal pulse and meningeal signs are not defined. What is the diagnosis in a patient?

a. *Meningoccocemia, infectious-toxic shockb. Epidemic Typhus, severe coursec. Measles, severe coursed. Scarlet fever, severe coursee. Flu, anaphylactic shock

652. A patient 25 years old, who returned from the Far East, suddenly has increased body temperature to 39 °C, a pain in the backbone, reddening skin type «hood», single hemorrhages on the skin. After 3 days, along with declining fever, weakness, thirst, decreased diuresis to 300 ml, decreased blood pressure. Much pronounced Pasternatskyj‘s symptom. What is the most likely diagnosis?

a. Leptospirosisb. Typhusc. Haemorrhagic fever Crimean-Congod. Acute glomerulonephritise. *Haemorrhagic fever with renal syndrome

653. Male 45 years old, fell ill after 2 weeks after returning from Afghanistan, where six months ago underwent malariA. In return were found lice. Suddenly, there were severe

Page 311: intranet.tdmu.edu.uaintranet.tdmu.edu.ua/data/kafedra/internal/infect_desease... · Web viewBest method to treat diarrhoea in child is: intra venous fluide *ORS antibiotics bowel

headache, weakness, body temperature 39 °C with a temporary decrease in the 4th day of illness, followed by general weakness, much intoxication, headache, appeared on the body abundance polymorphic rash. At the 7-day state of heavy, the phenomenon of encephalitis, excitation, hallucinations periodically. Objective – erythematos-petehial exanthemA. The temperature of the body 40,1 ?C, pulse was 136 for 1 min, blood pressure 120/70 mm RT art. Moderately enlarged liver and spleen. Paradoxical ishuriyA. In the blood analys neutrofil mild leukocytosis, eozinofilia, accelerated ESR. Which of the studies of blood will be positive?

a. At typhoid feverb. At the three-day malariac. Tropical malariad. *Epidemic typhus e. At brucellosis

654. Male 26 years old, who last month returned from Africa and was processing about lice, sick 5 days. Home was a sudden: severe headache, weakness, body temperature 38,8 °C, which persisted all day at a constant level, but at the 4th day of the disease declined for several hours. Then the patient condition has deteriorated significantly, a rash on the body, delay urine. OBJECTIVE: pulse was 110 for 1 min, blood pressure 115/70 mm RT. Art., temperature 39,7 °C. Face red, a significant injection of vascular sclera, cmall hemorrhages in the conjunctiva, mucous membrane soft palate, abundant rosy-petehialexanthema on the body. Signs interstitsial pneumonia, encephalitis. Enlarged liver and spleen. What kind of illness can you think of?

a. *Epidemic typhusb. Typhoid feverc. Tropical malariad. Leptospirosise. Yellow fever

655. Patient P., 68 years old, fell ill suddenly 7 days ago from the increase of temperature to 39,3 °C, appearances of headache and insomniA. Objectively: excited, inadequate. Face is hyperemic. The tongue is very dry, trembles at pulling out. On a trunk are polymorphic rash, tachycardia, and blood pressure low. HepatosplenomegaliA. Stool is absent. In age 10 years had epidemic typhus. What is the previous diagnosis?

a. Yersiniosisb. Typhoid feverc. *Brill‘s diseased. meningococcemiae. Sepsis

656. Patient P., 76 years old, complianed during 7 days for the permanent increase of temperature to 38,2-38,7 °C, headache, insomnia, horrific dreams, dry cough, myalgias, artalgias. Polymorphic rash appeared on a trunk on the 4th day of diseases. In childhood was ill by epidemic fever, three-day malariA. Temperature of body – 38,4 °C, pulse – 98 per a min, hepatolienal syndrome, bilateral pneumonia confirmed roentgenologic. In the analysis of blood moderate neutrophil leycocytosis. What is the preliminary diagnosis?

a. *Brill‘s diseaseb. Typhoid feverc. Lime‘s disease

Page 312: intranet.tdmu.edu.uaintranet.tdmu.edu.ua/data/kafedra/internal/infect_desease... · Web viewBest method to treat diarrhoea in child is: intra venous fluide *ORS antibiotics bowel

d. Malariae. Leptospirosis

657. Patients D., 30 years old, became ill saddenly, when the temperature of body rose to 40,2 °C, appeared headache, weakness, euphoria, injection of vascular sclera, hyperemia of face, appears the positive symptom of Govorov-Godele. Reaction with Rickettsia prowazekii 1:160, IgG – 87 %. What is your diagnosis?

a. Brill‘s diseaseb. Typhoid feverc. Paratyphoid Ad. Paratyphoid Be. *Epidemic typhus

658. Patients I., 78 years old, became ill saddenly, the temperature of body rose to 39,2 °C, euphoric, vessels of scleras are injection, hyperemia of face, appears enantema of Rozenberg. Reaction with Rickettsia prowazekii 1:160, IgG – 87 %. What is your diagnosis?

a. *Brill‘s diseaseb. Typhoid feverc. Meningococcal infectiond. Flue. Leptospirosis

659. At a patient with a pediculosis the temperature of body rose to 41,6 °C, appeared headache, euphoria, in 4th days from the beginning of illness – rozeola-petehial rash. Reaction with Rickettsia prowazekii 1:640, IGG – 89 %. What is your diagnosis?

a. Flub. Typhoid feverc. Meningococcal infectiond. *Epidemic typhus e. Leptospirosis

660. A patient’s temperature is 40 °C. There are also deep and unproductive cough, photophobia, face puffiness whitish points on the mucous membrane of cheeks opposite molar teeth. What is the most possible diagnosis?

a. Tuberculosis b. Meningococcemia c. *Measles d. Enteroviral infection e. Staphylococcus sepsis

661. A child after consuming food in a party complaints of vomiting and diarrhea within 1-5 hours. The diagnosis is:

a. *Staphylococcus aureusb. Streptococcusc. Clostridium Perfringensd. Clostridium Botulinume. Meningococcus

662. Patient A., 37 years old, entered to infectious hospital on the third days of disease in the severe condition. He complaints of the high fever with chills and sweat, general weakness, pain in right under a rib. Objectively: temperature of body 41 °С, icterus of

Page 313: intranet.tdmu.edu.uaintranet.tdmu.edu.ua/data/kafedra/internal/infect_desease... · Web viewBest method to treat diarrhoea in child is: intra venous fluide *ORS antibiotics bowel

skin, liver +2 cm, pain at palpation in abdomen, positive symptoms of Ortner and Mussy, a spleen is normal, tachycardiA. What is the preliminary diagnosis?

a. Malariab. *Cholangitisc. Viral hepatitisd. Sepsise. Leptospirosis

663. Patient W., 38 years old, entered to infectious hospital on the 5th days of disease in the severe condition. He complaints of the high fever with chills and sweat, general weakness, pain in sacrum. Objectively: temperature of body of 41 °С, tachycardia, positive symptom of Pasternacky, liver and spleen is not normal. Preliminary diagnosis?

a. Malariab. Cholangitisc. *Pyelonephritisd. Sepsise. Leptospirosis

664. Patient D., 39 years old, entered to infectious hospital on the second week of disease in the severe condition. She complaints of high fever with chills and sweat, general weakness, pharyngalgia at swallowing. Objectively: temperature of body 37,7 °С, hyperemia of mucus of pharynx, tonsils are enlarged, loose, festering raid in lakuns, enlarged submandibula, neck, axilars lymphonodules, icterus of sclera and skin, bradycardia, liver + 2 cm, spleen + 1 cm. Urine is color of beer, an excrement is discolored. What is the previous diagnosis?

a. Malariab. Infectious mononucleosisc. Viral hepatitisd. *Sepsise. Leptospirosis

665. Patient A., 37 years old, entered to infectious hospital on the third days of disease in the severe condition. He complaints of the high fever with chills and sweat, general weakness, pain in right under a rib. Objectively: temperature of body 41 °С, icterus of skin, liver + 2 cm, pain at palpation in abdomen, positive symptoms of Ortner and Mussy, a spleen is normal, tachycardiA. What methods is it possible to confirm a previous diagnosis by?

a. Global analysis of bloodb. *Ultrasonic researchc. Biochemical blood testd. Hemoculturee. Parazytoskopy of blood

666. Patient W., 38 years old, entered the infectious hospital on the 5th days of disease in the severe condition. He complaints of the high fever with chills and sweat, general weakness, pain in sacrum. Objectively: temperature of body of 41 °С, tachycardia, positive symptom of Pasternacky, liver and spleen not is normal. What is the previous diagnosis? What methods is it possible to confirm a previous diagnosis by?

a. General blood analysis b. Ultrasonic research

Page 314: intranet.tdmu.edu.uaintranet.tdmu.edu.ua/data/kafedra/internal/infect_desease... · Web viewBest method to treat diarrhoea in child is: intra venous fluide *ORS antibiotics bowel

c. Biochemical blood testd. Hemoculturee. *Parazytoskopy of blood

667. Patient of 52 years, fell ill sharply from a chill and head pain, fevers to 40°C. On a 3th day illnesses are nausea and vomiting, dark urine. On a 4th day a temperature went down to 37 °, but an icterus appeared and the amount of urine diminished to 600 ml. What disease is such development of symptoms characteristic for?

a. Hepatitis Ab. Hemorragic fever with a kidney syndromec. *Leptospirosisd. Sepsise. Acute glomerulonephritis

668. Animal technician 57 years, on the 3rd day of illness appealed to the doctor with complaints of head pain, high temperature, pain in gastrocnemius muscles, cmall of the back, icterus, dark urine and diminishing of its amount. Objectively: temperature 38,1 °C, injection of the sclera vessels, petechial rash on upper part of the thorax, hepatosplenomegaly. What is the most credible preliminary diagnosis?

a. Pseudotuberculosisb. Brucellosisc. Viral hepatitisd. *Leptospirosis e. Influenza

669. A patient I., 26 years old, delivered in a hospital on the 4th day of disease with complaints of fever, headache, pain in gastrocnemius muscles. Works as a specialist in land-reclamation. Has a lot of sexual contacts. Objectively: temperature – 39,7 °C. Severe common condition. Expressed icterosis of skins and sclerA. Hemorrhages in conjunctiva and sclerA. There is a hemorragic rash on a skin. A liver increased on 3 see below the costal arc, edge of spleen, the day's diuresis 300 ml. The etiologic factor of the disease most probable:

a. Rikkettsiab. Virus c. *Leptospirad. Spirochetae. Chlamidia

670. Patient 43, miner, on the 7th day of disease grumbles about a sharp weakness, high temperature, pain in the muscles of feet and cmall of the back, icterus, dark color of urine, headache. Fell ill sharply is a chill, temperature 40,2 °, there was a nose-bleed. A diuresis 200 ml. Credible diagnosis?

a. Sepsisb. Typhoidc. Viral hepatitisd. *Leptospirosis e. Malaria

671. A patient, 35 years, suffering sharply, complaints of a headache, pain in the muscles of lower extremities, increasing of temperature to 39,3 °C. Objectively on the 4th

Page 315: intranet.tdmu.edu.uaintranet.tdmu.edu.ua/data/kafedra/internal/infect_desease... · Web viewBest method to treat diarrhoea in child is: intra venous fluide *ORS antibiotics bowel

day of disease: the state is severy, hyperemic face, icterosis of skin and sclerA. Enlargment of liver and spleen. A diuresis is mionectic. What is most credible diagnosis?

a. Trichinosisb. Hepatitis Ac. Yersiniosisd. Infectious mononucleuse. *Leptospirosis

672. For a patient, workwoman of a pig farm, on a background of complete health a chill appeared suddenly, a temperature rose to 39,9 °C, there was a headache, nauseA. On the next day marked pains in the muscles of lower extremities, nose-bleeding began. On the 3rd day of illness, state became more severy. Face is hyperemic, scleritis, hyperemic scleritis. Liver +3 cm. Daily diuresis 700 ml. What is the preliminary diagnosis?

a. Yersiniosisb. Hepatitis Ac. Hemorrhagic fever with a kidney syndromed. Flue. *Leptospirosis

673. On the 3rd day of illness the sick is delivered in a severe condition with complaints of sudden rise of temperature, headache, repeated nose-bleed, pain in gastrocnemius muscles. Objectively: moderate icterus of sclera and skins, hepatospleenomegaly, оliguriA. What is most credible diagnosis?

a. *Leptospirosisb. Viral hepatitisc. Influenzad. Infectious mononucleosise. Malaria

674. A patient S., 45 years old, suffering suddenly from appearance of chill and increasing of temperature to 39,2 °C. In the evening pain appeared in a abdomen and gastrocnemius muscles. In 2 days noticed the ochrodermia of skin and sclerA. Objectively: the state is severy, temperature 39,9 °C. The tongue is covered. Moderate jaundice of the skin and sclerA. There is plural petachiae on a trunk. Superficial breathing 20 times per 1 min, pulse 102 per 1 min, AP 100/60 mm of Hg. A abdomen is soft, sickly in epigastrium, a liver on 3 cm comes from a costal arc. Daily diuresis 300 mm, urine is sad-coloured. What is your preliminary diagnosis?

a. Sepsisb. *Leptospirosisc. Influenzad. Hepatitis Be. Infectious mononucleus

675. A patient Z., 33 years old, miner, entered clinic of infectious diseases on the 7th day of disease with complaints of acute weakness, high temperature, pain in the muscles of feet and back, icterus, dark color of urine, headache. Became sick sharply from a chill, temperature rises up to 40,1 °C. On a 4th day there is an icterus, nose-bleeding, hemorrhages in sclerA. Duration of fever 6 days. Diuresis is 200 ml. What is credible diagnosis?

a. Typhoid fever

Page 316: intranet.tdmu.edu.uaintranet.tdmu.edu.ua/data/kafedra/internal/infect_desease... · Web viewBest method to treat diarrhoea in child is: intra venous fluide *ORS antibiotics bowel

b. *Leptospirosisc. Hepatitis Ad. Sepsise. Influenza

676. A patient Y., 25 years old, entered infectious department on the 3rd day of disease with complaints of headache, pain in the back, gastrocnemius muscles, high fever, chill. State is moderate. Icteruses of the scleras. Mucous membrane of soft palate is hyperemic. Tongue is dry, assessed with brown cover. The abdomen is swollen. Liver +2 cm, spleen is not megascopic. Painfull muscles, especially gastrocnemius. Urine is dark, excrements ordinary color. What is the most credible diagnosis?

a. Infectious moneucliousb. Hepatitis Ac. Malariad. *Leptospirosise. Yersiniosis

677. Patient A., 16 years, hospitalized in an infectious department with complaints of absence of motions in lower extremities. A disease was begun 2 days back with the increase of temperature to 38 °, dyarrhea is to 3-5 times per days. Melosalgias took a place, head pain. Objectively: temperature of body 36,8 °, active motions absent in lower extremities, in the area of defeat is areflexis, low blood pressure of muscles, a sensitiveness is stored. Meningeal symptoms are poorly expressed. What disease is it needed to think of?

a. Leptospirosisb. Meningococcal infectionc. *Poliomyelitis d. Parainfectional encephalitise. Tubercular meningo encephalitis

678. For patient A., 25 years, which returned from Far East, suddenly the temperature of body rose to 39 °, pain appeared in cmall of the back, hyperemia of person, neck, overhead half of trunk, single haemorrages as red strips on a neck and lateral surfaces of thorax. Nose-bleeds. Through 3 days a weakness, thirst, grew together with the decline of fever, diminished to 300 ml diuresis, an arteriotony went down. Positive symptom of Pasternackogo. What diagnosis is most credible?

a. Leptospirosisb. *Hemorrhagic fever with a kidney syndromec. Spotted feverd. Hemorrhagic fever Crimea-Congoe. Acute glomerulonephritis

679. Man 26 years, month ago returned from Africa, passed treatment concerning pediculosis. He is ill for 5 days. Beginning was sudden: great head pain, weakness, stationary temperature of body (38,8°C), which on a 4th day went down on a few clock. The state of patient was considerably worsened farther, a rash appeared on a trunk, coughing, uroschesis. Objectively: temperature of body 39,7°C, pulse 110 per 1 min, BP 115/70 mm Hg. Skin of person red, considerable injection of vessels of sclerotica, shallow hemorrhage on a conjunctiva, mucous membrane of soft palate, abundant roseol-petechial

Page 317: intranet.tdmu.edu.uaintranet.tdmu.edu.ua/data/kafedra/internal/infect_desease... · Web viewBest method to treat diarrhoea in child is: intra venous fluide *ORS antibiotics bowel

exanthema on a trunk. Signs of interstitial pneumonia, encephalitis. A liver and spleen is megascopic. What disease most probably?

a. Yellow feverb. Typhoid feverc. Tropical malariad. Leptospirosise. * Epidemic typhus

680. Patient A., 35 years, had ill sharply, complaints of high temperature of body, great head pain, sickliness in gastrocnemius muscles. Objectively on the 4th day of illness: the state is heavy, hyperemia of person, skin and sclerotica are icterus, a liver and spleen is megascopic. OliguriyA. What diagnosis is most credible?

a. Yersiniosisb. Viral hepatitisc. *Leptospirosis d. Poisoning of tetraetyl leade. Omsk hemorrhagic fever

681. At one on holiday-makers ashore lake in 6 days did a temperature rise to 38,5 °C, headacke, pain, appeared in muscles, sweating. In 3 days there was the sickly slight swelling in an inguinal areA. At examination in an inguinal area found out a dense, mobile, moderate sickly lymphonodus to 5 cm in a diameter. The skin above him is not changed. What diagnosis is most credible?

a. Leptospirosisb. Iersiniosisc. Infectious mononucleosisd. *Rabbit-fever e. Acute lymphadenitis

682. Patient of 62 years, in the past had been ill with spotted fever. Fell ill sharply: fever 39,5 °C, head pain, insomniA. On a 6th day on the skin of abdomen and lateral surfaces of thorax roseol appeared rash. A liver and spleen, tachycardia, deafness of tones of heart, low blood pressure is megascopic. What diagnosis is most credible?

a. *Illness of Brillab. Typhoidc. Leptospirosisd. Sepsise. Infectious mononucleosis

683. Patient I., 21 years old, entered hospital on the 7th day of illness with complaints of a sharp weakness, pains in muscles and joints, head pain, nauseA. A disease began with sore throat, cold, general weakness. Did not measure a temperature. Last was the state worsened 2 days, a chill appeared, high temperature 39-40°C, head pain, nauseA. Objectively: patient of adynamic, consciousness is kept, temperature of body 37,5°C, the state is extremely heavy, a skin is pale, lips and nail phalanxes of cianotic, on the skin of tiptoes and hands hemorragic rash, the scopes of heart are extended to the left on 1,5 cm, tones are deaf, pulse 130 per 1 mines, weak filling, BP 80/40 mm Hg. Breathing normal. A tongue is assessed, moist, phenomena of pharyngitis. Neck lymphonodus are megascopic. Meningeal signs are absent. OliguriA. What diagnosis is most credible?

a. Hemorrhagic fever

Page 318: intranet.tdmu.edu.uaintranet.tdmu.edu.ua/data/kafedra/internal/infect_desease... · Web viewBest method to treat diarrhoea in child is: intra venous fluide *ORS antibiotics bowel

b. Leptospirosisc. Epidemic typhusd. Rheumatoceilse. *Meningococcal infection. MeningococcemiA.

684. Animal technician 57 years, on the 3rd day of illness appealed to the doctor with complaints of head pain, high temperature, pain in gastrocnemius muscles, cmall of the back, icterus, dark urine and diminishing of its amount. Objectively: temperature 38,2°, injection of vessels of sclerotica, petechial rash on overhead part of thorax, megascopic hepar and spleen. Most credible preliminary diagnosis?

a. Brucellosisb. *Leptospirosisc. Viral hepatitisd. Pseudotuberculosise. Trichinosis

685. Patient 43, miner, on the 7th day of disease complains of a sharp weakness, high temperature, pain in the muscles of feet and cmall of the back, icterus, dark color of urine, head pain. Fell ill sharply is a chill, temperature 40°, there was a nose-bleed. A diuresis 200 ml. Credible diagnosis?

a. Malariab. Typhoidc. Viral hepatitisd. Sepsise. *Leptospirosis

686. Patient 35 years, which fell ill sharply, complains of headache myalgia, pain in the muscles of lower extremities, increase of temperature to 39 °C. Objectively on the 4th day of illness: state heavy. Face of hyperaemiA. Skin and sclera are icteric. The liver and spleen are enlarged. A diuresis is mionectic. Most credible diagnosis?

a. Yersiniosisb. Hepatitis Ac. *Leptospirosisd. Infectious mononucleosise. Trichinosisf. Leptospirosis

687. For a patient, workwoman of pig farm, on a background a complete health a chill appeared suddenly, a temperature rose to 39,9 °, there was a head myalgia, nauseA. The next day marked pains in the muscles of lower extremities, a nose-bleed began. At the reception of the permanent establishment, on the 3rd day of illness, state heavy. Face of hyperaemia, scleritis, subicterus of sclerA. Liver +3cm, a diuresis 700 ml. Preliminary diagnosis?

a. Hemorragic fever with a kidney syndromeb. Hepatitis of Ac. Yersiniosisd. Flue. *Leptospirosis

688. The patient 33 years, miner, entered clinic of infectious diseases on the 7th day of disease with complaints of a sharp weakness, high temperature, pain in the muscles of feet

Page 319: intranet.tdmu.edu.uaintranet.tdmu.edu.ua/data/kafedra/internal/infect_desease... · Web viewBest method to treat diarrhoea in child is: intra venous fluide *ORS antibiotics bowel

and cmall of the back, icterus, dark color of urine, head pain. Fell ill sharply from a chill, temperatures 40°. On a 4th day is an icterus, nose-bleed, hemorrhage in a scleroticA. Duration of fever 6 days. A diuresis 200 ml. Credible diagnosis?

a. *Leptospirosisb. Typhoidc. Hepatitis Ad. Sepsise. Iersiniosis

689. A patient is disturbed by attacks fevers which repeat oneself periodically every third day. The icterus of sclerotica and skin covers, Liver and spleen are enlarged. Which of the following diagnosis below is most correct?

a. Viral hepatitisb. Sepsisc. *Malariad. Yersiniosise. Leptospirosis

690. A patient 25 years, entered infectious separation on the 3rd day of disease with complaints of pain in gastrocnemius muscles, high fever, chill. State of middle weight. Sclera are icterus. Mucous membrane of soft palate is hyperaemiA. A tongue is dry, assessed a brown raid. A abdomen is swollen. Liver +2 cm A spleen is not megascopic. Palpation of muscles, especially gastrocnemius, painful. Wetting dark. Excrement of ordinary color. Name the most credible diagnosis:

a. Infectious mononucleosisb. Viral hepatitis Ac. Malariad. *Leptospirosise. Yersiniosis

691. In receiving department of infectious separation on the 3rd day of illness the sick is delivered in a grave condition with complaints of a suddenly arising up high temperature, head pain, repeated nose-bleed, pains in gastrocnemius muscles. Objectively: moderate icterus of sclerotica and skins, liver and spleen are enlarged oliguriA. What is the most credible diagnosis?

a. Infectious mononucleosisb. Viral hepatitisc. Yersiniosisd. *Leptospirosise. Malaria

692. Patient P., 45 years old, fell ill suddenly from appearance of chill and increase of temperature to 39,2 °C. In the evening myalgia appeared in a abdomen and gastrocnemius muscles. In 2 days noticed the ochrodermia of skin and scleroticA. Objectively: the state is heavy, temperature 39,9 °C. Skin and sclerotica moderate yellow. There is plural petechia on a trunk. Breathing is normal. FB-20/min, Pulse-102/min, BP 100/60 mm Hg. A abdomen is soft, sickly in an epigastrium, inreasing of liver. Daily diuresis 300 ml of urine of sad-coloured. What is preliminary diagnosis?

a. Sepsisb. *Leptospirosis

Page 320: intranet.tdmu.edu.uaintranet.tdmu.edu.ua/data/kafedra/internal/infect_desease... · Web viewBest method to treat diarrhoea in child is: intra venous fluide *ORS antibiotics bowel

c. Yersiniosisd. Hepatitis Be. Infectious mononucleosis

693. 76-years old patient during 7 days grumble about the permanent increase of temperature to 38-38,7 °C, moderate head pain, insomnia, horrific dreams, dry cough, myalgias, arthragiA. From the 4th day of illness – on a trunk polymorphic exanthema with predominance of roseol elements. In childhood was ill the spotted fever, three-day malariA. Temperature of body 38,4°, pulse 98 per 1 min, hepatolienal syndrome, bilateral pneumonia, confirmed roentgenological. There is moderate neutrophilic leykocytosis in the global analysis of blood, speed-up ESR. What from diagnosis is the most credible?

a. * Brill’s diseaseb. Typhoid feverc. Lime diseased. Malariae. Leptospirosis

694. Patient P., 24 years old, hospitalized in an infectious department in a severe condition. Complaints of head acke, pain in muscles and joints, vomiting. Objectively –a patient is excited, temperature of body 39 °C. BP 90/60 mm Hg, tachycardiA. hyperaestesia of skin appeared. Doubtful meningial signs. It is known from anamnesis, that lives in a mud flow, has an economy, there are rats. What preparations must be given?

a. Veroshpiron, euphyllinum, Dimedrolumb. *Mannitol, Lasixum, prednisolon, penicillinc. Analgin, Dimedrolum, acetophened. Manitol, acetophenee. Lasixum, analgin, ampicillin

695. Sick, 22 years old, hospitalized in an infectious separation with complaints of chill, fever, great head pain, pain in gastrocnemius muscles. The state is severe, hyperemia of face, skin and sclera are icteric, the iver and spleen are enlarged, oliguriA. What preparations are routinely given in this disease?

a. Introduction of wheyb. *Antibioticsc. Sulfanilamid preparationsd. Desintoxic therapye. Enterosorbtion

696. Patient T., 40 years, hospitalized in infectious separation with complaints of chill, fever, great head pain, pain in gastrocnemius muscles. The state is severe, hyperemia of face, skin and sclera are icteric, the liver and spleen are enlarged. BP 60/20 mm Hg, pulse 120 per 1 min Optimum chart of treatment?

a. Hormones, desintoxical therapy, antibioticsb. Diuretic, desintoxical therapy, antibioticsc. Sorbtions, diuretic, desintoxical therapyd. *Hormones, diuretic, desintoxical therapy, antibioticse. Hormones, diuretic, antibiotics

697. Patient A., 57 years, animal technician, on the 3rd day of illness appealed to the doctor with complaints of head pain, fever, pain in gastrocnemius muscles, cmall of the back, icterus, dark urine and diminishing of its amount; temperature of body 38,2 °C ,

Page 321: intranet.tdmu.edu.uaintranet.tdmu.edu.ua/data/kafedra/internal/infect_desease... · Web viewBest method to treat diarrhoea in child is: intra venous fluide *ORS antibiotics bowel

injection of vessels of sclerotica, petechial rash on overhead part of thorax, enlarged liver and spleen. What preparations must be appointed above all things?

a. Intravenous introduction of salt solutionsb. Transfusion of fresh-frozen placmac. *Setting of antibioticsd. Hyperbaric oxygenetione. Haemosorbtion, sympathomimetics

698. Patient of 43 years, miner, on the 4th day of illness complains of a sharp weakness, high fever, onychalgias and cmall of the back, dark color of urine, head pain. Fell ill sharply chill, temperature 40,3°, there was a nose-bleed. A diuresis 200 ml. What preparations from the listed is necessary to use for this disease?

a. *Specific immunoproteinb. Holinomimeticsc. Spacmolysantsd. Miorelakse. Vitamins of group B

699. A patient, 44 years, entered infectious separation with a diagnosis leptospirosis. On the 7th day of treatment his state was sharply worsened, pains appeared in cmall of the back, somnolence, languor, cramps, head pain, a diuresis diminished to 100 ml/days. In blood: red corpuscles 2,6 T/l, creatinine of 438 mcmoll/l, urea 13,0 mmol/l. What complication developed in the patient?

a. Heart attack of budsb. Acute hepatic insufficiencyc. Chronic pyelonephritisd. Ischemic strokee. *Acute kidney insufficiency

700. A patient 40 years complains of a sharp weakness, head pain, to pain in gastrocnemius muscles, cmall of the back, insomnia, vomiting. Fell ill sharply 4 days back, when did a chill appear, a temperature rose to 40°, myalgias, nose-bleeds, icterus. Bathed in the river, where a lot of rodents is. The state is severe, on lips is herpes, sclerotica injections, icterus. Hemorragic rash on a skin. Pulse 120 on 1 mines, BP 90/50 mm Hg. A liver and spleen is megascopic. The symptom of Pasternackogo is positive. For days selected 100 ml of urine. Specify the most credible urgent state.

a. *Acute kidney insufficiencyb. Infectiously-toxic shockc. Dehydratation shockd. Head cerebral edemae. Acute respiratory insufficiency

701. A patient 42, animal technician, fell ill sharply: chill, temperature to 40°, head pain, megalgias, is in muscles, especially gastrocnemius. Objectively: an icterus, hemorragic rash, is expressed on a body, a liver and spleen are enlarged, positive symptom of Pasternasky. Put a preliminary diagnosis.

a. Malariab. Ku-feverc. *Leptospirosisd. Spotted fever

Page 322: intranet.tdmu.edu.uaintranet.tdmu.edu.ua/data/kafedra/internal/infect_desease... · Web viewBest method to treat diarrhoea in child is: intra venous fluide *ORS antibiotics bowel

e. Fever of Ebola702. Man 45 years, works as on a stock-raising farm a veterinary. Delivered in an

induction centre with complaints of discoloration urine, diminishing of diuresis, fervescence to 39,3 °C, pain in muscles. It is ill during 5 days: the temperature of body rose suddenly, great pains appeared in the muscles of shins, head pain, rash on a skin, hemorrhage in a sclerotica, red color of urine. Pulse 56 per 1 mines, BP 90/60 mm Hg. Uranalysis: squirrel of a 0,99 gramme/l, eras. 25-30 cylinders hyalin 8-10. Urea of blood 20,5 mmol/l. What diagnosis most credible?

a. *Leptospirosisb. Acute glomerulonephritisc. Urolithiasisd. Cancer of urinary bladdere. Rheumatoceils

703. Sick, milkmaid diseased sharply, appeared great pain heads, broken, weakness, profuse sweats, loss of appetite, dry cough, insomnia, myalgias, pain, in a lumbar areA. A temperature from the first days rose to 39-40 °C. At a review it is found out hyperemia of face, injections of sclera vessels, hyperemia of mucous of nasopharyngs. Pulse 80 per minute. BP 90/60 mm/Hg. Lymphatic nodes are not palpable. Liver and spleen also not palpable. About what disease is it possible to think?

a. Typhoid feverb. Epidemic typhusc. *Leptospirosisd. Brucellosise. Flu

704. Sick, milkmaid became sick sharply, severe head ache appeared. Weakness, profuse sweats, loss of appetite, dry cough, insomnia, myalgias, pain in a lumbar area also appeared. A temperature from the first days rise to 39-40 °C. During inspection hyperemia of skin, injections of vessels of scleras, hyperemia of mucous membranes found out. Pulse 80 per min. BP is 90/60 mm/Hg. Lymphatic nodes are not palpable. Spleen and liver are enlarged. How is it possible to confirm a credible diagnosis?

a. Virologicalyb. *Bacteriologicallyc. Exposure of exciter in emptyingd. Research of hanging drop of bloode. Research of thick drop of blood

705. Sick, milkmaid diseased sharply, appeared great pain heads, broken, weakness, profuse sweats, loss of appetite, dry cough, insomnia, myalgias, pain, in a lumbar areA. A temperature from the first days rose to 39-40 °C. At a review it is found out hyperemia persons, injections of vessels of scleroticas, hyperemia of mucuse. Pulse 80/min. BP is 90/60 mm/Hg. Lymphatic nodes not palpable. Hepatoslpenomegaly. What etiotropic treatment should be given?

a. Benzylpenicillineb. *Теtracyclinc. Acyclovird. Delagilume. Ftalazol

Page 323: intranet.tdmu.edu.uaintranet.tdmu.edu.ua/data/kafedra/internal/infect_desease... · Web viewBest method to treat diarrhoea in child is: intra venous fluide *ORS antibiotics bowel

706. 32 years sick person, appealed to the doctor on the 7th day of illness with complaints of high temperature, head pain, pain in muscles especially gastrocnemius. Skin and sclera are icteric, on a skin hemorragic rash. HematuriA. 2 weeks ago rode on fishing. Reliable diagnosis?

a. *Leptospirosisb. Trichinosisc. Brucellosisd. Pyelonephritise. Hemorrhagic fever with a kidney syndrome

707. 45 years sick person, appealed to the hospital on the 5th day of illness. 8 days ago he arrived from Laos with complaints of fever, headache, general weakness. Objectively: temperature of body – 40,2 °C, skin moisture, scleroticas, subicteric, acrocyanosis, cardiac tones, deaf, increase of liver and spleen. What complication more frequent in all develops at the severe forms of this illness?

a. Comab. Edema of lungsc. *ID-syndromed. Acute hepatic insufficiencye. Acute kidney insufficiency

708. Patient A., 35 years old, became ill suddenly, complain on the high temperature, severe headache, pain in muscles of legs. Objectively (the 4th day of disease): the state is severe, hyperemia of face, skin and scleras are icterus, spleen and liver are enlarged, oligourhiA. What is the most credible diagnosis?

a. Yersiniosisb. Viral hepatitisc. *Leptospirosisd. Poisoning by a tetraethylleade. Omsk‘s hemorrhagic fever

709. Farmer, 57 years old, appealed to a doctor on the 3rd day of disease with complaints on headache, high temperature, pain in the muscles, icterus of skin, dark urine and decrease quantity of urine. Objectively: temperature – 38,2 °C, injection of scleras, petehial rash on the overhead part of thorax, hepatosplenomegaliA. What is the most reliable previous diagnosis?

a. Brucelosisb. *Leptospirosisc. Viral hepatitisd. Yersiniosise. Trichinosis

710. Chill appeared suddenly at the workwoman of pig farm, a temperature rose to 39,9 °C, there was a headache, nauseA. On the next day appeared pain in the muscles of lower extremities, began the nose bleeding. On the 3rd day of disease, state was severe. A face is hyperemic, scleritis, subicterous of scleras. Liver + 3 сm, diuresis – 700 ml. What is the previous diagnosis?

a. Hemorrhagic fever with a kidney syndromeb. Hepatitis Ac. Yersiniosis

Page 324: intranet.tdmu.edu.uaintranet.tdmu.edu.ua/data/kafedra/internal/infect_desease... · Web viewBest method to treat diarrhoea in child is: intra venous fluide *ORS antibiotics bowel

d. Flue. *Leptospirosis

711. Patient Q., 11 years old, complaints on weakness, headache, high temperature, pain in the muscles; at night suddenly the temperature of body rose to 39,5 °C, decrease level of urine. 4 days ago swimming in a lake, injured a leg. What preparations must be appointed?

a. Prednizolonum, hot foot-bathsb. Febrifugec. Euphyllin, vitamin Cd. *Antibioticse. Sorbents

712. Patient E., 24 years old, was hospitalized in the infectious department in the severe condition. He complains of headache, pain in muscles, joints, vomiting. Objectively: temperature of body 39 °C. BP is 90 and 60 mm Hg, appeared tachycardia, hyperesthesia of skin. Meningeal signs are doubtful. It is known from anamnesis, that he lives in a village, has a pet, rats. What preparations must be given?

a. Veroshpironum, euphilinum, dimedrolb. *Manitol, lazix, prednizolonum, penicillinc. Analgin, dimedrol, acetophend. Manitol, acetophene. Lazix, analgin, ampicillin

713. Patient T., 22 years old, was hospitalized in the infectious department with complaints of chill, fever, severe headache, pain in the gastrocnemius muscles. The state is severe, hyperemia of face, icterus of skin and scleras, liver and spleen are enlarged, olygouriA. What preparations must be given first?

a. Introduction of serumb. *Ftorchinolons preparationsc. Sulphanilamid preparationsd. Dethintoxikation therapye. Sorbents

714. Patient T., 22 years old, was hospitalized in the infectious department with complaints of chill, fever, severe headache, pain in the gastrocnemius muscles. The state is severe, hyperemia of face, icterus of skin and scleras, liver and spleen are enlarged, olygouriA. BP is 60 and 20 mm Hg, pulse – 120 per a min. What preparations must be given?

a. Hormones, dethintoxikation therapy, antibacterial preparationsb. Diuretic, dethintoxikation therapy, antibacterial preparationsc. Sorbents, diuretic, dethintoxikation therapyd. *Hormones diuretic, dethintoxikation therapy, antibacterial preparationse. Hormones, diuretic, antibacterial preparations

715. Patient A., 57 years old, farmer, came to the doctor on the 3rd day of illness with complaints of headache, pain in the gastrocnemius muscles, fever, icterus of skin and scleras, dark urine and decrease level of urine, temperature of body – 38,2 °C, petehial rash on overhead part of thorax, hepatosplenomegaliA. What preparations must be given?

a. Salts solutionsb. Transfusion of fresh-frozen placma

Page 325: intranet.tdmu.edu.uaintranet.tdmu.edu.ua/data/kafedra/internal/infect_desease... · Web viewBest method to treat diarrhoea in child is: intra venous fluide *ORS antibiotics bowel

c. *Antibioticsd. Hyperbaric oxygenatione. Hemosorbcion, sympathomimetics

716. Patient K., 43 years old, miner, on the 4th day of disease complained of weakness, headache, pain in the gastrocnemius muscles, fever, icterus of skin and scleras, dark urine, temperature of body – 38,2 °C, hepatosplenomegaliA. What preparations must be entered? There was the nose-bleed. Diuresis – 200 ml. What preparations must be given?

a. *Specific immunoglobulinb. Holynomimeticsc. Spacmolyticsd. Miorelaxantse. Vitamins of group B

717. At a patient, 39 years old: disease beginning suddenly, t° – 39,5 °C, pain in muscles (especially in gastrocnemius), hepatosplenomegly, icterus, hemorrhagic rash, oligouria, rigidity of the neck muscles. What is the most credible complications?

a. Renal insufficiency, reno-hepatic insufficiency, ІТS, ACVI, meningitisb. Renal insufficiency, hepatic insufficiency, ІТS, ACVI, meningitisc. Renal insufficiency, hepatic insufficiency, ІТS, ACVI, break of spleend. *Renal insufficiency, reno-hepatic insufficiency, ІТS, ACVI, uveite. Renal insufficiency, hepatic insufficiency, ІТS, ACVI, uveit

718. Patient K, 32 years old, of no fixed residence went to see a doctor for 5-days illness with complaints of fever, severe headache, insomniA. On body temperature 40 °C, pulse 110/min. The patient is excited and talkative. Hyperemic, scleritis. At whole over the body there is pink petehial rashes. Positive Govorov-Godele‘s symptom. Enlarged liver and spleen. What do you need to find out the epidemic status?

a. *The existence of head liceb. Use of poor foodc. Availability of parenteral interventionsd. Contact with rodentse. Drinking water from unhygienic sources

719. A patient L., 32 years old, who complains of severe headache and fever, on the 6th-day illness positive agglutination test with rickettsia reaction. What is the vector of the disease?:

a. Fleab. Flyc. Mosquitoesd. Beee. *Lice

720. Patient A., 37 years old, entered to infectious hospital on the third day of the disease in the severe condition. He complains of high fever with chills and sweat, general weakness, pain in right under a rib. Objectively: temperature of body 41 °С, icterus of skin, liver +2 cm, pain at palpation in abdomen, positive symptoms of Ortner and Mussy, a spleen is normal, tachycardiA. What is the previous diagnosis?

a. Malariab. *Cholangitisc. Viral hepatitis

Page 326: intranet.tdmu.edu.uaintranet.tdmu.edu.ua/data/kafedra/internal/infect_desease... · Web viewBest method to treat diarrhoea in child is: intra venous fluide *ORS antibiotics bowel

d. Sepsise. Leptospirosis

721. Patient W., 38 years old, entered to infectious hospital on the 5th days of disease in the severe condition. He complaints of the high fever with chills and sweat, general weakness, pain in sacrum. Objectively: temperature of body of 41 °С, tachycardia, positive symptom of Pasternasky, liver and spleen is not normal. Preliminary diagnosis?

a. Malariab. Cholangitisc. *Pyelonephritisd. Sepsise. Leptospirosis

722. Patient D., 39 years old, entered to infectious hospital on the second week of disease in the severe condition. Local habitant, nowhere arrived. She is complaints for the high fever with chills and sweat, general weakness. Attacks of fever without correct periodicity. Objectively: the temperature of body – 41 °С, subicterus of sclera, tachycardia, Pasrernacky‘s symptom positive, liver + 2 cm, spleen + 2 cm. What is the preliminary diagnosis?

a. Malariab. Cholangitisc. Pyelonephritisd. *Sepsise. Leptospirosis

723. Patient D., 33 years old, came to the infectious hospital on the second week of disease in the severe condition. She is complains of high fever with chills and sweat, general weakness, pharyngalgia at swallowing. Objectively: temperature of body 37,7 °С, hyperemia of mucus of pharynx, tonsils are enlarged, loose, festering raid in lacunas, enlarged submandibular, neck and axillar lymph nodes, icterus of sclera and skin, bradycardia, liver + 2 cm, spleen + 1 cm. Urine is color of beer, an excrement is discolored. What is the preliminary diagnosis?

a. Malariab. Infectious mononucleosisc. Viral hepatitisd. *Sepsise. Leptospirosis

724. Patient K, 32 years old, of no fixed residence went to see a doctor for 5-days illness with complaints of fever, severe headache, insomniA. Body temperature 40 °C, pulse 110/min. The patient is excited and talkative. Hyperemic, scleritis. The body is ocvered pink petehial rashes. Positive Govorova-Godele‘s symptom. Enlarged liver and spleen. What do you need to find out the epidemic status?

a. *The existence of head liceb. Use of poor foodc. Availability of parenteral interventionsd. Contact with rodentse. Drinking water from unhygienic sources

Page 327: intranet.tdmu.edu.uaintranet.tdmu.edu.ua/data/kafedra/internal/infect_desease... · Web viewBest method to treat diarrhoea in child is: intra venous fluide *ORS antibiotics bowel

725. A patient L., 32 years old, who complains of severe headache and fever, on the 6th-day of illness has positive agglutination test with rickettsia reaction. What is the vector of the disease?:

a. Fleab. Flyc. Mosquitoesd. Bee

E. *Lice726. A patient L, 72 years old, who complaints of severe headaches and fever, had 6-

day illness with positive agglutination test with rickettsia reaction. In the past he was sick with typhus. Vector of the disease is:

a. Fleab. Flyc. Mosquitoesd. Licee. *Carrier does not need a vector

727. A patient 29 years old, few days ago, had a chill, the temperature for 2-3 days increased to 39-40 °C. There was hyperemia and edema on the person, significant sclera like "drunk" person and "rabbit" eyes. On the third day of illness – on mucous of soft palate, parenthesis bright red enanthema is seen. In 3-4-days patient tried to protrude the tongue, there was hyper movement, tremor, moving it to the side. Diagnosis: epidemic typhus. Which disease is likely in the patient?

a. *Epidemic typhusb. Typhoid feverc. The disease Brilld. Paratyphoid Ae. Paratyphoid B

728. A patient on the 4th-day of fever has profuse rosy-petechial exanthema predominantly located on the lateral surface of the torso and limbs. Hyperemic, vascular conjunctivitis, petechial transition of anterior fold conjunctivA. What kind of illness do you think it is?

a. Typhoid feverb. *Epidemic typhusc. Measlesd. Hemorrhagic fever with renal syndromee. Crimean hemorrhagic fever

729. Patient B., complaints of headaches, delirium. When inspection: a body temperature of 39 °C, initiated, a person hyperemic, positive symptom Govorova-Godele, petechial rash on the trunk, limbs, tachycardia, hypotension, hepatosplenomegaly. Drunker, drug user, living in the basement room type. What is your preliminary diagnosis?

a. AIDS infectionb. Influenzac. Leptospirosisd. * Epidemic typhuse. Alcohol psychosis

Page 328: intranet.tdmu.edu.uaintranet.tdmu.edu.ua/data/kafedra/internal/infect_desease... · Web viewBest method to treat diarrhoea in child is: intra venous fluide *ORS antibiotics bowel

730. A patient 28 years old, of no fixed abode, hospitalized with a preliminary diagnosis «flu», a 5-day illness appeared rosy-petehia rashes on the body and interior surfaces of the extremities. The temperature of 41 °C, euphoria, hyperemia person, redness sclera, tongue tremor, tachycardia, enlarged spleen; excitement. What is the likely diagnosis?

a. Typhoid feverb. Leptospirosisc. Alcohol deliryd. Measlese. *Epidemic typhus

731. A watchman 42 years old, desperately ill. Entered the 6-day illness with fever 39,7 °C, severe headaches, noise in the ears, insomniA. OBJECTIVE: instituted, euphoric, talkative. Faceis red, his eyes shining, sclera and conjunctiva enhanced vascular pattern. On the inside shoulder and the side surfaces of chest rosy-petechia rash. Found on head lice and nits. Reaction Vidal 1:40. What is diagnosis?

a. Brill disease b. Malariac. *Epidemic typhusd. Typhoid fevere. Q-fever

732. A patient 28 years at the 4th-day of fever has profuse rosy-petechial exanthema predominantly located on the lateral surface of the torso and limbs flexion surfaces. An individual patient hyperemic expressed vessels conjunctival injection, petechial transition to fold conjunctivA. Analysis of urine protein single hyaline and granular cylinders. What kind of illness do you think it is?

a. Typhoid feverb. Hemorrhagic fever with renal syndromec. Crimean hemorrhagic feverd. Measlese. *Epidemic typhus

733. For a patients of 78 years acute illness began, which is characterized by fever with chilling, rash dominated rosy petechia elements. Agent and the source of infection is not detected, but it is known that 45 years ago patient had severe typhus. What is your diagnosis?

a. Yersiniab. *The Brill‘s diseasec. Typhoid feverd. Drug allergye. Food Allergies

734. The patient, a train conductor, 39 years old, hospitalized for a 4-day illness with complaints of headache, weakness, dizziness, sweating, insomnia, fever. Hyperemic, edema, conjunctivitis. At the transition fold conjunctiva – single petehies. At the skin torso, chest, abdomen, limbs abundance rosy-petechia rash. TachycardiA. AD 100 and 60. Tremor of the tongue. Palpable liver, spleen. What is the most likely diagnosis?

a. Leptospirosisb. Typhoid feverc. Influenza

Page 329: intranet.tdmu.edu.uaintranet.tdmu.edu.ua/data/kafedra/internal/infect_desease... · Web viewBest method to treat diarrhoea in child is: intra venous fluide *ORS antibiotics bowel

d. Meningococcemiae. *Epidemic typhus

735. A patient 30 years on 9th day of illness that began gradually, the slow rise of fever and intoxication, a painless rosy rashes on the skin of the abdomen. OBJECTIVE: pale, temperature 40 ° C, pulse 80/min, AD 100 and 65 mm RT. Art. Language is coached, abdomen bubbles, enlarged spleen and liver. For what disease you can suggest in the first place?

a. *Typhoid feverb. Epidemic typhusc. Measlesd. Scarlet fevere. Sepsis

736. Patient 20 years old, complained about the high temperatures of up to 39 ° C, headache in the frontal area, pain in the eyeball, photophobia, pain in muscles, dry cough. Acutely ill the day before. Objective: condition serious, hyperaemic, eyes shining, vascular injection sclerA. Pulse 96/min, rhythmical. Tone heart weakened. In the lungs scattered dry bubbling rale. Faces hyperemic, granular. Meningeal symptoms are not present. Blood tests: Leu. 3?109, eoz. 1 %, yang neu. 6 %, neu. 51 %, lymph. 35 %, mon. 7 %. What is the most likely diagnosis?

a. Measlesb. *Influenzac. Meningococcal diseased. Pneumoniae. Typhus

737. Often, in patient with epidemic typhus arise transition petechias in the conjunctivA. What term is it called?

a. Symptom of Hellerb. Conjunctivitisc. Symptom of Govorov-Godeled. *Symptom of Zorohovich-Chiari-Avtsynae. Exanthema Rosenberg‘s

738. Often, in patient with epidemic typhus arises petechia on mucosal soft palate. What term did it call?

a. Symptom of Hellerb. Conjunctivitisc. Symptom of Govorova-Godeled. Symptom of Zorohovich-Chiarie. *Exanthema Rosenberg‘s

739. Often, in patient with epidemic typhus the tongue is in tremor when protruded it sticks on the lower teeth. What term did it call?

a. Symptom of Hellerb. Conjunctivitisc. *Symptom of Govorova-Godeled. Symptom of Zorohovich-Chiarie. Exanthema Rosenberg‘s

Page 330: intranet.tdmu.edu.uaintranet.tdmu.edu.ua/data/kafedra/internal/infect_desease... · Web viewBest method to treat diarrhoea in child is: intra venous fluide *ORS antibiotics bowel

740. A patient 29 years, a few days ago, a chill, the temperature for 2-3 days increased to 39-40 ° C. There hyperemia and edema person, significant sclera like "drunk" person and "rabbit" eyes. On the third day of illness – on mucous of soft palate, parenthesis bright red exanthema is seen. At 3-4-day patient when trying to protrude the tongue, there was hyper movement, tremor, rejecting it to the side. Diagnosis: epidemic typhus. What is the mechanicm of infection?

a. Air-droppingb. Fecal-oralc. Contact Residentiald. Parenterale. *Vector borne

741. Patient A., 37 years old, entered to infectious hospital on the third days of disease in the severe condition. He complaints of the high fever with chills and sweat, general weakness, pain in right under a rib. Objectively: temperature of body 41 °С, icterus of skin, liver + 2 cm, pain at palpation in abdomen, positive symptoms of Ortner and Mussy, a spleen is normal, tachycardiA. What methods is it possible to confirm a previous diagnosis by?

a. General blood analysis b. *Ultrasonic researchc. Biochemical blood testd. Hem culturee. Parasitoscopy of blood

742. Patient W., 38 years old, entered to infectious hospital on the 5th days of disease in the severe condition. He complaints of the high fever with chills and sweat, general weakness, pain in sacrum. Objectively: temperature of body of 41 °С, tachycardia, and positive symptom of Pasternascky sign, liver and spleen not is normal. What is the previous diagnosis? What methods is it possible to confirm a previous diagnosis by?

a. Global analysis of bloodb. Ultrasonic researchc. Biochemical blood testd. Hem culturee. *Parasitoscopy of blood

743. Student of 22 years, gradually ill: hyperemia and erythema in area of right forearm, which was gradually increased to 10 cm in a diameter, appeared. Erythema has a center and raised bright red edges. The temperature of body is sub febrile, general state satisfactory. 5 days prior to beginning of illness gather mushrooms in the forest. What is our preliminary diagnosis?

a. Erysipelasb. *Lime diseasec. Ebolad. Erysipelase. Herpetic infection

744. Student F., 22 years old, gradually ill: hyperemia and erythema in area of right forearm, which was gradually increased to 10 cm in a diameter, appeared. Erythema has a center and raised bright red edges. The temperature of body sub febrile, general state is

Page 331: intranet.tdmu.edu.uaintranet.tdmu.edu.ua/data/kafedra/internal/infect_desease... · Web viewBest method to treat diarrhoea in child is: intra venous fluide *ORS antibiotics bowel

satisfactory. 5 days prior to beginning of illness walked in the forest after mushrooms. What laboratory methods will confirm the diagnosis?

a. Common analysis of bloodb. Hem culturec. parasitoscopy of bloodd. RMA with leptospirosise. *RIGA with borrelliea

745. Student of 22 years, gradually ill: hyperemia and erythema in area of right forearm, which was gradually increased to 10 cm in a diameter, appeared. Erythema has a center and raised bright red edges. The temperature of body is sub febrile, general state satisfactory. 5 days prior to beginning of illness gather mushrooms in the forest. What is etiotropic therapy?

a. *Tetracyclineb. Metronidazolec. Normal immunoglobulind. Glucocorticoidse. Enter sorbents

746. Patient A., 28 years old, was treated in a neurological department during 2 months concerning neuritis of facial nerve. In anamnesis - 2 years ago was treated due to polyarthritis (right knee and talocrural joints). Lives in a mud flow, likes to gather mushrooms. Presence of ticks does not deny. What is laboratory and instrumental methods of research will allow specifying a diagnosis?

a. Biological test on botulicmb. EEGc. Computer tomography of a braind. RNGA with borrellieae. *Lumbar puncture

747. Patient A., 28 years old, was treated in a neurological department during 2 months concerning neuritis of facial nerve. In anamnesis - 2 years ago was treated due to polyarthritis (right knee and talocrural joints). Lives in a mud flow, likes to gather mushrooms. Presence of ticks does not deny. What is etiotropic therapy?

a. *Benzyl penicillinb. Sulfanilamide’sc. Normal immunoglobulind. Glucocorticoidse. Antiviral drugs

748. Patient B., 38 years old, was treated from myocarditis in a cardiologic department. There are violations of cardiac conductivity on the type of atrioventricular blockade, tachycardiA. The temperature of body is sub febril. Arthritis of right knee-joint. Works as a forest ranger, likes to gather mushrooms and berries. What is treatment?

a. Benzyl penicillinb. Nonsteroid ant inflammatory drugsc. Glucocorticoidsd. Physical therapye. *All above enumerated

Page 332: intranet.tdmu.edu.uaintranet.tdmu.edu.ua/data/kafedra/internal/infect_desease... · Web viewBest method to treat diarrhoea in child is: intra venous fluide *ORS antibiotics bowel

749. Patient P., 35 years old, was treated from myocarditis in a cardiologic department. There are violations of cardiac conductivity on the type of atrioventricular blockade, tachycardiA. The temperature of body is. Arthritis of right knee-joint. Works as a forest ranger, likes to gather mushrooms and berries. What laboratory and instrumental methods of research will allow you to specify the diagnosis?

a. Serologyb. ECGc. X-rays investigationd. *RIGA with borrellieae. Biochemical blood test

750. A man 28 years old was appealed to the policlinic. The day before he found a tick in the surface of his skin. He was in-field 2 days ago. He feels satisfactory. What measures of Lime disease prophylaxis?

a. Treatment of the place of biteb. *Urgent antibiotic therapyc. Using of specific immunoglobulind. Enter sorbentse. Vitamins

751. Citizen A. was a participant in a business geologic expedition. Area is endemic to Lyme disease. How to prevent possible infection?

a. *Individual protecting from ticksb. Urgent antibiotic prophylaxis c. Vaccinationd. Using of specific immunoglobuline. All above enumerated

752. Patient of 52 years fell ill sharply from a chill and head pain, fevers to 40°C. On a 3th day illnesses are nausea and vomiting, dark urine. On a 4th day a temperature went down to 37 °, but an icterus appeared and the amount of urine diminished to 600 ml. What disease is such development of symptoms characteristic for?

a. Hepatitis Ab. Hemorrhagic fever with a kidney syndromec. *Leptospirosisd. Sepsise. Acute glomerulonephritis

753. A patient L, 72 years old, who complaints of severe headache and fever, the 6-day illness positive agglutination, test with rickettsia reaction. Past sick was typhus. Vector of the disease is:

a. Fleab. Flyc. Mosquitoesd. Licee. *Carrier does not need

754. A patient 29 years, a few days ago, a chill, the temperature for 2-3 days increased to 39-40 °C. There hyperemia and edema person, significant sclera like "drunk" person and "rabbit" eyes. On the third day of illness – on mucous of soft palate, parenthesis bright red enantema is seen. At 3-4-day patient when trying to protrude the tongue, there was

Page 333: intranet.tdmu.edu.uaintranet.tdmu.edu.ua/data/kafedra/internal/infect_desease... · Web viewBest method to treat diarrhoea in child is: intra venous fluide *ORS antibiotics bowel

hyper movement, tremor, rejecting it to the side. Diagnosis: epidemic typhus. Which disease is likely in a patient?

a. *Epidemic typhusb. Typhoid feverc. The disease Brilld. Paratyphoid Ae. Paratyphoid B

755. A patient at the 4-day fever has profuse rosy-petechial exanthema predominantly located on the lateral surface of the torso and limbs flexion surfaces. Hyperemic, vascular conjunctivitis, petechial transition of anterior fold conjunctivA. What kind of illness can think?

a. Typhoid feverb. *Typhusc. Measlesd. Hemorrhagic fever with renal syndromee. Crimean hemorrhagic fever

756. Patient B. complaints of headaches, delirium. When inspection: a body temperature of 39 ° C, initiated, a person hyperemic, positive symptom Govorova-Godele, petechial rash on the trunk, limbs, tachycardia, hypotension, hepatosplenomegaly. Drunker, drug user, living in the basement room type. What is your preliminary diagnosis?

a. AIDS infectionb. Influenzac. Leptospirosisd. *Typhuse. Alcohol psychosis

757. A patient 28 years old, of no fixed abode, hospitalized with preliminary diagnosis «flu», a 5-day illness appeared rosy-petechial rashes on the body and interior surfaces of the extremities. The temperature of 41 °C, euphoria, hyperemia person, redness sclera, tongue tremor, tachycardia, enlarged spleen; excitement. What is the likely diagnosis?

a. Typhoid feverb. Leptospirosisc. Alcohol deliriumd. Measlese. *Epidemic typhus

758. A patient 28 years at the 4-day fever has profuse rosy-petechial exanthema predominantly located on the lateral surface of the torso and limbs flexion surfaces. An individual patient hyperemic expressed vessels conjunctiva injection, petechias transition to fold conjunctivA. Analysis of urine protein single hyaline and granular cylinders. What kind of illness can think?

a. Typhoid feverb. Hemorrhagic fever with renal syndromec. Crimean hemorrhagic feverd. Measlese. *Epidemic typhus

759. For patients 78 years of acute illness began, which is characterized by fever with chilling, rash dominated rosy petechial elements. The condition of the patient violated

Page 334: intranet.tdmu.edu.uaintranet.tdmu.edu.ua/data/kafedra/internal/infect_desease... · Web viewBest method to treat diarrhoea in child is: intra venous fluide *ORS antibiotics bowel

moderately. Agent and the source of infection are not detected, but it is known that 45 years ago patient had severe typhus. What is your diagnosis?

a. Lyme diseaseb. * Brill‘s diseasec. Typhoid feverd. Drug allergye. Food Allergies

760. The patient, a train conductor, 39 years old, hospitalized for a 4-day illness with complaints of headache, weakness, dizziness, sweating, insomnia, fever. Hyperemic, edema, conjunctivitis. At the transition fold conjunctiva – single petechias. At the skin torso, chest, abdomen, limbs abundance rosy-petechial rash. TachycardiA. AD 100 and 60. Tremor of the tough. Palpable liver, spleen. Stool arrested. What is the most likely diagnosis?

a. Leptospirosisb. Typhoid feverc. Influenzad. Meningococcemiae. *Epidemic typhus

761. A patient 30 years on 9th day of illness that began gradually, the slow rise of fever and intoxication, a painless rosy rashes on the skin of the abdomen. OBJECTIVE: pale, temperature 40 ° C, pulse 80/min, AD 100 and 65 mm RT. Art. Language is coached, abdomen bubbles, enlarged spleen and liver. For what disease you can suggest in the first place?

a. *Typhoid feverb. Epidemic typhusc. Measlesd. Scarlet fevere. Sepsis

762. Patient 20 years, complained about the high temperatures of up to 39 ° C, headache in the frontal area, pain in the eyeball, photophobia, pain in muscles, dry cough. Acutely ill the day before. Objective: condition serious, hyperemic, eyes shining, vascular injection sclerA. Pulse 96/min, rhythmical. Tone heart weakened. In the lungs scattered dry bubbling rile. Faces hyperemic, granular. Meningeal symptoms are not present. Blood tests: Leu. 3*109, eoz. 1 %, yang neu. 6 %, neu. 51 %, lymph. 35 %, mon. 7 %. What is the most likely diagnosis?

a. Measlesb. *Influenzac. Meningococcal diseased. Pneumoniae. Typhus

763. Patient K., 23 years old., hospitalized on the 3rd day of illness, which was accompanied by mild running nose, high fever to 40,2 °C, headache and hemorrhagic rash on the skin. In the 2 hours after the introduction of penicillin blood pressure dropped to 40 and 10 mm RT. Art. Distal pulse and meningeal signs are not defined. What is the diagnosis in a patient?

a. *Meningococcemia, infectious-toxic shock

Page 335: intranet.tdmu.edu.uaintranet.tdmu.edu.ua/data/kafedra/internal/infect_desease... · Web viewBest method to treat diarrhoea in child is: intra venous fluide *ORS antibiotics bowel

b. Epidemic Typhus, severe coursec. Measles, severe coursed. Scarlet fever, severe coursee. Flu, anaphylactic shock

764. A patient 25 years old, who returned from the Far East, suddenly increased body temperature to 39 °C, a pain in backbone, reddening skin type «hood», single hemorrhages on the skin. After 3 days, along with declining fever, weakness, thirst, decreased diuresis to 300 ml, decreased blood pressure. Much pronounced Pasternatsky‘s symptom. What is the most likely diagnosis?

a. Leptospirosisb. Typhusc. Hemorrhagic fever Crimean-Congod. Acute glomerulonephritise. *Hemorrhagic fever with renal syndrome

765. Male 45 years old, fell ill after 2 weeks after returning from Afghanistan, where six months ago underwent malariA. In return were found lice. Suddenly, there were severe headache, weakness, body temperature 39 ?C with a temporary decrease in the 4th day of illness, followed by general weakness, much intoxication, headache, appeared on the body abundance polymorphic rash. At the 7-day state of heavy, the phenomenon of encephalitis, excitation, hallucinations periodically. Objective – erythematos-petechial exanthemA. The temperature of the body 40,1 °C, and pulse was 136 for 1 min, blood pressure 120/70 mm RT art. Moderately enlarged liver and spleen. In the blood analys neutrofil mild leukocytosis, eozinofilia, accelerated ESR. Which of the studies of blood will be positive?

a. At typhoid feverb. At the three-day malariac. Tropical malariad. *At epidemic typhus e. At brucellosis

766. Male 26 years old, who last month returned from Africa and was processing about lice, sick 5 days. Home was a sudden: severe headache, weakness, body temperature 38,8 °C, which persisted all day at a constant level, but at the 4th day of the disease declined for several hours. Then the patient condition has deteriorated significantly, a rash on the body, delay urine. OBJECTIVE: pulse was 110 for 1 min, blood pressure 115/70 mm RT. Art., temperature 39,7 °C. Face red, a significant injection of vascular sclera, and cmall hemorrhages in the conjunctiva, mucous membrane soft palate, and abundant rosy-petechial exanthema on the body. Signs interstitial pneumonia, encephalitis. Enlarged liver and spleen. What kind of illness can think?

a. *Epidemic typhusb. Typhoid feverc. Tropical malariad. Leptospirosise. Yellow fever

767. Patient P., 68 years old, fell ill suddenly 7 days ago from the increase of temperature to 39,3 °C, appearances of headache and insomniA. Objectively: excited, inadequate. Face is hyperemic. The tongue is very dry, trembles at pulling out. On a trunk

Page 336: intranet.tdmu.edu.uaintranet.tdmu.edu.ua/data/kafedra/internal/infect_desease... · Web viewBest method to treat diarrhoea in child is: intra venous fluide *ORS antibiotics bowel

are polymorphic rash, tachycardia, and blood pressure low. Hepatosplenomegaly. Stool is absent. In age 10 years had epidemic typhus. What is the previous diagnosis?

a. Yersiniosisb. Typhoid feverc. *Brill‘s diseased. meningococcemiae. Sepsis

768. Patient P., 76 years old, complied during 7 days for the permanent increase of temperature to 38,2-38,7 °C, headache, insomnia, horrific dreams, dry cough, myalgia’s, arthalgias. Polymorphic rash appeared on a trunk on the 4th day of diseases. In childhood was ill by epidemic fever, three-day malariA. Temperature of body – 38,4 °C, pulse – 98 per a min, bilateral pneumonia confirmed roentgenologic. In the analysis of blood moderate neutrophil leucocytosis. What is the previous diagnosis?

a. *Brill‘s diseaseb. Typhoid feverc. Lime‘s diseased. Malariae. Leptospirosis

769. Patients I., 78 years old, became ill saddenly, the temperature of body rose to 39,2 °C, euphoric, vessels of scleras are injection, hyperemia of face, appears enantema of Rozenberg. Reaction with Rickettsia prowazekii 1:160, IGG – 87 %. What is your diagnosis?

a. *Brill‘s diseaseb. Typhoid feverc. Meningococcal infectiond. Flue. Leptospirosis

770. Patients D., 30 years old, became ill saddenly, when the temperature of body rose to 40,2 °C, appeared headache, weakness, euphoria, injection of vascular sclera, hyperemia of face, appears the positive symptom of Govorov-Godele. Reaction with Rickettsia prowazekii 1:160, IgG – 87 %. What is your diagnosis?

a. Brill‘s diseaseb. Typhoid feverc. Paratyphoid Ad. Paratyphoid Be. *Epidemic typhus

771. A patient with a pediculosis the temperature of body rose to 41,6 °C, appeared headache, euphoria, in 4th days from the beginning of illness – rose-petechial rash. Reaction with Rickettsia prowazekii 1:640, IgG – 89 %. What is your diagnosis?

a. Flub. Typhoid feverc. Meningococcal infectiond. *Epidemic typhus e. Leptospirosis

772. 25 years sick, which got back from Far East, suddenly temperature of body rose up to 39,9 °C, pain appeared, rash as a “jacket”, single hemorrhages on a skin. In 3 days

Page 337: intranet.tdmu.edu.uaintranet.tdmu.edu.ua/data/kafedra/internal/infect_desease... · Web viewBest method to treat diarrhoea in child is: intra venous fluide *ORS antibiotics bowel

weakness, thirst, grew together with the decline of fever, diminished to 300 ml diuresis considerably expressed Pasternatsky symptom. What is most credible diagnosis?

a. *Hemorrhagic fever with a kidney syndromeb. Congo-Crimea hemorrhagic feverc. Epidemic typhusd. Glomerulonephritise. Leptospirosis

773. Person 26 years old, month ago got back from Africa and passed treatment on an occasion a pediculosis, are ill 5 days. Beginning was sudden: great head pain, a weakness, 38,8 °C, was saved all days at permanent level, but on the 4th day of illness went down on a few hours. Farther the state of patient became worse considerably, a rash, coughing, appeared on a trunk. Objectively: temperature – 39,7 °C, pulse – 110, BP - 115/70 mm/hg. The skin of person is red, considerable injection of sclera, shallow hemorrhages on a conjunctiva, mucous shell of soft palate, abundant petechial-еrythematos Signs of іnterstitial pneumonia, encephalitis. A liver and spleen is megascopic. What disease is most credible?

a. Dengue feverb. Typhoidc. Tropical malariad. Leptospirosise. *Epidemic typhus

774. Patient B., 45 years old, appealed to the hospital on the fourth day of illness with complaints of a high temperature, head pain, in muscles Two weeks ago a patient arrived from Far East. Objectively: temperature – 39,6 °C, face and neck hyperemic, vessels of the sclera’s and conjunctivitis. On a skin under collar-bones, shoulder-blades, necks, face is petechial rash in the type of chains. Pasternatsky‘s symptom is positive on both sides. What changes is characteristic for this disease in hemo gramme?

a. *Leukocytosisb. neutropeniac. Monocytosisd. ESR is lowe. Eosinophilia

775. Sick patient, 27 age, 3th day of illness, became ill sharply. Grumbles about a chill, head myalgia, pain in muscles and joints, nausea, vomit. Objectively: hyperemia and puffiness of person, neck, thorax, petechial, “rabbit eyes”, photophobiA. Pulse – 110. Hepatomegaly. A patient arrived from Zaire. Your diagnosis?

a. Viral hepatitisb. Fluc. Leptospirosisd. *Yellow fevere. Malaria

776. A patient, 39 years old, became ill sharply 3 days ago after return from JamaicA. Complaints on a high temperature, severe headache, nausea, vomit by “coffee- brown”, pain in the muscles and joints. Objectively: temperature 39,9 °C, face is swollen, red, conjunctivitis, photophobiA. Rashes. Sclerotic, and skin is yellow, petechial rash,

Page 338: intranet.tdmu.edu.uaintranet.tdmu.edu.ua/data/kafedra/internal/infect_desease... · Web viewBest method to treat diarrhoea in child is: intra venous fluide *ORS antibiotics bowel

acrocyanosis. A liver is latge3 сm, painful on palpation. Tachycardia, hypotension. In hemograme are leukopenia, thrombocytopeniA. Describe the most likely diagnosis.

a. *Yellow feverb. Viral hepatitis Bc. Malariad. Leptospirosise. Hemorrhagic fever with a kidney syndrome

777. At two nurses who care for the patients with fever from southern Sudan in a hospital, appeared a fever, headache, pain in muscles, joints, abdomen, general weakness and diarrheA. On the 4th day of the disease maculo-papular rash appeared on a body, on a 5th day was the bloody vomiting, melena, hypotension, and deafness of cardiac tones. The condition was extremely heavy. Put a previous diagnose.

a. Leptospirosisb. Spotted feverc. Dengue feverd. *Ebola-fevere. Malaria

778. A patient 26 years entered permanent establishment in 3 weeks upon termination of reaping. The state is heavy, chronotaraxis and space, high fever to 40 °C, on a skin petechial and еchimosis, on the back linear hemorrhages, sanguifluousness from a nose and gums, there was vomit by “coffee-grounds”. Pasternatsky symptom is sharply positive. At the laboratory inspection: neutrophilic leukocytosis, thrombocytes – 100 thousand in 1 ml, hematocrit – 0,55, day's amount of urine 70 ml, protienuria, еrythrocyturiA. What disease does it follow to suspect?

a. *Hemorrhagic fever with a kidney syndromeb. Leptospirosis, anicteric formc. Salmonellosis, septic formd. Lassa hemorrhagic fevere. Crimean hemorrhagic fever

779. A boy 12 years old is hospitalized in an infectious department. Became ill 7 days ago from appearance of a headache, hyperthermia up to 40,3 °C, pain in muscles, general weakness. In 2 days after normalization of temperature of body the appeared hemorrhagic rash on the skin and mucus’s, nasal bleeding, icterus, hepatospleenomegaly. Three weeks ago got back from rest in South AfricA. There have been repeated mosquito bites. What infectious disease does it follow to suspect above all things?

a. Dengue feverb. *Yellow feverc. Leptospirosisd. Malariae. Q-fever

780. A boy 10 years of age, who lives in Transcarpathia, on the 4th day of fever appeared painless hemorrhagic petechial rushes, sometimes in the form of red and purple stripes with overwhelming localization on a neck, thorax, in the axillary, above collar-bones. On hyperemic mucus of oro-pharing point hemorrhages, bleeding from a nose. In lungs was hard breathing, tones of the heart deaf, bradycardia, swelling and abdomen-ache, enlarged a liver and spleen. Oliguria, proteinuria, hematuria, cylindruriA. In blood

Page 339: intranet.tdmu.edu.uaintranet.tdmu.edu.ua/data/kafedra/internal/infect_desease... · Web viewBest method to treat diarrhoea in child is: intra venous fluide *ORS antibiotics bowel

neutrophilic leukocytosis, thrombocytopenia, accelerated ESR. What diagnose is previous the most reliable.

a. Meningococcal infectionb. *Hemorrhagic feverc. Rheumato cellsd. Leptospirosise. Thrombocytopenic purpura

781. In ambulance delivered a girl 14 years, which became ill 6 days ago from appearance of weakness, increase body temperature to 39,3 °C, enanthema appeared on the skin and mucous and had hemorrhagic character, icterus, nasal bleeding, increases of liver and spleen. 2 weeks ago got back from a festival in Brazil. What infectious pathology should be eliminated first?

a. *Yellow feverb. Q-feverc. Dengue fever d. Malariae. Viral hepatitis

782. A 30-year-old resident of Peru brought to the hospital at the 4th day of illness, there was vomiting with blood, nose bleeding, icteric skin, petechial. In urine revealed erythrocytes protein. Volume of urine decreased. What is the preliminary diagnosis?

a. *Yellow feverb. Dengue Fever c. Malariad. Lassa fevere. Ebola fever

783. Family week ago got back from the trip on AfricA. In a few days at a child 10 years the temperature of body rose to 40 °C, on a next day vomit, diarrhea with the admixtures of mucus, blood. The state gets worse gradually. On a 4th day on the skin of extremities, trunk appeared single hemorrhage, hemorrhage on a soft palate. What credible disease from will you settle tactic of conduct of patient coming?

a. *Hemorrhagic feversb. Grigoriev-Shiga dysenteryc. Leptospirosisd. Meningococcal infectione. Salmonellosis

784. The soldier, who arrived on vacation from Sierra Leone, was taken to hospital in connection with the febrile illness. Suspected Lassa fever. What kind of laboratory indicators is characteristic for this disease?

a. The decline in serum indicators KFK and LDGb. ESR greater after 60 mm/hourc. Neutrophil in liquord. Elevated levels of alkaline phosphatase in the serume. *High proteinuria

785. At patient, who acted to permanent establishment, on clinical epidemiological indexes of Lassa fever is suspected. What from the resulted clinical indexes are not characteristic for this disease?

Page 340: intranet.tdmu.edu.uaintranet.tdmu.edu.ua/data/kafedra/internal/infect_desease... · Web viewBest method to treat diarrhoea in child is: intra venous fluide *ORS antibiotics bowel

a. *Hemolytic icterusb. Generalized lymphadenopathyc. Conjunctivitisd. Ulcerous pharyngitise. Encephalopathy

786. At patient, who acted to permanent establishment, the especially dangerous hemorrhagic fever is suspected. For the selection of exciter can be used all materials, except for?

a. Bloodb. *Puncture from lymph nodesc. Urinesd. Pleura liquide. Swab from nasopharyngeal

787. In a patient who is at the hospital about high fever disease, which arose up suddenly, a diagnosis of Ebola fever. The severity of the state can be attributed to the development of the following emergency conditions, except for:

a. *Acute respiratory insufficiencyb. Infectious-toxic shockc. Hypovolemic shockd. Hemorrhagic shocke. Acute kidneys insufficiency

788. In a serviceman, who acted to permanent establishment, set previous diagnosis of especially dangerous hemorrhagic fever. What from the transferred terms of latent period does follow at determination of duration of primary disease measures?

a. 6 daysb. 10 daysc. 40 daysd. *21 daye. 72 hours

789. At a patient with the hemorrhagic fever with a kidney syndrome a diuresis decreased to 300 ml, the рН of blood is 7,0; increase the level of creatinine and urea on the blood. Intensive therapy is conducted. Which of the funds should not be considered appropriate?

a. *Hypotension drugsb. Heparinc. Introduction of entero sorbent solutiond. Introduction 4 % solution of sodium bicarbonatee. Saluretics

790. At a resident of Colombia on the 3th day of illness was a high temperature, head pain, marked pains in back and extremities appear. At the inspection: face is hyperemic, edema, scleritis. Tongue is assessed by the white coats, pulse frequent. Abdomen is soft, sickly in a еpigastric areA. 7 days ago got back from forests. What is previous diagnosis?

a. Ebola fever b. Dengue feverc. Hemorrhagic fever with a kidney syndromed. Lassa fever

Page 341: intranet.tdmu.edu.uaintranet.tdmu.edu.ua/data/kafedra/internal/infect_desease... · Web viewBest method to treat diarrhoea in child is: intra venous fluide *ORS antibiotics bowel

e. *Yellow fever791. You are flying in an airplane from the Republic of Zaire. Waitress asks to help a

child 12 years old. In inspection – at the torso, limbs, palms and soles are determined multi bubbles. Parents indicate that over 4 days before the child's fever was 40 °C. Rash developed gradually, on the first day of the face, on the second day on the body, on the third day in the limbs. What is the best tactic?

a. *The immediate message to ground air traffic controllers on the case, the suspect cmallpox

b. Reassure parents, said that the state is a manifestation of allergic reactionc. Immediately enter dexamethasoned. Immediately begin antibiotic therapye. Immediately enter vaccine

792. To you, as to the graduating student of medical university, is possibly to work upon termination of studies in CrimeA. What endemic ticks infections is in this territory?

a. *Crimean hemorrhagic fever, tick encephalitis, Q-feverb. Crimean hemorrhagic fever, malaria, epidemic typhusc. Tick encephalitis, Ebola fever, Lassa fever, Crimean hemorrhagic feverd. Q-fever, spotted fever, leptospirosise. Q-fever, Crimean hemorrhagic fever, psittacosis, tick encephalitis

793. Sick 37 years person on Crimea grumbles about a high temperature, non-permanent vomit head pain. On the 4th day of illness: the state is severe, temperature of body 38,9 °C, skin pale, mucous shells are hyperemic. On a soft palate is hemorrhagic enanthema, hemorrhages in sclera reveal. On lateral area of trunk – petechial rash. Pulse is weak, 120/min, BP – 90/50 mm/hg. Abdomen is soft, painless. What from therapeutic facilities not are prescribed in this case?

a. Cyclopheronb. *Biseptolc. Glucocorcoidsd. Suprastinee. Depiridamol

794. Patient 25 years, which got back from Far East, suddenly temperature of body raised to a 39 °C, pain appeared in back, erubescence as “jacket”, single hemorrhage on a skin. In 3 days a weakness, thirst, grew together with the decline of fever, diminished to a 300 ml diuresis, the arteriotony went down. Considerably expressed Pasternasky symptom. What diagnosis is most credible?

a. Typhoid feverb. Crimea-Congo hemorrhagic fever c. *Hemorrhagic fever with a kidneys syndromed. Acute glomerulonephritise. Leptospirosis

795. Patient 39 years, grumbles about the high temperature, sharp pain in a head, back, muscles of extremities, photophobiA. At a review is pallor of nasolabial triangle, hyperemia of person, neck, overhead half of trunk, by the positive Pasternatsky symptom on either side. Eyeing cracks lips, injection of sclera’s. Mucous of mouth bright red with point hemorrhages. On a neck, lateral surfaces of thorax, in arm-pits fossulas, above the

Page 342: intranet.tdmu.edu.uaintranet.tdmu.edu.ua/data/kafedra/internal/infect_desease... · Web viewBest method to treat diarrhoea in child is: intra venous fluide *ORS antibiotics bowel

collar-bones of petechial rushes in the form of asterisks and forms group as red or violet strips. What diagnosis is most credible?

a. Marburg feverb. Congo-Crimean hemorrhagic fever c. *Hemorrhagic fever with a kidney syndrome d. Ebola fevere. Lassa fever

796. Patient complaints of the increasing body temperature up to 40,2 °C, pain in the head, joints and muscles, limbs and back, abdomen, frequent vomits. Patient is excited. Neck, overhead part of breasts are hyperemic, scleratis and conjunctivatis of both eyes. Mucous of oral cavity is hyperemic with punctulate еnanthema, soft palate is filling out. On the 2-4th day of illness on the skin of lateral surface of trunk, and arm-pits areas, on abdomen and extremities appeared petechial rushes oval form with the clear contours of darkly-cherry blossom. Simultaneously mucosal bleeding appeared from gums, nose, lights, digestive channel uterus. Liver is megascopic. Positive Pasternaskiy‘s symptom. What diagnosis is most credible?

a. Yellow feverb. Congo-Crimea hemorrhagic feverc. *Hemorrhagic fever with a kidneys syndromed. Lassa fevere. Ebola fever

797. Sick person 34 years old, became ill suddenly – rose temperatures of body, vomiting. Patient disturbs pain in a head, back, joints, and bones. Hyperemia and puffiness of face and neck, eyes are poured by a blood. Patient is excited. The yellow coloring of skin and sclera’s appeared on the 3th day of illness, point hemorrhages on a skin, liver and spleen was multiplied. What diagnosis is most credible?

a. *Yellow feverb. Congo-Crimea hemorrhagic fever c. Hemorrhagic fever with a kidneys syndrome d. Lassa fevere. Ebola fever

798. Sick C., 33, appealed on the 5th day of illness. In second day the temperature of body was raised to 40 °C, delirium happened. An icterus grew. Vomits was dark-brown color, stool – dark (melena). Petechial and ecchymosis exanthema appeared on the skin of trunk and extremities. There was the nasal and uterine bleeding. Oliguria, blood and cylinders in urine, azotemiA. In the blood test – leukopenia – (1,5-2,0?109), neutropenia, increasing of ESR. Hyperbilirubinemia (due to both fractions of pigment), increasing of activity of aminotransferases, in urine was bilirubin, urobilin. What diagnosis is the most credible?

a. *Yellow feverb. Viral hepatitis Ac. Viral hepatitis Bd. Viral hepatitis hepatic encephalopathye. Crimean hemorrhagic fever

799. Patient, 30 years old, lives in Egypt. Became ill sharply. A temperature with a chill rose to 39 °C and was contained next 3 days. Disturbed headache, severe muscle pain. The

Page 343: intranet.tdmu.edu.uaintranet.tdmu.edu.ua/data/kafedra/internal/infect_desease... · Web viewBest method to treat diarrhoea in child is: intra venous fluide *ORS antibiotics bowel

skin of person is hyperemic, injection of sclera and conjunctiva vessels, hyperemia of nasopharyngs mucous. Pulse – 80/min, lowered blood pressure. Increasing of lymphatic nodes. Phenomena of bronchitis in lungs, cough insignificant, not productive. Petechial rush on the skin of overhead and lower extremities. Bleeding of gums, nose-bleeds. Enlarged liver. How do to confirm a credible diagnosis?

a. Bacteriologicalb. *Virologicc. Research of drop of blood in the dark fieldd. Research of hanging drop of bloode. Research of thick drop of blood

800. The Patient, a hunter, lives in Russia (Ural), arrived to Ukraine 2 weeks ago. Became ill sharply. A temperature rose to 39 °C. Disturbed severe headache, pains in the back, muscles, extremities. There was vomit, photophobiA. Decline of sharpness of sight. Consciousness is some darkened. Hyperemia of face, pallor of nasolabial triangle, injection of sclera’s vessels. Petechial rush pouring out trunk as strips. Pulse 110 per 1 min. Tones of heart is hypostheniA. Tongue is dry, abdomen is blown away. Megascopic liver. Sharply positive Pasternatskiy‘s symptom. OliguriA. In urine: protein, hailing, fibrin cylinders, renal epithelium cells. About what disease is it possible to think?

a. hemorrhagic feverb. Yellow feverc. Leptospirosisd. *Hemorrhagic fever with a kidneys syndromee. Crimea hemorrhagic fever

801. Sick, 35 years, habitant of CrimeA. Became ill sharply. With a chill a temperature rose to 39,5 °C. Disturbed pain of head, joints, in muscles, frequent vomits. Face, neck, overhead part of thorax, is hyperemic. Sclerotic, conjunctivitis. Petechial rush over the trunk. The state got worse. Uterine and intestinal nose-bleeds joined. Hyperemia of face changed by a pallor and puffiness. Patient a dynamic, consciousness is darkened. OliguriA. In spite of haematostatic therapy patient died. About what disease is it possible to think?

a. Lassa feverb. Yellow feverc. Q-feverd. *Crimea hemorrhagic fevere. Omsk hemorrhagic fever

802. A man, a resident of Nigeria, became ill sharply. Appeared a chill, repeated vomiting, body temperature had risen to 39,5 °C. A patient is excited. Hyperemia and puffiness was marked of face, expressed sclerotic, hyperemia of mycoses mouth cavity. Pulse 130 per 1 min. The icterus of skin and sclera appeared on a 3th day, hemorrhages on a skin. A liver and spleen is enlarged. After the brief improvement in the 5th day of illness the state became worse sharply. A temperature again rose to 39,9 °C. Raves. An icterus grew sharply. Vomit moderate. Vomiting and feces of black color. Nose bleeding. Petechial enanthemA. Tachycardia changed on bradycardiA. Blood pressure 80/50 mm/hg. OlyguriA.About what disease is it possible to think?

a. Lassa feverb. *Yellow fever

Page 344: intranet.tdmu.edu.uaintranet.tdmu.edu.ua/data/kafedra/internal/infect_desease... · Web viewBest method to treat diarrhoea in child is: intra venous fluide *ORS antibiotics bowel

c. Q-feverd. Crimea hemorrhagic fevere. Omsk hemorrhagic fever

803. A woman, a resident of Nigeria, became ill sharply. Appeared a chill, repeated vomiting, body temperature had risen to 39,7 °C. A patient is excited. Hyperemia and puffiness was marked of face, expressed sclerotic, hyperemia of mycoses mouth cavity. Pulse 130 per 1 min. The icterus of skin and sclera appeared on a 3th day, hemorrhages on a skin. A liver and spleen is enlarged. After the brief improvement in the 5th day of illness the state became worse sharply. A temperature again rose to 39,9 °C. Raves. An icterus grew sharply. Vomit moderate. Vomiting and feces of black color. Nose bleeding. Petechial enanthemA. Tachycardia changed on bradycardiA. Blood pressure 80/50 mm/hg. OliguriA.

a. Viral hepatitis Ab. Leptospirosisc. *Yellow feverd. Crimean fevere. Malaria

804. At workers laboratories, which looked after marmosets which were brought from Uganda, great pain appeared heads, myalgia’s, dryness in a mouth cavity, nausea, vomits, frequent watery emptying, afterwards pains joined in the area of lower ribs which increased at a cough. On the 4th day of disease a hemorrhagic syndrome appeared are hemorrhages in a conjunctiva, sanguifluousness of gums, blood in vomiting. Maculo-papular pouring out on a trunk. Credible diagnosis?

a. Lassa feverb. *Marburg feverc. Leptospirosisd. Ebola fevere. Crimean fever

805. A patient, habitant of Nigeria, grumbles about the increase of temperature, general weakness. Gradually a temperature rose to 38,9 °C, pain of head increased, pains appeared in muscles, nausea, and conjunctivitis. On a 3th day necrotizing ulcer pharyngitis developed. Ulcers appeared on soft palatine tonsils. The groups of lymphatic nodes of neck were multiplied. The state got worse. Abdomen-aches joined, vomit, diarrhea, pain in a thorax, cough. Relative bradycardiA. About what disease is it possible to think?

a. Flub. Typhoid feverc. *Hemorrhagic feverd. Quinsye. Spotted fever

806. Patient 45 years old, approached into the hospital on a 5th day of illness, 8 days ago he arrival from Laos, complaining of fever, headache, general weakness. OBJECTIVE: body temperature is 40,2 °C, skin moist, sclera inject, subichteric, acrocyanosis, cardinal tone dull, the increase of liver and spleen. What does the most often complications develop in malignant forms of this disease?

a. Comab. Lung edema

Page 345: intranet.tdmu.edu.uaintranet.tdmu.edu.ua/data/kafedra/internal/infect_desease... · Web viewBest method to treat diarrhoea in child is: intra venous fluide *ORS antibiotics bowel

c. *IVD syndromed. Acute liver failuree. Acute kidney failure

807. Patient G., complaints on the increased of body temperature to 39-40 °C, a sharp pain in the head, in the back pain, in the muscles of limbs, photophobiA. Objectively: nosolabial triangle was pallor; face, neck, half upper torso was hyperemic, a positive Pasternatskiy‘s symptom on both sides. Gaps narrowed eyes, sclerotic. Mucous membrane of faces bright was red with point hemorrhages. At the neck, thorax side surfaces, in axillary, over collarbone was petechial rash in the form of stars and grouped in the form of red or violet strips. What is the most likely diagnosis?

a. Yellow feverb. Crimean-Congo hemorrhagic fever c. *Hemorrhagic fever with renal syndromed. hemorrhagic fevere. Crimean hemorrhagic fever

808. Patient A., 25 years old, is being treated concerning tetanus. Choose the specific treatment.

a. Antibioticsb. *Immunoglobulinc. Anticonvulsant medicined. Cardiac preparationse. Respiratory analeptics

809. Patient D., 47 years old, veterinary doctor, admitted to infections department on 2nd day of disease in quite severe condition with expressed intoxication, high temperature (40 °C); general weakness, exhausting cough with runny bloody sputum. Objectively: cyanosis of mouth, pulse 128 per minute, mucous membranes of mouth cavity lightly hyperemic, due to significant edema of nasal and pharyngeal mucous present impediment breathing, during lungs auscultation – dry and moist rales, in permutation – areas of consolidations. Deaf heart tones, AP - 85/50 mmHg. From epidemiologic anamnesis revealed, that patient during a week till the onset of disease work with ill animals. Part of the animals died. What is the most possible diagnosis?

a. Community-acquired pneumoniab. *Anthrax, lung formc. Plague, lung formd. Atypical pneumonia (SARS)e. Q-fever

810. In a villager, on back surface of right fist appeared itching papule, in the center of which is present swelling with ichoric content. In next 2 days develops edema of fist and forearm. On 4th day increase of body temperature, in axillaries region of right revealed large painful lymphatic nodule. One day before the beginning of disease, patient had contact with dead calf. What is the most possible diagnosis?

a. Plague, skin-bubonic formb. *Anthrax ulcer, skin formc. Common carbuncled. Tularemia, skin-bubonic forme. Sepsis

Page 346: intranet.tdmu.edu.uaintranet.tdmu.edu.ua/data/kafedra/internal/infect_desease... · Web viewBest method to treat diarrhoea in child is: intra venous fluide *ORS antibiotics bowel

811. To villager an itching papula appeared on the back surface of right hand, in the center of which a bubble with ichor content appeared. During the next two days developed edema of hand and forearm . At 4th day appear body temperature, in right axillary region observe large painful lymph node. Patient examined a dead bull a day ago to beginning of illness. Most credible diagnosis?

a. Plague, skin-bubonic formb. *Anthrax, skin formc. Banal carbuncled. Tularemia, skin-bubonic forme. Sepsis

812. Young woman, a milkmaid, was admitted to the infectious department. Became ill 3 days ago. An itching node appeared on the neck. She continued to work, her state became worse on the eve of hospitalization, body temperature – 38,1?C, edema at neck and upper part of thorax, with serous-hemorrhagic excretions, not painful. No animal disease was registered during the last year in diary form, where she works. What diagnosis is it necessary to suspect?

a. Furuncleb. Tularemiac. Cu-feverd. *Anthraxe. Sap

813. 2 men admitted in the clinic of infectious diseases, attendant of cattle farm. Both complaints on increase of body temperature, headache, and injury of skin of hands like ulcers uncoated by crusty black covering. Became ill 3 days ago. At first at the skin of hands appeared 2-3 red spots, then in the place of spots appear bubbles, bursting with formation of the ulcers covered by a dense black scab. Around the scab appeared inflamed red colored secondary vesicles, all is surrounded by the soft studnevidnim edema but not painful. Patients care for the ill bull 4 days ago to illness. What is the most credible diagnosis?

a. Plagueb. Erysipelasc. Carbuncled. *Anthraxe. Tularemia

814. Men D., 44 years old, received a suspicious letter. A letter must be read, however a recipient fears that his content can be potentially dangerous in regard to anthrax. In this situation, it is necessary:

a. To through it, not unsealingb. *Cover a letter by a moist gauze and iron it on heated till 250°Cc. Burn out a letterd. Deliver it in sanepidemic unite. Send back

815. A patient has the suddenly expressed edema in half face and neck. In the center of edema is the black scab surrounded by cmall vesicles. Paracervicle and submandibular lymph nodes are increased in size. The changes on face are not accompanied by pain.

Page 347: intranet.tdmu.edu.uaintranet.tdmu.edu.ua/data/kafedra/internal/infect_desease... · Web viewBest method to treat diarrhoea in child is: intra venous fluide *ORS antibiotics bowel

During examination, patient has temperature 37,5 °C. What is the most credible diagnosis?

a. *Anthraxb. Plaguec. Tularemiad. Erysipelase. Furuncle

816. Patient A., 43 years old, was admitted in infectious hospital with complaints on a high body temperature and severe headache. He is ill 2 days. By examination observe carbuncle on forearm with the expressed edema around it and insignificant painful. Regional lymphadenitis. It is known from anamnesis that a patient works on a stock-raising farm. About what disease is it necessary to think about?

a. Erisipeliasb. Cancer of skinc. Erysipelasd. *Anthraxe. Eczema

817. A patient W., 40 years old, veterinary, is ill 3 day: insignificant weakness, body temperature – 38 °C, at right forearm – ulcer of diameter 1,5 cm, covered by dark brown scab, surrounded by hyperemia with cmall vesicles. Expressed edema of soft tissue of forearm. Enlarged and sensible lymph nodes on the right elbow and arm pit. On the hands fresh scratches. What is the most credible diagnosis?

a. *Anthraxb. Erysipelasc. Felinesd. Staphylococcal carbunclee. Tularemia, ulcerous-bubonic form

818. Patient A., complaints of redness of skin and edema at the right cheek. During examination: body temperature – 38,7 °C, painful and edematous submandibular lymph nodes on the right side, can clearly differentiate between hyperemia and healthy skin, bubbles with a dark liquid inwardly, painful palpation. Your preliminary diagnosis?

a. *Erysipelas, hemorrhagic formb. Anthrax, skin formc. Herpetic infectiond. Chicken-poxe. Phlegmon of cheek

819. Patient V., appealed to the surgeon with expressed edema of left half of neck. During examination a doctor observe a carbuncle on the lateral surface of neck and a few cmall bubbles near it, filled with a rather yellow liquid. Submandibular and anterior cervical lymph nodes are enlarged and painful. What diseases is the most credible?

a. *Anthraxb. Diphtheriac. edemad. Plaguee. Carbuncle of neck

Page 348: intranet.tdmu.edu.uaintranet.tdmu.edu.ua/data/kafedra/internal/infect_desease... · Web viewBest method to treat diarrhoea in child is: intra venous fluide *ORS antibiotics bowel

820. Patient P., 40 years old, is a farmer. He is ill during 3 days. He complaints of sub febrile temperature, headache. There is a large edema on the left arm. There is a vesicle (0,3x0,5 cm) in the center of it; which contains the serous and bloody liquid, painless, with considerable peripheral erythemA. What is clinical diagnose?

a. Plagueb. Erysipelasc. Carbuncled. Tularemiae. *Anthrax

821. Young woman, a milkmaid, was admitted to the infectious department. Became ill 3 days ago. An itching node appeared on the neck. She continued to work, her state became worse on the eve of hospitalization, body temperature – 38,1 °C, edema at neck and upper part of thorax, with serous-hemorrhagic excretions, not painful. No animal disease was registered during the last year in diary form, where she works.

a. What method is possible to confirm a diagnosis?b. Examination of blood in the dark fieldc. RIFA with anthrax antigend. *Bacteriologic examination of content of ulcere. Bacteriologic examination of blood

822. Patient C. has got significant edema of half of face and neck. In the middle of edema there is a black scab, surrounded by cmall vesicular. There are enlarged submandibular and frontal neck lymph nodes. The changes on face are not accompanied with pain. Temperature of body at examination is 37,5 °C. What is the most credible diagnosis:

a. *Anthraxb. Plaguec. Rabbit-feverd. Erysipelase. Furuncule

823. A teenager was bitten by a neighbor’s dog. This dog is not instilled; it’s on a leash and bit the boy after provocation in the area of hand. What should be done for the prophylaxis of rabies?

a. Elimination of dogb. Inoculations on vital testimoniesc. *Conditional course of inoculationsd. Surgical treatment of the wounde. All answers are correct

824. Woman of 33 years was attacked and bitten by a bat in the area of hand aggressively and bit a woman on her hand. During the laboratory research the diagnosis of rabies was confirmed. What should be done for the urgent prophylactic of the rabies in this case?

a. Human rabies immunoglobulin 18 doses of anti rabies vaccineb. 12 doses of anti rabies vaccinec. *Human rabies immunoglobulin and 6 doses of vaccined. Human rabies immunoglobulin and 21 dose of vaccinee. 2 doses of vaccine

Page 349: intranet.tdmu.edu.uaintranet.tdmu.edu.ua/data/kafedra/internal/infect_desease... · Web viewBest method to treat diarrhoea in child is: intra venous fluide *ORS antibiotics bowel

825. Wild dog bite man for fingers of the left hand. What kind of specific prophylaxis should be conducted for this patient?

a. *Human rabies immunoglobulin 6 doses of vaccineb. 12 doses of vaccinec. Human rabies immunoglobulin and 16 doses of vaccined. Human rabies immunoglobulin and 21 dose of vaccinee. 2 doses of vaccine

826. Patient A., 25 years old, is being treated concerning tetanus. Choose the specific treatment.

a. Antibioticsb. *Immunoglobulin’sc. Anticonvulsant medicined. Cardiac preparationse. Respiratory analeptics

827. During a walk in-field a fox attacked on a child, bit his feet and racemes of hands and disappear. An animal disappeared. What is medical tactic.

a. To process a wound surgicallyb. *To process a wound soapy solution, to enter an immunoprotein and vaccine c. Preventive vaccinationd. To process wounds surgically, to enter an human rabies immunoglobulin and

vaccinee. Preventive human rabies immunoglobulin

828. A 23 years old person, became ill sharply: fever 38.2 °C, moderate diffuse pharyngalgia at swallowing, pain and itching in the right eye. Objectively: tonsillitis, pharyngitis, conjunctivitis. What is previous diagnosis?

a. Adenoviral infection b. Enteroviral infection c. Para influenza d. Flu e. Acute respiratory infection

829. A 25 years old patient, fell ill rapidly, with chills and temperature rose to 39,9, headache appeared in front temporal regions, pain in eyeballs, dull pain in all trunk, closed nose. Dry cough after 2 days of illness, there was nose bleeding. Objectively: hyperemia with sputum, isolated petechial rash and shallow grittiness of soft palate. Difficult breathing in lungs. What is the most possible diagnosis?

a. Leptospirosis b. Adenoviral infection c. Typhoid fever d. Flu e. Epidemic typhus

830. A child 10 years old with temperature 38,0 °C, conjunctivitis, moist cough, hyperemia of the mucous membranes of cheeks and lips. Gums are pallor. What is your diagnosis?

a. Measles b. *Adenoviral infection c. Acute respiratory viral infection

Page 350: intranet.tdmu.edu.uaintranet.tdmu.edu.ua/data/kafedra/internal/infect_desease... · Web viewBest method to treat diarrhoea in child is: intra venous fluide *ORS antibiotics bowel

d. Enteroviral infection e. Infectious mononucleosis

831. A child 3 years old is found in the grave condition – naughty, forced breathing, dry «barking» coughing, voice is hoarse, perioral cyanosis. The third day, temperature of body is sub febrile, mild common cold. In lungs single dry wheezes can be heard. Moderate tachycardiA. For which disease these symptoms are characteristic?

a. Localized diphtheria of oropharynx b. Whooping-cough c. *Para influenza, false croup d. Bronchopneumonia e. Adenoviral infection

832. A child of age 2 years has temperature of body 37.3 °C, cold, hoarse voice “barking cough” appeared suddenly the anxiety, shortness of breath, appeared with participation of auxiliary muscles. Supposed diagnosis?

a. *Para influenza, false croup b. Diphtheria croup c. Allergic laryngitis, croup d. Flu, laryngitis e. Acute exudative pleurisies

833. A patient 14 years old, hospitalized in the infectious department in severe condition with considerable headache mainly in frontal and temporal area, pain in eyeballs, in muscles and joints. Objectively: patient is excited, temperature of the body is 39 °C. Bradycardia changed by tachycardiA. Muscles tonic and colonic cramps. Positive meningeal signs. It is found in epidemic anamnesis, his brother is also sick. What is your diagnosis?

a. *Flu with pneumonia and edema of brain b. Flu, typical course c. Para influenza, false croup d. Respiratory-syncytial infection e. Adenoviral infection, pneumonia

834. A patient 17 years old, became suddenly ill: temperature rose to 40,3 °C. Severe headache, motive excitation, frequent vomiting, tremor of fingers of extremities. Hemorrhagic spots of round form and different sizes, more frequently as stars, mainly on buttocks and trunk. Meningeal signs are positive. What is the most possible diagnosis?

a. Encephalitis b. Flu with a hemorrhagic syndrome c. *Meningococcal infection d. Measles e. Leptospirosis

835. A patient 17 years, 11th class student, were a lot of cases of ARI (acute respiratory infection) have happened, appealed to a doctor in clinic at 3rd day of disease with complaints of chills, general weakness, a moderate sore throat, running nose, swelling of face, watering from eyes. Objective examination: minor palatal hyperemia brackets and tonsillitis, on a background of moderate edema of tissues. Conjunctivitis. During palpation not painful enlarged inframaxillary lymph nodes, and enlarged neck lymph nodes were

Page 351: intranet.tdmu.edu.uaintranet.tdmu.edu.ua/data/kafedra/internal/infect_desease... · Web viewBest method to treat diarrhoea in child is: intra venous fluide *ORS antibiotics bowel

found. Crepitation can’t be found. Liver and spleen moderately increased. What is the most likely diagnosis?

a. Diphtheria b. *Adenoviral infection c. Meningococcal nasopharyngitis d. Influenza e. Infectious mononucleosis

836. A patient 18 years old, with complaints of headache, pharyngalgia, weakness, high temperature. Objectively: all groups of lymph nodes, 1-3 cm in a diameter, dense, elastic, enlarged, hepatospleenomegaly. Blood analysis: leukocytosis, mononuclear – 15 %. What is possible diagnosis?

a. *Infectious mononucleosis b. Adenoviral infection c. Angina d. Diphtheria e. Acute lymphocytosis

837. A patient 20 years old fell ill rapidly with increasing of temperature to 39.9 °C. Complaints of headache in front temporal region, pain in eyeballs, dull ache in whole trunk, closed nose, scrapes in the throat, dry cough. There was nose-bleeding. What diagnosis is most possible?

a. *Influenza b. Adenoviral infection c. Para influenza d. RS-infection e. Enteroviral infection

838. A patient 20 years old, complaints of increasing of temperature up to 39 act, headache in frontal area, pain in eyeballs, photophobia, pain in muscles, dry cough. Became ill sharply. Objectively: severe state. Face is hyperemic, eyes brilliant, injections of sclera’s. Pulse 96/min, rhythmic, tones of heart arehypostheniac. Meningeal symptoms are not present. Blood analysis: leuk 9*109, е 1 %, bands 6 %, seg 51 %, lymp 35 %, mono 7 %. What is the most possible diagnosis?

a. *Influenza b. Adenovirus infection c. Leptospirosis d. Pneumonia e. Epidemic typhus

839. A patient 26 years old, became sick rapidly: temperature 39.5 оC, severe headache, mainly in frontal and temporal areas, pain in muscles and joints. Examined on the 2nd day of illness: state of middle weight, skin is clean. Moderate hyperemia with cyanosis, pulse 120 per min, rhythmic. Heart activity is rhythmic, tones are muffled, in lungs there is vesicular breathing. What is the treatment of this patient?

a. Aspirin b. *Remantadin c. Ampicillin d. Ascorbic acid e. Ribonuclease

Page 352: intranet.tdmu.edu.uaintranet.tdmu.edu.ua/data/kafedra/internal/infect_desease... · Web viewBest method to treat diarrhoea in child is: intra venous fluide *ORS antibiotics bowel

840. A patient 27 years old, entered clinic on the 4th day of illness with a diagnosis ARVI, allergodermiA. Fell ill with the rise of temperature to 38,0 °C, headache, hyperemia of the throat, then barking cough appeared. On the 3rd day rash appeared on the skin and neck. Was treated by aspirin. Objectively: temperature 38.8 °C. Face is puffy, conjunctivitis. On the skin of neck and upper part of chest is an abundant red-papular rash as rings which do not itch. Mucosa of epiglottis is brightly hyperemic. Submandibular and neck lymphadenitis. Liver and spleen were not enlarged. What is your diagnosis?

a. Measles b. *Allergic dermatitis c. Infectious mononucleosis d. German measles e. Scarlet fever

841. A patient 56 years old, the day before felt easy indisposition, insignificant headache, and weakness. Afterwards the increasing of temperature appeared to 38,5 оC with chills, headache increased considerably, mainly in forehead and temples. Skin and conjunctiva is hyperemic, dry, barking cough. Pharynx is hyperemic. On soft palate present grainy granules, placed point hemorrhages. Difficult breathing. What is the most possible diagnosis?

a. Typhoid fever b. Leptospirosis c. Epidemic typhus d. *Flu e. Enteroviral infection

842. A patient becomes sick very fast: chills, increasing of temperature to 40,1 оC, headache in frontal and temporal regions, pain in eyeballs, close nose, dry cough and pain in the chest. The nose bleeding, nausea, double vomits. Objectively: conjunctivitis, hyperemia, edema, hemorrhages in mucous of otopharhynx, tachycardiA. Blood pressure is low. Difficult breathing . What is the most possible diagnosis?

a. Meningococcemia b. Epidemic typhus c. Leptospirosis d. *Flu e. Typhoid fever

843. A patient 56 years old, workwomen of pig farm, on a background chills appeared, the temperature rose to 39,9 °C, headache, and nauseA. The next day marked pains in the muscles of lower extremities appeared, the nose bleeding began. At the receipt, on the 3rd day common state deteriorated. Hyperemic spots, subecteric appeared. Liver +3 cm. daily – diuresis 700 ml. What is the previous diagnosis?

a. Hemorrhagic fever with a kidney syndrome b. Hepatitis A c. Escerichiosis d. Flu e. *Leptospirosis

844. A patient A., 30 years old, on the 4th day of illness a district doctor marked such subjective and objective data: insignificant indisposition, mild headache, hoarseness of voice, itching in throat, breaking dry cough, temperature of the body 37,4 °C. Pulse

Page 353: intranet.tdmu.edu.uaintranet.tdmu.edu.ua/data/kafedra/internal/infect_desease... · Web viewBest method to treat diarrhoea in child is: intra venous fluide *ORS antibiotics bowel

86/min., difficult nasal breathing, insignificant serous excretions from nose. Which acute respiratory infection does the patient carry?

a. Influenza b. *RS-viral infection c. Para influenza d. Adenoviral infection e. Enteroviral infection

845. A patient admitted in the infectious department with diagnosis of acute respiratory viral infection. Became ill suddenly, the disease is accompanied by the increase of temperature of body till 39 °C, by severe headache, mainly in area of frontal, temporal, above eyes, dryness in nose, itching in throat, dry cough, and dull pain in all body. He had bleeding from nose twice at home. Which acute respiratory disease has the patient?

a. Adenoviral infection b. RS-infection c. *Flu d. Para influenza e. Enteroviral infection

846. A patient C., was hospitalized on the 2nd day of illness with complaints of hoarseness of voice, rough barking cough, labored breathing. Objectively: the state is severe, uneasy, pallor, temperature 37.1 °C, BR 30/min., breathing is noisy, can hear from the distance, with participation of auxiliary musculature. Which viruses could cause development of similar state?

a. Rhino virus b. Influenza virus c. Adenovirus d. *Para influenza virus e. Cytomegalovirus

847. A patient caused a doctor home. Age – 75 years. Complaints of a sub febrile temperature, general weakness, pharyngalgia, conjunctivitis. In family a child is ill the acute adenoviral disease. A patient considers itself a patient the second day. At a review are found out the signs of acute blepharoconjunctivitis, pharyngitis. There are megascopic lymphatic knots: neck front and back, arm-pits and inguinal, to 1 cm in a diameter, soft, not is soldered between itself and with surrounding cellulose. A pharynx is hyperemic, tonsils are hypertrophied and hyperemic. In lights of wheezes it is not. Breathing clean. Tones of heart are muffled. BP is 140/80 mm Hg. Ps – 80 per 1 minute. Abdomen soft. A megascopic liver which comes forward on 3 cm below costal arc and spleen are palpated – soft, painless. Choose the most credible diagnosis:

a. *Adenoviral infection b. Flu c. Megablastoma d. Infectious mononucleosis e. Hepatitis A

848. A patient fell ill very rapidly: chills, increase of temperature to 40.1 °C, headache in front temporal regions, pain in eyeballs, close nose, dry cough, and chest pain. Nose bleeding, nausea, vomiting appeared after 4 hours. Objectively: conjunctivitis, hyperemia,

Page 354: intranet.tdmu.edu.uaintranet.tdmu.edu.ua/data/kafedra/internal/infect_desease... · Web viewBest method to treat diarrhoea in child is: intra venous fluide *ORS antibiotics bowel

edema, point hemorrhages in mucus of epiglottis, tachycardiA. Blood pressure is low. Weaken breathing in the lungs. What is the most possible diagnosis?

a. Leptospirosis b. Epidemic typhus c. *Flu d. Meningococcemia e. Enteroviral infection

849. A patient H., 22 years old, with flu was hospitalized into infectious department with the acute worsening of the common state. Consciousness is stored. The patient strangles. Pallor of skin with cyanosis. Respiratory rate 50 per min, AP 80/55 mmHg, pulse 110 per a min, temperature 39.8 оC. During percussion of lungs tympanic sound with dullness in lower quadrant was found. Crackles in the lower-back parts of lungs. What complication of influenza has developed in that patient?

a. Pneumonia b. Edema of lungs c. Edema of brain d. *Infectious-toxic shock e. Meningoencephalitis

850. A patient on the background of ARVI the fever developed to 40,1 °C, frequency of breathing is 40 for a minute. What measures are necessary?

a. *Decreasing of patients temperature b. Artificial ventilation c. Oxygen. inhalation d. Infusion therapy e. Antibiotic therapy

851. A patient P., 14 years old, is hospitalized in the infectious dept. in grave condition. Complaints on headache, mainly in frontal and temporal regions, superciliary arcs, vomiting on severe pain, pain on movement of eyeballs, in muscles, joints. Objectively – a patient is excited, temperature of the body 39 °C. BP 100/60 mmHg. Bradycardia was replaced by tachycardiA. Tonic cramps appeared. Doubtful meningeal signs. From anamnesis it is found that at home his brother has flu. What preparations must be injected?

a. Verospiron, euphyllin, dimedrol b. *Mannitol, paracetamol, prednisolone, euphyllin c. Analgin, dimedrol, aspirin, ampicillin d. Mannitol, aspirin e. Lasix, analgin, ampicillin

852. A patient with flu complicated by pneumonia, during some days there are the displays of infectious-toxic shock of ІІ degree. In BA the level of urea and keratinize increases. What from these preparations is not recommended to enter in such a situation?

a. *Adrenalin b. Prednisolone c. Polyconic solutions d. Dofaminum e. Heparin

853. A patient Т., 45 years old, was hospitalized at the 2nd day of disease. One week ago got back from India (sailor of the distant swimming). Complaints of temperature 41.3

Page 355: intranet.tdmu.edu.uaintranet.tdmu.edu.ua/data/kafedra/internal/infect_desease... · Web viewBest method to treat diarrhoea in child is: intra venous fluide *ORS antibiotics bowel

°C, great headache, shortness of breathing; cough with foamy pink color sputum. Objectively: pale of face, cyanosis of mucous, breath rate 24/min, tachycardiA. Lungs: breathing is hyposthenia, moist wheezes in both lungs, crepitation. What is possible diagnosis?

a. Flu b. Miliary tuberculosis c. *Plaque, pulmonary form d. Leptospirosis e. Sepsis

854. A patient, 20 years old, during few days complaints of pharyngalgias. After supercoiling the state became worse: sudden chills, increase of temperature to 40.6 °C, headache. On the skin of low extremities, trunk and buttocks there are a lot of different sizes hemorrhagic spots, acrocyanosis. Consciousness is preserved. Meningeal signs are absent. What is the previous diagnosis?

a. *Meningococcal infection b. Flu c. Epidemic typhus d. Hemorrhagic fever e. Leptospirosis

855. A student, 18 years old, for 7 days complaints of weakness, hyperthermia to 37.8 °C, mucous excretions from a nose, pharyngalgia at swallowing, pain in eyeballs. Objectively: increased lymph nodes of neck and mandible, lymphadenitis, edema and injection of conjunctiva, hyperemia of mucous of epiglottis, hypertrophy of tonsils. What is the most reliable diagnosis:

a. *Adenoviral infection b. Influenza c. Infectious mononuleosis d. Rhinoviral infection e. Parainfluenza

856. A woman 27 years old, complaints of the general weakness, absence of appetite, coughing, fever up to 37.5 °C for three weeks. Ulcerous illness of abdomen, myocarditis is in anamnesis. What inspection is primarily need to do?

a. Electrocardiography b. Fibrobronchoscopy c. Fibrogastroscopy d. *Fluorography e. Common blood analysis

857. In a patient of 16 years old, the disease began gradually, from the catarrhal syndrome. For 2-3 days the temperature of body increase till 38,5 °C, cold, severe cough with the negligible quantity of mucous sputum, «souring» eyes. Peripheral lymph nodes are soft, painless and some enlarged. Mucous of pharynx is hyperemic, granules on posterior part of pharynx. General state is satisfactory. What is the drug of choice?

a. Ascorbic acid b. Aspirin c. *Deoxyribonucleas d. Remantadin

Page 356: intranet.tdmu.edu.uaintranet.tdmu.edu.ua/data/kafedra/internal/infect_desease... · Web viewBest method to treat diarrhoea in child is: intra venous fluide *ORS antibiotics bowel

e. Aminocapronic acid 858. In girl V., 1 year old, appeared the thump of nose, dry cough, body temperature

rose till 37,5 °C. Next day cough become attack like with the excretion of cmall amount of viscid sputum. Noisy breathing. Sharply expressed expiratory dyspnea, breating rate 40 times/minute. During examination: acrocyanosis and emphysematous thorax, at lungs dissipated dry and single moist rales. Tear of frenulum of tongue. What will be the preliminary diagnosis?

a. *Parainfluenza b. Pneumonia c. Influenza d. Respiratory-syncytial infection e. Whooping-cough

859. A sick person, 45 years old, was hospitalized after 2 days of disease. On Sunday he came back from India (sailor). Complaints of increasing of temperature to 41 оC, severe headache, shortness of breath, cough, with sputum. Objectively: pallor, cyanosis of mucous, tachycardiA. Breathing is weaken, crackles in the lower-back parts of the lungs, crepitation. What is the possible diagnosis?

a. *Flu complicated by pneumonia b. Miliary tuberculosis c. Plague, pulmonary form d. Leptospirosis e. Sepsis

860. A sick woman, 42 years old, complaints of temperature 39.3 °C, headache in the frontal area, pain in the eyeballs, photophobia, pain in muscles, dry cough. Became ill suddenly one day before. Objectively: state is severe. Hyperemia of the face, eyes shinny, injection of scleras. Pulse 96/min., rhythmic. Tones of heart are hypotonic. Both lungs are dissipated. Dry wheezes. Mucosa of epiglottis is hyperemic, grainy, vessels are extended. Meningeal symptoms are not present. Analysis of blood: leuk – 3?109/l, еos – 1 %, band – 6 %, seg – 51 %, lymp – 35 %, mono – 7 %. What is the most possible diagnosis?

a. *Flu b. Measles c. Meningococcal infection d. Pneumonia e. Epidemic typhus

861. In a patient of 16 years old, the disease began gradually, from the catarrhal syndrome. For 2-3 days the temperature of body increase till 38,5 °C, cold, severe cough with the negligible quantity of mucous sputum, «souring» eyes. Peripheral lymph nodes are soft, painless and some enlarged. Mucous of pharynx is hyperemic, granules on posterior part of pharynx. General state is satisfactory. What is the drug of choice?

a. Ascorbic acid b. Aspirin c. *Desoxyribonucleas d. Remantadin e. Aminocapronic acid

862. Patient 22 years old, has increase temperature of body till 37,8 °C. Treated under the supervision of district doctor with a diagnosis of influenzA. On the 5th day of illness

Page 357: intranet.tdmu.edu.uaintranet.tdmu.edu.ua/data/kafedra/internal/infect_desease... · Web viewBest method to treat diarrhoea in child is: intra venous fluide *ORS antibiotics bowel

temperature remained the same; it began difficultly in opening eyes. On examination – edema on face, expressed conjunctivitis with film stratifications. Mucous pharynx is heperemia, on the back wall of gullet considerable graininess. Lymph nodes are enlarged in neck. The general state of patient is satisfactory. This disease is related to cold. What disease you suspect?

a. Leptospirosis b. Infectious mononucleosis c. *Adenoviral infection d. Allergic dermatitis e. Meningococcal infection

863. Patient A., 28 years old, hospitalized with a previous diagnosis of flu. On the 5th day of illness, rash appeared on the trunk and internal surfaces of extremities. Temperature 41.5 °C, hyperemia of sclera, tremor of tongue, tachycardia, spleenomegaly, excitation. What is the most possible diagnosis?

a. Measles b. Meningococcal infection c. Leptospirosis d. *Epidemic typhus e. Typhoid fever

864. Patient B., 20 years old, complaints of severe headache in temples and orbits, dull ache in the trunk, dry cough. Temperature of the body 39.6 °C. Inflammatory changes of mucous membrane of oropharynx. Normal breathing in the lungs. What is the most credible diagnosis?

a. Pneumonia b. Parainfluenza c. Respiratory micoplacma d. *Flu e. Meningococcal infection

865. Patient L., 18 years old is sick with fever till 38 °C which proceeds 5 days. he has moderate dry cough, common cold, badly opens eyes. On examination –edema on face, expressed conjunctivitis with film raids. Mucous of pharynx is hyperemic, posterior wall of pharynx is grainy. Internal organs are without pathology. What form of disease does the described picture correspond to?

a. Viral conjunctivitis b. Allergic dermatitis c. *Adenoviral infection d. Influenza e. Rhinoviral infection

866. Patient M., 11 years old, complaints on general weakness, cough, at night suddenly temperature rose till 39,5 °C, appeared restlessness, barking cough, noisy whistling breathing with drowing in supra- and subclavicular cavities, intercostal spaces. He was in contact with the patient acute respiratory viral infection. What should recommend him the first line?

a. *Prednisolon, hot foot-baths b. Seduxsen, euphylin c. Euphylin, vitamin C

Page 358: intranet.tdmu.edu.uaintranet.tdmu.edu.ua/data/kafedra/internal/infect_desease... · Web viewBest method to treat diarrhoea in child is: intra venous fluide *ORS antibiotics bowel

d. Antibiotics, dimedrol e. Astmopen, diazolin

867. Patient P., 14 years old, is hospitalized in the infectious department in the severe condition. Complaints on expressed headache, mainly in frontal and temporal regions, supercilliary arcs, origin of vomiting appear in condition of severe pain, pains by moving the eyeballs, in muscles and joints. Objectively: patient is excited, body temperature-39 ?C. BP-100/60 mmHg. Bradycardia was replaced by tachycardiA. Appeared tonic cramps. Doubtful meningeal signs. From anamnesis it is clear that his brother has flu at home. What will be your diagnosis?

a. Influenza, typical flow b. *Influenza with the phenomena of edema of brain c. Respiratory-syncytial infection d. Parainfluenza e. Adenoviral infection

868. Patient R., 16 years old, hospitalized for 5-day illness with complaints of moderate headache in fronto-temporal region, laid nose, sore throat, pain in the left eye, rise in temperature to 38.1-38.5 °C. General condition is satisfactory. Shortness of nasal breath, mucous discharging from the nose, hyperemia of face, enlargment of the neck and submaxillary lymph glands, left foamy conjunctivitis. What is preliminary diagnosis?

a. Influenza b. Infectious mononucleosis c. Enteroviral infection d. *Adenoviral infection e. parainfluenza

869. Sick M., 22 years old, complaints of increasing of body temperature to 39 оC, headache in frontal area, pain in eyeballs, photophobia, pain in a muscles, dry cough. Became ill suddenly. The state is heavy. Objectively face is hyperemic, injection of scleras. Pulse 96 per min, rhythmic. Tones of heart are hypotonic. In the lungs – dissipated dry wheezes. Mucous membrane of oropharynx is hyperemic, grainy, vessels are extended. menengial symptoms are not present. Analysis of blood: leukocytes 3*109/L, е 1 %, band neut. 6 %, seg. neut 51 %, lymphocytes 35 %, мonocytes 7 %. What is most probable diagnosis?

a. Measles b. *Flu c. Meningococcal disease d. Epidemic typhus e. Pneumonia

870. The patient P., 14 years old, is suffering from flu. He is hospitalized in infectious dept. due to worsening of his condition. He is conscious. A patient is suffocated. Pallor of skin covers with cyanosis, breathing rate 50 times/minute. BP-80/55 mmHg, pulse 110 times /minute. Body temperature-39,5 °C. Excretion of rose foamy sputum. On percussion of lungs there is tympanic sound with dullness in lower part of lung .On auscultation there is moist rales in lower posterior part of lungs. What complication of flu appeared in patient?

a. Bronchitis b. Edema of brain

Page 359: intranet.tdmu.edu.uaintranet.tdmu.edu.ua/data/kafedra/internal/infect_desease... · Web viewBest method to treat diarrhoea in child is: intra venous fluide *ORS antibiotics bowel

c. *Pneumonia d. Edema of lungs e. infectious-toxic shock

871. To the district doctor a patient, complaints of abundant excretions from a nose, moderate headache, hearing loss, perversion of taste. On examination – dry of skin, nose excoriation, in a pharynx – mild hyperemiA. Temperature of body is subfebrile. Pathological changes of internal organs are absent. Which acute respiratory viral infection carries the patient?

a. Adenoviral infection b. Parainfluenza c. *Rhinoviral infection d. RC-infection e. Influenza

872. At a child with the clinical displays of ARVI a generilized lymphadenopathy, one-sided conjunctivitis increase of liver and spleen, is marked. Most reliable diagnosis?

a. Infectious mononucleosisb. Leptospirosisc. *Adenoviral infectiond. Flue. Pseudotuberculosis

873. A patient caused a doctor home. Age – 75 years. Complaints of a subfebrile temperature, general weakness, pharyngalgia, conjunctivitis. In family a child is ill the acute adenoviral disease. A patient considers itself a patient on second day. At a review are found out the signs of acute blepharoconjunctivitis, pharyngitis. There are megascopic lymphatic knots: neck front and back, arm-pits and inguinal, to 1 cm in a diameter, soft, not is soldered between itself and with a surrounding cellulose. A pharynx is hyperemic, tonsills are hypertrophied and hyperemic. In lights of wheezes it is not. Breathing clean. Tones of heart are muffled. BP is 140/80 mm Hg. Ps – 80 per 1 minute. Abdomen soft. A megascopic liver which comes forward on 3 cm below costal arc and spleen are palpated – soft, painless. Choose the most credible diagnosis:

a. *Acute adenoviral infectionb. Fluc. Megacaryoblastomad. Infectious mononucleosise. Hepatitis A

874. Sick M., 22 years old, complaints of increasing of body temperature to 39 оC, headache in frontal area, pain in eyeballs, photophobia, pain in a muscles, dry cough. Became ill suddenly. The state is heavy. Objectively face is hyperemic, injection of scleras. Pulse 96 per min, rhythmic. Tones of heart are hypotonic. In the lungs – dissipated dry wheezes. Mucous membrane of oropharynx is hyperemic, grainy, vessels are extended. menengial symptoms are not present. Analysis of blood: leukocytes 3*109/L, е 1 %, band neut. 6 %, seg. neut 51 %, lymphocytes 35 %, мonocytes 7 %. What is most probable diagnosis?

a. Measlesb. *Fluc. Meningococcal disease

Page 360: intranet.tdmu.edu.uaintranet.tdmu.edu.ua/data/kafedra/internal/infect_desease... · Web viewBest method to treat diarrhoea in child is: intra venous fluide *ORS antibiotics bowel

d. Epidemic typhuse. Pneumonia

875. A patient 14 years old, hospitalized in the infectious department in severe condition with considerable headache mainly in frontal and temporal area, pain in eyeballs, in muscles and joints. Objectively: patient is excited, temperature of the body is 39 оC. Bradycardia changed by tachycardiA. Muscles tonic and clonic cramps. Positive meningeal signs. It is found in epidemic anamnesis, his brother is also sick. What is your diagnosis?

a. *Flu with pneumonia and edema of brainb. Flu, typical coursec. Parainfluenza, false crouped. Respiratory-sencytial infectione. Adenoviral infection, pneumonia

876. A patient H., 22 years old, with flu was hospitalized to infectious department with the acute worsening of the common state. Consiousness is clear. The patient strangles. Pallor of skin with cynosis. Respiratory rate 50 per min, AP 80/55 mmHg, pulse 110 per a min, temperature 39.8 оC. During percussion of lungs tympanic sound with dullness in lower quadrant was found. Crackles in the lower-back parts of lungs. What complication of influenza has developed in that patient?

a. Pneumoniab. *Edema of lungsc. Edema of braind. Infectious-toxic shocke. Meningoencephalitis

877. A sick, 54 years old, hospitalized in infectious department in the grave condition. Complaints are headache, mainly in frontal and temporal areas superciliary arcs, origin of vomiting on peak of pain. Objectively: patient is excited, temperature of body 39 оC, AP 100/60 mm Hg. Bradycardia changed to tachycardiA. Tonic cramps, meningeal signs appeared. From anamnesis it is known that father is also sick. What treatment should be prescribed?

a. *Mannitol, lasix, prednisolone, еuphyllin, suprastainb. Mannitol, acetophenec. Lasix, analgin, ampicillind. Verospiron, euphyllin, demidrole. Aspirin, analgin, demidrol

878. A patient becomes sick very fast: chills, increasing of temperature to 40,1 оC, headache in frontal and temporal regions, pain in eyeballs, close nose, dry cough and pain in the chest. The nose bleeding, nausea, double vomits. Objectively: conjunctivitis, hyperemia, edema, hemorrhages in mucous of otopharhynx, tachycardiA. Blood pressure is low. Difficult breathing . What is the most possible diagnosis?

a. Meningococcemiab. Epidemic typhusc. Leptospirosisd. *Flue. Typhoid fever

Page 361: intranet.tdmu.edu.uaintranet.tdmu.edu.ua/data/kafedra/internal/infect_desease... · Web viewBest method to treat diarrhoea in child is: intra venous fluide *ORS antibiotics bowel

879. A patient 56 years old, the day before he felt easy indisposition, insignificant headache, and weakness. Afterwards the increasing of temperature appeared to 38,5 оC with chills, headache increased considerably, mainly in forehead and temples. Skin and conjunctiva is hyperemic, dry, barking cough. Pharynx is hyperemic. On soft palate present grainy granules, placed point hemorrhages. Difficult breathing. What is the most possible diagnosis?

a. Typhoid feverb. Leptospirosisc. Epidemic typhusd. *Flue. Enteroviral infection

880. A sick person, 45 years old, was hospitalized after 2 days of disease. On Sunday he came back from India (sailor). Complaints of increasing of temperature to 41 оC, severe headache, shortness of breath, cough, with sputum. Objectively: pallor, cyanosis of mucous, tachycardiA. Breathing is weaken, crackles in the lower-back parts of the lungs, crepitation. What is the possible diagnosis?

a. *Flu complicated by pneumoniab. Miliary tuberculosisc. Plague, pulmonary formd. Leptospirosise. Sepsis

881. A 25 years old patient, fell ill rapidly, with chills and temperature rose to 39,9 оC, headache appeared in frontotemporal regions, pain in eyeballs, dull pain in all trunk, closed nose. Dry cough after 2 days of illness, there was nose bleeding. Objectively: hyperemia with sputum, isolated petechial rash and shallow grittiness of soft palate. Difficult breathing in lungs. What is the most possible diagnosis?

a. Leptospirosisb. Adenoviral infectionc. Typhoid feverd. *Flue. Epidemic typhus

882. A patient 26 years old, became sick rapidly: temperature 39.5 оC, severe headache, mainly in frontal and temporal areas, pain in muscles and joints. By examining on the 2nd day of illness: state of middle weight, skin is clean. Moderate hyperemia with cyanosis, pulse 120 per min, rhythmic. Heart activity is rhythmic, tones are muffled, in lungs there is vesicular breathing. What is the treatment of this patient?

a. Aspirinb. *Remantadinc. Ampicillind. Ascorbic acide. Ribonuclease

883. A patient fell ill very rapidly: chills, increase of temperature to 40.1 °C, headache in frontotemporal regions, pain in eyeballs, close nose, dry cough, and chest pain. Nose bleeding, nausea, vomiting appeared after 4 hours. Objectively: conjunctivitis, hyperemia, edema, point hemorrhages in mucus of epiglottis, tachycardiA. Blood pressure is low. Weaken breathing in the lungs. What is the most possible diagnosis?

Page 362: intranet.tdmu.edu.uaintranet.tdmu.edu.ua/data/kafedra/internal/infect_desease... · Web viewBest method to treat diarrhoea in child is: intra venous fluide *ORS antibiotics bowel

a. Leptospirosisb. Epidemic typhusc. *Flud. Мeningococcemiae. Enteroviral infection

884. A student, 18 years old, for 7 days complaints of weakness, hyperthermia to 37.8 °C, mucous excretions from a nose, pharyngalgia at swallowing, pain in eyeballs. Objectively: increased lymph nodes of neck and mandible, lymphadenitis, edema and injection of conjunctiva, hyperemia of mucous of epiglottis, hypertrophy of tonsils. What is the most reliable diagnosis?

a. *Adenoviral infection b. Influenzac. Infectious mononuleosisd. Rhinoviral infectione. Parainfluenza

885. A patient Т., 45 years old, was hospitalized on the 2nd day of disease. One week ago he came back from India (sailor of the distant swimming). Complaints of temperature 41.3 °C, great headache, shortness of breathing; cough with foamy pink colour sputum. Objectively: pale of face, cyanosis of mucous, breath rate 24/min, tachycardiA. Lungs: breathing is hyposthenic, moist wheezes in both lungs, crepitation. What is possible diagnosis?

a. Flub. Miliary tuberculosisc. *Plaque, pulmonary formd. Leptospirosise. Sepsis

886. A patient 20 years old sick rapidly with increasing of temperature to 39.9 °C. complaints of headache in frontotemporal region, pain in eyeballs, dull ache in whole trunk, closed nose, scrapes in the throat, dry cough. There was nose-bleeding. What diagnosis is most possible?

a. *Influenzab. Adenoviral infectionc. Parainfluenzad. RS-infectione. Enteroviral infection

887. 4 years old child complaints of: cough, temperature of body 38.1 °C. Conjunctiva is hyperemic. On mucous of cheeks there are points of hyperemia gum blushs. Weaken breathing in the lungs. What is the most possible diagnosis?

a. Scarlet feverb. Rubellac. *Measlesd. Herpetic infectione. Flu

888. A patient, 20 years old, during few days complaints of pharyngalgias. After supercooling the state became worse: sudden chills, increase of temperature to 40.6 °C, headache. On the skin of low extremities, trunk and buttocks there are a lot of different

Page 363: intranet.tdmu.edu.uaintranet.tdmu.edu.ua/data/kafedra/internal/infect_desease... · Web viewBest method to treat diarrhoea in child is: intra venous fluide *ORS antibiotics bowel

sizes hemorrhagic spots, acrocyanosis. Consiouness is preserved. Meningeal signs are absent. What is the previous diagnosis?

a. *Meningococcal infectionb. Fluc. Epidemic typhusd. Hemorrhagic fevere. Leptospirosis

889. A patient 56 years old, workwomen of pig farm, on a background chills appeared, the temperature rose to 39,9 °C, headache, nauseA. The next day marked pains in the muscles of lower extremities appeared, the nose bleeding began. At the receipt, on the 3rd day common state deteriorated. Hyperemic spots, subicteric appeared. Liver +3 cm. Daily – diuresis 700 ml. What is the previous diagnosis?

a. Hemorrhagic fever with a kidney syndromeb. Hepatitis Ac. Escerichiosisd. Flue. *Leptospirosis

890. A sick 19 years old, sick rapidly, when a temperature rose to 39,2 °C, coughing appeared, closed nose. Pains in muscles and joints. On the 3rd day of disease, shallow spots on the trunk appeared, extremities with hyperemia and edematous feet. Generalized lymphadenopathy, hyperemia of cheeks, enlargement of the liver were found out. What is previous diagnosis?

a. *Pseudotuberculosisb. Fluc. Infectious mononucleosisd. Herpetic infectione. Epidemic typhus

891. A woman who came back from a tour trip, the next day called emergency help. It is known from the anamnesis, that within a week the temperature of body was moderately high. Complaints of bad sleep and bad appetite, pain in the abdomen. During the assescment of the sick it is found out roseolas on the pale skin of breasts and abdomen. Pulse is normal, temperature of body 38,2 °C, hepatospleenomegaly. What is your previous diagnosis?

a. *Typhoid feverb. Epidemic typhusc. Flud. Enteroviral infectione. Leptospirosis

892. A patient C., 25 years old, fell suddenly ill. Every morning severe headache, frequent vomiting, temperature of the body is 39.9 °C. Adopted fatigue, then state got much worse. In the evening lost of consciousness. Expressed muscles pains of back and head. Positive Кеrning’s symptom. Leukocytes – 18,0?109. What is the most reliable diagnosis?

a. Flub. Epidemic typhus, typhus statec. Viral menigoencephalitis

Page 364: intranet.tdmu.edu.uaintranet.tdmu.edu.ua/data/kafedra/internal/infect_desease... · Web viewBest method to treat diarrhoea in child is: intra venous fluide *ORS antibiotics bowel

d. Sepsis, infectious-toxic shocke. *Bacterial menigoencephalitis

893. A patient 17 years old, became suddenly ill: temperature rose to 40,3 °C. Severe headache, motive excitation, frequent vomiting, tremor of fingers of extremities. Hemorrhagic spots of round form and different sizes, more frequently as stars, mainly on buttocks and trunk. Meningeal signs are positive. What is the most possible diagnosis?

a. Encephalitisb. Flu with a hemorrhagic syndromec. *Meningococcal infectiond. Measlese. Leptospirosis

894. Patient B., 20 years old, complaints of severe headache in temples and orbits, dull ache in the trunk, dry cough. Temperature of the body 39.6 °C. Inflammatory changes of mucous membrane of oropharynx. Normal breathing in the lungs. What is the most credible diagnosis?

a. Pneumoniab. Parainfluenzac. Respiratory micoplacmad. *Flue. Meningococcal infection

895. A sick woman, 42 years old, complaints of temperature 39.3 °C, headache in the frontal area, pain in the eyeballs, photophobia, pain in muscles, dry cough. Became ill suddenly one day before. Objectively: state is severe. Hyperemia of the face, eyes shinny, injection of scleras. Pulse 96/min., rhythmic. Tones of heart are hypotonic. Both lungs are dissipated. Dry wheezes. Mucosa of epiglottis is hyperemic, grainy, vessels are extended. Meningeal symptoms are not present. Analysis of blood: leuk – 3?109/l, еos – 1 %, band – 6 %, seg – 51 %, lymp – 35 %, mono – 7 %. What is the most possible diagnosis?

a. *Flub. Measlesc. Meningococcal infectiond. Pneumoniae. Epidemic typhus

896. Patient A., 28 years old, hospitalized with a previous diagnosis of flu. On the 5th day of illness, rash appeared on the trunk and internal surfaces of extremities. Temperature 41.5 °C, hyperemia of sclera, tremor of tongue, tachycardia, spleenomegaly, excitation. What is the most possible diagnosis?

a. Measlesb. Meningococcal infectionc. Leptospirosisd. *Epidemic typhuse. Typhoid

897. A sick explorer of train, 39 years old is hospitalized on the 4th day of illness with complaints of headache, weakness, dizziness, chills, insomnia, fever. The person is hyperemic, conjunctivitis. On the transitional fold of conjunctiva there is a single rash. On the skin of trunk, thorax, abdomen, extremities there are abundant red coloured rashes.

Page 365: intranet.tdmu.edu.uaintranet.tdmu.edu.ua/data/kafedra/internal/infect_desease... · Web viewBest method to treat diarrhoea in child is: intra venous fluide *ORS antibiotics bowel

TachycardiA. AP 100/60 mm of Hg. Tremor of tongue. Liver and spleen were enlarged. Stool fistula is detained. What is the most reliable diagnosis?

a. *Epidemic typhusb. Typhoidc. Flud. Меnigococcemiae. Leptospirosis

898. Sick, 52 years old, with complaints of pain in lumbar region, headache edema of chin. It is known from anamnesis that the sick suffers from obesity of ІІ degree. Recently carried heavy neurological stress and had flu. He has chronic bronchitis for 5 years, chronic gastritis for 8 years. Objectively: Temperature of the body 38.2 °C, AP – 140/90 mm Hg. It is proposed the diagnosis of acute glomerulonephritis. What transferred factors could be the reason of disease?

a. Neuro psycologic stressb. Chronic bronchitisc. Chronic gastritisd. Obesitye. *Flu

899. A patient C., was hospitalized on the 2nd day of illness with complaints of hoarseness of voice, rough barking cough, labored breathing. Objectively: the state is severe, uneasy, pallor, temperature 37.1 °C, BR 30/min., breathing is noisy, can hear from the distance, with participation of auxiliary musculature. Which viruses could cause development of similar state?

a. Rhino virusb. Influenza virusc. Adenovirusd. *Parainfluenza viruse. Cytomegalovirus

900. A sick 70 years old, became ill sharply, the temperature of body rose to 39.2 °C, excited, euphoric, hyperemia of face, Rozenberg’s exanthema appears. Ricketsia titer is 1:160, IgG – 87 %. What is diagnosis?

a. *Epidemic typhusb. Meningococcal infectionc. Epidemic spotted feverd. Flue. Parainfluenza

901. At patient with pediculosis rapidly rise temperature of body up to 41.2 °C, headache, euphoria appeared in 4 days from the beginning of illness. Red colour rash on the lateral thorax and back. Titer of Rickettsia antibodies 1:640, Ig M – 89 %. What is diagnosis?

a. Flub. Enteroviral infectionc. Brill-Zinsser diseased. *Epidemic typhuse. Parainfluenza

Page 366: intranet.tdmu.edu.uaintranet.tdmu.edu.ua/data/kafedra/internal/infect_desease... · Web viewBest method to treat diarrhoea in child is: intra venous fluide *ORS antibiotics bowel

902. A patient 27 years old, entered clinic on the 4th day of illness with a diagnosis ARVI, allergodermiA. Fell ill with the rise of temperature to 38,0 °C, headache, hyperemia of the throat, then barking cough appeared. On the 3rd day rash appeared on the skin and neck. Was treated by aspirin. Objectively: temperature 38.8 °C. Face is puffy, conjunctivitis. On the skin of neck and upper part of chest is abundant red-papular rashes as rings which does not itch. Mucosa of epiglottis is brightly hyperemic. Submandibular and neck lymphadenitis. Liver and spleen were not enlarged. What is your diagnosis?

a. *Measlesb. Allergic dermatitisc. Infectious mononucleosisd. German measlese. Scarlet fever

903. A patient K., 23 years old, with 3 days of moderate illness, with high temperature of body to 40.0 °C, headache and petechial rash on skin, is hospitalized. After introduction of penicillin at 2 o’clock, the BP fell down to 40/10 mm of hg. Peripheral pulse and мeningeal signs does not concerne. What is the diagnosis of the patient?

a. *Меningococcemia, infectious-toxic shockb. ARVI, anaphylactic shockc. Measles, severe coursed. Epidemic typhus, severe coursee. Scarlet fever, severe course

904. A patient, 75 years old, called a doctor at home. Rashes and subfebrile temperature, general weakness, pharyngalgia, conjunctivitis. In family a child is ill with acute adenoviral disease. A patient considers himself ill on the second day. At a review there are signs of pharyngitis. There are enlarged lymphatic nodes: of neck, front and back, armpits and inguinal up to 1 cm in diameter, soft. Pharynx is hyperemic, tonsils are hypertrophy and hyperemic. Both lungs have wheezing sounds. Not clean breathing. Tones of heart are muffled. AP 140/80 mm Hg. Heart rate 80 for 1 minute. Abdomen is soft. Enlarged liver 3 cm below costal arch and spleen is palpable. Palpation is soft, painless. Choose the most possible diagnosis?

a. *Adenoviral infectionb. Fluc. Hepatitis Bd. Infectious mononucleosise. Hepatitis A

905. A patient 18 years old, with complaints of headache, pharyngalgia, weakness, high temperature. Objectively: all groups of lymphonodes, 1-3 cm in a diameter, dense, elastic, enlarged, hepatospleenomegaly. Blood analysis: leukocytosis, mononuclear – 15 %. What is possible diagnosis?

a. *Infectious mononucleosisb. Adenoviral infectionc. Anginad. Diphtheriae. Acute lympoleycosis

Page 367: intranet.tdmu.edu.uaintranet.tdmu.edu.ua/data/kafedra/internal/infect_desease... · Web viewBest method to treat diarrhoea in child is: intra venous fluide *ORS antibiotics bowel

906. A child of age 2 years has temperature of body 37.3 °C, cold, hoarse voice “barking cough” appeared suddenly the anxiety, shortness of breath, appeared with participation of auxiliary muscles. Supposed diagnosis?

a. *Parainfluenza, false croupb. Diphtheria croupc. Allergic laryngitis, croup d. Flu, laryngitise. Acute exudative pleuritis

907. A patient of 5 years old, which treated at home on an occasion of flu by aspirin, calcium gluconatis, on the second day from the beginning of disease “coffee grounds” vomiting appeared, melenA. What complication arises?

a. Neurotoxicosisb. Pneumoniac. *Hemorrhagic syndromed. Infectious-toxice. Bowel obstruction

908. A patient with flu complicated by pneumonia, during some days there are the displays of infectious-toxic shock of ІІ degree. In BA the level of urea and creatinine increases. What from these preparations is not recommended to enter in such a situation?

a. *Adrenalinb. Prednisolonec. Polioniic solutionsd. Dofaminume. Heparin

909. A sick 15 years old, 3rd day of illness. On the background the catarrhal pneumonia, weakness in hands appeared, double vision, cross-eye. Voice is weak. Palatoplegia and extended extremities. Pulse 90/min. AP 130/90 mm Hg .What is your previous diagnosis?

a. Diphtheriab. *Poliomyelitisc. Botulicmd. Epidemic encephalitise. Enteroviral infection

910. A 23 years old person, became ill sharply: fever 38.2 °C, moderate diffuse pharyngalgia at swallowing, pain and itching in the right eye. Objectively: tonsillitis, pharyngitis, conjunctivitis. What is previous diagnosis?

a. *Adenoviral infection b. Enteroviral infectionc. Parainfluenzad. Flue. Acute respiratory infection

911. A patient 52 years old, hospitalized with the severe form of viral hepatitis B. The signs of flu appeared in the department. The indexes of bilirubin rose up and transaminase falls down. What complication can arise in that patient?

a. *Acute hepatic insufficiencyb. Infectious-toxic shockc. Gastric bleeding

Page 368: intranet.tdmu.edu.uaintranet.tdmu.edu.ua/data/kafedra/internal/infect_desease... · Web viewBest method to treat diarrhoea in child is: intra venous fluide *ORS antibiotics bowel

d. Neurotoxicosise. Cerebral comma

912. A child 10 years old with temperature 38,0 °C, conjunctivitis, moist cough, hyperemia of the mucous membranes of cheeks and lips. Gums are pallor. What is your diagnosis?

a. *Measlesb. Adenoviral infection c. Acute respiratory viral infectiond. Enteroviral infection e. Infectious mononucleosis

913. A patient with temperature of body 40.0 °C, nonproductive cough, photophobia, puffiness of face, dots on gums, blushes on the mucus of cheeks your diagnosis?

a. Tuberculosis b. Меningococcemiac. *Measlesd. Enteroviral infectione. Staphylococcal sepsis

914. A patient on the background of ARVI the fever developed to 40,1 °C, frequency of breathing is 40 for a minute. What measures are necessary?

a. *Decreasing of patients temperatureb. Artificial ventillationc. Oxygen. inhalationd. Infusion therapye. Antibioticotherapy

915. A woman 27 years old, complaints of the general weakness, absence of appetite, coughing, fever up to 37.5 °C for three weeks. Ulcerous illness of abdomen, myocarditis is in anamnesis. What inspection is primarily need to do?

a. Electrocardiographyb. Fibrobronchoscopyc. Fibrogastroscopyd. *Fluorographye. Common blood analysis

916. A patient 17 years, 11th class student, were a lot of cases of ARI (acute respiratory infection) have happened, appealed to a doctor in clinic at 3rd day of disease with complaints of chills, general weakness, a moderate sore throat, running nose, swelling of face, watering from eyes Objective examination: minor palatal hyperemia brackets and tonsillitis, on a background of moderate edema of tissues. Conjunctivitis. During palpation not painful enlarged inframaxillary lymph nodes, and enlarged neck lymph nodes were found. Crepitation can’t be find. Liver and spleen moderately increased. What is the most likely diagnosis?

a. Diphtheriab. *Adenoviral infectionc. Meningococcal nasopharyngitisd. Influenzae. Infectious mononucleosis

Page 369: intranet.tdmu.edu.uaintranet.tdmu.edu.ua/data/kafedra/internal/infect_desease... · Web viewBest method to treat diarrhoea in child is: intra venous fluide *ORS antibiotics bowel

917. Patient R., 16 years old, hospitalized for 5-day illness with complaints of moderate headache in fronto-temporal region, laid nose, sore throat, pain in the left eye, rise in temperature to 38.1-38.5 °C. General condition is satisfactory. Shortness of nasal breath, mucous discharging from the nose, hyperemia of face, enlargment of the neck and submaxillary lymph glands, left foamy conjunctivitis. What is preliminary diagnosis?

a. Influenzab. Infectious mononucleosisc. Enteroviral infectiond. *Adenoviral infectione. Influenza

918. Sick M., 22 years old, complaints of increasing of body temperature to 39 оC, headache in frontal area, pain in eyeballs, photophobia, pain in a muscles, dry cough. Became ill suddenly. The state is heavy. Objectively face is hyperemic, injection of scleras. Pulse 96 per min, rhythmic. Tones of heart are hypotonic. In the lungs – dissipated dry wheezes. Mucous membrane of oropharynx is hyperemic, grainy, vessels are extended. menengial symptoms are not present. Analysis of blood: leukocytes 3*109/L, е 1 %, band neut. 6 %, seg. neut 51 %, lymphocytes 35 %, мonocytes 7 %. What is most probable diagnosis?

a. Measlesb. *Fluc. Meningococcal diseased. Epidemic typhuse. Pneumonia

919. A patient 14 years old, hospitalized in the infectious department in severe condition with considerable headache mainly in frontal and temporal area, pain in eyeballs, in muscles and joints. Objectively: patient is excited, temperature of the body is 39 оC. Bradycardia changed by tachycardiA. Muscles tonic and clonic cramps. Positive meningeal signs. It is found in epidemic anamnesis, his brother is also sick. What is your diagnosis?

a. *Flu with pneumonia and edema of brainb. Flu, typical coursec. Parainfluenza, false crouped. Respiratory-sencytial infectione. Adenoviral infection, pneumonia

920. A patient H., 22 years old, with flu was hospitalized into infectious department with the acute worsening of the common state. Consiousness is stored. The patient strangles. Pallor of skin with cynosis. Respiratory rate 50 per min, AP 80/55 mmHg, pulse 110 per a min, temperature 39.8 оC. During percussion of lungs tympanic sound with dullness in lower quadrant was found. Crackles in the lower-back parts of lungs. What complication of influenza has developed in that patient?

a. Pneumoniab. *Edema of lungsc. Edema of braind. Infectious-toxic shocke. Meningoencephalitis

Page 370: intranet.tdmu.edu.uaintranet.tdmu.edu.ua/data/kafedra/internal/infect_desease... · Web viewBest method to treat diarrhoea in child is: intra venous fluide *ORS antibiotics bowel

921. A sick, 54 years old, hospitalized in infectious department in the grave condition. Complaints of expressed headache, mainly in frontal and temporal areas superciliary arcs, origin of vomiting on peak of pain. Objectively: patient is excited, temperature of body 39 оC, AP 100/60 mm Hg. Bradycardia changed to tachycardiA. Tonic cramps, meningeal signs appeared. From anamnesis it is known that father is also sick. What treatment should be prescribed?

a. *Mannitol, lasix, prednisolone, еuphyllin, suprastainb. Mannitol, acetophenec. Lasix, analgin, ampicillind. Verospiron, euphyllin, demidrole. Aspirin, analgin, demidrol

922. A patient becomes sick very fast: chills, increasing of temperature to 40,1 оC, headache in frontal and temporal regions, pain in eyeballs, close nose, dry cough and pain in the chest. The nose bleeding, nausea, double vomits. Objectively: conjunctivitis, hyperemia, edema, hemorrhages in mucous of otopharhynx, tachycardiA. Blood pressure is low. Difficult breathing . What is the most possible diagnosis?

a. Meningococcemiab. Epidemic typhusc. Leptospirosisd. *Flue. Typhoid fever

923. A patient 56 years old, the day before felt easy indisposition, insignificant headache, weakness. Afterwards the increasing of temperature appeared to 38,5 оC with chills, headache increased considerably, mainly in forehead and temples. Skin and conjunctiva is hyperemic, dry, barking cough. Pharynx is hyperemic. On soft palate present grainy granules, placed point hemorrhages. Difficult breathing. What is the most possible diagnosis?

a. Typhoid feverb. Leptospirosisc. Epidemic typhusd. *Flue. Enteroviral infection

924. A sick person, 45 years old, was hospitalized after 2 days of disease. On Sunday he came back from India (sailor). Complaints of increasing of temperature to 41 оC, severe headache, shortness of breath, cough, with sputum. Objectively: pallor, cyanosis of mucous, tachycardiA. Breathing is weaken, crackles in the lower-back parts of the lungs, crepitation. What is the possible diagnosis?

a. *Flu complicated by pneumoniab. Miliary tuberculosisc. Plague, pulmonary formd. Leptospirosise. Sepsis

925. A 25 years old patient, fell ill rapidly, with chills and temperature rose to 39,9 оC, headache appeared in frontotemporal regions, pain in eyeballs, dull pain in all trunk, closed nose. Dry cough after 2 days of illness, there was nose bleeding. Objectively:

Page 371: intranet.tdmu.edu.uaintranet.tdmu.edu.ua/data/kafedra/internal/infect_desease... · Web viewBest method to treat diarrhoea in child is: intra venous fluide *ORS antibiotics bowel

hyperemia with sputum, isolated petechial rash and shallow grittiness of soft palate. Difficult breathing in lungs. What is the most possible diagnosis?

a. Leptospirosisb. Adenoviral infectionc. Typhoid feverd. *Flue. Epidemic typhus

926. A patient 20 years old, complaints of increasing of temperature up to 39 оC, headache in frontal area, pain in eyeballs, photophobia, pain in muscles, dry cough. Became ill sharply. Objectively: severe state. Face is hyperemic, injections of scleras. Pulse 96/min, rhythmic, tones of heart are hyposthenic. Menengial symptoms are not present. Blood analysis: leuk 9?109, е 1 %, bands 6 %, seg 51 %, lymp 35 %, mono 7 %. What is the most possible diagnosis?

a. *Influenzab. Adenovirus infectionc. Leptospirosisd. Pneumoniae. Epidemic typhus

927. A patient 26 years old, became sick rapidly: temperature 39.5 оC, severe headache, mainly in frontal and temporal areas, pain in muscles and joints. Examined on the 2nd day of illness: state of middle weight, skin is clean. Moderate hyperemia with cyanosis, pulse 120 per min, rhythmic. Heart activity is rhythmic, tones are muffled, in lungs there is vesicular breathing. What is the treatment of this patient?

a. Aspirinb. *Remantadinc. Ampicillind. Ascorbic acide. Ribonuclease

928. A patient fell ill very rapidly: chills, increase of temperature to 40.1 °C, headache in frontotemporal regions, pain in eyeballs, close nose, dry cough, and chest pain. Nose bleeding, nausea, vomiting appeared after 4 hours. Objectively: conjunctivitis, hyperemia, edema, point hemorrhages in mucus of epiglottis, tachycardiA. Blood pressure is low. Weaken breathing in the lungs. What is the most possible diagnosis?

a. Leptospirosisb. Epidemic typhusc. *Flud. Мeningococcemiae. Enteroviral infection

929. A student, 18 years old, for 7 days complaints of weakness, hyperthermia to 37.8 °C, mucous excretions from a nose, pharyngalgia at swallowing, pain in eyeballs. Objectively: increased lymph nodes of neck and mandible, lymphadenitis, edema and injection of conjunctiva, hyperemia of mucous of epiglottis, hypertrophy of tonsils. What is the most reliable diagnosis:

a. *Adenoviral infection b. Influenzac. Infectious mononuleosis

Page 372: intranet.tdmu.edu.uaintranet.tdmu.edu.ua/data/kafedra/internal/infect_desease... · Web viewBest method to treat diarrhoea in child is: intra venous fluide *ORS antibiotics bowel

d. Rhinoviral infectione. Parainfluenza

930. A patient Т., 45 years old, was hospitalized at the 2nd day of disease. One week ago got back from India (sailor of the distant swimming). Complaints of temperature 41.3 °C, great headache, shortness of breathing; cough with foamy pink colour sputum. Objectively: pale of face, cyanosis of mucous, breath rate 24/min, tachycardiA. Lungs: breathing is hyposthenic, moist wheezes in both lungs, crepitation. What is possible diagnosis?

a. Flub. Miliary tuberculosisc. *Plaque, pulmonary formd. Leptospirosise. Sepsis

931. A patient 20 years old become sick rapidly with increasing of temperature to 39.9 °C. complaints of headache in frontotemporal region, pain in eyeballs, dull ache in whole trunk, closed nose, scrapes in the throat, dry cough. There was nose-bleeding. What diagnosis is most possible?

a. *Influenzab. Adenoviral infectionc. Parainfluenzad. RS-infectione. Enteroviral infection

932. A 4 years old child complaints of: cough, temperature of body 38.1 °C. Conjunctiva is hyperemic. On mucous of cheeks there are points of hyperemia gum blushs. Weaken breathing in the lungs. What is the most possible diagnosis?

a. Scarlet feverb. Rubellac. *Measlesd. Herpetic infectione. Flu

933. A patient, 20 years old, during few days complaints of pharyngalgias. After supercooling the state became worse: sudden chills, increase of temperature to 40.6 °C, headache. On the skin of low extremities, trunk and buttocks there are a lot of different sizes hemorrhagic spots, acrocyanosis. Consiouness is preserved. Meningeal signs are absent. What is the previous diagnosis?

a. *Meningococcal infectionb. Fluc. Epidemic typhusd. Hemorrhagic fevere. Leptospirosis

934. A patient 56 years old, workwomen of pig farm, on a background chills appeared, the temperature rose to 39,9 °C, headache, nauseA. The next day marked pains in the muscles of lower extremities appeared, the nose bleeding began. At the receipt, on the 3rd day common state deteriorated. Hyperemic spots, subecteric appeared. Liver +3 cm. Daily – diuresis 700 ml. What is the previous diagnosis?

a. Hemorrhagic fever with a kidney syndrome

Page 373: intranet.tdmu.edu.uaintranet.tdmu.edu.ua/data/kafedra/internal/infect_desease... · Web viewBest method to treat diarrhoea in child is: intra venous fluide *ORS antibiotics bowel

b. Hepatitis Ac. Escerichiosisd. Flue. *Leptospirosis

935. A sick 19 years old, fell ill rapidly, when a temperature rose to 39,2 °C, coughing appeared, closed nose. Pains in muscles and joints. On the 3rd day of disease, shallow spots on the trunk appeared, extremities with hyperemia and edematous feet. Generalized lymphadenopathy, hyperemia of cheeks, enlargement of the liver were found out. What is previous diagnosis?

a. *Pseudotuberculosisb. Fluc. Infectious mononucleosisd. Herpetic infectione. Epidemic typhus

936. A woman which came back from a tour trip, the next day she called emergency help. It is known from the anamnesis, that within a week the temperature of body was moderately high. Complaints of bad sleep and bad appetite, pain in the abdomen. During the assescment of the sick it is found out roseolas on the pale skin of breasts and abdomen. Pulse is normal, temperature of body 38,2 °C, hepatospleenomegaly. What is your previous diagnosis?

a. *Typhoid feverb. Epidemic typhusc. Flud. Enteroviral infectione. Leptospirosis

937. A patient C., 25 years old, fell suddenly ill. Every morning severe headache, frequent vomiting, temperature of the body is 39.9 °C. Adopted fatigue, then state got much worse. In the evening lost of consciousness. Expressed muscles pains of back and head. Positive Кеrning’s symptom. Leukocytes – 18,0*109. What is the most reliable diagnosis?

a. Flub. Epidemic typhus, typhus statec. Viral menigoencephalitisd. Sepsis, infectious-toxic shocke. *Bacterial menigoencephalitis

938. A patient 17 years old, became suddenly ill: temperature rose to 40,3 °C. Severe headache, motive excitation, frequent vomiting, tremor of fingers of extremities. Hemorrhagic spots of round form and different sizes, more frequently as stars, mainly on buttocks and trunk. Meningeal signs are positive. What is the most possible diagnosis?

a. Encephalitisb. Flu with a hemorrhagic syndromec. *Meningococcal infectiond. Measlese. Leptospirosis

939. Patient B., 20 years old, complaints of severe headache in temples and orbits, dull ache in the trunk, dry cough. Temperature of the body 39.6 °C. Inflammatory changes of

Page 374: intranet.tdmu.edu.uaintranet.tdmu.edu.ua/data/kafedra/internal/infect_desease... · Web viewBest method to treat diarrhoea in child is: intra venous fluide *ORS antibiotics bowel

mucous membrane of oropharynx. Normal breathing in the lungs. What is the most credible diagnosis?

a. Pneumoniab. Parainfluenzac. Respiratory micoplacmad. *Flue. Meningococcal infection

940. A patient B., 17 years old, student of 11th class, where cases of acute respiratory infection were noted came to the policlinic. On the 3rd day of illness, start with complaints of chills, general weakness, moderate pharyngalgia, cold, edema of face. Objectively: insignificant hyperemia of palatal tonsils. Moderate edema of the face, conjunctivitis. Enlargement of lymphatic nodes. Liver and spleen were moderately enlarged. What is the most possible diagnosis?

a. *Adenoviral infectionb. Diphtheriac. Meningococcal pharyngitisd. Flue. Infectious mononucleosis

941. A sick woman, 42 years old, complaints of temperature 39.3 °C, headache in the frontal area, pain in the eyeballs, photophobia, pain in muscles, dry cough. Became ill suddenly one day before. Objectively: state is severe. Hyperemia of the face, eyes shinny, injection of scleras. Pulse 96/min., rhythmic. Tones of heart are hypotonic. Both lungs are dissipated. Dry wheezes. Mucosa of epiglottis is hyperemic, grainy, vessels are extended. Meningeal symptoms are not present. Analysis of blood: leuk – 3?109/l, еos – 1 %, band – 6 %, seg – 51 %, lymp – 35 %, mono – 7 %. What is the most possible diagnosis?

a. *Flub. Measlesc. Meningococcal infectiond. Pneumoniae. Epidemic typhus

942. Patient A., 28 years old, hospitalized with a previous diagnosis of flu. On the 5th day of illness, rash appeared on the trunk and internal surfaces of extremities. Temperature 41.5 °C, hyperemia of sclera, tremor of tongue, tachycardia, spleenomegaly, excitation. What is the most possible diagnosis?

a. Measlesb. Meningococcal infectionc. Leptospirosisd. *Epidemic typhuse. Typhoid

943. A sick explorer of train, 39 years old is hospitalized on the 4th day of illness with complaints of headache, weakness, dizziness, chills, insomnia, fever. The person is hyperemic, conjunctivitis. On the transitional fold of conjunctiva there is a single rash. On the skin of trunk, thorax, abdomen, extremities there are abundant red coloured rashes. TachycardiA. AP 100/60 mm of Hg. Tremor of tongue. Liver and spleen were enlarged. Stool fistula is detained. What is the most reliable diagnosis?

a. *Epidemic typhus

Page 375: intranet.tdmu.edu.uaintranet.tdmu.edu.ua/data/kafedra/internal/infect_desease... · Web viewBest method to treat diarrhoea in child is: intra venous fluide *ORS antibiotics bowel

b. Typhoidc. Flud. Меnigococcemiae. Leptospirosis

944. Sick, 52 years old, with complaints of pain in lumbar region, headache edema of chin. It is known from anamnesis that the sick suffers from obesity of ІІ degree. Recently carried heavy neurological stress and had flu. He has chronic bronchitis for 5 years, chronic gastritis for 8 years. Objectively: Temperature of the body 38.2 °C, AP – 140/90 mm Hg. It is proposed the diagnosis of acute glomerulonephritis. What transferred factors could be the reason of disease?

a. Neuro psycologic stressb. Chronic bronchitisc. Chronic gastritisd. Obesitye. *Flu

945. A patient C., was hospitalized on the 2nd day of illness with complaints of hoarseness of voice, rough barking cough, labored breathing. Objectively: the state is severe, uneasy, pallor, temperature 37.1 °C, BR 30/min., breathing is noisy, can hear from the distance, with participation of auxiliary musculature. Which viruses could cause development of similar state?

a. Rhino virusb. Influenza virusc. Adenovirusd. *Parainfluenza viruse. Cytomegalovirus

946. A sick 70 years old, became ill sharply, the temperature of body rose to 39.2 °C, excited, euphoric, hyperemia of face, Rozenberg’s exanthema appears. Ricketsia titer is 1:160, IgG – 87 %. What is diagnosis?

a. *Epidemic typhusb. Meningococcal infectionc. Epidemic spotted feverd. Flue. Parainfluenza

947. At patient with pediculosis rapidly rise temperature of body up to 41.2 °C, headache, euphoria appeared in 4 days from the beginning of illness. Red colour rash on the lateral thorax and back. Titer of Rickettsia antibodies 1:640, Ig M – 89 %. What is diagnosis?

a. Flub. Enteroviral infectionc. Brill-Zinsser diseased. *Epidemic typhuse. Parainfluenza

948. A patient 27 years old, entered clinic on the 4th day of illness with a diagnosis ARVI, allergodermiA. Fell ill with the rise of temperature to 38,0 °C, headache, hyperemia of the throat, then barking cough appeared. On the 3rd day rash appeared on the skin and neck. Was treated by aspirin. Objectively: temperature 38.8 °C. Face is puffy,

Page 376: intranet.tdmu.edu.uaintranet.tdmu.edu.ua/data/kafedra/internal/infect_desease... · Web viewBest method to treat diarrhoea in child is: intra venous fluide *ORS antibiotics bowel

conjunctivitis. On the skin of neck and upper part of chest is abundant red-papular rashes as rings which does not itch. Mucosa of epiglottis is brightly hyperemic. Submandibular and neck lymphadenitis. Liver and spleen were not enlarged. What is your diagnosis?

a. *Measlesb. Allergic dermatitisc. Infectious mononucleosisd. German measlese. Scarlet fever

949. A patient K., 23 years old, with 3 days of moderate illness, with high temperature of body to 40.0 °C, headache and petechial rash on skin, is hospitalized. After introduction of penicillin at 2 o’clock, the BP fell down to 40/10 mm Hg. Peripheral pulse and мeningeal signs does not concerne. What is the diagnosis of the patient?

a. *Меningococcemia, infectious-toxic shockb. ARVI, anaphylactic shockc. Measles, severe coursed. Epidemic typhus, severe coursee. Scarlet fever, severe course

950. A patient, 75 years old, called a doctor at home. Rashes and subfebrile temperature, general weakness, pharyngalgia, conjunctivitis. In family a child is ill with acute adenoviral disease. A patient considers himself ill on the second day. At a review there are signs of pharyngitis. There are enlarged lymphatic nodes: of neck, front and back, armpits and inguinal up to 1 cm in diameter, soft. Pharynx is hyperemic, tonsils are hypertrophy and hyperemic. Both lungs have wheezing sounds. Not clean breathing. Tones of heart are muffled. AP 140/80 mm Hg. Heart rate 80 for 1 minute. Abdomen is soft. Enlarged liver 3 cm below costal arch and spleen is palpable. Palpation is soft, painless. Choose the most possible diagnosis:

a. *Adenoviral infectionb. Fluc. Hepatitis Bd. Infectious mononucleosise. Hepatitis A

951. A patient 18 years old, with complaints of headache, pharyngalgia, weakness, high temperature. Objectively: all groups of lymphonodes, 1-3 cm in a diameter, dense, elastic, enlarged, hepatospleenomegaly. Blood analysis: leukocytosis, mononuclear – 15 %. What is possible diagnosis?

a. *Infectious mononucleosisb. Adenoviral infectionc. Anginad. Diphtheriae. Acute lympoleycosis

952. A child of age 2 years has temperature of body 37.3 °C, cold, hoarse voice “barking cough” appeared suddenly the anxiety, shortness of breath, appeared with participation of auxiliary muscles. Supposed diagnosis?

a. *Parainfluenza, false croupb. Diphtheria croupc. Allergic laryngitis, croup

Page 377: intranet.tdmu.edu.uaintranet.tdmu.edu.ua/data/kafedra/internal/infect_desease... · Web viewBest method to treat diarrhoea in child is: intra venous fluide *ORS antibiotics bowel

d. Flu, laryngitise. Acute exudative pleuritis

953. A patient of 5 years old, which treated at home on an occasion of flu by aspirin, calcium gluconatis, on the second day from the beginning of disease “coffee grounds” vomiting appeared, melenA. What complication arises?

a. Neurotoxicosisb. Pneumoniac. *Hemorrhagic syndromed. Infectious-toxice. Bowel obstruction

954. A patient with flu complicated by pneumonia, during some days there are the displays of infectious-toxic shock of ІІ degree. In BA the level of urea and creatinine increases. What from these preparations is not recommended to enter in such a situation?

a. *Adrenalinb. Prednisolonec. Polioniic solutionsd. Dofaminume. Heparin

955. A sick 15 years old, 3rd day of illness. On the background the catarrhal pneumonia, weakness in hands appeared, double vision, cross-eye. Voice is weak. Palatoplegia and extended extremities. Pulse 90/min. AP 130/90 mm Hg .What is your previous diagnosis?

a. Diphtheriab. *Poliomyelitisc. Botulicmd. Epidemic encephalitise. Enteroviral infection

956. A 23 years old person, became ill sharply: fever 38.2 °C, moderate diffuse pharyngalgia at swallowing, pain and itching in the right eye. Objectively: tonsillitis, pharyngitis, conjunctivitis. What is previous diagnosis?

a. *Adenoviral infection b. Enteroviral infectionc. Parainfluenzad. Flue. Acute respiratory infection

957. A patient 52 years old, hospitalized with the severe form of viral hepatitis B. The signs of flu appeared in the department. The indexes of bilirubin rose up and transaminase falls down. What complication can arise in that patient?

a. *Acute hepatic insufficiencyb. Infectious-toxic shockc. Gastric bleeding d. Neurotoxicosise. Cerebral comma

958. A child 10 years old with temperature 38,0 °C, conjunctivitis, moist cough, hyperemia of the mucous membranes of cheeks and lips. Gums are pallor. What is your diagnosis?

a. *Measles

Page 378: intranet.tdmu.edu.uaintranet.tdmu.edu.ua/data/kafedra/internal/infect_desease... · Web viewBest method to treat diarrhoea in child is: intra venous fluide *ORS antibiotics bowel

b. Adenoviral infection c. Acute respiratory viral infectiond. Enteroviral infection e. Infectious mononucleosis

959. A patient with temperature of body 40.0 °C, nonproductive cough, photophobia, puffiness of face, dots on gums, blushes on the mucus of cheeks your diagnosis?

a. Tuberculosis b. Меningococcemiac. *Measlesd. Enteroviral infectione. Staphylococcal sepsis

960. A patient on the background of ARVI the fever developed to 40,1 °C, frequency of breathing is 40 for a minute. What measures are necessary?

a. *Decreasing of patients temperatureb. Artificial ventillationc. Oxygen. inhalationd. Infusion therapye. Antibioticotherapy

961. A woman 27 years old, complaints of the general weakness, absence of appetite, coughing, fever up to 37.5 °C for three weeks. Ulcerous illness of abdomen, myocarditis is in anamnesis. What inspection is primarily need to do?

a. Electrocardiographyb. Fibrobronchoscopyc. Fibrogastroscopyd. *Fluorographye. Common blood analysis

962. A man, 47 years old, became ill suddenly, appeared chills, fever 39,0 °C, head and muscular pain, nausea, vomiting and diarrheA. Cough, shortness of breath, appeared on the 3rd day of illness. Pulse is 68 per min, rhythmic. Breath rate 44/min. Physical examination revealed. Right sided pneumonia in roentgenologic diagnosed. Liver +2,5 cm. It is known that the patient, week prior to illness lived in a hotel with conditioners. What disease does it follow to think about?

a. Bacterial pneumoniab. Psittacosisc. Tuberculosisd. Myocard infarctione. *Legionellosis

963. A 55 years old businescman died from the edema of lungs on the background of severe bilateral pneumonia which is confirmed roentgenologically. A patient 5 days prior to beginning of illness got back from Sweden, where he was in the business trip for 2 weeks. In the hotel, where he lived, he used a swimming pool and conditioner. What etiology of pneumonia most possible in his death?

a. Tubercularb. Micoplacmac. Chlamydiad. *Legionellosis

Page 379: intranet.tdmu.edu.uaintranet.tdmu.edu.ua/data/kafedra/internal/infect_desease... · Web viewBest method to treat diarrhoea in child is: intra venous fluide *ORS antibiotics bowel

e. Pneumococcal964. Among the tourists who lived in a fashionable hotel, where was the outbreake of

respiratory disease in daylight saving time of the year. The illness of three patients have pneumonic form. In the blood after inspection found out with antibodies to L. pneumophila in a titer 1:256 (reaction of indirect immunofluorescence). Antibiotics of what group is the most expedient to appoint in this case?

a. Penicillinb. Aminoglycosidesc. *Macrolidesd. Теtracyclinese. Cephalosporines

965. A sick , 45 years old, is on treatment with a diagnosis of pneumoniA. Gets the antibiotics of penicillin group. Condition does not get better. In blood analysis found growth of titre of protomycoplacma antibodies in 4 times. What from the resulted preparations does follow to appoint for etiotropic treatment of patient?

a. *Теtracyclines, еrythromycinb. Ampicillin, levomycetinc. Acyclovir, zoviraxd. Gentamycin, penicilline. Furazolidon, nifuroksazid

966. A patient 23 years old with pregnancy of 12 weeks. Scars do not appear. She has passed the inspection in a woman consultation. Protomycoplacma antibodies in a titre of 1:10 was found in her analysis. What tactics of gynecologist should be?

a. *To repeat analysis with mycoplacma diagnostics in 14 daysb. To appoint tetracyclinc. To appoint еrythromycind. To cut pregnancy shorte. To repeat analysis with a mycoplacma diagnostic after childbirth

967. In a newborn, the septic state was accompanied with the increase of temperature to 40 oC which developed on 5th day, with pneumonia expressed intoxication: pallor, vomiting, shortness of breath, disturbance, cramps. Appeared rash (blisters with hemorrhages) on skin, mucous membrane of mouth cavity, throat and conjuctivA. The child suffers with congenital herpes. Lungs: difficult breathing and vesicular wheezing. Mild enlargement of liver. What is the most reliable diagnosis?

a. *Herpetic infectionb. Cytomegaloviral infectionc. Chicken poxd. Rubellae. AIDS

968. A new born child on 10th day of life became worse: Temperature 39.2 °C, no frequent vomiting, generalized cramps, violations of consciousness, spastic paresis of left extremities. Month prior to his birth herpes virus was present in the mother, which she did not treated. What disease is most possible?

a. *Herpetic encephalitisb. Meningococcal meningitisc. Subarechnoid hemorrhage

Page 380: intranet.tdmu.edu.uaintranet.tdmu.edu.ua/data/kafedra/internal/infect_desease... · Web viewBest method to treat diarrhoea in child is: intra venous fluide *ORS antibiotics bowel

d. Cerebral abscesse. Violation of blood cerebral circulation

969. A patient has herpetic meningitis. What preparation of specific therapy for viral neuro infection should be given?

a. *Acyclovirb. Cefataximec. Ceftriaxoned. Gentamycine. Furazolidon

970. A patient, 22 years old, became ill sharply. History showed fever up to 38.2 °C with headache, repeated vomiting, olfactory and tastes hallucinations. Quickly got complex of meningeal symptoms, pyramidal paresis. The general epileptic attack and comatose state also developed. Neurolymph is with mixed lymphocytosis, cytochrome, single red corpuscles. What is previous diagnosis?

a. Brain abscessb. Subdural empyemac. *Herpetic encephalitisd. Tumor of braine. Encephalopathy

971. A boy 11 years old, complaints of sickness at mastication, increasing of temperature to 37.1 °C, enlargment of parotid salivary glands . At the age of 8 years carried a paraflu infection. Objectively: in the region of right parotid salivary gland tubular sickly at palpation, a skin above it is not changed. A pharynx is moderate hyperemic, tonsils are not coated. What is your previous diagnosis?

a. *Cytomegaloviral syaloadenitisb. Lymphadenitisc. Parotitisd. Infectious mononucleosise. Cholylithiasis

972. At junior nurse, who works in child’s infectious department, herpes simplex was found. What manager of department must do?

a. *Create a quarantine in the departmentb. To appoint an immunoprotein to the childrenc. Discharge all children from the departmentd. To appoint immunomodulators with a prophylactic purposee. To inspect a junior nurse on a staphylococcus

973. A woman 65 years old had the disease beginning sharply from increase of temperature to 39.0 °C, weakness, and pain in the left part of thorax that increased with breathing motions. On 3rd day of disease vesicular breathing appeared after motion of rib on the left on a hyperemic background. Together with sick a grandchild lives 4 years. What measures of prophylaxis of disease need to be adopted?

a. Vaccinationb. Final disinfection c. Reception of specific immunoproteind. *Isolation of patiente. Acyclovir administration

Page 381: intranet.tdmu.edu.uaintranet.tdmu.edu.ua/data/kafedra/internal/infect_desease... · Web viewBest method to treat diarrhoea in child is: intra venous fluide *ORS antibiotics bowel

974. A patient 60 years old for 2 days has disturbed severe pain in a right arm. On 3rd day appeared blisters, pouring out as a chain on the skin of shoulder, forearm and brush. Sensitiveness in the area of pouring out is mionectic. What disease can be diagnosed?

a. Dermatitisb. *Herpetic ganglionitisc. Neck-pectoral redicalsd. Psoriasise. Allergy

975. The patient, 58 years old, was hospitalised in the infectious department with complaints of pain in the left half of thorax, fever. At a review: temperature of body 37.5 °C, in XI-XII intercostal area the grouped shallow blisters on a hyperemic-filling background are filled by transparent maintenance. Preparation of choice for treatment of this patient is:

a. Suprastinb. Prednisolonec. Biseptolum-480d. Semavine. *Laferon

976. A pregnant woman, 27 years (pregnancy ІІ, 8-10 weeks.), temperature of body increased. At the inspection on a TORCH-infection antibodies are found to the herpes virus, ІІ types of class IGM. What we must recommend to pregnant women?

a. *To cut pregnancyb. To prolong the supervisionc. Treatment with acyclovird. Symptomatic treatmente. Appoint of alpha-fetoprotein

977. A patient appealed to the doctor with complaints of difficulties in opening of the mouth. Two weeks ago fell down and head was hurt, did not have medication. At a review mouth opens on 1.5 cm, moderate expressed pain of muscles at the back of head. The paralysis of muscles of person, eyeballs, are more prominent. In a temporal area dry bloody crusts in the place of traumA. What are your diagnosis?

a. Neuritis of facial nerveb. Throat abscessc. *Facial paralytic stupor of Rоsеd. Bulbar encephalitise. Herpetic ganglionitis of knot of trifacial

978. A patient came with complaints of sickly erosions on his penis. From anamnesis frequent appearance of similar rashes is found out during a year. Objectively: on a balanus are the grouped blisters and erosions, soft on palpation. What is your diagnose?

a. *Recurrent herpes of ІІ typeb. Vulvar pemphigusc. Primary syphyllisd. Shankoform pyodermae. Recurrent herpes of ІІІ type

979. Sick patient, 25 years old, was consulted by a doctor on the third day of illness with complaints of pouring out on lips, wings of nose and ears, pain and swelling in the

Page 382: intranet.tdmu.edu.uaintranet.tdmu.edu.ua/data/kafedra/internal/infect_desease... · Web viewBest method to treat diarrhoea in child is: intra venous fluide *ORS antibiotics bowel

places of these pouring out. Objectively: temperature of body – 37.7 °C, unchanged skin of overhead lip, wings of nose, auricles, there is vesicular scars 1-2 mm with the group location. What disease you may think about?

a. Erysipelas, bullous formb. Anthraxc. *Herpetic infectiond. Anthrax, skin forme. Chicken pox

980. Sick A., complaints of turning red rash and edema on a right cheek. During a review: temperature of body – 38.7 °C, submandibular lymph nodes enlarged and painful, border between turning red and healthy skin is clear, there are blisters with a dark liquid inwardly, palpation is painful. What is your previous diagnosis?

a. *Erysipelas, hemorrhagic formb. Anthrax, skin formc. Herpetic infectiond. Chicken poxe. Phlegmon of cheek

981. A 32 years old patient appealed to a doctor on a background of fever and increase of temperature to 38.0 °C, pain in the right ear area, left side “deviation” appear during the next day . On examination: right side cmoothed out frontal and occipital skinning folds, right eyelids fissure is wider than left, a mouth is overtighten to the left, a right eyebrow does not rise upwards, and a cheek “hyperemia”, xerostomia and violation of taste receptors on front 2/3 right halves of tongue. Herpetic blisters in right external auditory canal and auricle. What is the most possible diagnosis?

a. *Hunt’s syndromeb. Post herpetic neuralgia of the I- and II- branches of right trifacial nervec. Post herpetic neuralgia of all branches of right trifacial nerved. Sludder’s syndromee. Rosolimo-Меlkerson-Rozantalia syndrome

982. A s 40 years old scientist, became ill sharply with chill, temperature of body – 39.8 °C, severe headache, vomiting, pain in muscles. Did not went to the doctor, the state had become worse, hyperemia of sclera, appeared, on lips herpes with hemorrhagic maintenance, the nose-bleeding, skin and sclera, became icteric, urine color is of strong tea, diuresis 200 ml, an anacholia was not present. What is the most reliable source of infection?

a. *Ratsb. Catsc. Infected peopled. Bacillocarriere. Mosquito

983. A patient came with complaints of erosions of his penis. From anamnesis frequent appearance of similar rashes during a year is found out. Objectively: on a balanus are the grouped blisters and erosions of polycyclic outlines, with clear margin, soft during palpation. What is your diagnosis?

a. *Recurrent congenital herpesb. Pemphigus vulgaris

Page 383: intranet.tdmu.edu.uaintranet.tdmu.edu.ua/data/kafedra/internal/infect_desease... · Web viewBest method to treat diarrhoea in child is: intra venous fluide *ORS antibiotics bowel

c. Primary syphillisd. Pyodermiae. Scabies

984. At a 36 years old sick person, 4 days ago a rash appeared on a skin that is accompanied by itching. Swelling of abdomen. Disturbed dull pain in right subcostal area had constipation. Day prior to it he ate the smoked meat. Atypical reactions appeared after the use of tomatoes, strawberry, chocolate. Objectively: on the skin of body, trunk, extremities a lot of rashes. Level of general IgE in normal. What is most possible diagnosis?

a. *Pseudoallergyb. Idiosyncrasyc. Food allergyd. Herpetic infectione. Erysipelas

985. Sick 65 years old patient, complaints of pain in a subscapular region. Objectively: on a skin surface of subscapular region the placed arcwise rose-red filling out hearths some infiltrative, with clear scopes was present. On-the-spot hearths grouped vesicles with transparent maintenance. What is the diagnosis?

a. Impetigob. Herpes simplexc. Erysipelasd. Allergodermiae. *Herpes zoster

986. A sick person, 65 years old, complaints of rash, pain in a subscapular region. Objectively: on a skin surface of the subscapular region present the arcwise rose-red filling out hearths some infiltrative, with clear scopes. On-the-spot hearths grouped vesicles with transparent maintenance. What preparation he should take?

a. Suprastynb. Prednisolonec. Biseptolum-480d. Loratidine. *Laferon

987. A patient N., 45 years old, complaints of headache, general weakness increasing of temperature, to 37.4 °C. In 2 days pain appeared in the pectoral region of spine with an irradiation in a right between scapular regions. After some time skin in this region turned red as a strip from a spine to the subarmpit line, and in 2 days red knots which through the set time grew into blisters with transparent maintenance appeared in this place. What is your diagnosis?

a. *Herpes zosterb. Thoracal rediculopathyc. Neurology of intercostal nervesd. Neurology of superscapular nervee. Herpetiform dermatitis

988. A 37 years old patient, 2 days ago a spot on a hand appeared, which for days grew into pustule with a black bottom, painless at touch, with the crown of daughters vesicles on periphery. There are painless edema on a hand and shoulder. Temperature rise to 39.0

Page 384: intranet.tdmu.edu.uaintranet.tdmu.edu.ua/data/kafedra/internal/infect_desease... · Web viewBest method to treat diarrhoea in child is: intra venous fluide *ORS antibiotics bowel

°C, dizziness appeared. Pulse – 100 beats per min, AP – 95/60 mm Hg. BR – 30 per a minute. What is the most possible diagnosis?

a. *Anthraxb. Plaguec. Tularemiad. Brucellosise. Herpes

989. A 70 years old sick person, after supercooling severy pain in the left half of head in the area of forehead and left eye appeared. 3 days ago the temperature of body increased to 37.6 °C, the blister of pouring out at the head and left overhead eyelid appeared. What disease can be diagnosed?

a. *Herpetic ganglionitisb. Encephalitisc. Allergyd. Dermatitise. Trifacial neuritis

990. A sick 3 years old child came to the doctor with symptoms of the fever, languor, waiver of meal. A boy is capricious, temperature of body 37.9 °C. On the mucus shell of soft palate, cheeks are single vesicle hypersalivation. What is the diagnosis?

a. *Herpetic stomatitisb. Candidosis of oral cavityc. Leucoplaciad. Follicular tonsillitise. Lacunar tonsillitis

991. Sick, 49 years old, came to the doctor with complaints of pain. On the 3rd day he became ill, first marked heartburn and pain in thorax, yesterday is pouring out. At examination: temperature of body 37.8 °C, after motion of the V-VI intercostals intervals on a hyperemic skin group of the blisters filled by transparent maintenance. What is the diagnosis?

a. *Herpes zosterb. Chicken poxc. Erysipelas, erythematous-bulous formd. Allergic dermatitise. Myositis

Page 385: intranet.tdmu.edu.uaintranet.tdmu.edu.ua/data/kafedra/internal/infect_desease... · Web viewBest method to treat diarrhoea in child is: intra venous fluide *ORS antibiotics bowel

Test to figures

1. A mechanism of transmission at this illness is (Fig. 49): A. Air-drop

B. Fecal-oral

C. * Contact

D. Transmissiv

E. Vertical

2. A mechanism of transmission at this illness is (Fig. 40):

A. Air-drop

B. Fecal-oral

C. * Contact

D. Transmissiv

E. Vertical

3. A mechanism of transmission at this illness is (Fig. 46):

A. Air-drop

B. Fecal-oral

C. * Contact

D. Transmissiv

E. Vertical

4. Appoint adequate therapy to this patient (Fig. 61) in blood of which is RNA HCV.

A. * Intron A 3 millions 3 times for a week

B. Intron A 3 millions 5 times for a week

C. Intron A 5 millions 3 times for a week

D. Intron A 5 millions 5 times for a week

E. Intron A 10 millions 3 times for a week

5. At what infectious disease does conduct this symptom (Fig. 58)?

A. Poisoning mushrooms

B. Meningo-encefalit

C. Poliomyelitis

D. Rabies

E. * Tetanus

6. Curent treatment for HIV infection (Fig. 3) consists of highly active artiretroviral therapy. Choose the correct combination of preparations:

Page 386: intranet.tdmu.edu.uaintranet.tdmu.edu.ua/data/kafedra/internal/infect_desease... · Web viewBest method to treat diarrhoea in child is: intra venous fluide *ORS antibiotics bowel

A. 1 NRTIs + 2 IP

B. 1 NRTIs + 1 IP + 2 NNRTI

C. * 3 NRTIs

D. 2 NRTIs + 2 IP

E. 2 NNRTIs + 1 IP

7. Curent treatment for HIV infection (Fig. 3) consists of highly active artiretroviral therapy. Choose the correct combination of preparations:

A. 1 NRTIs + 2 IP

B. 1 NRTIs + 1 IP + 2 NNRTI

C. 2 NRTIs

D. 2 NRTIs + 2 IP

E. * 2 NRTIs + 1 IP

8. Curent treatment for HIV infection (Fig. 3) consists of highly active artiretroviral therapy. Choose the correct combination of preparations:

A. 1 NRTIs + 2 IP

B. 1 NRTIs + 1 IP+ 2 NNRTI

C. * 2 NRTIs + 1 NNRTI

D. 2 NRTIs + 2 IP

E. 2 NNRTIs + 1 IP

9. Curent treatment for HIV infection (Fig. 3) consists of highly active artiretroviral therapy. Choose the correct combination of preparations:

A. 1 NRTIs + 2 IP

B. 1 NRTIs + 1 IP + 2 NNRTI

C. * 3 NRTIs

D. 2 NRTIs + 2 IP

E. 1 NNRTIs + 1 IP + 1 NNRTI

10. ?For what family of viruses does this exciter (Fig. 1) belong to?

A. Paramyxovirus

B. Reovirus

C. Rabdovirus

D. * Retrovirus

E. Herpesvirus

11. For what group does this infectious diseases caused by this exciter (Fig. 1) belong to by L. Gromashevskij classification?

Page 387: intranet.tdmu.edu.uaintranet.tdmu.edu.ua/data/kafedra/internal/infect_desease... · Web viewBest method to treat diarrhoea in child is: intra venous fluide *ORS antibiotics bowel

A. Intestinal infection

B. Infections of respiratory tract

C. Behave to all indicated groups

D. Blood infection

E. * Infection of external covers

12. For what infectious pathology is it characteristically (Fig. 51)?

A. Poisoning mushrooms

B. Meningo-encefalit

C. Poliomyelitis

D. Rabies

E. * Tetanus

13. For what infectious pathology is it characteristically (Fig. 52)?

A. Poisoning mushrooms

B. Encefalomeningit

C. Poliomyelitis

D. Rabies

E. * Tetanus

14. For what infectious pathology is it characteristically (Fig. 53)?

A. Poisoning mushrooms

B. Meningo-encefalit

C. Poliomyelitis

D. Rabies

E. * Tetanus

15. For what infectious pathology is it characteristically (Fig. 54)?

A. Poisoning mushrooms

B. Meningo-encefalit

C. Poliomyelitis

D. Rabies

E. * Tetanus

16. For what infectious pathology is it characteristically (Fig. 55)?

A. Poisoning mushrooms

B. Meningo-encefalit

Page 388: intranet.tdmu.edu.uaintranet.tdmu.edu.ua/data/kafedra/internal/infect_desease... · Web viewBest method to treat diarrhoea in child is: intra venous fluide *ORS antibiotics bowel

C. Poliomyelitis

D. Rabies

E. * Tetanus

17. For what infectious pathology is it characteristically (Fig. 56)?

A. Poisoning mushrooms

B. Meningo-encefalit

C. Poliomyelitis

D. Rabies

E. * Tetanus

18. For what infectious pathology is it characteristically (Fig. 57)?

A. Poisoning mushrooms

B. Meningo-encefalit

C. Poliomyelitis

D. Rabies

E. * Tetanus

19. How is this symptom named (Fig. 51)?

A. * Opisthotonos

B. Emprostotonus

C. Plevrostotonus

D. Risus sardonicus

E. Lockjaw

20. How is this symptom named (Fig. 53)?

A. Opisthotonos

B. Emprostotonus

C. Plevrostotonus

D. Risus sardonicus

E. Lockjaw

21. How is this symptom named (Fig. 54)?

A. Opisthotonos

B. Emprostotonus

C. Plevrostotonus

D. Risus sardonicus

Page 389: intranet.tdmu.edu.uaintranet.tdmu.edu.ua/data/kafedra/internal/infect_desease... · Web viewBest method to treat diarrhoea in child is: intra venous fluide *ORS antibiotics bowel

E. * Lockjaw

22. How is this symptom named (Fig. 55)?

A. Opisthotonos

B. Emprostotonus

C. Plevrostotonus

D. * Risus sardonicus

E. All right

23. How is this symptom named (Fig. 56)?

A. Opisthotonos

B. Emprostotonus

C. Plevrostotonus

D. All right

E. * Lockjaw

24. How is this symptom named (Fig. 57)?

A. Opisthotonos

B. Emprostotonus

C. Plevrostotonus

D. * Risus sardonicus

E. Lockjaw

25. How is this symptom named (Fig. 58)?

A. * Lorin-Epshteyn

B. Blyumberg

C. Murson

D. Rozenberg

E. Koplik

26. Man V., 26 years, has HIV infection 5 years. What is your diagnosis (Fig. 5)?

A. HIV infection. Pneumococal pneumonia

B. HIV infection. Megakaryoblastoma

C. * HIV infection. Pneumocystis pneumonia

D. HIV infection. Visceral leshmaniasis

E. HIV infection. Pneumococal pneumonia. Megakaryoblastoma

27. Man V., 26 years, has HIV infection 5 years. What is your diagnosis (Fig. 6)?

Page 390: intranet.tdmu.edu.uaintranet.tdmu.edu.ua/data/kafedra/internal/infect_desease... · Web viewBest method to treat diarrhoea in child is: intra venous fluide *ORS antibiotics bowel

A. Dementsia of AIDS

B. Patient has еncephalopathy (AIDS-related complex)

C. Rabies

D. Anxiously depressed syndrome for HIV infection

E. * Toxoplasmosis

28. Put a preliminary diagnosis (Fig. 40)?

A. * Erysipelas

B. Phlegmon

C. Sepsis

D. Herpetic infection

E. Erysipelotrix

29. Put a preliminary diagnosis (Fig. 41)?

A. * Erysipelas, erytematous form

B. Erysipelas, erytematous-bulous form

C. Erysipelas, erytematous-hemorrhagic form

D. Erysipelas, bulous-hemorrhagic form

E. Erysipelas, bulous form

30. Put a preliminary diagnosis (Fig. 42)?

A. * Erysipelas, erytematous form

B. Erysipelas, erytematous-bulous form

C. Erysipelas, erytematous-hemorrhagic form

D. Erysipelas, bulous-hemorrhagic form

E. Erysipelas, bulous form

31. Put a preliminary diagnosis (Fig. 44)?

A. Erysipelas, erytematous form

B. * Erysipelas, erytematous-bulous form

C. Erysipelas, erytematous-hemorrhagic form

D. Erysipelas, bulous-hemorrhagic form

E. Erysipelas, bulous form

32. Put a preliminary diagnosis (Fig. 45)?

A. * Erysipelas, erytematous form

B. Erysipelas, erytematous-bulous form

Page 391: intranet.tdmu.edu.uaintranet.tdmu.edu.ua/data/kafedra/internal/infect_desease... · Web viewBest method to treat diarrhoea in child is: intra venous fluide *ORS antibiotics bowel

C. Erysipelas, erytematous-hemorrhagic form

D. Erysipelas, bulous-hemorrhagic form

E. Erysipelas, bulous form

33. Put a preliminary diagnosis (Fig. 46)?

A. Erysipelas, erytematous form

B. Erysipelas, erytematous-bulous form

C. Erysipelas, erytematous-hemorrhagic form

D. * Erysipelas, bulous-hemorrhagic form

E. Erysipelas, bulous form

34. Put a preliminary diagnosis (Fig. 47)?

A. Erysipelas, erytematous form

B. Erysipelas, erytematous-bulous form

C. Erysipelas, erytematous-hemorrhagic form

D. * Erysipelas, bulous-hemorrhagic form

E. Erysipelas, bulous form

35. Put a preliminary diagnosis (Fig. 49)?

A. * Erysipelas

B. Phlegmon

C. Sepsis

D. Herpetic infection

E. Erysipelotrix

36. Put a preliminary diagnosis (Fig. 50)?

A. * Erysipelas

B. Phlegmon

C. Sepsis

D. Herpetic infection

E. Erysipelotrix

37. Put a previous diagnosis (Fig. 61)?

A. Viral hepatitis

B. Leptospirosis

C. Malaria

D. Amebiaz, visceral form

Page 392: intranet.tdmu.edu.uaintranet.tdmu.edu.ua/data/kafedra/internal/infect_desease... · Web viewBest method to treat diarrhoea in child is: intra venous fluide *ORS antibiotics bowel

E. All is right

38. Specify the measures of urgent prophylaxis of this desease (Fig. 15).

A. Anti-anthrax bacteriofag

B. * Penicillinum or tetracyclinum during 5 days

C. Vaccination

D. Medical supervision

E. Biseptolum 5 days

39. Specify the measures of urgent prophylaxis of this desease (Fig. 16).

A. Anti-anthrax bacteriofag

B. * Penicillinum or tetracyclinum during 5 days

C. Vaccination

D. Medical supervision

E. Biseptolum 5 days

40. Specify the measures of urgent prophylaxis of this desease (Fig. 18).

A. Anti-anthrax bacteriofag

B. * Penicillinum or tetracyclinum during 5 days

C. Vaccination

D. Medical supervision

E. Biseptolum 5 days

41. The duration of incubation period of the most possible disease (Fig. 20) is:

A. 3 to 8 days;

B. 2 to 12 days;

C. 2 to 10 days;

D. 1 to 8 days.

E. * 2 to 6 days;

42. The duration of incubation period of the most possible disease (Fig. 21) is:

A. 3 to 8 days;

B. 2 to 12 days;

C. 2 to 10 days;

D. 1 to 8 days.

E. * 2 to 6 days;

43. The duration of incubation period of the most possible disease (Fig. 22) is:

Page 393: intranet.tdmu.edu.uaintranet.tdmu.edu.ua/data/kafedra/internal/infect_desease... · Web viewBest method to treat diarrhoea in child is: intra venous fluide *ORS antibiotics bowel

A. 3 to 8 days;

B. 2 to 12 days;

C. 2 to 10 days;

D. 1 to 8 days.

E. * 2 to 6 days;

44. The experts of WHO consider suspicious in relation to AIDS (Fig. 12):

A. Increase 2 and more lymph nodes in two anatomical groups (except for inguinal), by sizes more than 2 cm in diameter, which lasts more than 2 months

B. Increase 3 and rmore lymph nodes in two anatomical topographic groups (except for inguinal) by sizes more than 2 cm in diameter, which lasts more than 3 months

C. Increase 2 and more lymph nodes in two аnatomical groups (except for inguinal), by sizes more than 1 cm in diameter, which lasts more than 2 months

D. * Increase 2 and more lymph nodes in two аnatomical groups (except for inguinal), by sizes more than 1 cm in diameter, which lasts more than 3 months

E. Increase 3 and more lymph nodes in two anatomical groups (except for inguinal), by sizes more than 2 cm in diameter, which lasts more than 1 months

45. The experts of WHO consider suspicious in relation to AIDS (Fig. 7):

A. Loss of weight (9 % and more)

B. Loss of weight (5 % and more)

C. Loss of weight (6 % and more)

D. * Loss of weight (10 % and more)

E. Loss of weight (3 % and more)

46. The experts of WHO consider suspicious in relation to AIDS (Fig. 8):

A. Diarhea which lasts more than 6 months

B. Diarhea which lasts more than 4 months

C. Diarhea which lasts more than 2 months

D. Diarhea which lasts more than 3 months

E. * Diarhea which lasts more than 1 months

47. What cells are infected by this virus (Fig. 1)?

A. CD 1

B. CD 2

C. CD 3

D. * CD 4

E. CD 5

Page 394: intranet.tdmu.edu.uaintranet.tdmu.edu.ua/data/kafedra/internal/infect_desease... · Web viewBest method to treat diarrhoea in child is: intra venous fluide *ORS antibiotics bowel

48. What clinical features of sarcoma Kaposhi in such (Fig. 2) patients?

A. A sarcoma metastasis in internal and marked high lethality

B. Pouring out with necrosis and ulceration

C. Primary elements appear on a head and trunk

D. Will strike the persons of young and middle age

E. All adopted features

49. What clinical features of sarcoma Kaposhi in such (Fig. 2) patients?

A. * A sarcoma metastasis in internal and marked high lethality

B. Pouring out without necrosis and ulceration

C. Primary elements appear on a leg

D. Will strike the persons of old age

E. All adopted features

50. What clinical features of sarcoma Kaposhi in such (Fig. 2) patients?

A. A sarcoma do not metastasis in internal and marked high lethality

B. * Pouring out with necrosis and ulceration

C. Primary elements appear on a leg

D. Will strike the persons of old age

E. All adopted features

51. What clinical features of sarcoma Kaposhi in such (Fig. 2) patients?

A. A sarcoma do not metastasis in internal and marked high lethality

B. Pouring out without necrosis and ulceration

C. * Primary elements appear on a head and trunk

D. Will strike the persons of old age

E. All adopted features

52. What clinical features of sarcoma Kaposhi in such (pic. 2) patients?

A. A sarcoma do not metastasis in internal and marked high lethality

B. Pouring out without necrosis and ulceration

C. Primary elements appear on a leg

D. * Will strike the persons of yung and middle age

E. All adopted features

53. What complication may appear only in patient with HIV infection (Fig. 5)?

A. Pneumococal pneumonia

Page 395: intranet.tdmu.edu.uaintranet.tdmu.edu.ua/data/kafedra/internal/infect_desease... · Web viewBest method to treat diarrhoea in child is: intra venous fluide *ORS antibiotics bowel

B. Megakaryoblastoma

C. * Pneumocystis pneumonia

D. Visceral leshmaniasis

E. Tularemia

54. What drug did use for the treatment of the most possible disease (Fig. 20)?

A. Amoxicillin

B. * Streptomycin

C. Penicillin

D. Biseptol

E. 5-NOK

55. What drug did use for the treatment of the most possible disease (Fig. 21)?

A. Amoxicillin

B. * Streptomycin

C. Penicillin

D. Biseptol

E. 5-NOK

56. What drug did use for the treatment of the most possible disease (Fig. 22)?

A. Amoxicillin

B. * Streptomycin

C. Penicillin

D. Biseptol

E. 5-NOK

57. What drug did use for the treatment of the most possible disease (Fig. 9)?

A. Merapenem

B. Flukonazol

C. * Interferon

D. Vitamin K

E. Levomicetyn

58. What drug did use for the treatment of the most possible disease (Fig. 10)?

A. Merapenem

B. Flukonazol

C. * Interferon

Page 396: intranet.tdmu.edu.uaintranet.tdmu.edu.ua/data/kafedra/internal/infect_desease... · Web viewBest method to treat diarrhoea in child is: intra venous fluide *ORS antibiotics bowel

D. Vitamin K

E. Levomicetyn

59. What is optimum preparation for the prophylaxis of relapse of this disease (Fig. 42)?

A. Bicillinum 1

B. Glyukokortikoides

C. Bicillinum 3 and glyukokortikoides

D. * Bicillinum 5

E. Bicillinum 5 and glyukokortikoides

60. What is optimum preparation for the prophylaxis of relapse of this disease (Fig. 50)?

A. Bicillinum 1

B. Glyukokortikoides

C. Bicillinum 3 and glyukokortikoides

D. * Bicillinum 5

E. Bicillinum 5 and glyukokortikoides

61. What is optimum preparation for treatment of this disease (Fig. 40)?

A. * Bicillinum

B. Glyukokortikoides

C. Levomicetin

D. Furagin

E. Tetracyclinum

62. What is optimum preparation for treatment of this disease (Fig. 41)?

A. * Bicillinum

B. Glyukokortikoides

C. Levomicetin

D. Furagin

E. Tetracyclinum

63. What is optimum preparation for treatment of this disease (Fig. 42)?

A. * Bicillinum

B. Glyukokortikoides

C. Levomicetin

D. Furagin

E. Tetracyclinum

Page 397: intranet.tdmu.edu.uaintranet.tdmu.edu.ua/data/kafedra/internal/infect_desease... · Web viewBest method to treat diarrhoea in child is: intra venous fluide *ORS antibiotics bowel

64. What is optimum preparation for treatment of this disease (Fig. 44)?

A. * Bicillinum

B. Glyukokortikoides

C. Levomicetin

D. Furagin

E. Tetracyclinum

65. What is optimum preparation for treatment of this disease (Fig. 45)?

A. * Bicillinum

B. Glyukokortikoides

C. Levomicetin

D. Furagin

E. Tetracyclinum

66. What is optimum preparation for treatment of this disease (Fig. 46)?

A. * Bicillinum

B. Glyukokortikoides

C. Levomicetin

D. Furagin

E. Tetracyclinum

67. What is optimum preparation for treatment of this disease (Fig. 47)?

A. * Bicillinum

B. Glyukokortikoides

C. Levomicetin

D. Furagin

E. Tetracyclinum

68. What is the most possible diagnosis (Fig. 10)?

A. Plague, septic form

B. Anthrax, septic form

C. * Hemorrhagic fever

D. Tularemia, septic form

E. Sepsis

69. What is the most possible diagnosis (Fig. 15)?

A. Plague, skin-bubonic form

Page 398: intranet.tdmu.edu.uaintranet.tdmu.edu.ua/data/kafedra/internal/infect_desease... · Web viewBest method to treat diarrhoea in child is: intra venous fluide *ORS antibiotics bowel

B. * Anthrax, skin form

C. Common carbuncle

D. Tularemia, skin-bubonic form

E. Sepsis

70. What is the most possible diagnosis (Fig. 16)?

A. Plague, skin-bubonic form

B. * Anthrax, skin form

C. Common carbuncle

D. Tularemia, skin-bubonic form

E. Sepsis

71. What is the most possible diagnosis (Fig. 18)?

A. Plague, skin-bubonic form

B. * Anthrax, skin form

C. Common carbuncle

D. Tularemia, skin-bubonic form

E. Sepsis

72. What is the most possible diagnosis (Fig. 20)?

A. * Plague, skin-bubonic form

B. Anthrax, skin form

C. Common carbuncle

D. Tularemia, skin-bubonic form

E. Sepsis

73. What is the most possible diagnosis (Fig. 21)?

A. * Plague, skin-bubonic form

B. Anthrax, skin form

C. Common carbuncle

D. Tularemia, skin-bubonic form

E. Sepsis

74. What is the most possible diagnosis (Fig. 22)?

A. * Plague, skin-bubonic form

B. Anthrax, skin form

C. Common carbuncle

Page 399: intranet.tdmu.edu.uaintranet.tdmu.edu.ua/data/kafedra/internal/infect_desease... · Web viewBest method to treat diarrhoea in child is: intra venous fluide *ORS antibiotics bowel

D. Tularemia, skin-bubonic form

E. Sepsis

75. What is the most possible diagnosis (Fig. 9)?

A. Plague, septic form

B. Anthrax, septic form

C. * Hemorrhagic fever

D. Tularemia, septic form

E. Sepsis

76. What is your diagnosis (Fig. 13)?

A. * Sarcoma Kaposhi

B. Candidos

C. CMV-infection

D. Toxoplasmosis

E. Diphtheria

77. What is your diagnosis (Fig. 2)?

A. * Sarcoma Kaposhi

B. Candidos

C. CMV-infection

D. Toxoplasmosis

E. Diphtheria

78. What is your diagnosis (Fig. 4)?

A. * Sarcoma Kaposhi

B. Toxoplasmosis

C. Chicken pox

D. CMV-infection

E. Small pox

79. What is your diagnosis (Fig. 4)?

A. Leycoplakia

B. * Sarcoma Kaposhi

C. Furunculosis

D. Rubella

E. CMV-infection

Page 400: intranet.tdmu.edu.uaintranet.tdmu.edu.ua/data/kafedra/internal/infect_desease... · Web viewBest method to treat diarrhoea in child is: intra venous fluide *ORS antibiotics bowel

80. What kind of specific prophylaxis should be conducted for this patient which was bitten (Fig. 60)?

A. Gamma-globulin and 18 doses of Kab

B. 12 doses of Kab

C. * 6 doses of Kab

D. Gamma-globulin and 21 dose of Kab

E. 2 doses of Kab

81. What kind of specific prophylaxis should be conducted for this patient which was bitten (Fig. 60)?

A. Gamma-globulin

B. * Vaccine

C. Vaccine and gamma-globulin

D. Gamma-globulin and serum

E. Serum

82. What kind of specific prophylaxis should be conducted for this patient which was bitten (Fig. 60)?

A. Gamma-globulin + tetanus antitoxin

B. * Antyrabies vaccine + tetanus antitoxin

C. Antyrabies vaccine and gamma-globulin

D. Gamma-globulin and serum

E. Serum + tetanus antitoxin

83. What mechanism of transmission of this virus (Fig. 1)?

A. Droplet

B. Fecal-oral

C. Transmissiv

D. * Contact

E. Vertical

84. What specific test is used for diagnostic of this disease (Fig. 15)?

A. Compliment fixation test

B. Indirect hemaglutination test

C. * Coetaneous test with antraxin

D. Hemaglutination test

E. RIFA with anthrax antigen

85. What specific test is used for diagnostic of this disease (Fig. 16)?

A. Compliment fixation test

Page 401: intranet.tdmu.edu.uaintranet.tdmu.edu.ua/data/kafedra/internal/infect_desease... · Web viewBest method to treat diarrhoea in child is: intra venous fluide *ORS antibiotics bowel

B. Indirect hemaglutination test

C. * Coetaneous test with antraxin

D. Hemaglutination test

E. RIFA with anthrax antigen

86. What specific test is used for diagnostic of this disease (Fig. 18)?

A. Compliment fixation test

B. Indirect hemaglutination test

C. * Coetaneous test with antraxin

D. Hemaglutination test

E. RIFA with anthrax antigen